Sei sulla pagina 1di 304

www.nicsoceanbook.

com/
OCEANBOOK@hotmail.com
Question 1 (1 point)
Which of the following transactions would occur in the dealer market?
Student Response: Percent Correct Student Answer Choices
Value Response Response
100.0% a. An investor sells $25,000 of
Government of Canada bonds.
0.0% b. An investor purchases 1,000
shares of Nortel Networks.
0.0% c. An investor sells 500 units of a
Canadian equity mutual fund.
0.0% d. An investor purchases 100
Royal Bank preferred shares.

General feedback: Almost all bonds and debentures are sold through the dealer market. Most common
and preferred shares are sold through the auction market while mutual funds are
usually purchased and sold from the mutual fund company.

Reference: Chapter 1 - Capital Markets and Financial Services - Dealer Markets - The
Unlisted Market
Score: 1/1

Question 2 (1 point)
Which of the following organizations is responsible for regulating and supervising banks, insurance, trust,
loan and investment companies at the federal level?
Student Response: Percent Correct Student Answer Choices
Value Response Response
0.0% a. The Bank of Canada.
0.0% b. The Superintendent of
Financial Institutions.
0.0% c. The Office of the Inspector
General of Canada.
100.0% d. The Office of the
Superintendent of Financial
Institutions.

General feedback: The Office of the Superintendent of Financial Institutions (OSFI) was established in
1987 by legislation that amalgamated the Department of Insurance and the Office of
the Inspector General of Banks. The same legislation established a committee
composed of the Superintendent of Financial Institutions, the Governor of the Bank of
Canada, the Deputy Minister of Finance and the Chairman of the Canada Deposit
Insurance Corporation.

Reference: CSC Textbook: Chapter 1 - Capital Markets and Financial Services -


Regulatory Organizations - The Office of the Superintendent of Financial Institutions
Score: 1/1
www.nicsoceanbook.com/
OCEANBOOK@hotmail.com
Question 3 (1 point)
Which of the following describes the process of an insurance company converting itself from a policyholder
company to a shareholder company?
Student Response: Percent Correct Student Answer Choices
Value Response Response
0.0% a. Forced conversion.
0.0% b. Debt-equity swap.
100.0% c. Demutualization.
0.0% d. Re-characterization.

General feedback: Demutualization is the process by which insurance companies, owned by


policyholders, reorganize into companies owned by shareholders. Policyholders in
effect become shareholders in an insurance corporation. The significance of
demutualization is that it provides insurance companies with access to capital
markets, allowing them to acquire other companies with equity, rather than cash,
making them better able to compete with other financial institutions such as banks.

Reference: CSC Textbook: Chapter 1 - The Capital Markets and Financial Services -
The Canadian Securities Industry - Recent Developments and Future Trends
Score: 1/1

Question 4 (1 point)
The Canadian securities industry is made up of 3 key elements that make it's functioning possible. What
are these 3 components?
Student Response: Percent Correct Student Answer Choices
Value Response Response
100.0% a. Financial products, financial
markets and financial
intermediaries.
0.0% b. Banks, trust companies and
insurance companies.
0.0% c. Federal, provincial and
municipal regulators.
0.0% d. Banks, pension funds and
investment dealers.

General feedback: The three key elements in the securities industry are financial products, financial
markets and financial intermediaries.

Reference: CSC Textbook: Chapter 1 - Capital Markets and Financial Services -


Introduction
Score: 1/1

Question 5 (1 point)
Which of the following statements concerning Alternative Trading Systems (ATS) is false?
www.nicsoceanbook.com/
OCEANBOOK@hotmail.com
Student Response: Percent Correct Student Answer Choices
Value Response Response
0.0% a. ATS lessen market
transparency.
0.0% b. ATS allow for a direct
negotiation of the price.
100.0% c. ATS increase the order flow
through the exchanges.
0.0% d. ATS facilitate global trading.

General feedback: When dealers set up their own ATS and deal from their own inventory of securities,
the number of orders flowing through the exchanges decreases, not increases as in
(c).

Reference: Chapter 1 - Capital Markets and Financial Services - Dealer Markets-The


Unlisted Market - The Unlisted Equity Market - Alternative Trading Systems
Score: 1/1

Question 6 (1 point)
Life insurance companies have historically been active in which of the following markets?
Student Response: Percent Correct Student Answer Choices
Value Response Response
100.0% a. Mortgage and long-term bond
markets.
0.0% b. Over-the-counter international
markets.
0.0% c. Stock exchange markets.
0.0% d. Derivatives markets.

General feedback: Since safety of principal is of prime importance to life insurance companies, they tend
to invest primarily in both mortgage and long-term bond markets.

Reference: Chapter 1 - Capital Markets and Financial Services - Life Insurance


Companies - Products and Services
Score: 1/1

Question 7 (1 point)
How is liquidity ensured in over-the-counter (OTC) trading?
Student Response: Percent Correct Student Answer Choices
Value Response Response
0.0% a. Through the availability of the
particular security on
alternative markets.
www.nicsoceanbook.com/
OCEANBOOK@hotmail.com
100.0% b. Through the willingness of the
market makers to quote bid
and ask prices.
0.0% c. Through the differences among
the quotations of the various
market makers.
0.0% d. Through the visibility of the
settlement amounts.

General feedback: The willingness of the market makers to quote bid and ask prices provides liquidity to
the system, although the market makers do have the right to refuse to trade at the
quoted price.

Reference: Chapter 1 - Capital Markets and Financial Services - Dealer Markets-The


Unlisted Market - The Mechanics of Trading
Score: 1/1

Question 8 (1 point)
What role is an investment firm most likely performing when it completes a sell transaction from its own
inventory of a security that is trading on a listed exchange?
Student Response: Percent Correct Student Answer Choices
Value Response Response
0.0% a. Agent.
0.0% b. Broker.
100.0% c. Principal.
0.0% d. Underwriter.

General feedback: The firm is functioning as a principal for this transaction. If the firm were functioning
as a broker and/or an agent, it would not have owned the security, but instead would
have been strictly functioning as an intermediary. As the shares had been trading on a
listed exchange for several years, it is unlikely that the firm would have been
performing the role of an underwriter.

Reference: CSC Textbook: Chapter 1 - Capital Markets and Financial Services - The
Canadian Securities Industry - Principal and Agency Functions
Score: 1/1

Question 9 (1 point)
The wealth management division of a securities firm most likely focuses on which of the following aspects
of business?
Student Response: Percent Correct Student Answer Choices
Value Response Response
0.0% a. Trading activities.
0.0% b. Institutional sales.
0.0% c. International sales.
www.nicsoceanbook.com/
OCEANBOOK@hotmail.com
100.0% d. Retail clients and small
businesses.

General feedback: The wealth management division focuses on retail clients and small businesses, both
from a banking perspective and a securities perspective.

Reference: Chapter 1 - Capital Markets and Financial Services - The Canadian


Securities Industry - Organization of Firms
Score: 1/1

Question 10 (1 point)
What is the term used to describe a fund that continuously issues shares to investors and redeems these
shares at net asset value?
Student Response: Percent Correct Student Answer Choices
Value Response Response
100.0% a. Open-end fund.
0.0% b. Closed-end fund.
0.0% c. Unrestricted fund.
0.0% d. Registered fund.

General feedback: Open-end funds or Mutual funds continuously issues shares to investors and redeems
these shares at net asset value.

Reference: Chapter 1 - Capital Markets and Financial Services - The Role of Financial
Intermediaries
Score: 1/1

Question 11 (1 point)
Which of the following events would likely prompt an increase in Canadian interest rates?
Student Response: Percent Correct Student Answer Choices
Value Response Response
100.0% a. A positive output gap.
0.0% b. A negative output gap
0.0% c. An increase in potential
output.
0.0% d. A decrease in potential output.

General feedback: The output gap measures inflation pressures in the economy by looking at the
difference between real GDP and potential GDP. A negative output gap occurs when
the economy is operating below capacity – inflationary pressures are low. A positive
output gap occurs when the economy is operating above capacity. At this point,
inflationary pressures are on the rise. When this occurs, the central bank would like
respond by raising short-term interest rates. Lenders want to be compensated for
inflation and inflation expectations, therefore, rising inflation will likely result in
www.nicsoceanbook.com/
OCEANBOOK@hotmail.com
increased interest rates.

Reference: CSC Textbook: Chapter 2 - The Canadian Economy - Interest Rates


Score: 1/1

Question 12 (1 point)
Overnight money is currently trading above the target of the operating band. The Bank of Canada wants
to implement a strategy to offset the impact this may have on the economy. What type of open market
operation would you recommend?
Student Response: Percent Correct Student Answer Choices
Value Response Response
0.0% a. An SRA.
100.0% b. An SPRA.
0.0% c. A drawdown.
0.0% d. A redeposit.

General feedback: When overnight money is trading above the target of the operating band, the Bank
may believe that this higher rate may dampen economic activity. To combat this, the
Bank intervenes and offers to lend at the upper limit of the operating band through a
Special Purchase and Resale Agreement (SPRA). An SRA, or sale and repurchase
agreement, would be conducted when the overnight rate it trading below the target of
the operating band. Drawdown and redeposits are cash management strategies, not
open market operations.
Reference: CSC Textbook: Chapter 2 - The Canadian Economy – Monetary Policy
Score: 1/1

Question 13 (1 point)
Which of the following is not a major function of the Bank of Canada?
Student Response: Percent Correct Student Answer Choices
Value Response Response
100.0% a. Determine taxation levels.
0.0% b. Conduct monetary policy.
0.0% c. Act as the government's fiscal
agent.
0.0% d. Act for the government in the
issuance and removal of bank
notes.

General feedback: Taxation levels are determined by the government, not the Bank of Canada.

Reference: CSC Textbook: Chapter 2 - The Canadian Economy - Monetary Policy - The
Bank of Canada - Role of the Bank of Canada
Score: 1/1

Question 14 (1 point)
www.nicsoceanbook.com/
OCEANBOOK@hotmail.com
What is the upper limit of the operating band for overnight financing set by the Bank of Canada officially
known as?
Student Response: Percent Correct Student Answer Choices
Value Response Response
100.0% a. The Bank Rate.
0.0% b. The prime rate.
0.0% c. The drawdown rate.
0.0% d. The benchmark Treasury bill
rate.

General feedback: The Bank Rate is the upper limit of the 50 basis point operating band for overnight
financing. Changes to the operating band and` therefore, to the Bank Rate are
announced by the Bank of Canada through a press release.

Reference: CSC Textbook: Chapter 2 - The Canadian Economy - Monetary Policy -


Implementing Monetary Policy
Score: 1/1

Question 15 (1 point)
Over the last two years, the price level for most goods has fallen and the annual change in the Consumer
Price Index (CPI) has been negative. What type of inflation is characterized by this scenario?
Student Response: Percent Correct Student Answer Choices
Value Response Response
0.0% a. Disinflation.
100.0% b. Deflation.
0.0% c. Cost-push inflation.
0.0% d. Potential inflation.

General feedback: Deflation is a sustained fall in prices where the annual change in the CPI is negative
year after year. In fact, deflation is just the opposite of inflation. Falling prices are
generally preferred over rising prices. Goods and services become cheaper, and our
income seems to go a little farther than it used to. Although true in the short-term,
there are negative consequences of deflation.
Reference: CSC Textbook: Chapter 2 - The Canadian Economy – Money and Inflation
Score: 1/1

Question 16 (1 point)
What is the expected longer-term outcome of sustained deflation in the economy?
Student Response: Percent Correct Student Answer Choices
Value Response Response
0.0% a. Rising living standards.
100.0% b. Falling domestic stock prices.
0.0% c. A slower rate of increase in
inflation.
www.nicsoceanbook.com/
OCEANBOOK@hotmail.com
0.0% d. A lower unemployment rate.

General feedback: Deflation occurs when the price level is steadily falling resulting in a negative
Consumer Price Index (CPI). Although lower prices are generally considered good for
the economy in the short-run, there are negative longer term implications. Faced with
lower prices, corporate profit levels fall and for the economy as a whole this leads to
higher unemployment, slower economic growth and consumers shift their focus from
spending to saving. Ultimately, declining company profits will negatively impact stock
prices.
Reference: CSC Textbook: Chapter 2 - The Canadian Economy – Money and Inflation
Score: 1/1

Question 17 (1 point)
Why was growth accounting, one of the techniques for measuring economic development, developed?
Student Response: Percent Correct Student Answer Choices
Value Response Response
100.0% a. To differentiate output growth
per worker due to capital
increase versus output growth
per worker due to
technological growth.
0.0% b. To prove that a higher rate of
savings will not lead to
sustained growth rates over
long periods of time.
0.0% c. To demonstrate that a
necessary condition for growth
opportunities is for workers to
have the necessary skills and
knowledge to handle new
developments in the economy.
0.0% d. To examine the pattern
between applied research and
product development.

General feedback: Growth accounting is a technique that was developed to differentiate output growth
per worker due to capital, versus output growth per worked due to technological
growth.

Reference: Chapter 2 - The Canadian Economy - The Economy in the Long Run - The
Determinants of Economic Growth
Score: 1/1

Question 18 (1 point)
Why does the Bank of Canada use Special Purchase and Resale Agreements (SPRAs)?
Student Response: Percent Correct Student Answer Choices
Value Response Response
100.0% a. To relieve undesired upward
www.nicsoceanbook.com/
OCEANBOOK@hotmail.com
pressure on overnight
financing rates.
0.0% b. To offset undesired downward
pressure on overnight
financing rates.
0.0% c. To relieve undesired upward
pressure on long-term
financing rates.
0.0% d. To offset undesired downward
pressure on long-term
financing rates.

General feedback: Special Purchase and Resale Agreements (SPRAs) are used by the Bank of Canada to
relieve undesired upward pressure on overnight financing rates. To offset undesired
downward pressure on overnight financing rates, the Bank of Canada uses Sale and
Repurchase Agreements (SRAs).

Reference: CSC Textbook: Chapter 2 - The Canadian Economy - Monetary Policy - The
Bank and Monetary Policy - Tools for Implementing Monetary Policy-Open Market
Operations
Score: 1/1

Question 19 (1 point)
The economy is moving into recession and the unemployment rate is rising. Which of the following
represents an automatic stabilizer that will soften the effect of the decrease in wage income?
Student Response: Percent Correct Student Answer Choices
Value Response Response
0.0% a. Income taxes.
0.0% b. Short-term interest rates.
100.0% c. Employment insurance
payments.
0.0% d. Government purchases of
goods and services.

General feedback: Automatic stabilizers are built-in fiscal measures that AUTOMATICALLY move counter
to the business cycle. In other words, they act to offset the effect of the business
cycle. Employment Insurance payments AUTOMATICALLY rise during a recession, and
help to offset some of the lost wage income that results from the unemployment that
occurs during a recession. As workers lose their jobs, they apply for and receive
Employment Insurance, allowing citizens to continue to purchase goods and services,
thus lessening the possible reduction in spending. Taxes are also an automatic
stabilizer, but work through lessening the burden of taxes on the economy, not
through softening the effect of the wage income decrease. Government purchases are
decisions made deliberately by the government, and are not automatic. Thus, even
though a government contract might increase wages, it is not an AUTOMATIC
stabilizer.

Reference: CSC Textbook: Chapter 2 - The Canadian Economy - Fiscal Policy - How
Fiscal Policy Affects the Economy
www.nicsoceanbook.com/
OCEANBOOK@hotmail.com
Score: 1/1

Question 20 (1 point)
How are prices in the economy behaving if it is reported that disinflation is occurring?
Student Response: Percent Correct Student Answer Choices
Value Response Response
0.0% a. Prices have been steadily
falling to the point where the
CPI is negative.
0.0% b. Prices are falling but
inflationary pressures remain
in the economy.
0.0% c. Prices are rising but higher
interest rates are causing
economic growth to slow faster
than expected.
100.0% d. Prices are rising but at a
slower rate than was recorded
over the last several quarters.

General feedback: Disinflation is a decline in the rate at which prices rise, or a decrease in the rate of
inflation. In fact, prices are still rising, but at a slower rate. In contrast, deflation
occurs when the price level is actually steadily falling resulting in a negative Consumer
Price Index (CPI).
Reference: CSC Textbook: Chapter 2 - The Canadian Economy – Money and Inflation
Score: 0/1

Question 21 (1 point)
Which of the following is an example of an "automatic stabilizer"?
Student Response: Percent Correct Student Answer Choices
Value Response Response
100.0% a. Increase in government
payments for employment
insurance during an economic
decline.
0.0% b. Increase in income tax rates in
times of economic decline.
0.0% c. Increase in government
spending in times of economic
growth.
0.0% d. Increase in interest rates in
times of downward trends of
the economy.

General feedback: When unemployment is rising, government payouts for employment insurance
increase and premiums from employers and employees decrease. Thus, government
transfers to persons increase at a time when wage income decreases and soften the
drop in disposable income and spending. Answers (b), (c) and (d) are incorrect and
www.nicsoceanbook.com/
OCEANBOOK@hotmail.com
are usually used to achieve the opposite effect of what is stated.

Reference: CSC Textbook: Chapter 2 - The Canadian Economy - Fiscal Policy - How
Fiscal Policy Affects the Economy
Score: 1/1

Question 22 (1 point)
Which of the following statements about inflation is correct?
Student Response: Percent Correct Student Answer Choices
Value Response Response
0.0% a. Inflation enhances the value of
money.
0.0% b. M1 is the leading indicator for
the rate of inflation.
0.0% c. Higher inflation improves real
asset returns.
100.0% d. Rising inflation decreases the
standard of living.

General feedback: Inflation results in the level of prices rising, which causes money to lose its value as
more funds are required to buy fewer goods. In other words, the value of money and
the real value of investments is eroded. M1 gives information on an economy's future
level of production while M2 provides a leading indicator of future inflation levels.
Reference: Chapter 2 – The Canadian Economy – Interest Rates and Inflation
Score: 1/1

Question 23 (1 point)
According to the Keynesian theory, when a country's economy is strong, what should its government do?
Student Response: Percent Correct Student Answer Choices
Value Response Response
0.0% a. Increase spending and run a
budget deficit.
0.0% b. Cut taxes and raise spending
to avoid over-expansion.
100.0% c. Cut spending and raise taxes
to avoid inflation.
0.0% d. Not use taxes to control
economic cycles.

General feedback: According to the Keynesian view, when the economy is strong, by cutting spending
and raising taxes the government may keep demand from exceeding supply in order
to avoid inflation.

Reference: CSC Textbook: Chapter 2 - Economic Theories - Keynesian Theory


Score: 1/1
www.nicsoceanbook.com/
OCEANBOOK@hotmail.com
Question 24 (1 point)
During a particular business day, the direct clearers in the Canadian Payments Association (CPA) demand
a higher quantity of settlement balances from the Bank of Canada than the Bank is willing to provide.
What short-term reaction in the Canadian money market is the Bank of Canada most likely trying to
achieve?
Student Response: Percent Correct Student Answer Choices
Value Response Response
0.0% a. Increase the money supply.
100.0% b. Increase short-term interest
rates.
0.0% c. Decrease short-term interest
rates.
0.0% d. Decrease the foreign exchange
rate.

General feedback: The Bank of Canada can exert a great deal of influence on short-term interest rates by
changing the quantity of settlement balances available to direct clearers. Direct
clearers are the CPA members that participate in the clearing and settling of cheques,
other paper items and electronic transfers between member institutions. By providing
fewer balances than the clearers demand, the Bank is reducing the money supply. The
clearers' actions to equalize their need for balances with the amount the Bank is
willing to provide puts upward pressure on interest rates. Typically, rising interest
rates increase the foreign exchange rate.

Reference: CSC Textbook: Chapter 2 - The Canadian Economy - Monetary Policy - The
Bank and Monetary Policy - Tools for Implementing Monetary Policy - Cash
Management
Score: 1/1

Question 25 (1 point)
The Self-Regulatory Organizations (SROs) require member firms and their investment advisors to comply
with a number of rules. Which of the following is not one of these rules?
Student Response: Percent Correct Student Answer Choices
Value Response Response
0.0% a. Learn the essential facts
relative to every client, order
or account accepted.
0.0% b. Ensure that orders are within
the bounds of good business
practice.
0.0% c. Follow the 'suitability principle'
for any recommendations they
provide to clients.
100.0% d. Ensure that the client's net
worth will increase.

General feedback: The Self-Regulatory Organizations (SROs) require that member firms and their
investment advisors:
www.nicsoceanbook.com/
OCEANBOOK@hotmail.com
* Learn the essential facts relative to every client, order or account accepted (the
'know your client rule')
* Ensure that acceptance of any order for any account is within the bounds of good
business practice
* Ensure that recommendations made for any account are appropriate for the client
and in keeping with his or her investment objective, personal circumstances and
tolerance to bearing risk - the 'suitability principle'

Reference: CSC Textbook: Chapter 3: Financing, Listing and Regulation - Regulation


and Investor Protection - Underlying Principles of Provincial Securities Legislation -
Know Your Client Rule
Score: 1/1

Question 26 (1 point)
Which of the following statements about bought deals is correct?
Student Response: Percent Correct Student Answer Choices
Value Response Response
0.0% a. There is less risk to the
principal dealer.
0.0% b. The spread between the
dealer's cost and the final
selling price is often large.
0.0% c. The distribution network is
often very large.
100.0% d. The dealer acts as a principal
in selling the issue.

General feedback: In bought deals, an investment dealer negotiates with the issuer directly and bids for
a specific new issue of securities. Under a bought deal the dealer assumes the risk of
the position; that is, acts as principal. The details of price and the type of issue is
decided either simultaneously with filing the short form prospectus or shortly
thereafter. Under a bought deal arrangement, the spread between the dealer’s cost
and the final selling price may be as low as one percent of the issue price, well below
traditional financing spreads. Once final regulatory approval is received, the bought
issue is sold by the investment dealer, either as a private placement to a select group
of investors or as a public issue under a short form prospectus.
Reference: Chapter 3 – Financing, Listing and Regulation - Financing
Score: 1/1

Question 27 (1 point)
What is the term used to refer to the issuance of securities by a company that has only previously
obtained financing through private means?
Student Response: Percent Correct Student Answer Choices
Value Response Response
0.0% a. Bought Deal.
0.0% b. Secondary issue.
0.0% c. Private Placement.
100.0% d. Initial public offering (IPO).
www.nicsoceanbook.com/
OCEANBOOK@hotmail.com
General feedback: An IPO is different from any other purchase of securities. Normally, if you purchase
securities, other than open-end mutual funds, they are trading on the secondary
market. With an IPO, you are essentially purchasing securities without a historical
track record, based on the marketing information provided to you by an investment
dealer. What the dealer is able to provide to you is strictly regulated. Whenever a new
issue of securities is to be offered to the public, a prospectus must be prepared in
accordance with the requirements of the relevant provincial acts and securities
commission. The success or failure of the IPO may have significant impact on the
investment dealer. The dealer could be left with large amounts of unsold securities in
inventory, depending on the arrangement of the underwriting (if the IPO is under-
subscribed for example) and also on the company, which may or may not be
successful in raising sufficient capital from the proceeds.

Reference: CSC Textbook: Chapter 3 - Financing, Listing and Regulation - Financing -


The Financing Process - The Method of Offering
Score: 1/1

Question 28 (1 point)
What document allows the dealer or agent for a new issue to determine the extent of possible public
interest prior to the actual distribution?
Student Response: Percent Correct Student Answer Choices
Value Response Response
0.0% a. Greensheet.
0.0% b. Final Prospectus.
0.0% c. Simplified Prospectus.
100.0% d. Preliminary prospectus.

General feedback: The Preliminary Prospectus - often referred to as the Red Herring Prospectus - is
available for distribution to potential purchases prior to the actual decision to
distribute securities. If insufficient interest is apparent, the issue may be cancelled.
The Greensheet is an internal information memorandum, used to give an investment
dealer's personnel background information that could be used in discussions with
potential purchasers. It is not an actual document for distribution to the public. The
Final Prospectus is prepared once the final decision is made to proceed with the issue
- based partly on the feedback received from the Preliminary Prospectus.

Reference: CSC Textbook: Chapter 3 - Financing, Listing and Regulation - Financing -


The Financing Process - Preliminary or Red Herring Prospectus
Score: 1/1

Question 29 (1 point)
Which of the following practices is not considered manipulative or prohibited?
Student Response: Percent Correct Student Answer Choices
Value Response Response
0.0% a. Selling a security at closing in
an effort to increase its market
price.
100.0% b. Providing account information
www.nicsoceanbook.com/
OCEANBOOK@hotmail.com
about the beneficial owner
when requested by a
regulator.
0.0% c. Confirming a transaction where
no trade has been executed.
0.0% d. Accepting an order that
involves no change in the
beneficial ownership of a
security.

General feedback: Exchange rules state that a member firm maintains the identification of the beneficial
owner financially responsible for the account, and must have this information available
at all times, upon the request of a stock exchange.

Reference: CSC Textbook: Chapter 3 - Financing, Listing and Regulation - Regulation


and Investor Protection - Necessity for Ethical Trading and Conduct - Examples of
Unethical Practices
Score: 1/1

Question 30 (1 point)
On which of the following markets can a new security be traded prior to its listing on a recognized
exchange?
Student Response: Percent Correct Student Answer Choices
Value Response Response
0.0% a. On the initial public offering
market.
0.0% b. On the exempt institution
market.
0.0% c. On the private placement
market.
100.0% d. On the over-the-counter grey
market.

General feedback: Sometimes, as a new share issue is brought to market, a market develops for the
security prior to actual exchange listing. This grey market is an unofficial OTC market
comprised of dealers wishing to execute customers' orders as well as to support the
issue until the official listing of the stock on a recognized exchange. The exempt
institution list is a determined list of companies who have different requirements re:
the provision, for example, of prospectuses on certain types of security issues, such
as the private placement market.

Reference: CSC Textbook: Chapter 3 - Financing, Listing and Regulation - The Listing
Process
Score: 1/1

Question 31 (1 point)
You agree to purchase 100 shares of a new equity issue. You receive the prospectus, and, immediately
upon reading it, contact your investment advisor, with whom you had placed the trade, to cancel the
www.nicsoceanbook.com/
OCEANBOOK@hotmail.com
purchase. What statutory right have you exercised?
Student Response: Percent Correct Student Answer Choices
Value Response Response
0.0% a. Right of Rescission.
100.0% b. Right of Withdrawal.
0.0% c. Right of Action for Damages.
0.0% d. Right of Full, True and Plain
Disclosure.

General feedback: In this instance, you are exercising the Right of Withdrawal. Most provincial securities
legislation provides a purchaser the right to withdraw from an agreement to purchase
securities during distribution. This right must be exercised within two business days
after receipt or deemed receipt of a prospectus by giving notice to the vendor or its
agent – in this case, the investment advisor. Open-ended Mutual Funds, which are
considered to be in continuous distribution, always are regulated by this rule.

Reference: CSC Textbook: Chapter 3 - Financing, Listing and Regulation - Regulation


and Investor Protection - Public Company Disclosure and Investor Rights - The
Purchasers' Statutory Rights
Score: 1/1

Question 32 (1 point)
Why might a company choose to distribute its new issue as a private placement?
Student Response: Percent Correct Student Answer Choices
Value Response Response
100.0% a. To avoid preparing and filing a
formal prospectus.
0.0% b. To distribute the securities
more broadly.
0.0% c. To give its employees a
priority in purchasing company
shares.
0.0% d. To ensure its shares will be
listed on a recognized
exchange.

General feedback: If a corporation chooses to distribute or sell a new issue as a private placement, an
exemption may be available so that the preparation and filing of a formal prospectus
may not be required.

Reference: CSC Textbook: Chapter 3 - Financing, Listing and Regulation - Financing -


The Financing Process - The Method of Offering
Score: 1/1

Question 33 (1 point)
What is the general underlying principle of Canadian securities legislation?
www.nicsoceanbook.com/
OCEANBOOK@hotmail.com
Student Response: Percent Correct Student Answer Choices
Value Response Response
0.0% a. Approval of the investment
merits of a particular issue of
securities.
0.0% b. Disapproval of the investment
merits of a particular issue of
securities based on standards
developed by the provincial
administrator.
100.0% c. Full, true and plain disclosure
of all pertinent facts for
securities offered for sale to
the public.
0.0% d. The prosperity of Canadian
investors.

General feedback: The general principle underlying the Canadian securities legislation is not the approval
or disapproval of the investment merits of a particular issue of securities by the
provincial administrator, but rather that of full, true and plain disclosure of all
pertinent facts by those offering the securities for sale to the public.

Reference: CSC Textbook: Chapter 3 - Financing, Listing and Regulation - Regulation


and Investor Protection - Underlying Principles of Provincial Securities Legislation
Score: 1/1

Question 34 (1 point)
Certain issuers may access capital markets without the necessity of preparing a full preliminary and final
prospectus prior to a distribution. By what description is this system recognized?
Student Response: Percent Correct Student Answer Choices
Value Response Response
0.0% a. Simplified Prospectus.
0.0% b. Continuous Disclosure.
0.0% c. Bought Deal Exemption.
100.0% d. Short Form Prospectus
Distribution.

General feedback: The Short Form Prospectus Distribution System (SFPD) has significantly reduced the
costs and time required to bring a new issue to market for certain qualified issuers –
otherwise known as reporting issuers. Such issuers may use a short-form prospectus -
different from the Simplified Prospectus system used by mutual fund companies - that
focuses on matters relating primarily to the securities being distributed, without the
extensive information contained in a full prospectus. Reference: CSC Textbook:
Chapter 3 - Financing, Listing and Regulation - Financing - The Financing Process -
Simplified or Short Form Prospectus
Score: 0/1

Question 35 (1 point)
www.nicsoceanbook.com/
OCEANBOOK@hotmail.com
For which of the following reasons may taxes reported on a company's income statement differ from taxes
reported on the company's tax return?
Student Response: Percent Correct Student Answer Choices
Value Response Response
0.0% a. Because of dividend income
received.
100.0% b. Because of different
amortization or depreciation
methods used.
0.0% c. Because of interest paid on
taxes.
0.0% d. Because of penalties levied on
taxes.

General feedback: Differences in the methods used for amortization or depreciation are only temporary
differences and are balanced over time.

Reference: CSC Textbook: Chapter 4 - Corporations and their Financial Statements -


Understanding The Balance Sheet - Classification of Liabilities - Future Income Taxes
Score: 1/1

Question 36 (1 point)
In which of the following situations may a public company be placed under the control of a voting trust?
Student Response: Percent Correct Student Answer Choices
Value Response Response
0.0% a. When the company is issuing
new shares through a private
placement.
100.0% b. When the company is
undergoing a restructuring due
to financial difficulties.
0.0% c. When the company is
acquiring an international
subsidiary.
0.0% d. When the company is merging
with an unlisted company.

General feedback: A corporation that is undergoing a restructuring due to financial difficulties may be
placed under the control of a few individuals through a voting trust.

Reference: CSC Textbook: Chapter 4 - Corporations and their Financial Statements -


Incorporated Businesses - Voting Trusts
Score: 1/1

Question 37 (1 point)
Which of the following is considered a current liability?
www.nicsoceanbook.com/
OCEANBOOK@hotmail.com
Student Response: Percent Correct Student Answer Choices
Value Response Response
0.0% a. Minority Interest.
0.0% b. Prepaid Expenses.
0.0% c. Future Income Taxes.
100.0% d. Accounts payable

General feedback: Current liabilities are liabilities that will be paid off in the near future, usually within
one year. Accounts payable are liabilities arising from the purchase of goods and
services. They’re considered a current liability because the company expects to pay
them off within one year. Both minority interest and future income taxes are
considered longer term liabilities.

Reference: CSC Textbook: Chapter 4 - Corporations and their Financial Statements -


Understanding The Balance Sheet - Classification of Assets - Current Assets
Score: 1/1

Question 38 (1 point)
AMZ Corporation is a large Canadian multi-national, with divisions operating in a number of geographical
areas and different industries. What kind of reporting within the financial statements would reveal the
scope of the various enterprises?
Student Response: Percent Correct Student Answer Choices
Value Response Response
0.0% a. Consolidated Balance Sheet.
0.0% b. Consolidated Statement of
Earnings.
0.0% c. Cash Flow Statement,
Operating Activities.
100.0% d. Notes to the Financial
Statements.

General feedback: The Notes to the Financial Statements are valuable addendums to the financial
statements. Rather than incorporating certain detailed reporting with the statements
themselves, the Notes are used to report significant items, not required to be included
on other statements, such as Segmented Results. An examination of this report allows
you to determine a company's exposure to different currency risks, the various
industries it is involved in, etc. - in each of its different segments. In general, if you
are doing an in-depth analysis of a company, you should closely review the Notes
section, as that is where you will find a great deal of potentially significant information
that would not be disclosed by a simple review of the financial statements.

Reference: CSC Textbook: Chapter 4 - Corporations and their Financial Statements -


Other Information in the Annual Report - Notes to the Financial Statements
Score: 0/1

Question 39 (1 point)
Which of the following information is not provided in the earnings statement of a company?
www.nicsoceanbook.com/
OCEANBOOK@hotmail.com
Student Response: Percent Correct Student Answer Choices
Value Response Response
0.0% a. How the income is spent.
100.0% b. The value of assets
accumulated.
0.0% c. Where the income is coming
from.
0.0% d. The adequacy of income to
support successful operations.

General feedback: The earnings statement of a company provides the following information about the
company: where the income is coming form and how it is spent, the adequacy of
earnings both to assure the successful operation of the company and to provide
income to the holders of its securities. The amount/value of the assets accumulated is
shown on the Balance Sheet.

Reference: CSC Textbook: Chapter 4 - Corporations and their Financial Statements -


Understanding The Earnings Statement - What It Shows
Score: 0/1

Question 40 (1 point)
Which of the following items measures the margin of profit between the cost of goods produced for sale
and net sales?
Student Response: Percent Correct Student Answer Choices
Value Response Response
0.0% a. Cost of goods sold.
0.0% b. Cost of goods manufactured.
100.0% c. Gross operating profit.
0.0% d. Operating profit.

General feedback: Gross operating profit is the difference between net sales and the cost of goods sold.

Reference: CSC Textbook: Chapter 4 - Corporations and their Financial Statements -


Understanding The Earnings Statement - The Operating Section - Gross Operating
Profit
Score: 1/1

Question 41 (1 point)
Company A owns 35% of the common shares of Company B and Company B earned income of $100
million and paid a common dividend of $20 million. How would the ownership of Company B be reflected
and what amount of income would be reflected on the Income Statement of Company A?
Student Response: Percent Correct Student Answer Choices
Value Response Response
0.0% a. Minority Interest of $42
million.
www.nicsoceanbook.com/
OCEANBOOK@hotmail.com
0.0% b. Dividend Income of $7 million.
100.0% c. Equity Income of $35 million.
0.0% d. Company A would not report
its ownership interest of
Company B on its Income
Statement.

General feedback: If a parent company (Company A) owns between 20% and 50% of the voting shares
of a subsidiary (Company B), the parent will use the equity accounting method to
reflect its ownership of the net income of the subsidiary. The parent's percentage
ownership of the net income of the subsidiary is added to the parent's Income
Statement using the Equity Income account. In this case, Company A will add Equity
Income of $35 million ($100 million x 35%) to its Income Statement.

Reference: CSC Textbook: Chapter 4 - Corporations and their Financial Statements -


Understanding The Earnings Statement - The Owners' Section
Score: 0/1

Question 42 (1 point)
Which of the following is a disadvantage of incorporating a business?
Student Response: Percent Correct Student Answer Choices
Value Response Response
0.0% a. Ability to transfer ownership.
0.0% b. Ability to raise capital.
100.0% c. Loss of flexibility.
0.0% d. Continuity of existence.

General feedback: A major disadvantage of incorporation is the loss of flexibility due to the many rules
imposed by various legal statutes. The ability to transfer ownership, the ability to
raise capital and the continuity of existence are all considered advantages of
incorporation.

Reference: CSC Textbook: Chapter 4 - Corporations and their Financial Statements -


Incorporated Businesses - Disadvantages of Incorporation
Score: 1/1

Question 43 (1 point)
Throughout the year, XYZ Inc. has experienced a significant drop in the price of the materials used in the
production of its main product. XYZ. uses the FIFO method for valuing its inventory. During the year, what
will the company likely report?
Student Response: Percent Correct Student Answer Choices
Value Response Response
0.0% a. Higher inventory and lower
income than if it had used the
LIFO method.
0.0% b. Higher inventory and higher
www.nicsoceanbook.com/
OCEANBOOK@hotmail.com
income than if it had used the
LIFO method.
0.0% c. Lower inventory and higher
income than if it had used the
LIFO method.
100.0% d. Lower inventory and lower
income than if it had used the
LIFO method.

General feedback: In periods of decreasing prices the FIFO method will produce lower inventory and
lower income than the LIFO method.

Reference: CSC Textbook - Chapter 4 - Corporations and their Financial Statements -


Understanding The Balance Sheet - Classsification of Assets - Current Assets
Score: 0/1

Question 44 (1 point)
A company uses cash on hand to build a warehouse and stock it with inventory. How will these
transactions be classified when they are accounted for in the cash flow statement?
Student Response: Percent Correct Student Answer Choices
Value Response Response
0.0% a. As investing activities.
0.0% b. As operating and financing
activities.
100.0% c. As operating and investing
activities
0.0% d. As operating, investing and
financing activities.

General feedback: The purchase of capital assets, which include property, plant and equipment, are
classified as investing activities on the cash flow statement. Changes in inventory are
classified as operating activities. If the company had borrowed money to finance the
construction of the warehouse, that would have been classified as a financing activity.

Reference: CSC Textbook: Chapter 4 - Corporations and their Financial Statements -


Understanding The Cash Flow Statement
Score: 1/1

Question 45 (1 point)
Bond A has a coupon of 10% and a maturity date of Jan. 1, 2015. Bond B has a coupon of 8% and a
maturity date of Jan. 1, 2015. If interest rates fall by 1%, what would be the expected comparative
changes in price for the 2 bonds, ignoring any other considerations?
Student Response: Percent Correct Student Answer Choices
Value Response Response
0.0% a. The price change will be
roughly the same amount for
both bonds.
www.nicsoceanbook.com/
OCEANBOOK@hotmail.com
0.0% b. The price of bond A will
increase more than the price of
bond B.
100.0% c. The price of bond B will
increase more than the price of
bond A.
0.0% d. The price of bond A will
decrease more than the price
of bond B.

General feedback: As interest rates move inversely with bond prices, the prices of the bonds will rise
when interest rates fall. Despite the identical maturity dates, the differing coupon
rates mean that changes in interest rates will affect the two bonds differently. As Bond
A has a higher coupon rate than Bond B, interest rate changes will affect the price of
the bond less than they will the price of Bond B. Note: Differences in credit quality,
etc., could also affect the relative change, but the question directed you to ignore any
such considerations.

Reference:
CSC Textbook: Chapter 5 - Fixed Income Securities - Bond Pricing Principles - Fixed-
Income Pricing Properties - Bond Prices are More Volatile when Interest Rates are Low
Score: 1/1

Question 46 (1 point)
A Canadian-based company issues a Swiss Franc-denominated bond in the Germany market. How would
this bond issue be classified in the German market?
Student Response: Percent Correct Student Answer Choices
Value Response Response
0.0% a. As a Swiss-Canadian bond.
100.0% b. As a Eurobond.
0.0% c. As a foreign bond.
0.0% d. As an international bond.

General feedback: Eurobonds are issued and sold outside a domestic market and are typically
denominated in a currency other than that of the domestic market. They are issued in
the Eurobond market or the international bond market and can be issued in any
number of different currencies.

Reference: CSC Textbook: Chapter 5 - Fixed Income Securities – Features and


Provisions of Bonds – Other Types of Fixed-Income Securities
Score: 1/1

Question 47 (1 point)
Company A needs to raise capital for its operations. If interest rates are expected to fall in the medium
term, what type of bond is the company most likely to issue?
Student Response: Percent Correct Student Answer Choices
Value Response Response
www.nicsoceanbook.com/
OCEANBOOK@hotmail.com
0.0% a. Extendible bond
100.0% b. Callable bond
0.0% c. Sinking fund bond
0.0% d. Collateral trust bond
??
??
??
??

General feedback: Bond issuers often reserve the right to pay off the bond before maturity, either to take
advantage of lower interest rates, or to use accumulated funds to eliminate interest
charges. A callable bond gives the company the opportunity to retire an outstanding
debt issue before maturity. Reference: CSC Text Chapter 5 – Fixed Income Securities
– Features and Provisions of Bonds.
Score: 1/1

Question 48 (1 point)
Assuming a current discount rate of 9% and semi-annual coupons, what is the present value of the first
coupon of an 8% five-year bond with a $100 par value?
Student Response: Percent Correct Student Answer Choices
Value Response Response
0.0% a. $2.58
100.0% b. $3.83
0.0% c. $3.85
0.0% d. $7.34

General feedback: The formula to calculate the present value of a coupon payment is: PV = FV/(1+r)n
where PV = Present Value, FV= Future Value (the coupon payment amount), r = the
discount rate and n = the number of compounding periods from present to the coupon
being calculated. Here, the rates need to be adjusted to reflect the semi-annual
compounding. r becomes 4.5%, and the FV (the coupon payment amount) becomes
$4.00, which is one-half of the annual coupon of $8.00 (8% x $100). As this is the
first coupon, n would be 1. If we were trying to calculate the 2nd payment, n would be
2, and so on.

Reference:
CSC Textbook: Chapter 5 - Fixed Income Securities - Bond Pricing Principles - The Use
of Present Value
Score: 1/1

Question 49 (1 point)
Fixed-income investors who set out to maximize income when the yield curve is inverted would select
which of the following types of bonds?
Student Response: Percent Correct Student Answer Choices
Value Response Response
0.0% a. long-term bonds
www.nicsoceanbook.com/
OCEANBOOK@hotmail.com
100.0% b. short-term bonds
0.0% c. mid-term bonds
0.0% d. full-term bonds

General feedback: An inverted yield curve implies that short-term rates are currently higher than long-
term rates. To maximize income in this type of yield environment, an investor would
benefit from purchasing short-term bonds. Reference: CSC Text Chapter 5 – Fixed
Income Securities –Bond Pricing Principles.
Score: 0/1

Question 50 (1 point)
To raise additional capital, a company's preference is to issue common shares, but, at present, markets
are more receptive to a debt offering. Additionally, the company would like to minimize the coupon
offered on any bond it offers. Which of the following types of debt security is most appropriate under
these circumstances?
Student Response: Percent Correct Student Answer Choices
Value Response Response
0.0% a. Extendible bond.
0.0% b. Foreign-pay bond.
100.0% c. Convertible debenture.
0.0% d. Floating-rate debenture.

General feedback: All of the above special features might be attached to a bond to make it more
saleable. Choosing which one is most appropriate requires analysis of market
conditions and a company's preferences. In this instance, the company's preference is
to issue equity, but market conditions are unreceptive to such an offering.
Additionally, the company's stated goal is to reduce the interest rate or coupon, they
must offer on the issue. A conversion privilege makes a debenture more marketable
and usually lowers the interest rate that a company must offer. It also enables the
company to raise equity capital indirectly, through the potential future conversion of
the bond to equity. Therefore, while any of the special privileges would make the bond
more attractive to purchasers, and therefore potentially lower the interest rate
offered, the conversion privilege is the most appropriate.

Reference:
CSC Textbook: Chapter 5 - Fixed Income Securities - Types of Bonds - Corporate
Bonds - Convertible Bonds and Debentures
Score: 1/1

Question 51 (1 point)
In a certain type of bond, a dealer acquires a block of high-quality bonds and separates the individual
future-dated interest coupons from the underlying bond's residue. The dealer then sells each coupon and
principal separately at a discount. What is the term used to describe this type of bond?
Student Response: Percent Correct Student Answer Choices
Value Response Response
100.0% a. Strip bonds.
www.nicsoceanbook.com/
OCEANBOOK@hotmail.com
0.0% b. Debentures.
0.0% c. Subordinated bonds.
0.0% d. Collateral trust bonds.

General feedback: Strip bonds.

Reference: CSC Textbook: Chapter 5 - Fixed-Income Securities - Types of Bonds -


Other Types of Fixed Income Securities - Strip Bonds
Score: 1/1

Question 52 (1 point)
Which of the following types of assets are used to secure a collateral trust bond?
Student Response: Percent Correct Student Answer Choices
Value Response Response
0.0% a. Inventory.
100.0% b. Securities.
0.0% c. Land.
0.0% d. Future earnings.

General feedback: A collateral trust bond is secured by a pledge of securities not by a pledge of property.
This type of bond is issued by companies, such as holding companies, that do not
have significant fixed assets but that own the securities of subsidiaries.

Reference: CSC Textbook: Chapter 5 - Fixed Income Securities - Types of Bonds -


Corporate Bonds - Collateral Trust Bonds
Score: 0/1

Question 53 (1 point)
When are floating rate debentures most beneficial to investors?
Student Response: Percent Correct Student Answer Choices
Value Response Response
0.0% a. When interest rates are falling.
100.0% b. When interest rates are rising.
0.0% c. When interest rates are stable.
0.0% d. When interest rates are
government-controlled.

General feedback: When interest rates are rising, the interest paid on floating rate debentures is
adjusted upwards every 6 months, which improves the price and yield of the
debentures.

Reference: CSC Textbook: Chapter 5 - Fixed-Income Securities - Types of Bonds -


Debentures
www.nicsoceanbook.com/
OCEANBOOK@hotmail.com
Score: 1/1

Question 54 (1 point)
Bond D has a duration of 4 and Bond E has a duration of 8. The Bank of Canada decides to pursue a policy
of higher interest rates. What impact will the increase in rates have on each bond?
Student Response: Percent Correct Student Answer Choices
Value Response Response
0.0% a. The price of Bond D will
increase by more than the
price of Bond E.
100.0% b. The price of Bond E will fall by
more than the price of Bond D.
0.0% c. The price of Bond E will fall by
less than the price of Bond D.
0.0% d. The prices of Bond D and Bond
E will fall by the same amount.

General feedback: Duration is a measure of the sensitivity of a bond’s price to changes in interest rates.
The higher the duration of a bond, the more it will react to changes in interest rates.
An increase in rates implies a decrease in the price of a bond. Since the duration of
Bond E is higher than the duration of Bond D, its price will fall by more than the fall in
price of Bond D.

Reference: CSC Textbook: Chapter 5 - Fixed Income Securities - Bond Pricing


Principles - Fixed-Income Pricing Properties – Duration
Score: 1/1

Question 55 (1 point)
A bond that pays interest semi-annually has 3 years left to maturity is purchased today with a yield to
maturity of 8.65%. After the purchase, interest rates begin to rise. Assuming that the bond is held to
maturity, how will the investor’s actual realized return compare to the original yield to maturity?
Student Response: Percent Correct Student Answer Choices
Value Response Response
0.0% a. The return will be the same.
100.0% b. The return will be higher.
0.0% c. The return will be lower.
0.0% d. The return is independent of
the yield to maturity.

General feedback: The following two assumptions must hold in order to realize the stated yield to
maturity (YTM) on the bond: (1) the coupon payments are reinvested at an interest
rate equal to the YTM at the time the bond was purchased, (2) the bond is held to
maturity. If the investor can reinvest the coupon income at 8.65% in each coupon
payment period until the bond matures, then the actual return realized will be equal to
the YTM. However, since interest rates fluctuate, the investor is faced with both
reinvestment risk and interest rate risk. If interest rates rise, the coupons will be
reinvested at a higher rate and the actual yield or return realized on the bond will be
www.nicsoceanbook.com/
OCEANBOOK@hotmail.com
higher than the original YTM when the bond was first purchased.

Reference: CSC Textbook: Chapter 5 - Fixed Income Securities - Bond Pricing


Principles - Fixed-Income Pricing Properties - Reinvestment Risk
Score: 1/1

Question 56 (1 point)
An investor purchases $200,000 par value of a 75-day Treasury bill at a price of $99.25. What would the
investor's yield be if he holds the T-bill until maturity?
Student Response: Percent Correct Student Answer Choices
Value Response Response
100.0% a. 3.68%
0.0% b. 3.65%
0.0% c. 4.12%
0.0% d. 4.86%

General feedback: The yield on a treasury bill is based on the difference between purchase price and
maturity value, as this is the only income an investor receives. To calculate the yield,
the investment income received (100-99.25) is divided by the price paid (99.25) and
then multiplied by (365/term of T-bill) then multiplied by 100. The resultant number is
the annualized return on the investment.

Reference: CSC Textbook: Chapter 5 - Fixed Income Securities - Bond Pricing


Principles - Bond Yield Calculations - Treasury Bill Yield
Score: 1/1

Question 57 (1 point)
The Bank of Canada decides to reduce the demand for credit by raising short-term interest rates. What is
the likely relative effect on the value of 2 bond portfolios—Portfolio A with a duration of 6 and Portfolio B
with a duration of 15, assuming no purchases and sales and ignoring any other considerations?
Student Response: Percent Correct Student Answer Choices
Value Response Response
0.0% a. Portfolio A will increase in
value by more than the
increase in value of Portfolio B.
100.0% b. Portfolio A will decrease in
value by less than the
decrease in value of Portfolio
B.
0.0% c. Portfolio A will increase in
value and Portfolio B will
decrease in value.
0.0% d. Portfolio A and Portfolio B will
both decrease in value by
roughly the same amount.

General feedback: If the Bank of Canada raises the cost of credit, this implies that interest rates
www.nicsoceanbook.com/
OCEANBOOK@hotmail.com
increase. When interest rates increase, bond prices fall. Bond portfolios with lower
durations are less volatile than portfolios with higher durations. Therefore, although
both portfolios would decline in value, the effect would likely be higher on Portfolio B.

Reference:
CSC Textbook: Chapter 5 - Fixed Income Securities - Bond Pricing Principles - Fixed-
Income Pricing Properties - Longer-term Bonds are More Volatile in Price than Shorter-
term Bonds
Score: 1/1

Question 58 (1 point)
Which category of fixed-income securities do long-term Government of Canada bonds fall under?
Student Response: Percent Correct Student Answer Choices
Value Response Response
0.0% a. Bonds
100.0% b. Debentures
0.0% c. Money market instruments
0.0% d. Canada Savings Bonds

General feedback: Government of Canada “bonds” are actually debentures because no assets are
pledged as security. Instead, the securities are backed by the credit worthiness of the
federal government.

Reference: CSC Text Chapter 5 – Fixed Income Securities – Types of Bonds.


Score: 1/1

Question 59 (1 point)
Nick buys $500,000 worth of AA Company 10.0% bonds that mature in 8 years at a price of 102 on
Friday, June 1. Assuming that all days in the current and following weeks are clearing days and the bond
pays interest semi-annually, when would this trade settle?
Student Response: Percent Correct Student Answer Choices
Value Response Response
0.0% a. June 1
0.0% b. June 4
0.0% c. June 5
100.0% d. June 6

General feedback: The knowledge of when a bond settles, depending on its type and time to maturity, is
critical for ensuring that funds and securities are available on the required day, and
that accrued interest is calculated correctly. Here, the bond in question has a term to
maturity of more than 3 years, meaning that it would settle on the 3rd clearing day -
June 6th, as the days in the weekend are not "counted" as clearing days.

Reference: Chapter 5 - Fixed-Income Securities - Delivery, Regulation and Settlement


- Bond Delivery
Score: 1/1
www.nicsoceanbook.com/
OCEANBOOK@hotmail.com
Question 60 (1 point)
What does a prior lien clause in a bond contract pertain to?
Student Response: Percent Correct Student Answer Choices
Value Response Response
0.0% a. The acquisition of property.
0.0% b. The disposition of property.
100.0% c. The issuance of securities.
0.0% d. The redemption of securities.

General feedback: The prohibition of prior lien requires that no securities may be issued by the company
which rank senior to these bonds as a claim on the mortgaged properties.

Reference: CSC Textbook: Chapter 5 - Fixed Income Securities - Features and


Provisions of Bonds - Protective Provisions - Prohibition of Prior Lien
Score: 1/1

Question 61 (1 point)
Which of the following bond obligations would be expected to retire the largest amount of an issue by
maturity?
Student Response: Percent Correct Student Answer Choices
Value Response Response
0.0% a. A purchase fund obligation.
100.0% b. A sinking fund obligation.
0.0% c. A retractable obligation.
0.0% d. A negative pledge obligation.

General feedback: Sinking funds are sums of money that are set aside out of earnings each year to
provide for the repayment of all or part of a debt issue by maturity. Sinking fund
provisions are as binding on the issuer as any mortgage provision. In contrast, a
purchase fund is set up to retire a specified amount of the outstanding bonds or
debentures through purchases in the market, if these purchases can be made at or
below a stipulated price. Therefore, because of the binding nature of a sinking fund, it
will retire a greater portion of the issue than a purchase fund.

Reference: CSC Textbook: Chapter 5 - Fixed Income Securities – Features and


Provisions of Bonds – Sinking Funds and Purchase Funds
Score: 1/1

Question 62 (1 point)
What is the approximate market price of a $1,000 convertible debenture convertible into 30 common
shares, with a coupon of 7%, current market yield of 7.25%, and a current stock price of $45?
Student Response: Percent Correct Student Answer Choices
Value Response Response
www.nicsoceanbook.com/
OCEANBOOK@hotmail.com
0.0% a. $975
0.0% b. $1,250
100.0% c. $1,350
0.0% d. $1,500

General feedback: When the equity value is below the bond value, the convertible price will be based on
its bond value. When the equity value is above the bond value, the convertible price
will be based on the equity value. In this case, the equity value is $1,350 (30 shares x
$45). The bond value is slightly less than $1,000 because the market yield is slightly
higher than the coupon. Recall that bond prices fall as market yields rise. Therefore,
the convertible price will be determined by the equity value and be approximately
equal to $1,350.

Reference:
CSC Textbook: Chapter 5 - Fixed-Income Securities - Bond Pricing Principles - Fixed-
Income Pricing Properties
Score: 1/1

Question 63 (1 point)
From an issuer’s perspective, which of the following types of debt securities would be the least expensive
to maintain over the short term in a high interest rate environment?
Student Response: Percent Correct Student Answer Choices
Value Response Response
0.0% a. Extendible bond.
0.0% b. Foreign-pay bond.
100.0% c. Convertible debenture.
0.0% d. Floating-rate debenture.

General feedback: All of the above special features might be attached to a bond to make it more
saleable. Choosing which one is most appropriate requires analysis of market
conditions and a company's preferences. A conversion privilege makes a debenture
more marketable and usually lowers the interest rate that a company must offer. It
also enables the company to raise equity capital indirectly, through the potential
future conversion of the bond to equity. Therefore, while any of the special privileges
would make the bond more attractive to purchasers, and therefore potentially lower
the interest rate offered, the conversion privilege is the most appropriate.

Reference: CSC Textbook: Chapter 5 - Fixed Income Securities - Types of Bonds -


Corporate Bonds - Convertible Bonds and Debentures
Score: 0/1

Question 64 (1 point)
In what way are Canada Savings Bonds (CSBs) different from most provincial savings bonds?
Student Response: Percent Correct Student Answer Choices
Value Response Response
0.0% a. CSBs can be acquired by
www.nicsoceanbook.com/
OCEANBOOK@hotmail.com
investors on domestic and
international markets, while
provincial savings bonds
cannot.
0.0% b. CSBs are transferable, while
provincial savings bonds are
not.
100.0% c. CSBs can be redeemed any
business day, while provincial
savings bonds can be
redeemed only twice a year.
0.0% d. CSBs are RRSP-eligible, while
certain restrictions apply in
case of provincial savings
bonds.

General feedback: Both CSB and Provincial Savings Bonds are RRSP-eligible. CSB are not marketable,
and thus are not transferable. Purchasers of CSBs must be bona fide Canadian
residents with a Canadian address for registration purposes. Provincial Savings Bonds
can only be purchased by residents of the province. CSBs can be redeemed any
business day, while provincial savings bonds are redeemed twice a year only (e.g.
OSBs are redeemed in June and December only).

Reference: Fixed-Income Securities - Types of Bonds


CSC Textbook: Chapter 5 - Fixed-Income Securities - Types of Bonds - Provincial
Government Securities and Guarantees - Provincial Savings Bonds
Score: 1/1

Question 65 (1 point)
You have completed an Interest Rate Analysis that has lead you to conclude that there will be a bullish
move in interest rates of approximately 0.5%, and a maximum possible bearish change of 1%. Using a
6%, 5-year bond with a 5.2% yield to maturity, you calculate that the expected gain/loss in capital is -
4.14% to +2.16%.

What is the expected range of the change in interest rates?


Student Response: Percent Correct Student Answer Choices
Value Response Response
100.0% a. +1% to -0.5%
0.0% b. -1% to +-0.5%
0.0% c. +1% to +1.5%
0.0% d. -1.5% to 1.0%

General feedback: The critical understanding in this question is that a bullish move in interest rates is a
decline, and a bearish move is an increase. This is based on the inverse relationship
between bond prices and interest rates: as rates rise, bond prices fall, and vice versa.
Here, we see a positive move in interest rates of .5%, meaning a .5% decline, and a
maximum possible negative change of 1%, meaning a 1% increase. Therefore, the
range would be from 1% to -0.5%.
www.nicsoceanbook.com/
OCEANBOOK@hotmail.com
Chapter 9 – The Portfolio Approach – Monitoring the Economy, The Markets, The
Portfolio, and The Client – Fixed-Income Securities
Score: 0/1

Question 66 (1 point)
An investor purchases a 6% semi-annual bond with 2 years to maturity with the intention of holding the
bond to maturity. The yield to maturity (YTM) at the time of purchase was 7.25%. After the purchase,
interest rates decline in each of the two remaining years. At maturity, the yield to maturity quoted at the
time the bond was purchased will be ______ and the overall return on the bond will be ______.
Student Response: Percent Correct Student Answer Choices
Value Response Response
0.0% a. Overstated; higher.
100.0% b. Overstated; lower.
0.0% c. Understated; higher.
0.0% d. Understated; lower.

General feedback: Yield to Maturity is calculated with the assumption that all interest received from
coupon bonds is reinvested or compounded at the same interest rate as was in
existence when the bond was purchased. However, since interest rates fluctuate, it is
unlikely that the investor will be able to reinvest his/her coupon payments at exactly
this rate. If coupon payments are reinvested, on average, at a lower rate, the overall
return on the bond will decrease. As a result, the yield to maturity quoted at the time
the bond was purchased will be overstated.

Reference:
CSC Textbook: Chapter 5 - Fixed Income Securities - Bond Pricing Principles - Bond
Yield Calculations - Approximate Yield to Maturity
Score: 1/1

Question 67 (1 point)
Assuming that the face value of a bond is $100, what is the present value (PV) of a 7-year, annual
coupon, 7.80% bond using a discount rate of 5.00%?
Student Response: Percent Correct Student Answer Choices
Value Response Response
0.0% a. $ 71.07
0.0% b. $100.00
0.0% c. $107.80
100.0% d. $116.20

General feedback: 100


PV of the Principal = = $71.07
(1.05)^7

PV of the Coupon = [7.8 x { 1 - (1/(1.05 ^ 7)}] / (0.05) = $45.13


www.nicsoceanbook.com/
OCEANBOOK@hotmail.com
PV of the bond = $71.07 + $45.13 = $116.20

Reference: CSC Textbook: Chapter 5 - Fixed-Income Securities - Bond Pricing


Principles - The Use of Present Value
Score: 1/1

Question 68 (1 point)
You have purchased 500 shares of B2 Company, which is not a security eligible for reduced margin, for
$1.85 per share, on margin. Ignoring any other costs or commissions, how much must you deposit to
your margin account to meet minimum margin requirements?
Student Response: Percent Correct Student Answer Choices
Value Response Response
0.0% a. $277.50
0.0% b. $462.50
100.0% c. $555.00
0.0% d. $740.00

General feedback: As these shares are not eligible for reduced margin, they do not qualify for the
reduced margin requirement where a broker may provide up to 70% of the current
market value of a security as a loan. For shares trading between $1.75 and $1.99, the
broker may provide up to 40% of the current market value as a loan. Therefore, the
amount you must deposit is the difference between the amount you paid for the
shares and the amount the broker will loan: $925.00 - $370 = $555.00. If the shares
change in price afterwards, you would receive a margin call if the shares decline in
price, or be able to withdraw part of your deposit if you wish if the shares rise in price.
If the shares move into a different price range for loan value, that loan value
percentage will apply for the new calculation. Remember that you pay interest on the
loan from the broker.

Reference:
CSC Textbook: Chapter 6 - Equities - Cash and Margin Accounts - Margin Accounts -
Maximum Loan Values
Score: 0/1

Question 69 (1 point)
A client places a stop-buy order on LUC Corp. at $38. What kind of order does this become once LUC
reaches or exceeds $38 in price?
Student Response: Percent Correct Student Answer Choices
Value Response Response
100.0% a. A limit order.
0.0% b. A market order.
0.0% c. A day order.
0.0% d. A GTC order.

General feedback: Once a stop buy order, or its opposite, the stop loss order, reaches its trigger price,
according to TSX rules it automatically becomes a limit order, in that it will be filled at
a specified price or better.
www.nicsoceanbook.com/
OCEANBOOK@hotmail.com
Reference: CSC Textbook: Chapter 6 - Equities - Buy and Sell Orders - At the Market
Order
Score: 1/1

Question 70 (1 point)
BIG Inc. declared dividends payable on July 8 to shareholders of record on June 17. Anita purchased 200
BIG. shares on June 15. Assuming these dates are all business dates, which of the following statements is
correct?
Student Response: Percent Correct Student Answer Choices
Value Response Response
0.0% a. Anita is entitled to receive the
dividends since she purchased
the stock within the cum rights
period.
100.0% b. Anita is not entitled to receive
the dividends since the trade
will settle after the record
date.
0.0% c. Anita is entitled to receive the
dividends since she purchased
the stock before the ex
dividend date.
0.0% d. Anita is not entitled to receive
the dividends since she
purchased the stock after the
dividends were declared.

General feedback: Since trades settle on the third business day after a trade, Anita’s purchase of shares
2 business days before the record date on June 15 would not settle until the day after
the record date. Therefore Anita would not be a shareholder of record for purposes of
receiving the dividend.

Reference: CSC Textbook: Chapter 6 - Equities - Rights and Advantages of Common


Share Ownership - Dividends - Ex-Dividend/Cum Dividend
Score: 1/1

Question 71 (1 point)
You have very high hopes for the common shares of a new internet company that has just begun trading
on the TSX at $10 a share. You believe that the shares will rise to $50 in the next 30 days. To take
advantage of this opportunity right now, what kind of order would you enter?
Student Response: Percent Correct Student Answer Choices
Value Response Response
0.0% a. A day order.
0.0% b. An any part order.
0.0% c. A fill or kill order.
100.0% d. An at the market order.
www.nicsoceanbook.com/
OCEANBOOK@hotmail.com
General feedback: As with many types of buy and sell orders, the names, which have evolved over the
years, are usually descriptive of the way in which the order works. Here, the correct
term is at the market - meaning buy the shares at the market price, without limit,
regardless of the current price.

Reference: CSC Textbook: Chapter 6 - Equities - Buy and Sell Orders - At the Market
Order
Score: 1/1

Question 72 (1 point)
An investor establishes a margin account with an investment broker and then purchases 300 shares of
ABC Company at $11.00 per share, making the minimum margin deposit. Two weeks later, the price of
ABC shares rises to $15.00 per share. Ignoring any other costs and considerations, how will this price
increase affect the margin deposit required from the investor?
Student Response: Percent Correct Student Answer Choices
Value Response Response
0.0% a. There will be no effect on the
amount of the margin deposit
required.
0.0% b. A margin call will be issued.
100.0% c. The investor may withdraw
funds from the account.
0.0% d. The investor will be required to
close his/her position.

General feedback: When a long position is established on margin, sufficient funds (or securities with
excess loan value) must be in the account to cover the purchase. The dealer lends
some of these funds to the client, the client being responsible for the balance. As the
shares have risen in value, the maximum loan value a broker will give for the shares
increases. This means that the investor's required margin deposit will decrease. The
investor may withdraw the difference between the original deposit and the new
requirement, or leave it in the account.

Reference:
CSC Textbook: Chapter 6 - Equities - Cash and Margin Accounts - Margin Accounts -
Margining Long Positions on Listed Equities
Score: 1/1

Question 73 (1 point)
A client instructs you to purchase 3,000 shares of DAR at $3.50 or better. What type of an order is this?
Student Response: Percent Correct Student Answer Choices
Value Response Response
0.0% a. Market order.
0.0% b. Stop loss order.
100.0% c. Limit order.
0.0% d. Any part order.
www.nicsoceanbook.com/
OCEANBOOK@hotmail.com
General feedback: A limit order is an order in which the client sets a specific price at which the
transaction may be executed; or at a better price if the trader is able to obtain it.

Reference: CSC Textbook: Chapter 6 - Equities - Buy and Sell Orders - Limit Order
Score: 0/1

Question 74 (1 point)
In which of the following ways does the S&P500 overcome some of the disadvantages associated with the
Dow Jones Industrial Average (DJIA)?
Student Response: Percent Correct Student Answer Choices
Value Response Response
0.0% a. The S&P500 is considered less
volatile than the DJIA.
0.0% b. The S&P500 represents a
narrower range of industry
group stocks.
0.0% c. The S&P500 includes American
company stocks and many
foreign stocks for greater
diversification, while the DJIA
is composed only of American
company stocks.
100.0% d. The S&P500 is composed of
industrial, financial, utility and
transportation stocks, while
the DJIA is composed of 30
blue chip company stocks.

General feedback: The DJIA is composed of only 30 blue chip companies and is not truly representative
of broad market activity. The S&P500 is a better indicator of the performance of the
broader market because it is composed of 500 stocks from several sectors - industrial,
financial, utility and transportation.

Reference: CSC Textbook: Chapter 6 - Equities - Stock Indexes and Averages - U.S.
Stock Market Indexes
Score: 1/1

Question 75 (1 point)
Which U.S. index is price-weighted versus being weighted according to market capitalization?
Student Response: Percent Correct Student Answer Choices
Value Response Response
0.0% a. The NASDAQ Composite Index.
0.0% b. The Standard and Poor's 500
Composite Index.
0.0% c. The New York Stock Exchange
Index.
100.0% d. Dow Jones Industrial Average.
www.nicsoceanbook.com/
OCEANBOOK@hotmail.com

General feedback: of the North American markets are weighted to market capitalization, which many
believe is a more reliable weighting scheme. The DJIA is calculated by adding the
prices of each stock and dividing the sum by a specific divisor, so higher priced stocks
carry a heavier weighting. This is referred to as being price-weighted. Taking the price
of each stock in the index and multiplying that value by the number of shares
outstanding for each issue and then adding all of the totals together is how index
levels are calculated based on market capitalization. This results in stocks with the
highest market capitalization having the greatest effect on the level of the index.
Reference: Chapter 6 – Equity Securities – Equity Indexes and Averages
Score: 1/1

Question 76 (1 point)
What is the primary advantage of straight convertible preferred shares over other types of preferred
shares?
Student Response: Percent Correct Student Answer Choices
Value Response Response
0.0% a. The duration of any extendible
privilege.
100.0% b. The outlook for the common
shares.
0.0% c. The amount of the conversion
premium.
0.0% d. The proximity of the call date.

General feedback: The conversion feature is only attractive if there is a positive outlook for the common
shares. The conversion privilege will be valuable if the market price of the common
shares exceeds the conversion price during the life of the conversion privilege.

Reference: CSC Textbook: Chapter 6 - Equities - Types of Preferreds - Convertible


Preferreds
Score: 1/1

Question 77 (1 point)
Company A has a 5.8% cumulative preferred share issue outstanding that pays dividends annually. In
2003 the company paid no dividends and in July 2004, the Board of Directors declared a 6.8% dividend
rate as a partial repayment of the dividends in arrears. John purchased these shares in 2000 and sold
them at the beginning of 2004, before the dividends were declared. What dividend payment is John
entitled to?
Student Response: Percent Correct Student Answer Choices
Value Response Response
0.0% a. Payment at 1.7% dividend
rate.
0.0% b. Payment at 5.8% dividend
rate.
0.0% c. Payment at 6.8% dividend
rate.
www.nicsoceanbook.com/
OCEANBOOK@hotmail.com
100.0% d. John is not entitled to any
dividends.

General feedback: No payments are made to preferred shareholders who previously sold their stock. If a
partial or complete payment of arrears materializes, payment is made only to the
preferred shareholders owning shares at the time of repayment.

Reference: CSC Textbook: Chapter 6: Equities - Preferred Shares - Features of


Preferred Shares - Cumulative and Non-cumulative
Score: 1/1

Question 78 (1 point)
A Canadian Stock Exchange publishes the following dividend announcement:

Payment ($) When Payable Shareholders of Ex Dividend Date


Record

X Ltd. .35 June 1 May 15 May 13

If an investor bought 300 shares of X Ltd. on May 12th, what is the amount of the taxable dividend?
Student Response: Percent Correct Student Answer Choices
Value Response Response
0.0% a. $0.00
0.0% b. $105.00
0.0% c. $113.75
100.0% d. $131.25

General feedback: The investor purchased the shares during the cum-dividend period (i.e. before the ex-
dividend date of May 13), and is entitled to the full dividend of (300 x .35) = $105.
However, the question asks for the taxable amount of the dividend. Individual
taxpayers receive preferential tax treatment on dividends received from taxable
Canadian corporations. The tax calculation requires that first you “gross-up” the actual
dividend received, which in this case would be $105, by 25%. (105 x 1.25) =
$131.25. This is the amount upon which income tax is calculated.

Reference: Chapter 6 - Equities - Rights and Advantages of Common Share Ownership


- Tax Treatment
Score: 1/1

Question 79 (1 point)
In which of the following scenarios would a client be best advised to purchase variable rate preferred
shares over straight preferred shares?
Student Response: Percent Correct Student Answer Choices
Value Response Response
www.nicsoceanbook.com/
OCEANBOOK@hotmail.com
0.0% a. The price of the common
shares is expected to remain
stable.
0.0% b. The dividend on the common
shares is expected to increase.
0.0% c. Market interest rates are
expected to fall.
100.0% d. The inflation rate continues to
show an upward trend.

General feedback: When inflation rates begin to rise, investor expectations should change to include the
possibility that the Bank of Canada will alter monetary policy in favour of higher
interest rates. The dividend paid on a floating rate preferred share will increase as
market interest rates increase according to a specified formula. This is advantageous
because investors will partially maintain the purchasing power of their investment in a
period of increasing interest rates. The price of the floating rate preferred share is less
responsive to interest rate changes than a straight preferred share because the
dividend adjusts to interest rate changes.

Reference: CSC Textbook: Chapter 6 - Equities - Types of Preferreds - Variable or


Floating Rate Preferreds
Score: 0/1

Question 80 (1 point)
As indexes move up and down, which of the following indicators is a more accurate reflection of market
performance?
Student Response: Percent Correct Student Answer Choices
Value Response Response
100.0% a. Percentage change.
0.0% b. Net point change.
0.0% c. Closing values.
0.0% d. Movers and shakers.

General feedback: As indexes move up and down, the percentage change is a more accurate reflection of
market performance than net point changes.

Reference: CSC Textbook: Chapter 6 - Equities - Stock Indexes and Averages -


Canadian Market Indexes - The S&P/TSX Composite Index
Score: 1/1

Question 81 (1 point)
One of your clients has done research into PQR Company that has led her to believe that the company's
profits will be rising sharply over the next few years. As an elderly, conservative, strongly risk-averse
investor, she wishes to benefit from this rise while taking the minimum possible risk. An examination of
the company's capital structure shows that there are outstanding Class A common shares, Series J
participating preferred shares, Series L retractable preferred shares, and Series M cumulative non-
redeemable preferred shares. In which of these classes of shares should she invest to pursue her
investment strategy?
www.nicsoceanbook.com/
OCEANBOOK@hotmail.com
Student Response: Percent Correct Student Answer Choices
Value Response Response
0.0% a. Class A common shares.
100.0% b. Series J participating preferred
shares.
0.0% c. Series L retractable preferred
shares.
0.0% d. Series M cumulative non-
redeemable preferred shares.

General feedback: Common shares are the primary investment choice when an investor wishes to
participate in the rising profits of a company. However, this investor is extremely
uncomfortable with risk. Retractable preferred shares would allow her to sell her
shares back to the company; however, this would not allow her to participate in the
growth of the company's revenues. The cumulative preferreds would help protect her
if the company chose not to pay dividends due to lack of profits; however, if her
research is accurate, this is unlikely. Only one type of preferred share allows an
investor to participate in the growth of a company's revenues through additional
dividend payments- participating preferreds. These shares have certain rights to a
share in the earnings of a company over and above the specified dividend rate.
Therefore, answer choice (B) would be the most appropriate recommendation.

Reference:
CSC Textbook: Chapter 6 - Equities - Types of Preferred Shares - Other Types of
Preferreds - Participating Preferreds
Score: 1/1

Question 82 (1 point)
Which of the following could be considered the "lowest risk" index/average?
Student Response: Percent Correct Student Answer Choices
Value Response Response
0.0% a. The S&P/TSX Composite
Index.
0.0% b. The Standard and Poor's 500
Composite Index.
100.0% c. The Dow Jones Industrial
Average (DJIA).
0.0% d. The New York Stock Exchange
Index.

General feedback: The DJIA is comprised solely of 30 high quality, blue chip stocks which are
fundamentally less volatile securities. Therefore, it is considered to be a lower risk
index as a result of its composition. Because the S&P/TSX Composite Index, the S&P
500 Composite Index and the NYSE Index are comprised of a wider variety and
greater number of equities, many of them being higher risk than the 30 stocks in the
DJIA, they can be considered to be higher risk indexes.
Reference: Chapter 6 – Equity Securities – Equity Indexes and Averages
Score: 1/1
www.nicsoceanbook.com/
OCEANBOOK@hotmail.com
Question 83 (1 point)
Which of the following features of a preferred share is generally most advantageous to an issuer?
Student Response: Percent Correct Student Answer Choices
Value Response Response
100.0% a. Redeemable.
0.0% b. Convertible.
0.0% c. Retractable.
0.0% d. Variable Rate.

General feedback: Each of the above is a special feature that can be attached to a preferred share, or a
bond. When it is the investor who decides whether the security will be redeemed, the
shares are retractable. If it were the issuer, the shares would be redeemable.
Convertible refers to the ability to convert the shares to another security, usually
common shares; variable rate refers to the interest paid on the shares. Retractable is
of benefit to the investor; redeemable to the issuer.

Reference: CSC Textbook: Chapter 6 - Equities - Types of Preferred Shares -


Retractable Preferreds
Score: 1/1

Question 84 (1 point)
Which of the following statements correctly describes the effect on a company's equity value per common
share of a 2-for-1 split of its common shares, ignoring any other considerations?
Student Response: Percent Correct Student Answer Choices
Value Response Response
0.0% a. There will be no effect on the
equity value per common
share.
100.0% b. The equity value per common
share will decrease by 50%.
0.0% c. The equity value per common
share will decrease by 100%.
0.0% d. The equity value per common
share will increase by 50%.

General feedback: Although a stock split does not affect the proportionate ownership of a company for
the holder of the common share, the amount of equity attributable to each common
share does fall as a result of a stock split. As the denominator of the ratio is now twice
as large, the final answer is 50% smaller. However, as the equity value per common
share fell for the same amount for all shareholders, the proportionate ownership
remains constant (the shareholder now owns twice as many shares with 1/2 as much
equity per common share).

Reference: CSC Textbook: Chapter 6 - Equities - Stock Splits and Consolidations


Score: 1/1
www.nicsoceanbook.com/
OCEANBOOK@hotmail.com
Question 85 (1 point)
Ronaldo purchased 1,000 WOW common shares (not eligible for reduced margin) 3 months ago at $2.75
in his margin account. To keep the account in good standing, he deposited the required margin before the
transaction settled. Assume that the stock has sharply declined to $1.80 and Ronaldo has no extra cash in
his account. Excluding any commission charges, what additional amount would Ronaldo have to deposit to
avoid a margin call on the WOW shares?
Student Response: Percent Correct Student Answer Choices
Value Response Response
100.0% a. $ 655
0.0% b. $ 720
0.0% c. $1,375
0.0% d. $2,030

General feedback: At the time of purchase:


Total cost to buy WOW stock (1,000 x $2.75) $2,750
Less: Dealer's maximum loan (50% x 1,000 x $2.75) $1,375
Margin $1,375
Current values:
Original Total cost to buy WOW stock $2,750
Less: Dealer's revised maximum loan (40% x 1,000 x $1.80) $ 720
Gross Margin requirement: $2,030
Less: Client's original margin deposit $1,375
Net margin deficiency $ 655

Reference: CSC Textbook: Chapter 6 - Equities - Cash and Margin Accounts - Margin
Accounts
Score: 1/1

Question 86 (1 point)
An investor’s FAR shares have appreciated quite significantly over the past few months. She is planning to
take a three-month European vacation but wants to protect her paper profit on the FAR shares. What type
of order would you recommend she place on the FAR shares while she is on vacation?
Student Response: Percent Correct Student Answer Choices
Value Response Response
0.0% a. Limit order.
0.0% b. Any part order.
0.0% c. Stop buy order.
100.0% d. Stop loss order.

General feedback: A stop order is one that becomes effective as a limit order when the price of a board
lot of a stock reaches or, in the case of a stop loss order declines below a stated limit
and in the case of a stop buy order, goes above a stated limit. In this case, the
investor would place a stop loss order - in other words, an order that is intended to
act to stop more losses, by selling the investor's shares as soon as possible before the
shares potentially decline further. The stop buy order is used for short sales - the
order is to buy shares to stop further losses as share prices rise (remember that short
www.nicsoceanbook.com/
OCEANBOOK@hotmail.com
sales carry the risk of unlimited losses, as the investor may have to purchase shares
to cover a position at a price far higher than the amount received on the initial short
sale.) Stop orders are protective orders. Limit orders and any part orders are orders
that have specific instructions attached to them for the broker. The limit order states
the specific price at which a trade must be executed (although the broker is allowed
to, if possible, obtain a better price). The any part order means that the investor will
accept a trade that is “any part” of the requested purchase or sale, up to the
maximum amount of shares specified for the trade.

Reference: CSC Textbook: Chapter 6 - Equities - Buy and Sell Orders - Stop Loss and
Stop Buy Orders - Stop Loss Order
Score: 1/1

Question 87 (1 point)
You have purchased 1,000 shares of Pixie Corp., a security not eligible for reduced margin, for $1.80 per
share, on margin. Ignoring any other costs or commissions, how much must you deposit to your margin
account to meet minimum margin requirements?
Student Response: Percent Correct Student Answer Choices
Value Response Response
0.0% a. $462.50
0.0% b. $887.00
100.0% c. $1,080.00
0.0% d. $2.040.00

General feedback: As these shares are not eligible for reduced margin, the broker may provide up to
40% of the current market value as a loan. Therefore, the amount you must deposit is
the difference between the amount you paid for the shares and the amount the broker
will loan: $1,800 × 40% = $720. The client’s margin requirement is $1,080 ($1,800 -
$720). If the shares change in price afterwards, you would receive a margin call if the
shares decline in price, or be able to withdraw part of your deposit if you wish if the
shares rise in price. If the shares move into a different price range for loan value, that
loan value percentage will apply for the new calculation. Remember that you pay
interest on the loan from the broker.

Reference: CSC Textbook: Chapter 6 - Equities – Cash and Margin Accounts – Margin
Accounts – Margining Long Positions in Listed Equities
Score: 1/1

Question 88 (1 point)
A client purchased 2,000 shares of DDD Inc., a security eligible for reduced margin at $12.38 in her
margin account. She already had a credit balance of $2,000 in the account. Assuming no commission was
paid on the transaction and there are no additional securities held long in the account, how much
additional margin would the client have to deposit to keep her account in good standing?
Student Response: Percent Correct Student Answer Choices
Value Response Response
0.0% a. $4,904
100.0% b. $5,428
0.0% c. $7,428
0.0% d. $9,904
www.nicsoceanbook.com/
OCEANBOOK@hotmail.com

General feedback: 2,000 x $12.38 = $24,760 x 30% = $7,428 - $2,000 = $5,428

Reference:
CSC Textbook - Chapter 6 - Equities - Cash and Margin Accounts - Margin Accounts -
Margining Long Positions in Listed Equities
Score: 1/1

Question 89 (1 point)
Which of the following statements about market performance is correct?
Student Response: Percent Correct Student Answer Choices
Value Response Response
0.0% a. The most accurate reflection of
market performance are net
point changes in stock indexes.
100.0% b. The most accurate reflection of
market performance are
percentage changes in stock
indexes.
0.0% c. Currency values overall have
no effect on market
performance.
0.0% d. Relative strength indicators
frequently give false signals
about market performance.

General feedback: Historically, it has been shown that percentage changes in market indexes is a more
accurate reflection of market performance than net point changes. Currency values
can have a definite impact on index changes and relative strength indicators are often
used in the investment community to compare index performance.
Reference: Chapter 6 – Equity Securities – Equity Indexes and Averages
Score: 1/1

Question 90 (1 point)
What margin is required if an investor purchased 5,000 shares of XYZ, a security not eligible for reduced
margin, for $25.75 per share?
Student Response: Percent Correct Student Answer Choices
Value Response Response
0.0% a. $25,750
0.0% b. $38,625
100.0% c. $64,375
0.0% d. $90,125

General feedback: The broker is willing to lend the investor 50% of the market value of the securities in
this example. Therefore, the investor must provide margin of $64,375 (5,000 shares x
www.nicsoceanbook.com/
OCEANBOOK@hotmail.com
$25.75 x 50%).

Reference:
CSC Textbook: Chapter 6 - Equities - Cash and Margin Accounts - Margin Accounts -
Maximum Loan Values
Score: 1/1

Question 91 (1 point)
HNF Company shares are currently trading at $67.20. An investor purchases a September 65 call at
$2.25. At what price will this position become profitable for the investor?
Student Response: Percent Correct Student Answer Choices
Value Response Response
0.0% a. At a price below $62.75
0.0% b. At a price equal to $64.95
100.0% c. At a price above $67.25
0.0% d. At a price above $69.45

General feedback: A call gives an investor the right, but not the obligation, to buy a specific quantity of a
specific underlying security. An investor would purchase a call if he/she expected the
price of the underlying security to rise. When you purchase a call, you pay a price
known as a premium. To earn back your premium, the stock must rise far enough so
that its price is higher than the exercise price by the amount of the premium paid. If
the stock rises past this point, you begin to earn a profit. Here, the exercise price is
$65. At a stock price of $67.25 ($65 + $2.25), the investor breaks even. Above this
price the investor begins to earn a profit on the position.

Reference: CSC Textbook: Chapter 7- Derivatives - Options - Basic Option Strategies


for Individual and Institutional Investors – Buying Put Options
Score: 1/1

Question 92 (1 point)
When does delivery take place on a futures contract?
Student Response: Percent Correct Student Answer Choices
Value Response Response
0.0% a. Never.
100.0% b. At expiry.
0.0% c. When traded.
0.0% d. When exercised.

General feedback: The reality is that only a minute number of futures contracts are ever actually
tendered for delivery. However, the understanding of when it may be delivered is
crucial to understanding the principle behind these instruments. If they were
"available" to be exercised at any time, similar to American style options, then the
concept of using these as a way to guarantee a price in the future for an individual's
production would not be met. Additionally, the price paid for the future contract in
some cases is actually dependent on when the commodity will be "delivered" with
futures contracts for delivery of a commodity during a season when it is in short
www.nicsoceanbook.com/
OCEANBOOK@hotmail.com
supply will likely be more expensive than one for delivery during a period when it is in
abundant supply. Therefore, delivery, if it is actually made, occurs at the stated expiry
date, or as soon as practical thereafter, as specified within the contract specifications
of the futures contract.

Reference: CSC Textbook: Chapter 7 - Derivatives - Forwards - Futures: Key Terms


and Definitions
Score: 1/1

Question 93 (1 point)
Which of the following is an advantage of investing in forward contracts rather than future contracts?
Student Response: Percent Correct Student Answer Choices
Value Response Response
0.0% a. Forward contracts are more
liquid.
100.0% b. Forward contracts can be
tailored to meet the needs of
the parties involved.
0.0% c. Forward contracts are cleared
through a clearing corporation,
which guarantees the
performance of the contract.
0.0% d. Forward contracts have very
little risk of default.

General feedback: A major advantage of investing in forward contracts is that they can be tailored to
meet the needs of the parties involved. On the other hand, forward contracts are
usually more illiquid than futures. Forward contracts are backed only by the credit-
worthiness of the 2 parties and thus have the risk of a default. Futures, not forwards,
are cleared through a clearing corporation, which guarantees the performance of the
contract.

Reference: Derivative Securities - Futures and Forwards


CSC Textbook - Chapter 9 - Derivative Securities - Futures and Forwards
Score: 1/1

Question 94 (1 point)
How are rights and warrants different from call options?
Student Response: Percent Correct Student Answer Choices
Value Response Response
100.0% a. Rights and warrants are
usually issued by a company to
raise capital.
0.0% b. Rights and warrants are
usually issued by one investor
to another.
0.0% c. Rights and warrants are issued
by the Canadian Derivatives
Clearing Corporation (CDCC).
www.nicsoceanbook.com/
OCEANBOOK@hotmail.com
0.0% d. Rights and warrants can be
called by the issuing company.

General feedback: Unlike options, rights and warrants are usually issued by a company as a method of
raising capital.

Reference: CSC Textbook: Chapter 7 - Rights and Warrants


Score: 1/1

Question 95 (1 point)
When can a European-style option be exercised?
Student Response: Percent Correct Student Answer Choices
Value Response Response
0.0% a. Never.
100.0% b. At expiry only.
0.0% c. At any time on or before
expiry.
0.0% d. At any time after expiry.

General feedback: A European-style option can be exercised only at expiry. The other primary type of
option is the American-style option, which can be exercised at any time on or before
expiry.

Reference: CSC Textbook: Chapter 7 - Derivatives - Options - Key Terms and


Definitions
Score: 1/1

Question 96 (1 point)
An investor owns 300 shares of ABC Company, with an average cost per share of $14.25. In order to earn
additional income, he writes 3 ABC 14.75 calls at $2.75 while ABC shares are trading at $14.50. What is
the intrinsic value of the calls?
Student Response: Percent Correct Student Answer Choices
Value Response Response
100.0% a. $0.00
0.0% b. $0.25
0.0% c. $0.50
0.0% d. $2.75

General feedback: An option price, just like a warrant price or a right price, is made up of two factors: a
time value and an intrinsic value. If calls are "out-of-the-money" they do not have an
intrinsic value. Here, as the exercise price of the calls is above the market price of the
shares, they are "out-of-the-money" meaning that the entire $2.75 is time value.
There is no "real" value for an "out-of-the-money" option. You would not pay $14.75
for shares that you could purchase for $14.50. However, if ABC shares rose to $15.00,
www.nicsoceanbook.com/
OCEANBOOK@hotmail.com
you could purchase the shares through exercising the options for $0.25 less.
Assuming that the price of the calls was $2.85 at that time, the intrinsic value would
be $0.25, and the time value $2.60.
Reference: CSC Textbook: Chapter 7 - Derivatives- Options - Writing Call Options
Score: 1/1

Question 97 (1 point)
ABC Company shares are currently trading at $37.50. An investor purchases a Sep 33.50 put at $2.25. At
what price will this position become profitable for the investor?
Student Response: Percent Correct Student Answer Choices
Value Response Response
100.0% a. At a price below $31.25.
0.0% b. At a price equal to $35.25.
0.0% c. At a price above $35.75.
0.0% d. At a price above $39.75

General feedback: A put gives an investor the right, but not the obligation, to sell a specific quantity of a
specific underlying security. An investor would purchase a put if he/she expected the
price of the underlying security to decline. When you purchase a put, you pay a price -
known as a premium. To earn back your premium, the stock must decline far enough
that the exercise price is above the stock price by the premium paid. If the stock
declines past this point, you begin to earn a profit. This position becomes profitable at
a price below $31.25 ($33.50 - $2.25). At $31.25 the investor breaks even. If the
price falls below this price the investor begins to make a profit. The actual decline in
the stock price would be ($37.50 - $31.25) = $6.25. The investor is speculating that
the shares will decline by at least $6.25.

Reference: Derivative Securities - Options


CSC Textbook: Chapter 7 - Derivatives - Options - Buying Put Options
Score: 1/1

Question 98 (1 point)
Which of the following risks does a covered-call writer assume?
Student Response: Percent Correct Student Answer Choices
Value Response Response
0.0% a. Unlimited loss potential if the
stock price increases.
0.0% b. Loss of the entire premium.
0.0% c. No risk because the investor
owns the stock.
100.0% d. Loss of a potential capital gain
if the option is assigned.

General feedback: The call writer owns the stock and must sell it to the call buyer at the strike price if
called. The covered writer loses the potential capital gain if the stock price rises above
the strike price at expiry and is assigned. The writer has potential opportunity loss of
((market price - (strike price + option premium)). If the investor had not written the
www.nicsoceanbook.com/
OCEANBOOK@hotmail.com
call and the market price of the stock increased substantially, the investor could
realize a large capital gain.

Reference: CSC Textbook: Chapter 7 - Derivatives - Options - Writing Call Options


Score: 0/1

Question 99 (1 point)
Jose sells 10 VNC December 27.50 puts at a premium of $2.50 to establish a cash-secured put write when
VNC shares are selling at $27 a share. If Jose is assigned on his put position, what is his effective
purchase price on the VNC shares?
Student Response: Percent Correct Student Answer Choices
Value Response Response
0.0% a. $29.50
0.0% b. $27.50
0.0% c. $27.00
100.0% d. $25.00

General feedback: If Jose is assigned on his short put position, he will have to buy the underlying shares.
The price he must pay is the strike price of the puts, or $27.50 a share. When Jose
wrote the puts, he received a premium of $2.50, so his effective purchase price is
actually $25 a share ($27.50 – $2.50).

Reference: CSC Textbook: Chapter 7 - Derivatives - Options – Basic Option Strategies


for Individual and Institutional Investors – Writing Put Options
Score: 0/1

Question 100 (1 point)


Investors have the following information concerning the warrants for the shares of Company B:

Market Price of Shares Market Price of Exercise Price of


Warrants Warrants

Company B $45 $0.75 $50.00

What is the intrinsic value of the Company B warrants?


Student Response: Percent Correct Student Answer Choices
Value Response Response
0.0% a. -$5.00
100.0% b. $0.00
0.0% c. $0.75
0.0% d. $5.00

General feedback: The intrinsic value is the amount by which a warrant's exercise price is less than the
market price of the stock, and is calculated by subtracting the exercise price from the
market price ($45.00-$50.00) = -$5.00. In this case, the exercise price is above the
price of the stock, meaning that, if you exercised the warrant, you would be paying
www.nicsoceanbook.com/
OCEANBOOK@hotmail.com
more for the stock than what you would have to pay on the open market. Thus, this
warrant has no intrinsic value. Why would the intrinsic value not be -$5.00? If the
calculation is negative, the intrinsic value is deemed to be zero.

Reference: CSC Textbook: Chapter 7 - Derivatives - Rights and Warrants - Warrants -


Valuing Warrants
www.nicsoceanbook.com/
OCEANBOOK@hotmail.com
Question 1 (1 point)
Which of the following types of borrowing is not considered short-term?
Student Response: Percent Correct Student Answer Choices
Value Response Response
0.0% a. Commercial paper.
0.0% b. Bankers' Acceptances.
0.0% c. Bank of Canada purchase and
resale agreements.
100.0% d. Capital from shareholders.

General feedback: Capital from shareholders would be considered long-term and is not a type of
borrowing.

Reference: Chapter 1 - Capital Markets and Financial Services - The Canadian


Securities Industry - Financing Securities Houses
Score: 1/1

Question 2 (1 point)
Which of the following organizations has the primary responsibility for enacting legislation and regulation
to control the securities industry?
Student Response: Percent Correct Student Answer Choices
Value Response Response
100.0% a. The provincial regulators.
0.0% b. The Canadian Investor
Protection Fund.
0.0% c. The Office of the
Superintendent of Financial
Institutions.
0.0% d. The federal government.

General feedback: In Canada, the regulation of the securities business is a provincial responsibility. The
provincial regulators, recognizing the complexity of their task, work with other
regulators, such as the Canadian Investor Protection Fund and the SROs, to maintain
high standards.

Reference: CSC Textbook: Chapter 1 - Capital Markets and Financial Services -


Regulatory Organizations - The Provincial Regulators
Score: 1/1

Question 3 (1 point)
Which of the following examples best describes the process of demutualization?
Student Response: Percent Correct Student Answer Choices
Value Response Response
0.0% a. Two or more Canadian-owned
www.nicsoceanbook.com/
OCEANBOOK@hotmail.com
insurance companies merging
into a single entity.
0.0% b. A Canadian-owned insurance
company acquiring a trust or a
loan company.
100.0% c. An insurance company, owned
by policyholders, reorganizing
into a company owned by
shareholders.
0.0% d. A mutual insurance company
acquiring a Schedule II bank.

General feedback: Demutualization is the process by which an insurance company, owned by


policyholders, reorganizes into a company owned by shareholders. Demutualization
allows an insurance company to acquire other companies with equity, rather than
cash.

Reference: Chapter 1 - Capital Markets and Financial Services - Life Insurance


Companies - Recent Developments and Future Trends
Score: 1/1

Question 4 (1 point)
Which of the following correctly describes the process of underwriting or financing?
Student Response: Percent Correct Student Answer Choices
Value Response Response
0.0% a. Evaluating the market risk of a
specific security.
100.0% b. Purchasing, from a
government body or company,
a new issue of securities on a
given date at a specified price.
0.0% c. Discounting the market rating
of a government or corporate
bond due to changes in the
solvency of the entity.
0.0% d. Securing a bond issue with
financial assets.

General feedback: In the securities business, underwriting or financing has come to mean the purchase
from a government body or company of a new issue of securities on a given date at a
specified price.

Reference: Chapter 1 - Capital Markets and Financial Services - The Canadian


Securities Industry - Dealer, Principal and Agency Transactions -
Underwriting/Financing
Score: 0/1

Question 5 (1 point)
www.nicsoceanbook.com/
OCEANBOOK@hotmail.com
Which of the following statements concerning Alternative Trading Systems (ATS) is false?
Student Response: Percent Correct Student Answer Choices
Value Response Response
0.0% a. ATS lessen market
transparency.
0.0% b. ATS allow for a direct
negotiation of the price.
100.0% c. ATS increase the order flow
through the exchanges.
0.0% d. ATS facilitate global trading.

General feedback: When dealers set up their own ATS and deal from their own inventory of securities,
the number of orders flowing through the exchanges decreases, not increases as in
(c).

Reference: Chapter 1 - Capital Markets and Financial Services - Dealer Markets-The


Unlisted Market - The Unlisted Equity Market - Alternative Trading Systems
Score: 1/1

Question 6 (1 point)
To which of the following bodies would an investor apply to recover funds lost as the result of the
insolvency of a member firm on the TSX?
Student Response: Percent Correct Student Answer Choices
Value Response Response
0.0% a. Toronto Stock Exchange.
0.0% b. Investment Dealers
Association.
100.0% c. Canadian Investor Protection
Fund.
0.0% d. Canadian Depository Insurance
Corporation.

General feedback: The firm would be a member of a Sponsoring Self-Regulatory Organization - in this
case the Toronto Stock Exchange. The Canadian Investor Protection Fund (CIPF) was
set up to protect customers in the event of the insolvency of these firms. The IDA and
the TSX are self-regulatory organizations themselves; the Canadian Depository
Insurance Corporation (CDIC) offers protection to investors in chartered bank
products.

Reference:
CSC Textbook: Chapter 1 - Capital Markets and Financial Services - Regulatory
Organizations - Canadian Investor Protection Fund
Score: 1/1

Question 7 (1 point)
Which of the following represents the Canadian investment industry as both a national regulator and a
trade association?
www.nicsoceanbook.com/
OCEANBOOK@hotmail.com
Student Response: Percent Correct Student Answer Choices
Value Response Response
0.0% a. Toronto Stock Exchange.
0.0% b. Canadian Depository for
Securities.
100.0% c. Investment Dealers
Association.
0.0% d. The Office of the
Superintendent of Financial
Institutions.

General feedback: The IDA (Investment Dealers Association) is charged with "to protect investors and
enhance the efficiency and competitiveness of the Canadian capital markets.” It acts
in a dual role, both as an industry regulator and a trade association. It is important to
know and distinguish between the various self-regulatory and affiliated organizations.
Each serves a different and important purpose, and represent a different segment of
the industry.

Reference: CSC Textbook: Chapter 1 - Capital Markets and Financial Services -


Regulatory Organizations - The Self-Regulatory Organizations
Score: 1/1

Question 8 (1 point)
What important function do financial intermediaries play in the marketplace?
Student Response: Percent Correct Student Answer Choices
Value Response Response
0.0% a. They ensure the integrity and
accuracy of prospectuses.
0.0% b. They arbitrate disputes
between customers and
financial institutions.
0.0% c. They regulate the settlement
dates on securities
transactions.
100.0% d. They assist in the transfer of
capital from savers to users.

General feedback: Financial intermediaries play the important role of facilitating the trading or movement
of the financial instruments that transfer capital between suppliers and users.

Reference: CSC Textbook: Chapter 1 – Capital Markets – Financial Markets


Score: 1/1

Question 9 (1 point)
How is liquidity ensured in over-the-counter (OTC) trading?
www.nicsoceanbook.com/
OCEANBOOK@hotmail.com
Student Response: Percent Correct Student Answer Choices
Value Response Response
0.0% a. Through the availability of the
particular security on
alternative markets.
100.0% b. Through the willingness of the
market makers to quote bid
and ask prices.
0.0% c. Through the differences among
the quotations of the various
market makers.
0.0% d. Through the visibility of the
settlement amounts.

General feedback: The willingness of the market makers to quote bid and ask prices provides liquidity to
the system, although the market makers do have the right to refuse to trade at the
quoted price.

Reference: Chapter 1 - Capital Markets and Financial Services - Dealer Markets-The


Unlisted Market - The Mechanics of Trading
Score: 1/1

Question 10 (1 point)
Which of the following sources of income is not used by Canadian exchanges to meet their operating and
development costs?
Student Response: Percent Correct Student Answer Choices
Value Response Response
0.0% a. Transaction fees paid by
members for transactions
executed.
100.0% b. Fees on the spread between
bid and offer prices.
0.0% c. Annual fees for sustaining a
listing on an exchange.
0.0% d. Fees paid for changes in the
capital structure of a listed
company.

General feedback: Some of the sources of income used to finance the operating and development costs
of an exchange are transaction fees paid by members for orders executed, fees paid
for the original listing of a corporation, annual sustaining fees, fees paid for changes in
the capital structure of a listed company, etc. Fees on the spread between bid and
offering prices are not charged by the Canadian exchanges.

Reference: Chapter 1 - Capital Markets and Financial Services - The Role of Financial
Markets - How Exchanges are Financed
Score: 1/1
www.nicsoceanbook.com/
OCEANBOOK@hotmail.com
Question 11 (1 point)
Canada gives $10,000,000 in aid to third world countries. What account in the balance of payments would
this activity affect, and would it increase or decrease the account?
Student Response: Percent Correct Student Answer Choices
Value Response Response
0.0% a. It would increase the Current
Account.
100.0% b. It would decrease the Current
Account.
0.0% c. It would increase the Capital
Account.
0.0% d. It would decrease the Capital
Account.

General feedback: The key difference to determining if a transaction is a capital or current account
transaction lies in identifying if the transaction results in the acquisition or disposition
of an asset and the right to any income that it earns. Obviously, third-world aid would
not fall into this category. It would, in fact, be a part of the service component of the
current account, in this case as a non-merchandise import. As it is a payment from
the account, it would decrease the balance.

Reference: CSC Textbook: Chapter 2 - The Canadian Economy - External Sector - The
Balance of Payments
Score: 1/1

Question 12 (1 point)
Assume that the prices of Canadian goods stay unchanged. What impact will a decline in the value of the
Canadian dollar have on the volume of exports and imports?
Student Response: Percent Correct Student Answer Choices
Value Response Response
100.0% a. Exports will rise and imports
will fall.
0.0% b. Imports will rise and exports
will fall.
0.0% c. Imports and exports will rise.
0.0% d. Imports and exports will fall.

General feedback: A falling Canadian dollar lowers the price of Canadian exports in foreign markets and
raises the price of imports in Canada. This leads to higher exports and lower imports.

Reference: CSC Textbook: Chapter 2 - The Canadian Economy - The External Sector -
The Exchange Rate and its Impact on the Economy
Score: 1/1

Question 13 (1 point)
Which of the following is not a major function of the Bank of Canada?
www.nicsoceanbook.com/
OCEANBOOK@hotmail.com
Student Response: Percent Correct Student Answer Choices
Value Response Response
100.0% a. Determine taxation levels.
0.0% b. Conduct monetary policy.
0.0% c. Act as the government's fiscal
agent.
0.0% d. Act for the government in the
issuance and removal of bank
notes.

General feedback: Taxation levels are determined by the government, not the Bank of Canada.

Reference: CSC Textbook: Chapter 2 - The Canadian Economy - Monetary Policy - The
Bank of Canada - Role of the Bank of Canada
Score: 1/1

Question 14 (1 point)
How are prices in the economy behaving if it is reported that disinflation is occurring?
Student Response: Percent Correct Student Answer Choices
Value Response Response
0.0% a. Prices have been steadily
falling to the point where the
CPI is negative.
0.0% b. Prices are falling but
inflationary pressures remain
in the economy.
0.0% c. Prices are rising but higher
interest rates are causing
economic growth to slow faster
than expected.
100.0% d. Prices are rising but at a
slower rate than was recorded
over the last several quarters.

General feedback: Disinflation is a decline in the rate at which prices rise, or a decrease in the rate of
inflation. In fact, prices are still rising, but at a slower rate. In contrast, deflation
occurs when the price level is actually steadily falling resulting in a negative Consumer
Price Index (CPI).
Reference: CSC Textbook: Chapter 2 - The Canadian Economy – Money and Inflation
Score: 1/1

Question 15 (1 point)
In general, under what conditions will inflation fall or remain steady?
Student Response: Percent Correct Student Answer Choices
Value Response Response
0.0% a. When actual output exceeds
www.nicsoceanbook.com/
OCEANBOOK@hotmail.com
potential output.
0.0% b. When labour becomes a scarce
resource.
100.0% c. When actual output is below
potential levels.
0.0% d. When companies start
operating well above normal
capacity.

General feedback: When actual output is below potential levels, unemployed workers and unused plant
and equipment can be called into service without having to raise wages or prices.
Inflation will thus generally fall or be steady. Statements (a), (b), and (d) will
generally lead to an increase in inflation.

Reference: CSC Textbook: Chapter 2 - The Canadian Economy - Money and Inflation -
The Causes of Inflation
Score: 1/1

Question 16 (1 point)
If the Consumer Price Index (CPI) was 130.6 in 2003 and 136.9 in 2004, what was inflation rate over the
year?
Student Response: Percent Correct Student Answer Choices
Value Response Response
0.0% a. 4.60
100.0% b. 4.82
0.0% c. 5.31
0.0% d. 6.30

General feedback: To calculate the rate of inflation over a period of time one must subtract the CPI at
the beginning of the period from the CPI at the end of the period and then divide the
result by the CPI at the beginning of the period. In this example the solution can be
derived as follows: (136.9 – 130.6) / 130.6.
Reference: Chapter 2 – The Canadian Economy – Interest Rates and Inflation
Score: 1/1

Question 17 (1 point)
Which of the following is an example of an "automatic stabilizer"?
Student Response: Percent Correct Student Answer Choices
Value Response Response
100.0% a. Increase in government
payments for employment
insurance during an economic
decline.
0.0% b. Increase in income tax rates in
times of economic decline.
0.0% c. Increase in government
www.nicsoceanbook.com/
OCEANBOOK@hotmail.com
spending in times of economic
growth.
0.0% d. Increase in interest rates in
times of downward trends of
the economy.

General feedback: When unemployment is rising, government payouts for employment insurance
increase and premiums from employers and employees decrease. Thus, government
transfers to persons increase at a time when wage income decreases and soften the
drop in disposable income and spending. Answers (b), (c) and (d) are incorrect and
are usually used to achieve the opposite effect of what is stated.

Reference: CSC Textbook: Chapter 2 - The Canadian Economy - Fiscal Policy - How
Fiscal Policy Affects the Economy
Score: 1/1

Question 18 (1 point)
The economy is moving into recession and the unemployment rate is rising. Which of the following
represents an automatic stabilizer that will soften the effect of the decrease in wage income?
Student Response: Percent Correct Student Answer Choices
Value Response Response
0.0% a. Income taxes.
0.0% b. Short-term interest rates.
100.0% c. Employment insurance
payments.
0.0% d. Government purchases of
goods and services.

General feedback: Automatic stabilizers are built-in fiscal measures that AUTOMATICALLY move counter
to the business cycle. In other words, they act to offset the effect of the business
cycle. Employment Insurance payments AUTOMATICALLY rise during a recession, and
help to offset some of the lost wage income that results from the unemployment that
occurs during a recession. As workers lose their jobs, they apply for and receive
Employment Insurance, allowing citizens to continue to purchase goods and services,
thus lessening the possible reduction in spending. Taxes are also an automatic
stabilizer, but work through lessening the burden of taxes on the economy, not
through softening the effect of the wage income decrease. Government purchases are
decisions made deliberately by the government, and are not automatic. Thus, even
though a government contract might increase wages, it is not an AUTOMATIC
stabilizer.

Reference: CSC Textbook: Chapter 2 - The Canadian Economy - Fiscal Policy - How
Fiscal Policy Affects the Economy
Score: 1/1

Question 19 (1 point)
In which of the following ways is the Bank Rate set?
Student Response: Percent Correct Student Answer Choices
www.nicsoceanbook.com/
OCEANBOOK@hotmail.com
Value Response Response
100.0% a. The Bank Rate is set at the
upper limit of the operating
band for overnight financing by
the Bank of Canada.
0.0% b. The Bank Rate is calculated as
the 3-month T-bill rate minus
0.5%.
0.0% c. The Bank Rate is calculated as
the 3-month T-bill rate plus
0.5%.
0.0% d. The Bank Rate is set at the
mid-point of the operating
band for overnight financing by
the Bank of Canada.

General feedback: The Bank Rate is the upper limit of the 50 basis point operating band for overnight
financing. Changes to the operating band and, therefore, to the Bank Rate are
announced by the Bank of Canada through a press release.

Reference: CSC Textbook: Chapter 2 - The Canadian Economy - Monetary Policy - The
Bank and Monetary Policy - Implementing Monetary Policy
Score: 1/1

Question 20 (1 point)
Due to expansion and growth, an economy experiences an unemployment rate that is below the level of
the natural unemployment rate. What will be the likely effect of this level of unemployment on this
economy's inflation rate and interest rates, assuming all other factors remain constant?
Student Response: Percent Correct Student Answer Choices
Value Response Response
0.0% a. Inflation and interest rates will
both fall.
100.0% b. Inflation and interest rates will
both rise.
0.0% c. The inflation rate will fall and
interest rates will rise.
0.0% d. The inflation rate will rise and
interest rates will fall.

General feedback: An economy that experiences a level of economic activity that results in an
unemployment rate of this level is likely in, or moving towards, the peak of the
business cycle - when demand has begun to outstrip the capacity for the economy to
supply it. In the peak, typically inflation and interest rates would both rise.

Reference: CSC Textbook: Chapter 2 - The Canadian Economy - The Economy in the
Short Run
Score: 1/1
www.nicsoceanbook.com/
OCEANBOOK@hotmail.com
Question 21 (1 point)
Which of the following is not considered an automatic economic stabilizer?
Student Response: Percent Correct Student Answer Choices
Value Response Response
0.0% a. Taxes.
0.0% b. Government transfers to
individuals.
100.0% c. Government purchases of
goods and services.
0.0% d. Employment insurance.

General feedback: Automatic stabilizers are built-in fiscal measures that AUTOMATICALLY move counter
to the business cycle. In other words, it acts to offset the effect of the business cycle.
Employment Insurance payments AUTOMATICALLY rise during a recession, and help
to offset some of the lost wage income that results from the unemployment that
occurs during a recession. As workers lose their jobs, they apply for and receive
Employment Insurance, allowing citizens to continue to purchase goods and services,
thus lessening the possible reduction in spending. Taxes are also an automatic
stabilizer, but work through lessening the burden of taxes on the economy, not
through softening the effect of the wage income decrease. Government purchases are
decisions made deliberately by the government, and are not automatic. Thus, even
though a government contract might increase wages, it is not an AUTOMATIC
stabilizer.

Reference: CSC Textbook: Chapter 2 - The Canadian Economy - Fiscal Policy - How
Fiscal Policy Affects the Economy
Score: 1/1

Question 22 (1 point)
An analysis of a country's current economy reveals the following:

• demand has begun to outstrip capacity;


• labour and product shortages are apparent; and
• inflation has begun to rise.

Into which of the following business cycles would this economy be entering?
Student Response: Percent Correct Student Answer Choices
Value Response Response
100.0% a. Peak phase.
0.0% b. Recovery phase.
0.0% c. Expansion phase.
0.0% d. Recession phase.

General feedback: The business cycle has five phases, each of which has typical characteristics. In
recovery, the economy has excess unused capacity; therefore, the indicators are not
suggestive of this phase. In recession, the level of economic activity is declining,
meaning that excess capacity is being created - clearly not the situation here. If your
www.nicsoceanbook.com/
OCEANBOOK@hotmail.com
analysis had shown that growth was steady, as evidenced by stable inflation, rising
corporate profits, steady job creation, and other like activities, it would have
suggested expansion. However, the indication that the economy is in a situation
where it cannot meet all of the demands being made upon it indicates that it is the
final stage of the expansion - the peak.

Reference: CSC Textbook: Chapter 2 - The Canadian Economy - The Economy in the
Short Run - Phases of the Business Cycle
Score: 1/1

Question 23 (1 point)
Which of the following is not a leading business cycle indicator?
Student Response: Percent Correct Student Answer Choices
Value Response Response
0.0% a. Housing starts.
0.0% b. Manufacturers' new orders for
durables.
100.0% c. Level of inventories.
0.0% d. Spot commodity prices.

General feedback: The level of inventories is classified as a lagging indicator, since it changes after the
economy as a whole changes.

Reference: Macroeconomics - The Economy


CSC Textbook: Chapter 2 - The Canadian Economy - The Economy in the Short Run -
Business Cycle Indicators
Score: 0/1

Question 24 (1 point)
According to the Keynesian theory, when a country's economy is strong, what should its government do?
Student Response: Percent Correct Student Answer Choices
Value Response Response
0.0% a. Increase spending and run a
budget deficit.
0.0% b. Cut taxes and raise spending
to avoid over-expansion.
100.0% c. Cut spending and raise taxes
to avoid inflation.
0.0% d. Not use taxes to control
economic cycles.

General feedback: According to the Keynesian view, when the economy is strong, by cutting spending
and raising taxes the government may keep demand from exceeding supply in order
to avoid inflation.

Reference: CSC Textbook: Chapter 2 - Economic Theories - Keynesian Theory


www.nicsoceanbook.com/
OCEANBOOK@hotmail.com
Score: 1/1

Question 25 (1 point)
Why might a company choose to distribute its new issue as a private placement?
Student Response: Percent Correct Student Answer Choices
Value Response Response
100.0% a. To avoid preparing and filing a
formal prospectus.
0.0% b. To distribute the securities
more broadly.
0.0% c. To give its employees a
priority in purchasing company
shares.
0.0% d. To ensure its shares will be
listed on a recognized
exchange.

General feedback: If a corporation chooses to distribute or sell a new issue as a private placement, an
exemption may be available so that the preparation and filing of a formal prospectus
may not be required.

Reference: CSC Textbook: Chapter 3 - Financing, Listing and Regulation - Financing -


The Financing Process - The Method of Offering
Score: 1/1

Question 26 (1 point)
A Company approaches you as an underwriter, seeking your advice on a new security issue. The
company's debt/equity ratio is quite high, while its interest coverage ratio is low. While market interest in
the company's product is intense, its net income and cash flow have, in fact, been fairly low. The
company's balance sheet shows little to no assets, as its product is one of intellectual capital. In your
opinion as an underwriter which of the following security issues would you recommend?
Student Response: Percent Correct Student Answer Choices
Value Response Response
0.0% a. Bonds.
0.0% b. Debentures.
0.0% c. Preferred Shares.
100.0% d. Common Shares.

General feedback: Every parameter of the company as stated above leads towards the same
recommendation: common shares. The company's debt/equity ratio is high, meaning
that it would likely have a problem assuming more debt, particularly as the company
is currently having problems keeping the interest coverage at an appropriate level.
The market is positive on the company, meaning that an equity issue would likely be
well received, at a good premium to market; however, the fact that net income and
cash flow are low means that the commitment to a preferred share issue would be
inappropriate. The company also has no assets, meaning that a secured bond would
not be possible.
www.nicsoceanbook.com/
OCEANBOOK@hotmail.com
Reference: CSC Textbook: Chapter 3 - Financing - The Financing Process - Advice on
Protective Provisions
Score: 1/1

Question 27 (1 point)
A publicly-traded company wants to increase both the number of its outstanding shares and the dollar
amount of its equity. How can this be accomplished?
Student Response: Percent Correct Student Answer Choices
Value Response Response
0.0% a. Through a stock split.
100.0% b. Through a secondary offering
of common stock.
0.0% c. Through stock consolidation.
0.0% d. Through a retractable
debenture offering.

General feedback: A secondary offering of common stock increases the number of common shares
outstanding and the dollar amount of the equity of the company.

Reference: CSC Textbook: Chapter 3 - Financing, Listing and Regulation - Financing


Score: 1/1

Question 28 (1 point)
Who has the right to vote proxies, unless delegated otherwise, for shares held in street form?
Student Response: Percent Correct Student Answer Choices
Value Response Response
0.0% a. Issuing Company.
0.0% b. Investment Dealer.
100.0% c. Beneficial Owner.
0.0% d. Canadian Depository for
Securities.

General feedback: The right to vote is a right that is owned by the beneficial owner of the shares,
although this individual or entity can assign this right to another individual or entity.
Even when shares are held in a street form (e.g., all of Investment Firm ABC's clients
holdings in XYZ common shares are included in one position with the Canadian
Depository for Securities) the individual holders of shares included in this bulk
certificate are the holders of the right to vote. The Investment Dealer or the
management of the issuing company cannot vote on their behalf without specific
direction. In some instances, proxies are forwarded to shareholders containing
language such as "failure to return this proxy will constitute a vote in favour of
management." However, as it is a requirement that proxy materials be forwarded to
all shareholders, even if their shares are part of a bulk certificate, it is still the
shareholder who is making this voting decision.
www.nicsoceanbook.com/
OCEANBOOK@hotmail.com
Reference: CSC Textbook: Chapter 3 - Financing, Listing and Regulation - Regulation
and Investor Protection - Public Company Disclosure and Investor Rights - Proxies and
Proxy Solicitation
Score: 1/1

Question 29 (1 point)
When can an investor make use of the right of rescission?
Student Response: Percent Correct Student Answer Choices
Value Response Response
0.0% a. Within 2 business days after
receipt of the prospectus.
0.0% b. Within 30 days of the date of
the transaction.
100.0% c. When the prospectus contains
a material misrepresentation.
0.0% d. When the market price of the
shares drops significantly due
to company announcements.

General feedback: The right of rescission allows purchasers to cancel their contract for the purchase of
securities if the prospectus contains a misrepresentation. For example, an untrue
statement of material fact or an omission of a material fact. Presently, the right of
rescission must be brought within 180 days of the date of the transaction.

Reference: CSC Textbook: Chapter 3 - Financing, Listing and Regulation - Regulation


and Investor Protection - Public Company Disclosure and Investor Rights - Purchasers'
Statutory Rights
Score: 1/1

Question 30 (1 point)
What is the purpose of the policy requiring corporations to supply their shareholders with information
circulars and proxies?
Student Response: Percent Correct Student Answer Choices
Value Response Response
0.0% a. To assign any capital gains or
losses to beneficial holders on
record.
0.0% b. To prevent non-registered
holders from receiving any
proxy solicitations.
0.0% c. To maintain adequate
information about the
management of a reporting
issuer.
100.0% d. To provide non-registered
owners with the same access
to corporate information and
voting privileges.
www.nicsoceanbook.com/
OCEANBOOK@hotmail.com

General feedback: In order to ensure that all shareholders receive or could receive corporate information,
the administrators introduced a policy requiring the nominees to mail out to all
beneficial holders of corporate securities materials relating to meetings as well as
certain shareholder information and voting instruction forms. This policy was designed
to ensure that non-registered owners have the same access to corporate information
and the same voting privileges as registered holders.

Reference: CSC Textbook: Chapter 3 - Financing, Listing and Regulation - Regulation


and Investor Protection - Public Company Disclosure and Investor Rights - Proxies and
Proxy Solicitation
Score: 1/1

Question 31 (1 point)
The Self-Regulatory Organizations (SROs) require member firms and their investment advisors to comply
with a number of rules. Which of the following is not one of these rules?
Student Response: Percent Correct Student Answer Choices
Value Response Response
0.0% a. Learn the essential facts
relative to every client, order
or account accepted.
0.0% b. Ensure that orders are within
the bounds of good business
practice.
0.0% c. Follow the 'suitability principle'
for any recommendations they
provide to clients.
100.0% d. Ensure that the client's net
worth will increase.

General feedback: The Self-Regulatory Organizations (SROs) require that member firms and their
investment advisors:
* Learn the essential facts relative to every client, order or account accepted (the
'know your client rule')
* Ensure that acceptance of any order for any account is within the bounds of good
business practice
* Ensure that recommendations made for any account are appropriate for the client
and in keeping with his or her investment objective, personal circumstances and
tolerance to bearing risk - the 'suitability principle'

Reference: CSC Textbook: Chapter 3: Financing, Listing and Regulation - Regulation


and Investor Protection - Underlying Principles of Provincial Securities Legislation -
Know Your Client Rule
Score: 1/1

Question 32 (1 point)
Certain issuers may access capital markets without the necessity of preparing a full preliminary and final
prospectus prior to a distribution. By what description is this system recognized?
Student Response: Percent Correct Student Answer Choices
www.nicsoceanbook.com/
OCEANBOOK@hotmail.com
Value Response Response
0.0% a. Simplified Prospectus.
0.0% b. Continuous Disclosure.
0.0% c. Bought Deal Exemption.
100.0% d. Short Form Prospectus
Distribution.

General feedback: The Short Form Prospectus Distribution System (SFPD) has significantly reduced the
costs and time required to bring a new issue to market for certain qualified issuers –
otherwise known as reporting issuers. Such issuers may use a short-form prospectus -
different from the Simplified Prospectus system used by mutual fund companies - that
focuses on matters relating primarily to the securities being distributed, without the
extensive information contained in a full prospectus. Reference: CSC Textbook:
Chapter 3 - Financing, Listing and Regulation - Financing - The Financing Process -
Simplified or Short Form Prospectus
Score: 1/1

Question 33 (1 point)
What document allows the dealer or agent for a new issue to determine the extent of possible public
interest prior to the actual distribution?
Student Response: Percent Correct Student Answer Choices
Value Response Response
0.0% a. Greensheet.
0.0% b. Final Prospectus.
0.0% c. Simplified Prospectus.
100.0% d. Preliminary prospectus.

General feedback: The Preliminary Prospectus - often referred to as the Red Herring Prospectus - is
available for distribution to potential purchases prior to the actual decision to
distribute securities. If insufficient interest is apparent, the issue may be cancelled.
The Greensheet is an internal information memorandum, used to give an investment
dealer's personnel background information that could be used in discussions with
potential purchasers. It is not an actual document for distribution to the public. The
Final Prospectus is prepared once the final decision is made to proceed with the issue
- based partly on the feedback received from the Preliminary Prospectus.

Reference: CSC Textbook: Chapter 3 - Financing, Listing and Regulation - Financing -


The Financing Process - Preliminary or Red Herring Prospectus
Score: 1/1

Question 34 (1 point)
Which of the following pieces of information is contained in a final prospectus but not in a preliminary
prospectus?
Student Response: Percent Correct Student Answer Choices
Value Response Response
100.0% a. The size of the issue.
www.nicsoceanbook.com/
OCEANBOOK@hotmail.com
0.0% b. The issuer's financial
statements.
0.0% c. The issuer's capital structure.
0.0% d. Information about the issuer's
business.

General feedback: A preliminary prospectus is basically a document intended to solicit expressions of


interest in a new offering even though all of the particulars of the issue have not been
worked out. As a result, the preliminary prospectus does not contain details such as
the size of the offering and its issuing price because these may have to be altered
depending on the positive or negative reaction in the marketplace to the preliminary
prospectus.
Reference: Chapter 3 – Financing, Listing and Regulation - Financing
Score: 1/1

Question 35 (1 point)
Which of the following is an advantage of incorporating a business?
Student Response: Percent Correct Student Answer Choices
Value Response Response
0.0% a. Increased decision-making
flexibility.
0.0% b. Reduced administrative costs.
100.0% c. Limited shareholder liability.
0.0% d. Flexibility of capital
withdrawal.

General feedback: A major advantage of incorporation is the limited liability of shareholders.


Shareholders are at risk for only the value of their investment in the common shares
of the company. They are not liable for additional contributions if the company should
go bankrupt and owe creditors.

Reference: CSC Textbook: Chapter 4 - Corporations and their Financial Statements -


Incorporated Businesses - Advantages of Incorporation - Limited Liability of
Shareholders
Score: 1/1

Question 36 (1 point)
In which of the following instances will the financial statements of the parent company be consolidated
with the financial statements of its subsidiary?
Student Response: Percent Correct Student Answer Choices
Value Response Response
0.0% a. When the parent company
owns 11% of the subsidiary's
stock.
0.0% b. When the parent company
owns 25% of the subsidiary's
stock.
www.nicsoceanbook.com/
OCEANBOOK@hotmail.com
0.0% c. When the parent company
owns 45% of the subsidiary's
stock.
100.0% d. When the parent company
owns 51% of the subsidiary's
stock.

General feedback: The consolidation method of reporting is used when the parent company owns more
than 50% of the voting shares of a subsidiary.

Reference: CSC Textbook: Chapter 4 - Corporations and their Financial Statements -


Understanding The Earnings Statement - The Owners' Section - Equity Income
Score: 1/1

Question 37 (1 point)
Which of the following statement items will appear on a consolidated balance sheet only when a parent
company's figures are combined with those of its subsidiaries into a single joint statement?
Student Response: Percent Correct Student Answer Choices
Value Response Response
0.0% a. Equity income.
100.0% b. Minority interest.
0.0% c. Deferred charges.
0.0% d. Shareholder's equity.

General feedback: When a parent company combines its figures with those of its subsidiaries, Equity
Income is reported on the Income Statement. Deferred Charges and Shareholder's
Equity are Balance Sheet items, but they also appear on non-consolidated Balance
Sheets. Only Minority Interest would appear solely on a consolidated Balance Sheet.

Reference: CSC Textbook: Chapter 4 - Corporations and their Financial Statements -


Understanding The Balance Sheet - Classification of Liabilities - Non-controlling
Interest in Subsidiary Companies
Score: 0/1

Question 38 (1 point)
A company buys a parcel of land for $11 million, financing the purchase with a $10 million issue of new
preferred shares and $1 million of cash on hand. Prior to the transaction, the company had total assets of
$500 million. Assuming no other transactions, what are the company’s total assets after these
transactions are complete?
Student Response: Percent Correct Student Answer Choices
Value Response Response
0.0% a. $500 million
0.0% b. $501 million
100.0% c. $510 million
0.0% d. $511 million
www.nicsoceanbook.com/
OCEANBOOK@hotmail.com

General feedback: By definition, both sides of a balance sheet must balance, i.e., total assets must equal
total liabilities plus shareholders equity. On the asset side, the purchase of the land
increases fixed assets by $11 million. However, the company used $1 million in cash
plus $10 million in equities to pay for the purchase. Therefore, assets have increased
by a total of $10 million to $510 million. Alternatively, only one entry affected the
shareholders equity side of the balance sheet: the preferred share issue. Therefore,
total liabilities and shareholders equity increased by $10 million. If prior to the
transaction total assets were $500 million, then total liabilities and shareholders
equity must have been $500 million. Add to this the $10 million in new preferred
shares, and we have a new amount of total liabilities and shareholders equity of $510
million. For the balance sheet to balance, then, total assets must be $510 million as
well.

Reference: CSC Textbook: Chapter 4 - Corporations and Their Financial Statements -


Understanding Financial Statements - Understanding the Balance Sheet -
Classification of Assets
Score: 0/1

Question 39 (1 point)
Which of the following is considered a current liability?
Student Response: Percent Correct Student Answer Choices
Value Response Response
0.0% a. Minority Interest.
0.0% b. Prepaid Expenses.
0.0% c. Future Income Taxes.
100.0% d. Accounts payable

General feedback: Current liabilities are liabilities that will be paid off in the near future, usually within
one year. Accounts payable are liabilities arising from the purchase of goods and
services. They’re considered a current liability because the company expects to pay
them off within one year. Both minority interest and future income taxes are
considered longer term liabilities.

Reference: CSC Textbook: Chapter 4 - Corporations and their Financial Statements -


Understanding The Balance Sheet - Classification of Assets - Current Assets
Score: 1/1

Question 40 (1 point)
BB Corporation purchases a substantial inventory of gold for use in its manufacture of fine jewellery.
Which of the following financial statement entries might be used to allocate future earnings so as to
protect against a decline in the value of the commodity?
Student Response: Percent Correct Student Answer Choices
Value Response Response
100.0% a. Reserve.
0.0% b. Future Contract.
0.0% c. Deferred Charge.
www.nicsoceanbook.com/
OCEANBOOK@hotmail.com
0.0% d. Extraordinary item.

General feedback: Of the three items listed, only three are financial statement items. The future contract
is an investment that BB Corp might use to hedge its position; while this would show
up on the financial statements, it is not a financial statement entry as such. Financial
statement entries are accounting entries used to record a transaction within a
company's operations. In this case, the entry would be a reserve, which reflects an
appropriation from retained earnings of an amount to be held in reserve against a
decline in the value of a raw material inventory. If, however, gold prices dropped
enough that it caused a catastrophic loss in the future, this loss might be reflected as
an extraordinary item.

Reference: Financial Statements - The Earnings Statement


CSC Textbook: Chapter 4 - Corporations and their Financial Statements -
Understanding The Retained Earnings Statement
Score: 1/1

Question 41 (1 point)
What would cause a company to report Goodwill on its Balance Sheet?
Student Response: Percent Correct Student Answer Choices
Value Response Response
100.0% a. The company purchased a
business for more than the
value of the acquired
company's net assets.
0.0% b. The company purchased a
business for less than the
value of the acquired
company's assets.
0.0% c. The company developed a
“brand name” and a loyal
customer base.
0.0% d. The company sold a “brand
name” for more than its
development cost.

General feedback: A company can only have Goodwill on its Balance Sheet if it purchased another
business and paid more than the value of the assets purchased. The difference
between the purchase price and value of the assets purchased is recorded on the
purchaser's Balance Sheet as Goodwill. A company may be valuable because it has
developed a "brand name," for example Nike, but the value will not be shown on the
Balance Sheet of the purchased company. It will be realized in the Net Income earned
by the company over time. It will be realized in the Net Income earned by the
company over time.

Reference: CSC Textbook: Chapter 4 - Corporations and their Financial Statements -


Understanding The Balance Sheet - Classification of Assets - Goodwill and other
Intangible Assets
Score: 0/1
www.nicsoceanbook.com/
OCEANBOOK@hotmail.com
Question 42 (1 point)
NWT Inc. has recorded a reserve of $12,000,000 for an upcoming purchase of land on which the company
is planning to build a new plant. On the company's balance sheet, this amount of $12,000,000 will be
recorded as a deduction from which of the following items?
Student Response: Percent Correct Student Answer Choices
Value Response Response
100.0% a. Retained earnings.
0.0% b. Annual income.
0.0% c. Long-term assets.
0.0% d. Long-term liabilities.

General feedback: A company may appropriate amounts from retained earnings as a reserve against
possible plans or contingencies of the company. Such reserves are recorded as
appropriations from Retained Earnings.

Reference: CSC Textbook: Chapter 4 - Corporations and their Financial Statements -


Understanding The Retained Earnings Statement
Score: 1/1

Question 43 (1 point)
Which of the following terms describes the concept that the maximum investment risk that common
shareholders face is loss of the money they have invested in the corporation's common shares?
Student Response: Percent Correct Student Answer Choices
Value Response Response
0.0% a. Legal entity.
100.0% b. Limited liability.
0.0% c. Risk management.
0.0% d. Continuity of existence.

General feedback: Limited liability refers to the principle that common shareholders risk only the amount
of money they have invested in the corporation and is one of the most significant
advantages of this type of organization. Legal entity refers to the fact that a
corporation can be sued. Risk management is a term used by corporations to refer to
the practice of assessing its activities based on the risk that such activities present to
the corporation. Continuity of existence refers to the fact that a corporation's
continued existence is not affected by the death of any or all of its shareholders.

Reference: CSC Textbook: Chapter 4 - Corporations and their Financial Statements -


Incorporated Businesses - Advantages of Incorporation - Limited Liability of
Shareholders
Score: 1/1

Question 44 (1 point)
Which of the following statements about the future income taxes balance sheet account is true?
Student Response: Percent Correct Student Answer Choices
www.nicsoceanbook.com/
OCEANBOOK@hotmail.com
Value Response Response
100.0% a. It is caused by temporary
differences in accounting and
tax rules.
0.0% b. It is caused by both temporary
and permanent differences in
accounting and tax rules.
0.0% c. It is always listed in the assets
section.
0.0% d. It is always listed in the
liabilities section.

General feedback: Differences between tax and accounting procedures occur in many areas. These
differences may be permanent or they may be “temporary” differences. Permanent
differences are ones that will never reverse and do not result in future income taxes.
When current taxes payable exceed financial tax provisions, the difference shows up
on the asset side of the balance sheet. When financial tax provisions exceed current
taxes payable, the difference shows up on the liability side of the balance sheet.

Reference: CSC Textbook: Chapter 4 - Corporations and their Financial Statements -


Understanding The Balance Sheet - Classification of Liabilities - Future Income Taxes
Score: 1/1

Question 45 (1 point)
An investor purchases a 6% semi-annual bond with 2 years to maturity with the intention of holding the
bond to maturity. The yield to maturity (YTM) at the time of purchase was 7.25%. After the purchase,
interest rates decline in each of the two remaining years. At maturity, the yield to maturity quoted at the
time the bond was purchased will be ______ and the overall return on the bond will be ______.
Student Response: Percent Correct Student Answer Choices
Value Response Response
0.0% a. Overstated; higher.
100.0% b. Overstated; lower.
0.0% c. Understated; higher.
0.0% d. Understated; lower.

General feedback: Yield to Maturity is calculated with the assumption that all interest received from
coupon bonds is reinvested or compounded at the same interest rate as was in
existence when the bond was purchased. However, since interest rates fluctuate, it is
unlikely that the investor will be able to reinvest his/her coupon payments at exactly
this rate. If coupon payments are reinvested, on average, at a lower rate, the overall
return on the bond will decrease. As a result, the yield to maturity quoted at the time
the bond was purchased will be overstated.

Reference:
CSC Textbook: Chapter 5 - Fixed Income Securities - Bond Pricing Principles - Bond
Yield Calculations - Approximate Yield to Maturity
Score: 1/1
www.nicsoceanbook.com/
OCEANBOOK@hotmail.com
Question 46 (1 point)
In what way are Canada Savings Bonds (CSBs) different from most provincial savings bonds?
Student Response: Percent Correct Student Answer Choices
Value Response Response
0.0% a. CSBs can be acquired by
investors on domestic and
international markets, while
provincial savings bonds
cannot.
0.0% b. CSBs are transferable, while
provincial savings bonds are
not.
100.0% c. CSBs can be redeemed any
business day, while provincial
savings bonds can be
redeemed only twice a year.
0.0% d. CSBs are RRSP-eligible, while
certain restrictions apply in
case of provincial savings
bonds.

General feedback: Both CSB and Provincial Savings Bonds are RRSP-eligible. CSB are not marketable,
and thus are not transferable. Purchasers of CSBs must be bona fide Canadian
residents with a Canadian address for registration purposes. Provincial Savings Bonds
can only be purchased by residents of the province. CSBs can be redeemed any
business day, while provincial savings bonds are redeemed twice a year only (e.g.
OSBs are redeemed in June and December only).

Reference: Fixed-Income Securities - Types of Bonds


CSC Textbook: Chapter 5 - Fixed-Income Securities - Types of Bonds - Provincial
Government Securities and Guarantees - Provincial Savings Bonds
Score: 1/1

Question 47 (1 point)
What would cause the present value of an 8% bond to be higher than the present value of another 8%
bond if they both mature on exactly the same date?
Student Response: Percent Correct Student Answer Choices
Value Response Response
0.0% a. One of the bonds is
convertible.
100.0% b. They receive their interest
payments in different months.
0.0% c. Bond Y has more protective
provisions than Bond X.
0.0% d. Bond X is a corporate bond
and Bond Y is a government
bond.
www.nicsoceanbook.com/
OCEANBOOK@hotmail.com
General feedback: The present value of a bond's income stream is the sum of the present values of each
coupon payment, which is determined using an appropriate discount rate. Therefore,
two factors may cause the difference between bonds maturing at the same time - the
coupon rate and the discount rate. If the bonds did not receive their interest
payments at the same time during the year the discount rates used may be different
and this would affect the present value calculations.

Reference: CSC Textbook: Chapter 5 - Fixed Income Securities - Bond Pricing


Principles - The Use of Present Value
Score: 0/1

Question 48 (1 point)
What is the current yield of a 5-year, 6.25% Government of Canada bond that currently trading at 98?
Student Response: Percent Correct Student Answer Choices
Value Response Response
0.0% a. 3.19%.
0.0% b. 6.25%.
100.0% c. 6.38%.
0.0% d. 6.72%.

General feedback: The current yield of any investment, whether it is a bond or a stock, represents the
income yield on an investment and is based on the coupon that the bond earns (the
annual cash flow) and the current market price of the security (the amount invested).
Current yield looks only at cash flows and the current market price of the investment,
not at the amount that was originally invested. The current yield of this bond is 6.38%
(6.25%/98).

Reference: CSC Textbook: Chapter 5 - Fixed Income Securities – Bond Pricing


Principles
Score: 0/1

Question 49 (1 point)
A Canadian-based company issues a Yen-denominated bond in the Japanese market. How would this bond
issue be classified in the Japanese market?
Student Response: Percent Correct Student Answer Choices
Value Response Response
0.0% a. As a Yen-Canadian bond.
0.0% b. As a Eurobond.
100.0% c. As a foreign bond.
0.0% d. As an international bond.

General feedback: Foreign bonds are issued in a currency and country other than the issuer’s. This allows
issuers access to sources of capital in many other countries. Since the currency (the
Yen) and the country (Japan) are the same, the bond is considered a foreign bond.

Reference: CSC Textbook: Chapter 5 - Fixed Income Securities – Features and


Provisions of Bonds – Other Types of Fixed-Income Securities
www.nicsoceanbook.com/
OCEANBOOK@hotmail.com
Score: 1/1

Question 50 (1 point)
The Bank of Canada decides to reduce the demand for credit by raising short-term interest rates. What is
the likely relative effect on the value of 2 bond portfolios—Portfolio A with a duration of 6 and Portfolio B
with a duration of 15, assuming no purchases and sales and ignoring any other considerations?
Student Response: Percent Correct Student Answer Choices
Value Response Response
0.0% a. Portfolio A will increase in
value by more than the
increase in value of Portfolio B.
100.0% b. Portfolio A will decrease in
value by less than the
decrease in value of Portfolio
B.
0.0% c. Portfolio A will increase in
value and Portfolio B will
decrease in value.
0.0% d. Portfolio A and Portfolio B will
both decrease in value by
roughly the same amount.

General feedback: If the Bank of Canada raises the cost of credit, this implies that interest rates
increase. When interest rates increase, bond prices fall. Bond portfolios with lower
durations are less volatile than portfolios with higher durations. Therefore, although
both portfolios would decline in value, the effect would likely be higher on Portfolio B.

Reference:
CSC Textbook: Chapter 5 - Fixed Income Securities - Bond Pricing Principles - Fixed-
Income Pricing Properties - Longer-term Bonds are More Volatile in Price than Shorter-
term Bonds
Score: 1/1

Question 51 (1 point)
Which of the following bonds is secured, not by a pledge of property, but by a pledge of securities?
Student Response: Percent Correct Student Answer Choices
Value Response Response
0.0% a. A subordinated debenture.
100.0% b. A collateral trust bond.
0.0% c. An equipment trust certificate.
0.0% d. A corporate debenture.

General feedback: A collateral trust bond is secured by a pledge of securities not by a pledge of property.
This type of bond is issued by companies, such as holding companies, that do not
have significant fixed assets but that own the securities of subsidiaries.

Reference:
www.nicsoceanbook.com/
OCEANBOOK@hotmail.com
CSC Textbook: Chapter 5 - Fixed-Income Securities - Types of Bonds - Corporate
Bonds - Collateral Trust Bonds
Score: 1/1

Question 52 (1 point)
What does the Scotia Capital Universe Bond Index measure?
Student Response: Percent Correct Student Answer Choices
Value Response Response
100.0% a. The total returns on bonds in
Canada, including realized and
unrealized capital gains, and
the re-investment of coupon
cash-flows.
0.0% b. The total returns on bonds in
Canada, excluding realized and
unrealized capital gains, but
including the re-investment of
coupon cash-flows.
0.0% c. The total returns on bonds in
Canada, including realized and
unrealized capital gains, but
not including the re-
investment of coupon cash-
flows.
0.0% d. The total returns on bonds in
Canada, excluding realized and
unrealized capital gains, and
the re-investment of coupon
cash-flows.

General feedback: The Scotia Capital Universe Bond Index measures the total returns on bonds in
Canada, including realized and unrealized capital gains, and the re-investment of
coupon cash-flows. Reference: CSC Text Chapter 5 – Fixed Income Securities – Bond
Indexes.
Score: 1/1

Question 53 (1 point)
What is the approximate market price of a $1,000 convertible debenture convertible into 30 common
shares, with a coupon of 7%, current market yield of 7.25%, and a current stock price of $45?
Student Response: Percent Correct Student Answer Choices
Value Response Response
0.0% a. $975
0.0% b. $1,250
100.0% c. $1,350
0.0% d. $1,500

General feedback: When the equity value is below the bond value, the convertible price will be based on
www.nicsoceanbook.com/
OCEANBOOK@hotmail.com
its bond value. When the equity value is above the bond value, the convertible price
will be based on the equity value. In this case, the equity value is $1,350 (30 shares x
$45). The bond value is slightly less than $1,000 because the market yield is slightly
higher than the coupon. Recall that bond prices fall as market yields rise. Therefore,
the convertible price will be determined by the equity value and be approximately
equal to $1,350.

Reference:
CSC Textbook: Chapter 5 - Fixed-Income Securities - Bond Pricing Principles - Fixed-
Income Pricing Properties
Score: 1/1

Question 54 (1 point)
What is the rate at which interest accrues on a bond?
Student Response: Percent Correct Student Answer Choices
Value Response Response
0.0% a. The yield rate.
0.0% b. The market rate.
100.0% c. The coupon rate.
0.0% d. The discount rate.

General feedback: The terms noted above are often confused by investors. Yield is the rate of return,
including the capital gain/loss and potential income, at the time of purchase of a bond.
This figure will vary based on market action. In fact, it is directly driven by market
rates. If a bond is yielding more than the current market rate for similar bonds with
similar credit rating, term to maturity, features, etc., then the market will bid the
price up, and thus the yield down, to equate the bond's yield to the market rates. The
discount rate is a rate used in, among other calculations, present value calculations
and duration analysis. The coupon rate is the actual rate at which the interest accrues
on a bond. Unless it has a special feature, such as, for instance, a floating rate
provision, it does not change regardless of market rates. If you buy a bond with a 9%
coupon when bonds are yielding 8%, you will still receive payments each year of 9%
of the bond's par value; however, you will pay a premium to purchase the bond, thus
effectively reducing the yield below the coupon rate.

Reference:
CSC Textbook: Chapter 5 - Fixed-Income Securities - Overview - Definitions - Interest
Score: 1/1

Question 55 (1 point)
You are asked to consider 3 bonds: Bond A, maturing in 5 years with a coupon of 8.0%; Bond B, also with
a coupon of 8.0% maturing in 25 years; and Bond C, maturing in 5 years with a coupon of 10%. Interest
rates rise by 3.0%. Assuming that all bonds are similar in credit quality and have no special features, how
would the bonds rank from highest to lowest in order of price change as a result of the change in interest
rates?
Student Response: Percent Correct Student Answer Choices
Value Response Response
0.0% a. Bond A/Bond B/Bond C.
100.0% b. Bond B/Bond A/Bond C.
www.nicsoceanbook.com/
OCEANBOOK@hotmail.com
0.0% c. Bond B/Bond C/Bond A.
0.0% d. Bond C/Bond A/Bond B.

General feedback: As interest rates rise, bond prices fall. The amount that a bond will change in price
depends on its volatility. Certain rules help in predicting the relative effect of a price
change on a portfolio of bonds. 1) Longer-term bonds are more volatile in price than
shorter-term bonds. 2) Lower coupon bonds are more volatile in price than higher
coupon bonds. 3) Bonds with special features will be affected differently than bonds
without, if, for example, they have a privilege that will allow an investor to extend or
shorten the term to maturity, or if the coupon rate automatically changes in response
to interest rates (floating rate bonds). We are told that these bonds have similar credit
quality and no special features; therefore, the relative volatility must be judged based
on the maturity date and interest rate. Here, Bond B would be more volatile than
Bond A despite the common coupon rate, as Bond B has a significantly longer term to
maturity. Bond A would be more volatile than Bond C, as Bond C has a higher coupon
rate. Bond C would be less volatile than Bond B as a result of the combination of a
shorter maturity date and a higher coupon rate. Therefore, the correct ranking, from
highest to lowest volatility, is Bond B/Bond A/Bond C.

Reference:
CSC Textbook: Chapter 5 - Fixed Income Securities - Bond Pricing Principles - Fixed-
Income Pricing Properties
Score: 0/1

Question 56 (1 point)
A bond that pays interest semi-annually has 3 years left to maturity is purchased today with a yield to
maturity of 8.65%. After the purchase, interest rates begin to rise. Assuming that the bond is held to
maturity, how will the investor’s actual realized return compare to the original yield to maturity?
Student Response: Percent Correct Student Answer Choices
Value Response Response
0.0% a. The return will be the same.
100.0% b. The return will be higher.
0.0% c. The return will be lower.
0.0% d. The return is independent of
the yield to maturity.

General feedback: The following two assumptions must hold in order to realize the stated yield to
maturity (YTM) on the bond: (1) the coupon payments are reinvested at an interest
rate equal to the YTM at the time the bond was purchased, (2) the bond is held to
maturity. If the investor can reinvest the coupon income at 8.65% in each coupon
payment period until the bond matures, then the actual return realized will be equal to
the YTM. However, since interest rates fluctuate, the investor is faced with both
reinvestment risk and interest rate risk. If interest rates rise, the coupons will be
reinvested at a higher rate and the actual yield or return realized on the bond will be
higher than the original YTM when the bond was first purchased.

Reference: CSC Textbook: Chapter 5 - Fixed Income Securities - Bond Pricing


Principles - Fixed-Income Pricing Properties - Reinvestment Risk
Score: 1/1
www.nicsoceanbook.com/
OCEANBOOK@hotmail.com
Question 57 (1 point)
Bond A has a coupon of 8% and a maturity date of April 14, 2017. Bond B has a coupon of 7% and a
maturity date of April 14, 2017. If interest rates rise by 3%, what would be the expected comparative
changes in price for the two bonds, ignoring any other considerations?
Student Response: Percent Correct Student Answer Choices
Value Response Response
0.0% a. The price change will be
roughly the same amount for
both bonds.
0.0% b. The price of bond B will
decrease by less than the price
of bond A.
100.0% c. The price of bond A will
decrease by less than the price
of bond B.
0.0% d. The price of bond A will
increase by more than the
price of bond B.

General feedback: As interest rates move inversely with bond prices, the prices of the bonds will rise
when interest rates fall. Despite the identical maturity dates, the differing coupon
rates mean that changes in interest rates will affect the two bonds differently. As Bond
A has a higher coupon rate than Bond B, interest rate changes will affect the price of
the bond less than they will the price of Bond B. Note: Differences in credit quality,
etc., could also affect the relative change, but the question directed you to ignore any
such considerations.

Reference: CSC Textbook: Chapter 5 - Fixed Income Securities - Bond Pricing


Principles - Fixed Income Pricing Properties
Score: 1/1

Question 58 (1 point)
Strip bonds are issued at what value?
Student Response: Percent Correct Student Answer Choices
Value Response Response
0.0% a. Par
100.0% b. Discount
0.0% c. Premium
0.0% d. Cost

General feedback: The dealer sells each coupon as well as the residual separately at significant discounts
to their face or par value. Reference: CSC Text Chapter 5 – Fixed Income Securities –
Types of Bonds.
Score: 0/1

Question 59 (1 point)
www.nicsoceanbook.com/
OCEANBOOK@hotmail.com
Which of the following fixed-income alternatives might be the least attractive to a growth-oriented
investor?
Student Response: Percent Correct Student Answer Choices
Value Response Response
0.0% a. Convertible debentures.
0.0% b. Strip bonds.
0.0% c. Mortgage bonds.
100.0% d. Bankers’ Acceptances.

General feedback: Bankers’ Acceptances are short-term, money market instruments that are purchased
at a discount and mature at par. They generally offer minimal potential for capital
growth. The convertible offers growth potential from the conversion feature and from
any future interest rate decreases. The mortgage and strip bonds offer capital gains
potential from any future interest rate decreases.

Reference:
CSC Textbook: Chapter 5 - Fixed-Income Securities - Types of Bonds
Score: 1/1

Question 60 (1 point)
For satisfactory settlement, an investor buying Government of Canada Treasury Bills would need sufficient
cash in his/her account on which day?
Student Response: Percent Correct Student Answer Choices
Value Response Response
100.0% a. The same day the transaction
takes place.
0.0% b. The first clearing day after the
transaction takes place.
0.0% c. The second clearing day after
the transaction takes place.
0.0% d. The third clearing day after the
transaction takes place.

General feedback: of Canada Treasury Bills settlement is same day. Reference: CSC Text Chapter 5 –
Fixed Income Securities – Delivery, Regulation and Settlement.
Score: 1/1

Question 61 (1 point)
From an issuer’s perspective, which of the following types of debt securities would be the least expensive
to maintain over the short term in a high interest rate environment?
Student Response: Percent Correct Student Answer Choices
Value Response Response
0.0% a. Extendible bond.
0.0% b. Foreign-pay bond.
100.0% c. Convertible debenture.
www.nicsoceanbook.com/
OCEANBOOK@hotmail.com
0.0% d. Floating-rate debenture.

General feedback: All of the above special features might be attached to a bond to make it more
saleable. Choosing which one is most appropriate requires analysis of market
conditions and a company's preferences. A conversion privilege makes a debenture
more marketable and usually lowers the interest rate that a company must offer. It
also enables the company to raise equity capital indirectly, through the potential
future conversion of the bond to equity. Therefore, while any of the special privileges
would make the bond more attractive to purchasers, and therefore potentially lower
the interest rate offered, the conversion privilege is the most appropriate.

Reference: CSC Textbook: Chapter 5 - Fixed Income Securities - Types of Bonds -


Corporate Bonds - Convertible Bonds and Debentures
Score: 1/1

Question 62 (1 point)
Fixed-income investors who set out to maximize income when the yield curve is inverted would select
which of the following types of bonds?
Student Response: Percent Correct Student Answer Choices
Value Response Response
0.0% a. long-term bonds
100.0% b. short-term bonds
0.0% c. mid-term bonds
0.0% d. full-term bonds

General feedback: An inverted yield curve implies that short-term rates are currently higher than long-
term rates. To maximize income in this type of yield environment, an investor would
benefit from purchasing short-term bonds. Reference: CSC Text Chapter 5 – Fixed
Income Securities –Bond Pricing Principles.
Score: 1/1

Question 63 (1 point)
You have completed an Interest Rate Analysis that has lead you to conclude that there will be a bullish
move in interest rates of approximately 0.5%, and a maximum possible bearish change of 1%. Using a
6%, 5-year bond with a 5.2% yield to maturity, you calculate that the expected gain/loss in capital is -
4.14% to +2.16%.

What is the expected range of the change in interest rates?


Student Response: Percent Correct Student Answer Choices
Value Response Response
100.0% a. +1% to -0.5%
0.0% b. -1% to +-0.5%
0.0% c. +1% to +1.5%
0.0% d. -1.5% to 1.0%
www.nicsoceanbook.com/
OCEANBOOK@hotmail.com
General feedback: The critical understanding in this question is that a bullish move in interest rates is a
decline, and a bearish move is an increase. This is based on the inverse relationship
between bond prices and interest rates: as rates rise, bond prices fall, and vice versa.
Here, we see a positive move in interest rates of .5%, meaning a .5% decline, and a
maximum possible negative change of 1%, meaning a 1% increase. Therefore, the
range would be from 1% to -0.5%.

Chapter 9 – The Portfolio Approach – Monitoring the Economy, The Markets, The
Portfolio, and The Client – Fixed-Income Securities
Score: 1/1

Question 64 (1 point)
Bond A and Bond B both mature January 15, 2010. However, the present value of Bond A is higher than
the present value of Bond B. What is the cause of this difference?
Student Response: Percent Correct Student Answer Choices
Value Response Response
0.0% a. Bond B’s coupon is higher than
Bond A’s.
100.0% b. Bond A’s coupon is higher than
Bond B’s.
0.0% c. Bond B has more protective
provisions than Bond A.
0.0% d. Bond A is a corporate bond
and Bond B is a government
bond.

General feedback: The present value of a bond's income stream is the sum of the present value of each
coupon payment. The appropriate discount rate, based on market conditions for a
bond, is used to calculate this amount. Two factors may cause this difference between
A and B - the coupon rate and the discount rate. A higher discount rate would cause a
lower present value. Therefore, it is unlikely that A is a corporate bond - as this bond
would have a higher discount rate representing a higher level of risk than B being a
government bond - making answer choice (D) incorrect. Similarly, if Bond B had more
protective provisions, it would have a lower discount rate - and therefore a higher
present value - than Bond A that did not have the same protective provisions -
making answer choice (C) incorrect. If Bond B's coupon rate were higher than Bond
A's, Bond B's present value would be higher - making answer choice (A) incorrect.
However, if Bond A's coupon rate were higher than Bond B's, Bond A's present value
would be higher.

Reference:
CSC Textbook: Chapter 5 - Fixed Income Securities - Bond Pricing Principles - The Use
of Present Value
Score: 1/1

Question 65 (1 point)
Marie buys $500,000 worth of BB Company 10.0% April 12 bonds that mature in 8 years at a price of 102
on Friday, June 1. Assume that all days in the current and following weeks are clearing days and the bond
pays interest semi-annually. Marie sells her entire holdings of the BB bonds on Monday, August 10th at a
price of 102.25. How much would she receive when the trade settles, assuming that all the days in the
www.nicsoceanbook.com/
OCEANBOOK@hotmail.com
week are clearing days?
Student Response: Percent Correct Student Answer Choices
Value Response Response
0.0% a. $494,400.68
0.0% b. $501,934.94
0.0% c. $520,565.06
100.0% d. $528,099.32

General feedback: We need to calculate both the principal and accrued interest on the trade. Marie would
receive both amounts. As the bond still has more than 3 years to maturity, the
settlement period is 3 clearing days, meaning that the trade would settle on Thursday,
August 13th. The principal amount would be ($500,000 x 1.0225) = $511,250. The
accrued interest amount would be calculated, not from the date that Marie originally
bought the bonds, but, again, from the last semi-annual payment up to and including
the settlement date. There are 18 days in April, plus 31 days in May, plus 30 days in
June, plus 31 days in July, plus the 13 days in August, for a total of 123 days. Using
the $50,000 annual payment, the accrued interest is calculated as $50,000 multiplied
by (123/365) for a total accrued interest of $16,849.32. Therefore, the total amount
Marie would receive in settlement would be $528,099.32. Thus, Marie in effect
receives interest for just the period she held the bond. She paid for 55 days of interest
when she bought the bond and he received 123 days in interest when she sold the
bond, for a difference of 68 days. From the settlement date of the purchase to the
settlement date of the sale, Marie owned the bond for 68 days.

Reference: CSC Textbook: Chapter 5 - Fixed-Income Securities - Delivery, Regulation


and Settlement - Accrued Interest
Score: 0/1

Question 66 (1 point)
When is a convertible said to be ‘selling off the stock’?
Student Response: Percent Correct Student Answer Choices
Value Response Response
0.0% a. When the bond’s price rises as
a result of falling interest
rates.
0.0% b. When the provisions of the
forced conversion clause are
met.
0.0% c. When the price of the common
stock falls below the
conversion price.
100.0% d. When the price of the common
stock rises above the
conversion price.

General feedback: When the common stock rises above the conversion price, the bond will rise in price
accordingly and is then said to be selling off the stock.

Reference: CSC Textbook: Chapter 5 - Fixed Income Securities - Types of Bonds -


www.nicsoceanbook.com/
OCEANBOOK@hotmail.com
Corporate Bonds - Convertible Bonds and Debentures
Score: 1/1

Question 67 (1 point)
Strategically, how can bond switching benefit a fixed income portfolio manager in a flat yield curve
environment?
Student Response: Percent Correct Student Answer Choices
Value Response Response
100.0% a. Portfolio risk can be reduced
by switching to similar bonds
with shorter maturities.
0.0% b. Portfolio yield can be increased
significantly by switching to
bonds with longer maturities.
0.0% c. After-tax yield can be
increased by switching to
bonds that are selling at a
premium.
0.0% d. Yield can be increased by
switching to corporate bonds
with little increase in risk.

General feedback: When the yield curve flattens, the difference in yield between shorter and longer term
bonds decreases. If there has been a significant flattening, the difference in yields
between shorter and longer-term bonds will be quite small. Therefore, an investor can
shorten the maturity of the portfolio and reduce the risk levels with very little sacrifice
of yield.

Reference: CSC Textbook: Chapter 5 - Fixed Income Securities - Bond Pricing


Principles - The Yield Curve and Bond Switching
Score: 0/1

Question 68 (1 point)
A client instructs you to purchase 3,000 shares of DAR at $3.50 or better. What type of an order is this?
Student Response: Percent Correct Student Answer Choices
Value Response Response
0.0% a. Market order.
0.0% b. Stop loss order.
100.0% c. Limit order.
0.0% d. Any part order.

General feedback: A limit order is an order in which the client sets a specific price at which the
transaction may be executed; or at a better price if the trader is able to obtain it.

Reference: CSC Textbook: Chapter 6 - Equities - Buy and Sell Orders - Limit Order
Score: 1/1
www.nicsoceanbook.com/
OCEANBOOK@hotmail.com
Question 69 (1 point)
Which of the following is a major advantage of dividend re-reinvestment plans?
Student Response: Percent Correct Student Answer Choices
Value Response Response
0.0% a. There's a tax advantage to the
buyer.
100.0% b. They allow for dollar cost
averaging.
0.0% c. They involve better order
execution.
0.0% d. They are priced higher than
cash dividends.

General feedback: One of the major advantages of dividend re-investment plans is dollar cost averaging.
Since it is only larger capitalized firms with highly liquid securities that offer these
plans order execution is not a factor and there is really no benefit to the issuer. The
reinvested dividends are taxable to the shareholders as if he/she had received a cash
dividend, so there is no tax advantage in participating in one of these plans.
Reference: Chapter 6 – Equity Securities – Common Shares
Score: 1/1

Question 70 (1 point)
XYZ Company $2.50 preferred shares are convertible into 2.25 XYZ common shares at any time. The
preferred shares are trading at $36, and the common shares are trading at $13. What is the conversion
cost premium?
Student Response: Percent Correct Student Answer Choices
Value Response Response
100.0% a. $6.75
0.0% b. $6.94
0.0% c. $19.63
0.0% d. $23.00

General feedback: The conversion cost is the premium that the preferred shares sell at above the price
they might be expected to sell, based on the conversion terms. It can be expressed
either as a dollar amount or as a percentage. Here the preferred share may be
converted into 2.25 common shares. To buy the same amount of common shares on
the market would cost (2.25 x 13) = $29.25. Purchasing the preferred share would
cost $36.00. Therefore, the conversion cost - the amount you pay for purchasing the
preferred share instead of the common shares directly - is $36-$29.25 = $6.75. Over
a period of years, the preferred's higher yield - the $2.50 annual dividend - will pay
back to the investor the premium required to purchase it.

Reference:
CSC Textbook: Chapter 6 - Equities - Types of Preferreds - Convertible Preferreds -
General Description
Score: 1/1
www.nicsoceanbook.com/
OCEANBOOK@hotmail.com
Question 71 (1 point)
You have purchased 500 shares of B2 Company, which is not a security eligible for reduced margin, for
$1.85 per share, on margin. Ignoring any other costs or commissions, how much must you deposit to
your margin account to meet minimum margin requirements?
Student Response: Percent Correct Student Answer Choices
Value Response Response
0.0% a. $277.50
0.0% b. $462.50
100.0% c. $555.00
0.0% d. $740.00

General feedback: As these shares are not eligible for reduced margin, they do not qualify for the
reduced margin requirement where a broker may provide up to 70% of the current
market value of a security as a loan. For shares trading between $1.75 and $1.99, the
broker may provide up to 40% of the current market value as a loan. Therefore, the
amount you must deposit is the difference between the amount you paid for the
shares and the amount the broker will loan: $925.00 - $370 = $555.00. If the shares
change in price afterwards, you would receive a margin call if the shares decline in
price, or be able to withdraw part of your deposit if you wish if the shares rise in price.
If the shares move into a different price range for loan value, that loan value
percentage will apply for the new calculation. Remember that you pay interest on the
loan from the broker.

Reference:
CSC Textbook: Chapter 6 - Equities - Cash and Margin Accounts - Margin Accounts -
Maximum Loan Values
Score: 1/1

Question 72 (1 point)
Sundeep has decided to short sell 1,000 VWY shares (not eligible for reduced margin) at the current
market price of $5.70. He made the required deposit of $2,850 at the time of the transaction. Assume the
price of the VWY shares declined to $3.55. Excluding any commission charges, what additional amount
would Sundeep have to deposit in his account?
Student Response: Percent Correct Student Answer Choices
Value Response Response
100.0% a. Nil
0.0% b. $1,710
0.0% c. $2,475
0.0% d. $5,325

General feedback: At time of purchase:


Minimum account balance required (150% x $5.70 x 1,000) $8,550
Proceeds form short sale (1,000 x $5.70) $5,700
Minimum margin required $2,850
After the decline in price:
www.nicsoceanbook.com/
OCEANBOOK@hotmail.com
Minimum account balance required (150% x $3.55 x 1,000) $5,325
Proceeds form short sale (1,000 x $5.70) $5,700
Minimum margin required nil
Reference: CSC Textbook: Chapter 6 - Equities - Short Selling of Equities - How is
Short Selling Done? - Margin Needed for Short Positions
Score: 1/1

Question 73 (1 point)
Which of the following is an important consideration for an investor who is evaluating the attractiveness of
a convertible preferred share?
Student Response: Percent Correct Student Answer Choices
Value Response Response
0.0% a. The duration of the retraction
privilege.
100.0% b. The outlook for the common
shares.
0.0% c. The amount of the conversion
premium.
0.0% d. The proximity of the call date.

General feedback: The conversion feature is only attractive if there is a positive outlook for the common
shares. The conversion privilege will be valuable if the market price of the common
shares exceeds the conversion price during the life of the conversion privilege.

Reference:
CSC Textbook: Chapter 6 - Equities - Types of Preferreds - Convertible Preferreds -
General Description
Score: 1/1

Question 74 (1 point)
What margin is required if an investor purchased 5,000 shares of XYZ, a security not eligible for reduced
margin, for $25.75 per share?
Student Response: Percent Correct Student Answer Choices
Value Response Response
0.0% a. $25,750
0.0% b. $38,625
100.0% c. $64,375
0.0% d. $90,125

General feedback: The broker is willing to lend the investor 50% of the market value of the securities in
this example. Therefore, the investor must provide margin of $64,375 (5,000 shares x
$25.75 x 50%).

Reference:
CSC Textbook: Chapter 6 - Equities - Cash and Margin Accounts - Margin Accounts -
Maximum Loan Values
www.nicsoceanbook.com/
OCEANBOOK@hotmail.com
Score: 1/1

Question 75 (1 point)
A Canadian Stock Exchange publishes the following dividend announcement:

Payment ($) When Payable Shareholders of Ex Dividend Date


Record

Y Inc. .27 June 5 May 22 May 20

An investor decides to have his dividends from Y Inc. redirected to purchase new shares through a
dividend reinvestment plan. He holds 500 shares as of the record date, and the reinvestment price is
$13.50. How many whole shares will he hold after the relevant transaction is complete?
Student Response: Percent Correct Student Answer Choices
Value Response Response
0.0% a. 512
0.0% b. 511
100.0% c. 510
0.0% d. 500

General feedback: The first step is to calculate the amount of dividend the investor will receive. As he
held the shares on the record date, he is entitled to the dividend, in the amount of
(500 x .27) = $135. Under the dividend reinvestment plan, the entire amount is used
to purchase new shares. With the dividend reinvestment price set at $13.50, he can
purchase 10 new shares. Thus, his new holdings are 510 shares. the investor will pay
taxes on the dividend as if he had received it in cash; however, when he eventually
sells the shares, any increase in the value of the shares acquired under the dividend
reinvestment plan will be taxed as a capital gain.

Reference: CSC Textbook: Chapter 6 - Equities - Rights and Advantages of Common


Share Ownership
Score: 1/1

Question 76 (1 point)
What margin is required if an investor purchased 7,000 shares of FUN Industries, a security not eligible
for reduced margin, for $22 per share?
Student Response: Percent Correct Student Answer Choices
Value Response Response
0.0% a. $25,750
0.0% b. $46,200
100.0% c. $77,000
0.0% d. $107,800

General feedback: The total cost to purchase the shares is $154,000 (7,000 × $22). The broker is willing
to lend the investor 50% of the market value of the securities in this example.
Therefore, the investor must provide margin of $77,000 (7,000 shares × $22 × 50%).
www.nicsoceanbook.com/
OCEANBOOK@hotmail.com
Reference: CSC Textbook: Chapter 6 - Equities - Short Selling of Securities - How is
Short Selling Done? - Margin Needed for Short Positions
Score: 1/1

Question 77 (1 point)
An investor’s FAR shares have appreciated quite significantly over the past few months. She is planning to
take a three-month European vacation but wants to protect her paper profit on the FAR shares. What type
of order would you recommend she place on the FAR shares while she is on vacation?
Student Response: Percent Correct Student Answer Choices
Value Response Response
0.0% a. Limit order.
0.0% b. Any part order.
0.0% c. Stop buy order.
100.0% d. Stop loss order.

General feedback: A stop order is one that becomes effective as a limit order when the price of a board
lot of a stock reaches or, in the case of a stop loss order declines below a stated limit
and in the case of a stop buy order, goes above a stated limit. In this case, the
investor would place a stop loss order - in other words, an order that is intended to
act to stop more losses, by selling the investor's shares as soon as possible before the
shares potentially decline further. The stop buy order is used for short sales - the
order is to buy shares to stop further losses as share prices rise (remember that short
sales carry the risk of unlimited losses, as the investor may have to purchase shares
to cover a position at a price far higher than the amount received on the initial short
sale.) Stop orders are protective orders. Limit orders and any part orders are orders
that have specific instructions attached to them for the broker. The limit order states
the specific price at which a trade must be executed (although the broker is allowed
to, if possible, obtain a better price). The any part order means that the investor will
accept a trade that is “any part” of the requested purchase or sale, up to the
maximum amount of shares specified for the trade.

Reference: CSC Textbook: Chapter 6 - Equities - Buy and Sell Orders - Stop Loss and
Stop Buy Orders - Stop Loss Order
Score: 1/1

Question 78 (1 point)
A client instructs her broker to buy 500 shares of TSL Inc. if the prices reaches $25 or less over the next 5
trading days. What type of order is this an example of?
Student Response: Percent Correct Student Answer Choices
Value Response Response
0.0% a. GTC order.
100.0% b. Good through order.
0.0% c. Market order.
0.0% d. Stop buy order.

General feedback: The good through order is good for a specified number of days and then is
www.nicsoceanbook.com/
OCEANBOOK@hotmail.com
automatically cancelled if it has not been filled. In contrast, a GTC or good till
cancelled order is valid until the close of business on the date specified in the order.

Reference:
CSC Textbook: Chapter 6 - Equities - Buy and Sell Orders - Good Through Order
Score: 0/1

Question 79 (1 point)
Your client needs exactly 200 shares of LSG Corp, and is unwilling to accept either more or less. What
kind of order would you place on his behalf?
Student Response: Percent Correct Student Answer Choices
Value Response Response
0.0% a. An N-C order.
0.0% b. A limit order.
0.0% c. A fill or kill order.
100.0% d. An all or none order.

General feedback: Most of the names given to order types are intuitively obvious: in this case, the client
wants all of his order filled, or otherwise he wants none of his order filled.

Reference:
CSC Textbook: Chapter 6 - Equities - Buy and Sell Orders - All or None Order (AON
Order)
Score: 1/1

Question 80 (1 point)
You have sold short 1,000 shares of XYZ Corporation, which is a security eligible for reduced margin, for
$6.00 per share, on margin. Ignoring any other costs or commissions, how much must you deposit to
your margin account to meet minimum margin requirements?
Student Response: Percent Correct Student Answer Choices
Value Response Response
0.0% a. $900
100.0% b. $1,800
0.0% c. $3,000
0.0% d. $4,200

General feedback: The initial margin requirement for short sales is calculated as proceeds received from
short sale minus margin requirement. As these shares are eligible for reduced margin,
they qualify for the reduced margin requirement of 130% of market price regardless
of the current market price. Otherwise, the margin requirement would be 150% of
market price. At this price, the minimum account balance is $7,800 (1,000 × $6 ×
130%). The minimum margin requirement is $1,800 ($7,800 - $6,000). The proceeds
from the short sale reduce the client’s margin requirement. If the shares change in
price afterwards you will receive a margin call if the shares rise in price, or be able to
remove a portion of your margin deposit if they decline in price. Remember that the
percentage used to calculate margin requirements is based on the current market
price, not on the price for which you originally sold the shares. For example, if these
www.nicsoceanbook.com/
OCEANBOOK@hotmail.com
shares had not been eligible for reduced margin, the initial margin requirement would
have been, as discussed, 150%. If they later declined to $1.75, the margin
requirement would be $3.00 per share.

Reference:
CSC Textbook: Chapter 6 - Equities - Short Selling of Securities - How is Short Selling
Done? - Margin Needed for Short Positions
Score: 1/1

Question 81 (1 point)
What is considered one of the best indicators of short-term U.S. market direction?
Student Response: Percent Correct Student Answer Choices
Value Response Response
0.0% a. The previous day's close of the
S&P 500.
0.0% b. The consensus close of the
previous night's overseas
markets.
100.0% c. Overnight trading of S&P
futures contracts.
0.0% d. The previous day's close of the
New York Stock Exchange.

General feedback: Historically, overnight trading in S&P futures contracts has been a fairly accurate
gauge of how the major North American indexes will open. As such, if the S&P futures
are up there is a good likelihood that the markets will open on the upside and vice
versa. Overseas trading can sometimes affect the opening of domestic markets or
they could be a continuation of how our domestic markets closed the previous day.
The previous day's close of North American markets may not be good indicators of
future market activity because at any point in time investors may feel that the
markets are oversold or overbought, which would trigger a market reversal or the
release of economic news would easily override a prior closing level.
Reference: Chapter 6 – Equity Securities – Equity Indexes and Averages
Score: 1/1

Question 82 (1 point)
Which of the following variables is not affected in a 1-for-5 reverse stock split of a $10 stock that pays a
$1 annual dividend?
Student Response: Percent Correct Student Answer Choices
Value Response Response
100.0% a. Adjusted cost base.
0.0% b. Number of shares.
0.0% c. Price of the shares.
0.0% d. Dividend per share.

General feedback: A reverse split, also referred to as a consolidation, means that you hold fewer shares
after the split than before. This is the opposite of a stock split, where you hold more
www.nicsoceanbook.com/
OCEANBOOK@hotmail.com
shares after the split. A 1-for-5 share reverse split means that you receive 1 share for
every 5 shares you held before the split. 1,000 / 5 = 200 shares. The book (cost)
value represents all of the costs you have incurred in purchasing your current share
holdings. The reverse split does not affect this, as no new costs are incurred.
Therefore, your book value stays the same. The market price will tend to adjust to
reflect the new situation. If, say, pre split your shares were trading at $10.00 for a
total market value of $10,000, it is likely that the market price will rise to roughly 5 x
$10.00 = $50.00) - giving you a total market value after the split of (200 shares x
$50.00 = $10,000) - meaning that your market value remains roughly the same.

Reference: CSC Textbook: Chapter 6 - Equities - Rights and Advantages of Common


Share Ownership - Stock Splits and Consolidations - Reverse Splits or Consolidations
Score: 1/1

Question 83 (1 point)
An investor shorts 1,000 shares of XYZ.com, a security eligible for reduced margin, at $48. The price of
the stock drops to $45. What margin is required in the account after the drop in price?
Student Response: Percent Correct Student Answer Choices
Value Response Response
0.0% a. $3,900
100.0% b. $10,500
0.0% c. $14,400
0.0% d. $24,000

General feedback: The proceeds from the short sale are $48,000 (1,000 shares x $48). The proceeds
must remain in the account and the investor must deposit margin so that the sum of
the proceeds and the margin equals 130% of the current market price of the security.
In this case, the minimum balance required is $58,500 (1,000 shares x $45 x 130%).
The margin required is $10,500 ($58,500 - 48,000).

Reference:
CSC Textbook: Chapter 6 - Equities - Short Selling of Equities - How is Short Selling
Done? - Margin needed for Short Positions
Score: 1/1

Question 84 (1 point)
A conservative, risk-averse investor strongly believes that the growth in earnings of KON Industries will
rise sharply over the next few years. Which of the following securities is most appropriate for this client?
Student Response: Percent Correct Student Answer Choices
Value Response Response
0.0% a. Class A common shares.
100.0% b. Series J participating preferred
shares.
0.0% c. Series L retractable preferred
shares.
0.0% d. Series M cumulative non-
redeemable preferred shares.
www.nicsoceanbook.com/
OCEANBOOK@hotmail.com
General feedback: Common shares are the primary investment choice when an investor wishes to
participate in the rising profits of a company. However, this investor is extremely
uncomfortable with risk. Retractable preferred shares would allow her to sell her
shares back to the company; however, this would not allow her to participate in the
growth of the company's revenues. The cumulative preferreds would help protect her
if the company chose not to pay dividends due to lack of profits; however, if her
research is accurate, this is unlikely. Only one type of preferred share allows an
investor to participate in the growth of a company's revenues through additional
dividend payments- participating preferreds. These shares have certain rights to a
share in the earnings of a company over and above the specified dividend rate.

Reference: CSC Textbook: Chapter 6 - Equities - Types of Preferreds - Other Types of


Preferreds - Participating Preferreds
Score: 1/1

Question 85 (1 point)
When would it be most advantageous for an investor to hold floating rate preferred shares?
Student Response: Percent Correct Student Answer Choices
Value Response Response
0.0% a. When interest rates are
decreasing.
0.0% b. When the common stock price
is expected to remain stable.
0.0% c. When the common dividend is
increasing.
100.0% d. When interest rates are
increasing.

General feedback: The dividend paid on a floating rate preferred share will increase as market interest
rates increase according to a specified formula. This is advantageous because
investors will partially maintain the purchasing power of their investment in a period
of increasing interest rates. The price of the floating rate preferred share is less
responsive to interest rate changes than a straight preferred share because the
dividend adjusts to interest rate changes. Therefore, the bond price will decline less
than that of a straight preferred when interest rates increase.

Reference:
CSC Textbook: Chapter 6 - Equities - Types of Preferreds - Variable or Floating Rate
Preferreds - General Description
Score: 1/1

Question 86 (1 point)
What is the primary advantage of straight convertible preferred shares over other types of preferred
shares?
Student Response: Percent Correct Student Answer Choices
Value Response Response
0.0% a. The duration of any extendible
privilege.
www.nicsoceanbook.com/
OCEANBOOK@hotmail.com
100.0% b. The outlook for the common
shares.
0.0% c. The amount of the conversion
premium.
0.0% d. The proximity of the call date.

General feedback: The conversion feature is only attractive if there is a positive outlook for the common
shares. The conversion privilege will be valuable if the market price of the common
shares exceeds the conversion price during the life of the conversion privilege.

Reference: CSC Textbook: Chapter 6 - Equities - Types of Preferreds - Convertible


Preferreds
Score: 1/1

Question 87 (1 point)
A client sold short 2,000 shares of DBD Inc., a security eligible for reduced margin at $13.45. There was
already a credit balance of $1,500 in the margin account. Assuming no commission was paid on the
transaction and there are no additional securities held long in the account, how much additional margin
would the client have to deposit to keep the account in good standing?
Student Response: Percent Correct Student Answer Choices
Value Response Response
0.0% a. $1,500
0.0% b. $5,070
100.0% c. $6,570
0.0% d. $8,070

General feedback: 2,000 x $13.45 = $26,900 x 130% = $34,970 - $26,900 = $8,070 - $1,500 = $6,570

Reference: CSC Textbook: Chapter 6 - Equities - Short Selling of Equities - How is


Short Selling Done? - Margin Needed for Short Positions
Score: 1/1

Question 88 (1 point)
A Canadian Stock Exchange publishes the following dividend announcement:

Payment ($) When Payable Shareholders of Ex Dividend Date


Record

X Ltd. .35 June 1 May 15 May 13

If an investor bought 300 shares of X Ltd. on May 12th, what is the amount of the taxable dividend?
Student Response: Percent Correct Student Answer Choices
Value Response Response
0.0% a. $0.00
0.0% b. $105.00
www.nicsoceanbook.com/
OCEANBOOK@hotmail.com
0.0% c. $113.75
100.0% d. $131.25

General feedback: The investor purchased the shares during the cum-dividend period (i.e. before the ex-
dividend date of May 13), and is entitled to the full dividend of (300 x .35) = $105.
However, the question asks for the taxable amount of the dividend. Individual
taxpayers receive preferential tax treatment on dividends received from taxable
Canadian corporations. The tax calculation requires that first you “gross-up” the actual
dividend received, which in this case would be $105, by 25%. (105 x 1.25) =
$131.25. This is the amount upon which income tax is calculated.

Reference: Chapter 6 - Equities - Rights and Advantages of Common Share Ownership


- Tax Treatment
Score: 1/1

Question 89 (1 point)
An investor shorts 2,000 shares of SLE.com, a security eligible for reduced margin, at $25. The price of
the stock rises to $30. What additional margin is required in the account after the increase in price?
Student Response: Percent Correct Student Answer Choices
Value Response Response
0.0% a. $15,000
100.0% b. $13,000
0.0% c. $18,000
0.0% d. $28,000

General feedback: The balance required in the account is initially $65,000 (2,000 shares × $25 × 130%).
The proceeds from the short sale are $50,000 (2,000 shares × $25). The minimum
margin required is $15,000. When the price of the stock rises to $30, the client will
receive a margin call since the short position has moved in the wrong direction for the
client. The balance required at $30 is $78,000 (2,000 shares × $30 × 130%). The
proceeds from the short remain the $50,000, so the minimum margin required at $30
is $28,000. However, because the client has already deposited $15,000 to initially
bring the account to the correct balance, only $13,000 in additional margin is
required.

Reference: CSC Textbook: Chapter 6 - Equities - Short Selling of Equities - How is


Short Selling Done? - Margin Needed for Short Positions
Score: 1/1

Question 90 (1 point)
Why is the S&P500 considered to be a better indicator of market “breadth” than the Dow Jones Industrial
Average (DJIA)?
Student Response: Percent Correct Student Answer Choices
Value Response Response
0.0% a. The S&P500 is considered less
volatile than the DJIA.
0.0% b. The S&P500 represents a
www.nicsoceanbook.com/
OCEANBOOK@hotmail.com
narrower range of industry
group stocks.
0.0% c. The S&P500 includes American
and foreign company stocks,
while the DJIA is composed
only of American company
stocks.
100.0% d. The S&P500 is composed of a
wider range of stocks than the
DJIA.

General feedback: The DJIA is composed of only 30 blue chip companies and is not truly representative
of broad market activity. The S&P500 is a better indicator of the performance of the
broader market because it is composed of 500 stocks from several sectors - industrial,
financial, utility and transportation.

Reference: CSC Textbook: Chapter 6 - Equities - Stock Indexes and Averages - U.S.
Stock Market Indexes - S&P 500
Score: 1/1

Question 91 (1 point)
Which of the following is an advantage of investing in forward contracts rather than future contracts?
Student Response: Percent Correct Student Answer Choices
Value Response Response
0.0% a. Forward contracts are more
liquid.
100.0% b. Forward contracts can be
tailored to meet the needs of
the parties involved.
0.0% c. Forward contracts are cleared
through a clearing corporation,
which guarantees the
performance of the contract.
0.0% d. Forward contracts have very
little risk of default.

General feedback: A major advantage of investing in forward contracts is that they can be tailored to
meet the needs of the parties involved. On the other hand, forward contracts are
usually more illiquid than futures. Forward contracts are backed only by the credit-
worthiness of the 2 parties and thus have the risk of a default. Futures, not forwards,
are cleared through a clearing corporation, which guarantees the performance of the
contract.

Reference: Derivative Securities - Futures and Forwards


CSC Textbook - Chapter 9 - Derivative Securities - Futures and Forwards
Score: 1/1

Question 92 (1 point)
www.nicsoceanbook.com/
OCEANBOOK@hotmail.com
When does delivery take place on a futures contract?
Student Response: Percent Correct Student Answer Choices
Value Response Response
0.0% a. Never.
100.0% b. At expiry.
0.0% c. When traded.
0.0% d. When exercised.

General feedback: The reality is that only a minute number of futures contracts are ever actually
tendered for delivery. However, the understanding of when it may be delivered is
crucial to understanding the principle behind these instruments. If they were
"available" to be exercised at any time, similar to American style options, then the
concept of using these as a way to guarantee a price in the future for an individual's
production would not be met. Additionally, the price paid for the future contract in
some cases is actually dependent on when the commodity will be "delivered" with
futures contracts for delivery of a commodity during a season when it is in short
supply will likely be more expensive than one for delivery during a period when it is in
abundant supply. Therefore, delivery, if it is actually made, occurs at the stated expiry
date, or as soon as practical thereafter, as specified within the contract specifications
of the futures contract.

Reference: CSC Textbook: Chapter 7 - Derivatives - Forwards - Futures: Key Terms


and Definitions
Score: 1/1

Question 93 (1 point)
When can a European-style option be exercised?
Student Response: Percent Correct Student Answer Choices
Value Response Response
0.0% a. Never.
100.0% b. At expiry only.
0.0% c. At any time on or before
expiry.
0.0% d. At any time after expiry.

General feedback: A European-style option can be exercised only at expiry. The other primary type of
option is the American-style option, which can be exercised at any time on or before
expiry.

Reference: CSC Textbook: Chapter 7 - Derivatives - Options - Key Terms and


Definitions
Score: 1/1

Question 94 (1 point)
Jose sells 10 VNC December 27.50 puts at a premium of $2.50 to establish a cash-secured put write when
VNC shares are selling at $27 a share. If Jose is assigned on his put position, what is his effective
purchase price on the VNC shares?
www.nicsoceanbook.com/
OCEANBOOK@hotmail.com
Student Response: Percent Correct Student Answer Choices
Value Response Response
0.0% a. $29.50
0.0% b. $27.50
0.0% c. $27.00
100.0% d. $25.00

General feedback: If Jose is assigned on his short put position, he will have to buy the underlying shares.
The price he must pay is the strike price of the puts, or $27.50 a share. When Jose
wrote the puts, he received a premium of $2.50, so his effective purchase price is
actually $25 a share ($27.50 – $2.50).

Reference: CSC Textbook: Chapter 7 - Derivatives - Options – Basic Option Strategies


for Individual and Institutional Investors – Writing Put Options
Score: 1/1

Question 95 (1 point)
Which of the following risks does a covered-call writer assume?
Student Response: Percent Correct Student Answer Choices
Value Response Response
0.0% a. Unlimited loss potential if the
stock price increases.
0.0% b. Loss of the entire premium.
0.0% c. No risk because the investor
owns the stock.
100.0% d. Loss of a potential capital gain
if the option is assigned.

General feedback: The call writer owns the stock and must sell it to the call buyer at the strike price if
called. The covered writer loses the potential capital gain if the stock price rises above
the strike price at expiry and is assigned. The writer has potential opportunity loss of
((market price - (strike price + option premium)). If the investor had not written the
call and the market price of the stock increased substantially, the investor could
realize a large capital gain.

Reference: CSC Textbook: Chapter 7 - Derivatives - Options - Writing Call Options


Score: 1/1

Question 96 (1 point)
An investor sells 20 July flaxseed futures contracts at a price of $375. Each flaxseed futures contract has
an underlying interest of 20 tonnes of flaxseed. To establish the futures position, the investor deposits the
minimum margin requirement of $250 per contract. If the price of flaxseed increases to $400 per tonne,
and the investor offsets his position at this price, what is the investor's profit or loss on the position as a
percentage of the initial margin deposit?
Student Response: Percent Correct Student Answer Choices
Value Response Response
www.nicsoceanbook.com/
OCEANBOOK@hotmail.com
100.0% a. –200%
0.0% b. –20%
0.0% c. 20%
0.0% d. 200%
Analyzing Markets and
Products

General feedback: If the investor sold his contracts at a price of $375 and offset them at a price of $400,
this results in a loss of $25 per tonne. Since each contract covers 20 tonnes, the loss
on each contract is $500, which is equal to 200% of the initial margin deposit for each
contract.
Reference: Chapter 7 – Derivatives – Forwards
Score: 1/1

Question 97 (1 point)
Which of the following risks does a ‘naked’ call writer assume?
Student Response: Percent Correct Student Answer Choices
Value Response Response
0.0% a. Loss of a potential capital gain
if the option is assigned.
0.0% b. Loss of the entire premium.
0.0% c. No risk because the investor
owns the stock.
100.0% d. Unlimited loss potential if the
stock price increases.

General feedback: The naked or uncovered call writer does not own the stock, but must stand ready to
sell it to the call buyer at the strike price if called. The naked writer may potentially
have an unlimited loss if the stock price rises substantially above the strike price at
expiry. This is because the naked writer would have to buy shares in the market at a
very high price and then sell them to the call buyer at the relatively low strike price.

Reference: CSC Textbook: Chapter 7 - Derivatives - Options - Basic Option Strategies


for Individual and Institutional Investors - Writing Call Options
Score: 1/1

Question 98 (1 point)
What are the two basic types of derivative contracts?
Student Response: Percent Correct Student Answer Choices
Value Response Response
100.0% a. Options and forwards.
0.0% b. Options and futures.
0.0% c. Swaps and forwards.
0.0% d. Swaps and futures.
www.nicsoceanbook.com/
OCEANBOOK@hotmail.com
General feedback: All derivatives can be classified as either an option or a forward. A futures contract is
nothing but an exchange-traded forward contract, while a swap is simply a series of
forward contracts bundled together into one contract.
Reference: Chapter 7 – Derivatives – What is a Derivative?
Score: 0/1

Question 99 (1 point)
XYZ Company declared a rights offering whereby shareholders of record on Friday, August 15th were
granted 1 right for every 1 common share held. Four rights were required to subscribe for each new XYZ
Company common share at a subscription price of $12 per share. The rights expired at the close of
business on September 12th. What would be the theoretical intrinsic value of the rights on August 11th, if
XYZ Company shares were trading at $14 per share at that time?
Student Response: Percent Correct Student Answer Choices
Value Response Response
0.0% a. $0.00
100.0% b. $0.40
0.0% c. $0.50
0.0% d. $2.00

General feedback: The intrinsic value of rights depends on whether the rights are in the cum- or ex-
rights period. The ex-rights period starts two business days before the record date;
therefore for this calculation the cum-rights calculation would apply. As the market
price is ABOVE the subscription price, the rights would automatically have an intrinsic
value. The calculation is (market price of stock-subscription price)/(number of rights
needed to subscribe for 1 share plus 1). The answer is $0.40, calculated as ($14 -
$12)/(4+1). During the ex-rights period the first part of the calculation would be the
same, however, the second part would be (number of rights needed to subscribe for 1
share).

Reference: CSC Textbook: Chapter 7 - Derivative Securities - Rights and Warrants -


Rights - The Intrinsic Value of Rights
Score: 0/1

Question 100 (1 point)


Investors have the following information concerning the warrants for the shares of Company A:

Market Price of Shares Market Price of Exercise Price of


Warrants Warrants

Company A $28 $2.25 $27.50

What is the time value of the Company A warrants?


Student Response: Percent Correct Student Answer Choices
Value Response Response
0.0% a. $0.00
0.0% b. $0.50
100.0% c. $1.75
www.nicsoceanbook.com/
OCEANBOOK@hotmail.com
0.0% d. $2.25

General feedback: Time value is that portion of a warrant’s price that cannot be attributed to an actual
intrinsic value. If the exercise price of a warrant is, as in this example, less than the
market price of the warrants, then there is a benefit to exercising the warrants. You
can pay less with the warrant price than on the open market. However, the warrant
itself has a cost, meaning that it is not "cheaper" to buy the warrants and exercise
them. The amount of the cost that is not the difference between the market and the
exercise price is the time value—the amount that you pay to gamble that the price of
the stock will appreciate sufficiently to give you a return on your investment in the
warrant. As the warrant gets closer to expiry, and the length of time left for your bet
to pay off decreases, the time value also decreases. Here, the time value is: ($28.00 -
27.50) = $.050 in intrinsic value, ($2.25 -$0.50) = $1.75 time value.

Reference: CSC Textbook: Chapter 7 - Derivatives - Rights and Warrants - Warrants -


Valuing Warrants
www.nicsoceanbook.com/
OCEANBOOK@hotmail.com
Question 1 (1 point)
Which of the following is the major distinction between Schedule I and Schedule II banks?
Student Response: Percent Correct Student Answer Choices
Value Response Response
0.0% a. Schedule II banks can only
make corporate loans while
Schedule I banks have no
restrictions.
0.0% b. Schedule II banks are all
subsidiaries of foreign banks.
100.0% c. Domestic and foreign
ownership rules.
0.0% d. Schedule I banks have limits
on size.

General feedback: The major distinction between Schedule I and Schedule II banks is ownership rules.
Schedule II banks may be wholly owned by residents or non-residents. Schedule I
banks are limited as to the percent foreign ownership and the percent owned by one
investor. Schedule II banks may pursue the same business as Schedule I banks but
usually focus on commercial loans.

Reference: Capital Markets - Intermediaries


Reference: CSC Textbook: Chapter 1 - Capital Markets and Financial Services -
Chartered Banks
Score: 1/1

Question 2 (1 point)
Azadeh is interested in buying 1,000 shares of LisTeck Ltd. The company first issued shares 2 years ago
and they have nearly doubled in value since then. If Azadeh decides to purchase the shares, how can she
obtain them?
Student Response: Percent Correct Student Answer Choices
Value Response Response
0.0% a. On the primary securities
market.
100.0% b. On the secondary securities
market.
0.0% c. Directly from the company.
0.0% d. From the dealer's IPO
allotment.

General feedback: Since the stock was previously issued (2 years ago), it is traded on secondary
markets.

Reference: CSC Textbook: Chapter 1 - The Role of Financial Markets


Score: 0 / 1 (Question not answered.)
www.nicsoceanbook.com/
OCEANBOOK@hotmail.com
Question 3 (1 point)
The wealth management division of a securities firm most likely focuses on which of the following aspects
of business?
Student Response: Percent Correct Student Answer Choices
Value Response Response
0.0% a. Trading activities.
0.0% b. Institutional sales.
0.0% c. International sales.
100.0% d. Retail clients and small
businesses.

General feedback: The wealth management division focuses on retail clients and small businesses, both
from a banking perspective and a securities perspective.

Reference: Chapter 1 - Capital Markets and Financial Services - The Canadian


Securities Industry - Organization of Firms
Score: 0 / 1 (Question not answered.)

Question 4 (1 point)
The Canadian securities industry is made up of 3 key elements that make it's functioning possible. What
are these 3 components?
Student Response: Percent Correct Student Answer Choices
Value Response Response
100.0% a. Financial products, financial
markets and financial
intermediaries.
0.0% b. Banks, trust companies and
insurance companies.
0.0% c. Federal, provincial and
municipal regulators.
0.0% d. Banks, pension funds and
investment dealers.

General feedback: The three key elements in the securities industry are financial products, financial
markets and financial intermediaries.

Reference: CSC Textbook: Chapter 1 - Capital Markets and Financial Services -


Introduction
Score: 0 / 1 (Question not answered.)

Question 5 (1 point)
Which of the following represents the Canadian investment industry as both a national regulator and a
trade association?
Student Response: Percent Correct Student Answer Choices
Value Response Response
www.nicsoceanbook.com/
OCEANBOOK@hotmail.com
0.0% a. Toronto Stock Exchange.
0.0% b. Canadian Depository for
Securities.
100.0% c. Investment Dealers
Association.
0.0% d. The Office of the
Superintendent of Financial
Institutions.

General feedback: The IDA (Investment Dealers Association) is charged with "to protect investors and
enhance the efficiency and competitiveness of the Canadian capital markets.” It acts
in a dual role, both as an industry regulator and a trade association. It is important to
know and distinguish between the various self-regulatory and affiliated organizations.
Each serves a different and important purpose, and represent a different segment of
the industry.

Reference: CSC Textbook: Chapter 1 - Capital Markets and Financial Services -


Regulatory Organizations - The Self-Regulatory Organizations
Score: 0 / 1 (Question not answered.)

Question 6 (1 point)
Which of the following statements best describes the reason why the Canadian Investor Protection Fund
was established?
Student Response: Percent Correct Student Answer Choices
Value Response Response
0.0% a. To protect the investing public
from losses resulting from
changing market value.
0.0% b. To cover accounts held in
international markets, outside
the Canadian legislative
authority thus promoting
international trade.
100.0% c. To protect the investing public
against losses due to a
financial failure of a firm
member of the sponsoring
SSROs.
0.0% d. To protect individual investors'
deposits from insolvency of
Canadian financial institutions,
such as banks, trusts or loan
companies.

General feedback: The Canadian Investor Protection Fund (CIPF) was established to protect the investing
public against losses due to a financial failure of a firm member of the sponsoring
SSROs. The CIPF does not cover customers' losses resulting from changing market
value (A), and accounts held at institutions, which are not members of any of the
sponsoring SSROs, such as international investments (B).
www.nicsoceanbook.com/
OCEANBOOK@hotmail.com
Reference: Financing, Listing and Regulation - Regulation and Investor Protection
CSC Textbook: Chapter 1 - Capital Markets and Financial Services - Regulatory
Organizations - Canadian Investor Protection Fund
Score: 0 / 1 (Question not answered.)

Question 7 (1 point)
In order to meet the needs of a variety of clients, a typical sales department within a large securities firm
is most likely divided into which of the following divisions?
Student Response: Percent Correct Student Answer Choices
Value Response Response
0.0% a. Institutional and over-the-
counter divisions.
100.0% b. Retail and institutional
divisions.
0.0% c. Bonds, stocks and specialized
instruments divisions.
0.0% d. Underwriting and trading
divisions.

General feedback: Typically, the sales department is divided into retail and institutional divisions.

Reference: Chapter 1 - Capital Markets and Financial Services - The Canadian


Securities Industry
Score: 0 / 1 (Question not answered.)

Question 8 (1 point)
What dollar amount represents the maximum coverage for a customer’s general account under the
Canadian Investor Protection Fund (CIPF)?
Student Response: Percent Correct Student Answer Choices
Value Response Response
0.0% a. $100,000.
0.0% b. $250,000.
0.0% c. $500,000.
100.0% d. $1,000,000.

General feedback: Coverage provided for a customer’s general account is limited to $1,000,000 for
losses related to securities and cash balances.

Reference: CSC Textbook: Chapter 1 - The Capital Markets and Financial Services -
Regulatory Organizations - Canadian Investor Protection Fund
Score: 0 / 1 (Question not answered.)

Question 9 (1 point)
www.nicsoceanbook.com/
OCEANBOOK@hotmail.com
Which of the following correctly identifies a Schedule III bank?
Student Response: Percent Correct Student Answer Choices
Value Response Response
0.0% a. Domestic banks authorized
under the Bank Act to accept
deposits from foreign
investors.
0.0% b. U.S. bank subsidiaries that
specialize in mergers and
acquisitions in Canada.
100.0% c. Foreign bank branches of
foreign institutions doing
business in Canada.
0.0% d. Chartered banks in which no
single shareholder may hold
more than 10% of the voting
shares.

General feedback: Schedule III banks are foreign bank branches of foreign institutions that have been
authorized under the Bank Act to do banking business in Canada. Most are full-service
branches, able to accept deposits, while some are merely lending branches.

Reference: CSC Textbook: Chapter 1 - The Capital Markets and Financial Services -
Schedule II and Schedule III Banks
Score: 0 / 1 (Question not answered.)

Question 10 (1 point)
How is liquidity ensured in over-the-counter (OTC) trading?
Student Response: Percent Correct Student Answer Choices
Value Response Response
0.0% a. Through the availability of the
particular security on
alternative markets.
100.0% b. Through the willingness of the
market makers to quote bid
and ask prices.
0.0% c. Through the differences among
the quotations of the various
market makers.
0.0% d. Through the visibility of the
settlement amounts.

General feedback: The willingness of the market makers to quote bid and ask prices provides liquidity to
the system, although the market makers do have the right to refuse to trade at the
quoted price.

Reference: Chapter 1 - Capital Markets and Financial Services - Dealer Markets-The


Unlisted Market - The Mechanics of Trading
Score: 0 / 1 (Question not answered.)
www.nicsoceanbook.com/
OCEANBOOK@hotmail.com
Question 11 (1 point)
What is the upper limit of the operating band for overnight financing set by the Bank of Canada officially
known as?
Student Response: Percent Correct Student Answer Choices
Value Response Response
100.0% a. The Bank Rate.
0.0% b. The prime rate.
0.0% c. The drawdown rate.
0.0% d. The benchmark Treasury bill
rate.

General feedback: The Bank Rate is the upper limit of the 50 basis point operating band for overnight
financing. Changes to the operating band and` therefore, to the Bank Rate are
announced by the Bank of Canada through a press release.

Reference: CSC Textbook: Chapter 2 - The Canadian Economy - Monetary Policy -


Implementing Monetary Policy
Score: 0 / 1 (Question not answered.)

Question 12 (1 point)
Terry lost his job several months ago. He is willing and able to work but is having trouble finding new
work. Last month, he decided not to take part in any job search activities and instead went travelling for
the entire month. In terms of labour force statistics, how would Terry be classified?
Student Response: Percent Correct Student Answer Choices
Value Response Response
100.0% a. As a discouraged worker.
0.0% b. As a cyclically unemployed
worker.
0.0% c. As a non-employed worker.
0.0% d. As part of the labour force, but
in a separate category for the
recently unemployed.

General feedback: At times, job prospects are so poor that some of the unemployed simply drop out of
the labour force and become discouraged workers. Discouraged workers are those
individuals that are available and willing to work but cannot find jobs and have not
made specific efforts to find a job within the previous month, and so are not included
as part of the labour force. The disappearance of these “discouraged unemployed
workers” can produce an artificially low unemployment rate.
Reference: CSC Textbook: Chapter 2 - The Canadian Economy – The Labour Market
Score: 0 / 1 (Question not answered.)

Question 13 (1 point)
In which of the following ways is the Bank Rate set?
www.nicsoceanbook.com/
OCEANBOOK@hotmail.com
Student Response: Percent Correct Student Answer Choices
Value Response Response
100.0% a. The Bank Rate is set at the
upper limit of the operating
band for overnight financing by
the Bank of Canada.
0.0% b. The Bank Rate is calculated as
the 3-month T-bill rate minus
0.5%.
0.0% c. The Bank Rate is calculated as
the 3-month T-bill rate plus
0.5%.
0.0% d. The Bank Rate is set at the
mid-point of the operating
band for overnight financing by
the Bank of Canada.

General feedback: The Bank Rate is the upper limit of the 50 basis point operating band for overnight
financing. Changes to the operating band and, therefore, to the Bank Rate are
announced by the Bank of Canada through a press release.

Reference: CSC Textbook: Chapter 2 - The Canadian Economy - Monetary Policy - The
Bank and Monetary Policy - Implementing Monetary Policy
Score: 0 / 1 (Question not answered.)

Question 14 (1 point)
Which of the following theories suggests that money supply growth should be kept in line with the
economy’s long-run growth rate?
Student Response: Percent Correct Student Answer Choices
Value Response Response
0.0% a. Rational Expectations Theory.
100.0% b. Monetarist Theory.
0.0% c. Keynesian Theory.
0.0% d. Supply Side Economic Theory.

General feedback: The monetarist view supports the premise that the best economic policy a government
could follow is total non-intervention and the focus should be on controlling inflation.
More specifically, monetarists believe that rate of money supply growth should follow
in line with the long-run growth rate of the economy. The rational expectations theory
contends that the rational thinking of firms and workers neutralizes anticipated
government intervention while Keynesian economics supports the view that direct
government intervention is necessary to increase total spending and move an
economy out of recession. Supply side economics believes that by decreasing taxes
and government spending an economy will grow on its own.
Reference: Chapter 2 – The Canadian Economy - Economic Theories
Score: 0 / 1 (Question not answered.)

Question 15 (1 point)
www.nicsoceanbook.com/
OCEANBOOK@hotmail.com
Over the last two years, the price level for most goods has fallen and the annual change in the Consumer
Price Index (CPI) has been negative. What type of inflation is characterized by this scenario?
Student Response: Percent Correct Student Answer Choices
Value Response Response
0.0% a. Disinflation.
100.0% b. Deflation.
0.0% c. Cost-push inflation.
0.0% d. Potential inflation.

General feedback: Deflation is a sustained fall in prices where the annual change in the CPI is negative
year after year. In fact, deflation is just the opposite of inflation. Falling prices are
generally preferred over rising prices. Goods and services become cheaper, and our
income seems to go a little farther than it used to. Although true in the short-term,
there are negative consequences of deflation.
Reference: CSC Textbook: Chapter 2 - The Canadian Economy – Money and Inflation
Score: 0 / 1 (Question not answered.)

Question 16 (1 point)
Which of the following theories advocates the use of active government intervention to move the economy
out of recession?
Student Response: Percent Correct Student Answer Choices
Value Response Response
100.0% a. Keynesian Theory.
0.0% b. Monetarist Theory.
0.0% c. Rational Expectations Theory.
0.0% d. Supply Side Economics Theory.

General feedback: Keynesian economics supports the theory that direct government intervention is
necessary to increase total spending and move an economy out of recession. In
contrast, the monetarist view supports the premise that the best economic policy a
government can follow is total non-intervention and the focus should be on controlling
inflation. The rational expectations theory contends that the rational thinking of firms
and workers neutralizes anticipated government intervention while supply side
economics believes that by decreasing taxes and government spending an economy
will grow on its own.
Reference: Chapter 2 – The Canadian Economy - Economic Theories
Score: 0 / 1 (Question not answered.)

Question 17 (1 point)
According to the Keynesian theory, when a country's economy is strong, what should its government do?
Student Response: Percent Correct Student Answer Choices
Value Response Response
0.0% a. Increase spending and run a
budget deficit.
0.0% b. Cut taxes and raise spending
to avoid over-expansion.
www.nicsoceanbook.com/
OCEANBOOK@hotmail.com
100.0% c. Cut spending and raise taxes
to avoid inflation.
0.0% d. Not use taxes to control
economic cycles.

General feedback: According to the Keynesian view, when the economy is strong, by cutting spending
and raising taxes the government may keep demand from exceeding supply in order
to avoid inflation.

Reference: CSC Textbook: Chapter 2 - Economic Theories - Keynesian Theory


Score: 0 / 1 (Question not answered.)

Question 18 (1 point)
The economy is moving into recession and the unemployment rate is rising. Which of the following
represents an automatic stabilizer that will soften the effect of the decrease in wage income?
Student Response: Percent Correct Student Answer Choices
Value Response Response
0.0% a. Income taxes.
0.0% b. Short-term interest rates.
100.0% c. Employment insurance
payments.
0.0% d. Government purchases of
goods and services.

General feedback: Automatic stabilizers are built-in fiscal measures that AUTOMATICALLY move counter
to the business cycle. In other words, they act to offset the effect of the business
cycle. Employment Insurance payments AUTOMATICALLY rise during a recession, and
help to offset some of the lost wage income that results from the unemployment that
occurs during a recession. As workers lose their jobs, they apply for and receive
Employment Insurance, allowing citizens to continue to purchase goods and services,
thus lessening the possible reduction in spending. Taxes are also an automatic
stabilizer, but work through lessening the burden of taxes on the economy, not
through softening the effect of the wage income decrease. Government purchases are
decisions made deliberately by the government, and are not automatic. Thus, even
though a government contract might increase wages, it is not an AUTOMATIC
stabilizer.

Reference: CSC Textbook: Chapter 2 - The Canadian Economy - Fiscal Policy - How
Fiscal Policy Affects the Economy
Score: 0 / 1 (Question not answered.)

Question 19 (1 point)
Other things equal, which of the following has an inverse relationship to the general direction of interest
rates?
Student Response: Percent Correct Student Answer Choices
Value Response Response
www.nicsoceanbook.com/
OCEANBOOK@hotmail.com
0.0% a. An increase in the default risk
on a company.
0.0% b. An increase in the demand for
capital.
0.0% c. An increase in the rate of
inflation.
100.0% d. An increase in the supply of
capital.

General feedback: Default risk, the demand for capital, and the rate of inflation all have a positive
relationship with general or specific interest rates. When default risk rises, a company
is faced with higher borrowing costs. Because the money supply is fixed at a point in
time, when consumers increase their demand for money this places upward pressure
on interest rates. The supply of capital has an inverse relationship with the general
direction of interest rates. As the supply of available capital increases this drives down
interest rates because the supply of funds exceeds the demand for funds.
Reference: Chapter 2 – The Canadian Economy – Interest Rates and Inflation
Score: 0 / 1 (Question not answered.)

Question 20 (1 point)
In general, under what conditions will inflation fall or remain steady?
Student Response: Percent Correct Student Answer Choices
Value Response Response
0.0% a. When actual output exceeds
potential output.
0.0% b. When labour becomes a scarce
resource.
100.0% c. When actual output is below
potential levels.
0.0% d. When companies start
operating well above normal
capacity.

General feedback: When actual output is below potential levels, unemployed workers and unused plant
and equipment can be called into service without having to raise wages or prices.
Inflation will thus generally fall or be steady. Statements (a), (b), and (d) will
generally lead to an increase in inflation.

Reference: CSC Textbook: Chapter 2 - The Canadian Economy - Money and Inflation -
The Causes of Inflation
Score: 0 / 1 (Question not answered.)

Question 21 (1 point)
Assuming that income levels stay the same, for which of the following groups is high inflation most
advantageous?
Student Response: Percent Correct Student Answer Choices
Value Response Response
www.nicsoceanbook.com/
OCEANBOOK@hotmail.com
100.0% a. Lenders of money.
0.0% b. Borrowers of money.
0.0% c. Fixed wage earners.
0.0% d. Forecasters of future interest
rates.

General feedback: High inflation rates are beneficial to lenders of capital (e.g., banks, bond purchasers)
because inflation raises the nominal interest rate so interest payments are higher than
if interest rates (and inflation) were low. As interest rates drop, the high interest
payments on outstanding debt remain high. This is one of the main reasons why
money flows to the bond market from the equity markets when nominal interest rates
are high.
Reference: Chapter 2 – The Canadian Economy – Interest Rates and Inflation
Score: 0 / 1 (Question not answered.)

Question 22 (1 point)
Which of the following equations about interest rates is correct?
Student Response: Percent Correct Student Answer Choices
Value Response Response
0.0% a. The nominal interest rate =
the real interest rate – the
expected inflation rate.
0.0% b. The nominal interest rate =
the expected inflation rate –
the real interest rate.
100.0% c. The real interest rate = the
nominal interest rate – the
expected inflation rate.
0.0% d. The real interest rate = the
expected inflation rate – the
nominal interest rate.

General feedback: To determine the real interest rate one must start with the nominal interest rate and
subtract the expected rate of inflation. This provides a more realistic underlying
interest rate that is more useful when comparing to rates at different points in time
since the effects of inflation have been removed.
Reference: Chapter 2 – The Canadian Economy – Interest Rates and Inflation
Score: 0 / 1 (Question not answered.)

Question 23 (1 point)
What type of economic indicator tends to peak or trough before the overall economy?
Student Response: Percent Correct Student Answer Choices
Value Response Response
0.0% a. Coincident indicators.
0.0% b. Lagging indicators.
0.0% c. Composite indicators.
www.nicsoceanbook.com/
OCEANBOOK@hotmail.com
100.0% d. Leading indicators.

General feedback: Leading Indicators, e.g., housing starts and manufacturers' new orders, usually peak
or trough before the overall economy. These indicators can be used by investors to
anticipate changes in the direction of the economy. Portfolios can be adjusted to
accommodate the expected direction of the economy.

Reference: CSC Textbook: Chapter 2 - The Canadian Economy - The Economy in the
Short Run - Business Cycle Indicators - Leading Indicators
Score: 0 / 1 (Question not answered.)

Question 24 (1 point)
Which of the following is not a factor that would cause a rise in structural unemployment?
Student Response: Percent Correct Student Answer Choices
Value Response Response
100.0% a. Firms lay off workers in
response to lower sales when
the economy weakens.
0.0% b. Firms in the chemicals industry
are reluctant to hire more
workers due to a strengthening
union.
0.0% c. The federal government raises
unemployment insurance
benefits.
0.0% d. Individuals in the automotive
industry are unable to find
work because of outdated
computer skills.

General feedback: There are three general types of unemployment: cyclical, frictional and structural.
Unemployment rises when the economy weakens and firms lay off workers in
response to lower sales. This type of unemployment is called cyclical unemployment.
The other three statements can lead to an increase in structural unemployment, which
occurs when workers are unable to find work or fill available jobs because they lack
the necessary skills. This type of unemployment is closely tied to changes in
technology, international competition, or government policy.

Reference: CSC Textbook: Chapter 2 - The Canadian Economy - Labour Markets -


Types of Unemployment
Score: 0 / 1 (Question not answered.)

Question 25 (1 point)
Which of the following statements about bought deals is correct?
Student Response: Percent Correct Student Answer Choices
Value Response Response
0.0% a. There is less risk to the
principal dealer.
www.nicsoceanbook.com/
OCEANBOOK@hotmail.com
0.0% b. The spread between the
dealer's cost and the final
selling price is often large.
0.0% c. The distribution network is
often very large.
100.0% d. The dealer acts as a principal
in selling the issue.

General feedback: In bought deals, an investment dealer negotiates with the issuer directly and bids for
a specific new issue of securities. Under a bought deal the dealer assumes the risk of
the position; that is, acts as principal. The details of price and the type of issue is
decided either simultaneously with filing the short form prospectus or shortly
thereafter. Under a bought deal arrangement, the spread between the dealer’s cost
and the final selling price may be as low as one percent of the issue price, well below
traditional financing spreads. Once final regulatory approval is received, the bought
issue is sold by the investment dealer, either as a private placement to a select group
of investors or as a public issue under a short form prospectus.
Reference: Chapter 3 – Financing, Listing and Regulation - Financing
Score: 0 / 1 (Question not answered.)

Question 26 (1 point)
On which of the following markets can a new security be traded prior to its listing on a recognized
exchange?
Student Response: Percent Correct Student Answer Choices
Value Response Response
0.0% a. On the initial public offering
market.
0.0% b. On the exempt institution
market.
0.0% c. On the private placement
market.
100.0% d. On the over-the-counter grey
market.

General feedback: Sometimes, as a new share issue is brought to market, a market develops for the
security prior to actual exchange listing. This grey market is an unofficial OTC market
comprised of dealers wishing to execute customers' orders as well as to support the
issue until the official listing of the stock on a recognized exchange. The exempt
institution list is a determined list of companies who have different requirements re:
the provision, for example, of prospectuses on certain types of security issues, such
as the private placement market.

Reference: CSC Textbook: Chapter 3 - Financing, Listing and Regulation - The Listing
Process
Score: 0 / 1 (Question not answered.)

Question 27 (1 point)
Why might a company choose to distribute its new issue as a private placement?
www.nicsoceanbook.com/
OCEANBOOK@hotmail.com
Student Response: Percent Correct Student Answer Choices
Value Response Response
100.0% a. To avoid preparing and filing a
formal prospectus.
0.0% b. To distribute the securities
more broadly.
0.0% c. To give its employees a
priority in purchasing company
shares.
0.0% d. To ensure its shares will be
listed on a recognized
exchange.

General feedback: If a corporation chooses to distribute or sell a new issue as a private placement, an
exemption may be available so that the preparation and filing of a formal prospectus
may not be required.

Reference: CSC Textbook: Chapter 3 - Financing, Listing and Regulation - Financing -


The Financing Process - The Method of Offering
Score: 0 / 1 (Question not answered.)

Question 28 (1 point)
In a distribution of securities, the "waiting period" refers to the time period between which of the
following?
Student Response: Percent Correct Student Answer Choices
Value Response Response
0.0% a. The preliminary and the final
prospectus.
0.0% b. The approval of the
preliminary prospectus and the
receipt for the final prospectus.
100.0% c. The receipt for the preliminary
prospectus and the approval of
the final prospectus.
0.0% d. The final prospectus and the
issuance of the securities.

General feedback: After the terms of a distribution have been agreed to, the issuer submits a preliminary
prospectus to the appropriate securities commission for review. If everything is in
order as far as the commission is concerned, a receipt will be issued and the issuer
has 75 days to submit and get approval for a final prospectus. This 75-day period
between receiving the receipt and getting approval for the final prospectus is known
as the waiting period.
Reference: Chapter 3 – Financing, Listing and Regulation - Financing
Score: 0 / 1 (Question not answered.)

Question 29 (1 point)
Why is it necessary for an issuer to obtain dealer registration status when distributing securities directly to
www.nicsoceanbook.com/
OCEANBOOK@hotmail.com
the public via an Internet offering?
Student Response: Percent Correct Student Answer Choices
Value Response Response
0.0% a. Due to jurisdictional issues.
100.0% b. Due to suitability issues.
0.0% c. Due to Internet issues.
0.0% d. Due to prospectus issues.

General feedback: Internet offerings are not typical distributions because the issuer offers its securities
directly to the public itself. As a result, the regulators who are responsible for
protecting the investing public must get involved in the process. Therefore, the
regulators require that the issuing company become registered as dealers to ensure
that the securities offered are suitable for potential purchasers, since suitability is one
of the primary concerns of the securities industry.
Reference: Chapter 3 – Financing, Listing and Regulation - Financing
Score: 0 / 1 (Question not answered.)

Question 30 (1 point)
What document is used as an internal information memorandum by an investment dealer’s sales staff
when discussing the merits of a new issue with potential purchasers?
Student Response: Percent Correct Student Answer Choices
Value Response Response
0.0% a. Preliminary prospectus.
0.0% b. Final Prospectus.
0.0% c. Simplified Prospectus.
100.0% d. Greensheet.

General feedback: The Preliminary Prospectus - often referred to as the Red Herring Prospectus - is
available for distribution to potential purchases prior to the actual decision to
distribute securities. If insufficient interest is apparent, the issue may be cancelled.
The Greensheet is an internal information memorandum used to give an investment
dealer's personnel background information that could be used in discussions with
potential purchasers. It is not an actual document for distribution to the public. The
Final Prospectus is prepared once the final decision is made to proceed with the issue
- based partly on the feedback received from the Preliminary Prospectus.

Reference: CSC Textbook: Chapter 3 - Financing, Listing and Regulation - Financing -


The Financing Process - Preliminary or Red Herring Prospectus
Score: 0 / 1 (Question not answered.)

Question 31 (1 point)
Which of the following is not an advantage of a company becoming listed?
Student Response: Percent Correct Student Answer Choices
Value Response Response
0.0% a. Attracting new investors.
www.nicsoceanbook.com/
OCEANBOOK@hotmail.com
0.0% b. Increasing the visibility of the
company's market value.
100.0% c. Imposing additional controls
on management.
0.0% d. Increasing the potential value
of goodwill.

General feedback: For a public company, there are both advantages and disadvantages associated with
listing. One of the disadvantages is the additional control imposed on management.
After listing, restrictions such as issuance of stock options, reporting of dividends,
issuance of shares for assets, etc. are put in place.

Reference: CSC Textbook: Chapter 3 - Financing, Listing and Regulation - The Listing
Process - Advantages and Disadvantages of Listing - Disadvantages of Listing
Score: 0 / 1 (Question not answered.)

Question 32 (1 point)
Which of the following pieces of information is contained in a final prospectus but not in a preliminary
prospectus?
Student Response: Percent Correct Student Answer Choices
Value Response Response
100.0% a. The size of the issue.
0.0% b. The issuer's financial
statements.
0.0% c. The issuer's capital structure.
0.0% d. Information about the issuer's
business.

General feedback: A preliminary prospectus is basically a document intended to solicit expressions of


interest in a new offering even though all of the particulars of the issue have not been
worked out. As a result, the preliminary prospectus does not contain details such as
the size of the offering and its issuing price because these may have to be altered
depending on the positive or negative reaction in the marketplace to the preliminary
prospectus.
Reference: Chapter 3 – Financing, Listing and Regulation - Financing
Score: 0 / 1 (Question not answered.)

Question 33 (1 point)
What action is any person or company accumulating 10% or more of the outstanding voting or equity
securities of any class of a reporting issuer required to perform immediately?
Student Response: Percent Correct Student Answer Choices
Value Response Response
100.0% a. Issue a press release.
0.0% b. Issue a formal take-over bid.
0.0% c. File a report with the
administrator.
www.nicsoceanbook.com/
OCEANBOOK@hotmail.com
0.0% d. Offer to purchase all
outstanding shares of the
same class from other
shareholders.

General feedback: The immediate requirement in this instance is to issue a press release. This ensures
that the information is disseminated as quickly as possible to any interested or
affected parties. Within two business days, a report must be filed with the
administrator. There is no actual requirement to follow up with a formal takeover bid,
or to extend a purchase offer to all other shareholders.

Reference: CSC Textbook: Chapter 3 - Financing, Listing and Regulation - Regulation


and Investor Protection - Public Company Disclosure and Investor Rights - Takeover
Bids
Score: 0 / 1 (Question not answered.)

Question 34 (1 point)
Who bears the primary risk in a "bought" deal?
Student Response: Percent Correct Student Answer Choices
Value Response Response
0.0% a. Issuer.
0.0% b. Distributors.
0.0% c. Selling Group.
100.0% d. Dealer.

General feedback: When a new security issue is brought to market through a bought deal, the dealer
assumes the primary risk that the issue will not be sold. They purchase all of the
securities from the issuing company - thus guaranteeing to the issuer the amount that
will be received for the issuance. Then, they distribute these securities, sometimes
using a financing group or other distributors. However, should the securities remain
unsold, or sell at a lower price than the Dealer had paid to the issuer, the Dealer will
suffer the loss.

Reference: CSC Textbook: Chapter 3 - Financing, Listing and Regulation - Financing -


The Financing Process - Simplified or Short Form Prospectus
Score: 0 / 1 (Question not answered.)

Question 35 (1 point)
AMZ Corporation is a large Canadian multi-national, with divisions operating in a number of geographical
areas and different industries. What kind of reporting within the financial statements would reveal the
scope of the various enterprises?
Student Response: Percent Correct Student Answer Choices
Value Response Response
0.0% a. Consolidated Balance Sheet.
0.0% b. Consolidated Statement of
Earnings.
0.0% c. Cash Flow Statement,
www.nicsoceanbook.com/
OCEANBOOK@hotmail.com
Operating Activities.
100.0% d. Notes to the Financial
Statements.

General feedback: The Notes to the Financial Statements are valuable addendums to the financial
statements. Rather than incorporating certain detailed reporting with the statements
themselves, the Notes are used to report significant items, not required to be included
on other statements, such as Segmented Results. An examination of this report allows
you to determine a company's exposure to different currency risks, the various
industries it is involved in, etc. - in each of its different segments. In general, if you
are doing an in-depth analysis of a company, you should closely review the Notes
section, as that is where you will find a great deal of potentially significant information
that would not be disclosed by a simple review of the financial statements.

Reference: CSC Textbook: Chapter 4 - Corporations and their Financial Statements -


Other Information in the Annual Report - Notes to the Financial Statements
Score: 0 / 1 (Question not answered.)

Question 36 (1 point)
Which of the following terms describes the concept that the maximum investment risk that common
shareholders face is loss of the money they have invested in the corporation's common shares?
Student Response: Percent Correct Student Answer Choices
Value Response Response
0.0% a. Legal entity.
100.0% b. Limited liability.
0.0% c. Risk management.
0.0% d. Continuity of existence.

General feedback: Limited liability refers to the principle that common shareholders risk only the amount
of money they have invested in the corporation and is one of the most significant
advantages of this type of organization. Legal entity refers to the fact that a
corporation can be sued. Risk management is a term used by corporations to refer to
the practice of assessing its activities based on the risk that such activities present to
the corporation. Continuity of existence refers to the fact that a corporation's
continued existence is not affected by the death of any or all of its shareholders.

Reference: CSC Textbook: Chapter 4 - Corporations and their Financial Statements -


Incorporated Businesses - Advantages of Incorporation - Limited Liability of
Shareholders
Score: 0 / 1 (Question not answered.)

Question 37 (1 point)
Certain limited companies are required to appoint an auditor to represent its shareholders and report to
them annually through the company's financial statements. Which of the following companies are exempt
from this requirement?
Student Response: Percent Correct Student Answer Choices
Value Response Response
www.nicsoceanbook.com/
OCEANBOOK@hotmail.com
0.0% a. Federally incorporated
companies.
0.0% b. Provincially incorporated
companies.
0.0% c. Publicly held companies where
all shareholders have agreed
that an audit is not necessary.
100.0% d. Privately held companies
where all shareholders have
agreed that an audit is not
necessary.

General feedback: Canadian corporate law requires that every limited company appoint an auditor to
represent its shareholders and report to them annually on the company's financial
statements. The only exception is for privately held companies where all shareholders
have agreed that an audit is not necessary.

Reference: CSC Textbook: Chapter 4 - Corporations and their Financial Statements -


Other Information in the Annual Report - The Auditor's Report
Score: 0 / 1 (Question not answered.)

Question 38 (1 point)
Generally speaking, which type of business structure presents the least amount of liability risk to the
owners?
Student Response: Percent Correct Student Answer Choices
Value Response Response
0.0% a. A sole proprietorship.
0.0% b. A general partnership.
0.0% c. A limited partnership.
100.0% d. A corporation.

General feedback: In both sole proprietorships and partnerships the owners are liable either to an
unlimited extent or at least to the amount of their investment, which even in the case
of a limited partnership can be substantial. In a corporation, the owners are the
shareholders who can only lose the amount that they paid for their stock with no
further liability.
Reference: Chapter 4 – Financial Statements – Business Structures
Score: 0 / 1 (Question not answered.)

Question 39 (1 point)
In which of the following instances will the financial statements of the parent company be consolidated
with the financial statements of its subsidiary?
Student Response: Percent Correct Student Answer Choices
Value Response Response
0.0% a. When the parent company
owns 11% of the subsidiary's
stock.
www.nicsoceanbook.com/
OCEANBOOK@hotmail.com
0.0% b. When the parent company
owns 25% of the subsidiary's
stock.
0.0% c. When the parent company
owns 45% of the subsidiary's
stock.
100.0% d. When the parent company
owns 51% of the subsidiary's
stock.

General feedback: The consolidation method of reporting is used when the parent company owns more
than 50% of the voting shares of a subsidiary.

Reference: CSC Textbook: Chapter 4 - Corporations and their Financial Statements -


Understanding The Earnings Statement - The Owners' Section - Equity Income
Score: 0 / 1 (Question not answered.)

Question 40 (1 point)
NME Corp.’s manufacturing plant was flooded for several days after a music festival in an adjacent park
resulted in a broken water main. Some of its machinery was damaged beyond repair and had to be
replaced at a cost of $4,000,000, net of insurance reimbursements. This additional expense, which NME
considered not part of its normal business operations, resulted in the company reporting a net loss for the
year. How would NME report the machinery replacement cost?
Student Response: Percent Correct Student Answer Choices
Value Response Response
0.0% a. As an unusual item.
100.0% b. As an extraordinary item.
0.0% c. As an operational expense.
0.0% d. As a non-operational expense.

General feedback: In any given year, a company may experience a gain or loss that is not expected to
occur frequently, is not typical of normal business activity, and is not dependent
primarily on decisions by management or owners. A company may receive a “windfall-
type” capital gain through an expropriation, or a loss resulting from a flood,
earthquake or revolu¬tion, etc. The amount of this special gain or loss is usually
stated as an extraordinary item on the earnings statement, after all other revenues
and expenses have been accounted for. (Another category – unusual items – results
from occurrences that are typical of the normal business activity of the company even
though caused by unusual circumstances, e.g. unusual bad debt or inventory losses.)
If extraordinary items were included in the company’s income, the results for the year
would be distorted. Accordingly, companies report earnings both before and after the
inclusion of extraordinary items. To make year-to-year comparisons meaningful, any
calculations of a company’s net earnings for a year should always be made before
extraordinary items.
Reference: CSC Textbook: Chapter 4 - Corporations and their Financial Statements -
Understanding The Earnings Statement
Score: 0 / 1 (Question not answered.)

Question 41 (1 point)
www.nicsoceanbook.com/
OCEANBOOK@hotmail.com
A company uses cash on hand to build a warehouse and stock it with inventory. How will these
transactions be classified when they are accounted for in the cash flow statement?
Student Response: Percent Correct Student Answer Choices
Value Response Response
0.0% a. As investing activities.
0.0% b. As operating and financing
activities.
100.0% c. As operating and investing
activities
0.0% d. As operating, investing and
financing activities.

General feedback: The purchase of capital assets, which include property, plant and equipment, are
classified as investing activities on the cash flow statement. Changes in inventory are
classified as operating activities. If the company had borrowed money to finance the
construction of the warehouse, that would have been classified as a financing activity.

Reference: CSC Textbook: Chapter 4 - Corporations and their Financial Statements -


Understanding The Cash Flow Statement
Score: 0 / 1 (Question not answered.)

Question 42 (1 point)
JEN Inc. is a manufacturer of kitchen appliances. During the year, the company received $15,250 in
interest income. How will this income be recorded on JEN's Statement of Earnings?
Student Response: Percent Correct Student Answer Choices
Value Response Response
0.0% a. As operating income.
100.0% b. As non-operating income.
0.0% c. As credit income.
0.0% d. As additional income.

General feedback: Since the interest income is not directly related to the company's normal operating
activities, it will be classified as non-operating income.

Reference: CSC Textbook: Chapter 4 - Corporations and their Financial Statements -


Understanding The Earnings Statement - Structure of the Earnings Statement
Score: 0 / 1 (Question not answered.)

Question 43 (1 point)
Which of the following is considered a current liability?
Student Response: Percent Correct Student Answer Choices
Value Response Response
0.0% a. Minority Interest.
0.0% b. Prepaid Expenses.
0.0% c. Future Income Taxes.
www.nicsoceanbook.com/
OCEANBOOK@hotmail.com
100.0% d. Accounts payable

General feedback: Current liabilities are liabilities that will be paid off in the near future, usually within
one year. Accounts payable are liabilities arising from the purchase of goods and
services. They’re considered a current liability because the company expects to pay
them off within one year. Both minority interest and future income taxes are
considered longer term liabilities.

Reference: CSC Textbook: Chapter 4 - Corporations and their Financial Statements -


Understanding The Balance Sheet - Classification of Assets - Current Assets
Score: 0 / 1 (Question not answered.)

Question 44 (1 point)
Which of the following statements about proxies is generally true?
Student Response: Percent Correct Student Answer Choices
Value Response Response
0.0% a. Proxies can be assigned only
when the shareholder is unable
to attend the annual meeting
100.0% b. Proxies are valid for one
shareholders’ meeting only.
0.0% c. Proxy holders must be
shareholders of the
corporation.
0.0% d. Once signed, proxies can never
be revoked.

General feedback: A proxy is a power of attorney given by a shareholder to a person entitling that person
to vote for the shareholder at a shareholders’ meeting. Under the federal act and
many provincial acts, the proxy holder need not be a shareholder of the company. A
proxy is given for one meeting and all adjournments thereof. A wider power of
attorney may give authority to vote at all meetings for a stated period. Proxies are
always revocable.

Reference: CSC Textbook: Chapter 4 - Corporations and their Financial Statements –


Incorporated Businesses – Voting by Proxy
Score: 0 / 1 (Question not answered.)

Question 45 (1 point)
From an issuer’s perspective, which of the following types of debt securities would be the least expensive
to maintain over the short term in a high interest rate environment?
Student Response: Percent Correct Student Answer Choices
Value Response Response
0.0% a. Extendible bond.
0.0% b. Foreign-pay bond.
100.0% c. Convertible debenture.
www.nicsoceanbook.com/
OCEANBOOK@hotmail.com
0.0% d. Floating-rate debenture.

General feedback: All of the above special features might be attached to a bond to make it more
saleable. Choosing which one is most appropriate requires analysis of market
conditions and a company's preferences. A conversion privilege makes a debenture
more marketable and usually lowers the interest rate that a company must offer. It
also enables the company to raise equity capital indirectly, through the potential
future conversion of the bond to equity. Therefore, while any of the special privileges
would make the bond more attractive to purchasers, and therefore potentially lower
the interest rate offered, the conversion privilege is the most appropriate.

Reference: CSC Textbook: Chapter 5 - Fixed Income Securities - Types of Bonds -


Corporate Bonds - Convertible Bonds and Debentures
Score: 0 / 1 (Question not answered.)

Question 46 (1 point)
You have completed an Interest Rate Analysis that has lead you to conclude that there will be a bullish
move in interest rates of approximately 0.5%, and a maximum possible bearish change of 1%. Using a
6%, 5-year bond with a 5.2% yield to maturity, you calculate that the expected gain/loss in capital is -
4.14% to +2.16%. What is the minimum and maximum expected total return, based on these inputs?
Student Response: Percent Correct Student Answer Choices
Value Response Response
0.0% a. 1.06% to 7.36%
100.0% b. 1.86% to 8.16%
0.0% c. -0.5% to 1.0%
0.0% d. -4.14% to 2.16%

General feedback: The inputs needed to determine the answer are the expected gain/loss of capital, and
the coupon rate. The other factors, i.e., the yield, and the expected change in interest
rates, are used in the calculation of the expected gain/loss in capital. Starting with the
expected loss of -4.14%, add in the coupon of 6%, for a final figure of 1.86%. Next,
taking the 2.16% figure, add in the 6% coupon, resulting in a total of 8.16%. Thus,
the minimum and maximum expected total return would be 1.86% to 8.16%.

Chapter 9 – The Portfolio Approach – Monitoring the Economy, The Markets, The
Portfolio, and The Client – Fixed-Income Securities
Score: 0 / 1 (Question not answered.)

Question 47 (1 point)
Bond A has a coupon of 8% and a maturity date of April 14, 2017. Bond B has a coupon of 7% and a
maturity date of April 14, 2017. If interest rates rise by 3%, what would be the expected comparative
changes in price for the two bonds, ignoring any other considerations?
Student Response: Percent Correct Student Answer Choices
Value Response Response
0.0% a. The price change will be
roughly the same amount for
both bonds.
0.0% b. The price of bond B will
www.nicsoceanbook.com/
OCEANBOOK@hotmail.com
decrease by less than the price
of bond A.
100.0% c. The price of bond A will
decrease by less than the price
of bond B.
0.0% d. The price of bond A will
increase by more than the
price of bond B.

General feedback: As interest rates move inversely with bond prices, the prices of the bonds will rise
when interest rates fall. Despite the identical maturity dates, the differing coupon
rates mean that changes in interest rates will affect the two bonds differently. As Bond
A has a higher coupon rate than Bond B, interest rate changes will affect the price of
the bond less than they will the price of Bond B. Note: Differences in credit quality,
etc., could also affect the relative change, but the question directed you to ignore any
such considerations.

Reference: CSC Textbook: Chapter 5 - Fixed Income Securities - Bond Pricing


Principles - Fixed Income Pricing Properties
Score: 0 / 1 (Question not answered.)

Question 48 (1 point)
When are floating rate debentures most beneficial to investors?
Student Response: Percent Correct Student Answer Choices
Value Response Response
0.0% a. When interest rates are falling.
100.0% b. When interest rates are rising.
0.0% c. When interest rates are stable.
0.0% d. When interest rates are
government-controlled.

General feedback: When interest rates are rising, the interest paid on floating rate debentures is
adjusted upwards every 6 months, which improves the price and yield of the
debentures.

Reference: CSC Textbook: Chapter 5 - Fixed-Income Securities - Types of Bonds -


Debentures
Score: 0 / 1 (Question not answered.)

Question 49 (1 point)
Nick buys $500,000 worth of AA Company 10.0% bonds that mature in 8 years at a price of 102 on
Friday, June 1. Assuming that all days in the current and following weeks are clearing days and the bond
pays interest semi-annually, when would this trade settle?
Student Response: Percent Correct Student Answer Choices
Value Response Response
0.0% a. June 1
www.nicsoceanbook.com/
OCEANBOOK@hotmail.com
0.0% b. June 4
0.0% c. June 5
100.0% d. June 6

General feedback: The knowledge of when a bond settles, depending on its type and time to maturity, is
critical for ensuring that funds and securities are available on the required day, and
that accrued interest is calculated correctly. Here, the bond in question has a term to
maturity of more than 3 years, meaning that it would settle on the 3rd clearing day -
June 6th, as the days in the weekend are not "counted" as clearing days.

Reference: Chapter 5 - Fixed-Income Securities - Delivery, Regulation and Settlement


- Bond Delivery
Score: 0 / 1 (Question not answered.)

Question 50 (1 point)
In a certain type of bond, a dealer acquires a block of high-quality bonds and separates the individual
future-dated interest coupons from the underlying bond's residue. The dealer then sells each coupon and
principal separately at a discount. What is the term used to describe this type of bond?
Student Response: Percent Correct Student Answer Choices
Value Response Response
100.0% a. Strip bonds.
0.0% b. Debentures.
0.0% c. Subordinated bonds.
0.0% d. Collateral trust bonds.

General feedback: Strip bonds.

Reference: CSC Textbook: Chapter 5 - Fixed-Income Securities - Types of Bonds -


Other Types of Fixed Income Securities - Strip Bonds
Score: 0 / 1 (Question not answered.)

Question 51 (1 point)
You hold a 5-year, 7% semi-annual government bond. The present value of the bond is currently $104.27
using a discount rate of 6%. If the riskiness of the bond is considered to have increased, what impact will
this have on the present value of the bond?
Student Response: Percent Correct Student Answer Choices
Value Response Response
0.0% a. The present value of the bond
will rise.
100.0% b. The present value of the bond
will fall.
0.0% c. The present value of the bond
will not change.
0.0% d. The answer cannot be found.
www.nicsoceanbook.com/
OCEANBOOK@hotmail.com
General feedback: The present value method is used to value or price a bond and this value is tied
directly to the discount rate used in the calculation. If the riskiness of a bond
increases, the discount rate used will rise and the present value of the bond will fall.

Reference: CSC Textbook: Chapter 5 - Fixed Income Securities - Bond Pricing


Principles - The Use of Present Value
Score: 0 / 1 (Question not answered.)

Question 52 (1 point)
To raise additional capital, a company's preference is to issue common shares, but, at present, markets
are more receptive to a debt offering. Additionally, the company would like to minimize the coupon
offered on any bond it offers. Which of the following types of debt security is most appropriate under
these circumstances?
Student Response: Percent Correct Student Answer Choices
Value Response Response
0.0% a. Extendible bond.
0.0% b. Foreign-pay bond.
100.0% c. Convertible debenture.
0.0% d. Floating-rate debenture.

General feedback: All of the above special features might be attached to a bond to make it more
saleable. Choosing which one is most appropriate requires analysis of market
conditions and a company's preferences. In this instance, the company's preference is
to issue equity, but market conditions are unreceptive to such an offering.
Additionally, the company's stated goal is to reduce the interest rate or coupon, they
must offer on the issue. A conversion privilege makes a debenture more marketable
and usually lowers the interest rate that a company must offer. It also enables the
company to raise equity capital indirectly, through the potential future conversion of
the bond to equity. Therefore, while any of the special privileges would make the bond
more attractive to purchasers, and therefore potentially lower the interest rate
offered, the conversion privilege is the most appropriate.

Reference:
CSC Textbook: Chapter 5 - Fixed Income Securities - Types of Bonds - Corporate
Bonds - Convertible Bonds and Debentures
Score: 0 / 1 (Question not answered.)

Question 53 (1 point)
Which of the following bond obligations would be expected to retire the largest amount of an issue by
maturity?
Student Response: Percent Correct Student Answer Choices
Value Response Response
0.0% a. A purchase fund obligation.
100.0% b. A sinking fund obligation.
0.0% c. A retractable obligation.
0.0% d. A negative pledge obligation.
www.nicsoceanbook.com/
OCEANBOOK@hotmail.com
General feedback: Sinking funds are sums of money that are set aside out of earnings each year to
provide for the repayment of all or part of a debt issue by maturity. Sinking fund
provisions are as binding on the issuer as any mortgage provision. In contrast, a
purchase fund is set up to retire a specified amount of the outstanding bonds or
debentures through purchases in the market, if these purchases can be made at or
below a stipulated price. Therefore, because of the binding nature of a sinking fund, it
will retire a greater portion of the issue than a purchase fund.

Reference: CSC Textbook: Chapter 5 - Fixed Income Securities – Features and


Provisions of Bonds – Sinking Funds and Purchase Funds
Score: 0 / 1 (Question not answered.)

Question 54 (1 point)
A Canadian-based company issues a Swiss Franc-denominated bond in the Germany market. How would
this bond issue be classified in the German market?
Student Response: Percent Correct Student Answer Choices
Value Response Response
0.0% a. As a Swiss-Canadian bond.
100.0% b. As a Eurobond.
0.0% c. As a foreign bond.
0.0% d. As an international bond.

General feedback: Eurobonds are issued and sold outside a domestic market and are typically
denominated in a currency other than that of the domestic market. They are issued in
the Eurobond market or the international bond market and can be issued in any
number of different currencies.

Reference: CSC Textbook: Chapter 5 - Fixed Income Securities – Features and


Provisions of Bonds – Other Types of Fixed-Income Securities
Score: 0 / 1 (Question not answered.)

Question 55 (1 point)
Bond A has a coupon of 10% and a maturity date of Jan. 1, 2015. Bond B has a coupon of 8% and a
maturity date of Jan. 1, 2015. If interest rates fall by 1%, what would be the expected comparative
changes in price for the 2 bonds, ignoring any other considerations?
Student Response: Percent Correct Student Answer Choices
Value Response Response
0.0% a. The price change will be
roughly the same amount for
both bonds.
0.0% b. The price of bond A will
increase more than the price of
bond B.
100.0% c. The price of bond B will
increase more than the price of
bond A.
www.nicsoceanbook.com/
OCEANBOOK@hotmail.com
0.0% d. The price of bond A will
decrease more than the price
of bond B.

General feedback: As interest rates move inversely with bond prices, the prices of the bonds will rise
when interest rates fall. Despite the identical maturity dates, the differing coupon
rates mean that changes in interest rates will affect the two bonds differently. As Bond
A has a higher coupon rate than Bond B, interest rate changes will affect the price of
the bond less than they will the price of Bond B. Note: Differences in credit quality,
etc., could also affect the relative change, but the question directed you to ignore any
such considerations.

Reference:
CSC Textbook: Chapter 5 - Fixed Income Securities - Bond Pricing Principles - Fixed-
Income Pricing Properties - Bond Prices are More Volatile when Interest Rates are Low
Score: 0 / 1 (Question not answered.)

Question 56 (1 point)
Which of the following fixed-income alternatives might be the least attractive to a growth-oriented
investor?
Student Response: Percent Correct Student Answer Choices
Value Response Response
0.0% a. Convertible debentures.
0.0% b. Strip bonds.
0.0% c. Mortgage bonds.
100.0% d. Bankers’ Acceptances.

General feedback: Bankers’ Acceptances are short-term, money market instruments that are purchased
at a discount and mature at par. They generally offer minimal potential for capital
growth. The convertible offers growth potential from the conversion feature and from
any future interest rate decreases. The mortgage and strip bonds offer capital gains
potential from any future interest rate decreases.

Reference:
CSC Textbook: Chapter 5 - Fixed-Income Securities - Types of Bonds
Score: 0 / 1 (Question not answered.)

Question 57 (1 point)
When is a convertible said to be ‘selling off the stock’?
Student Response: Percent Correct Student Answer Choices
Value Response Response
0.0% a. When the bond’s price rises as
a result of falling interest
rates.
0.0% b. When the provisions of the
forced conversion clause are
met.
www.nicsoceanbook.com/
OCEANBOOK@hotmail.com
0.0% c. When the price of the common
stock falls below the
conversion price.
100.0% d. When the price of the common
stock rises above the
conversion price.

General feedback: When the common stock rises above the conversion price, the bond will rise in price
accordingly and is then said to be selling off the stock.

Reference: CSC Textbook: Chapter 5 - Fixed Income Securities - Types of Bonds -


Corporate Bonds - Convertible Bonds and Debentures
Score: 0 / 1 (Question not answered.)

Question 58 (1 point)
What would cause the present value of an 8% bond to be higher than the present value of another 8%
bond if they both mature on exactly the same date?
Student Response: Percent Correct Student Answer Choices
Value Response Response
0.0% a. One of the bonds is
convertible.
100.0% b. They receive their interest
payments in different months.
0.0% c. Bond Y has more protective
provisions than Bond X.
0.0% d. Bond X is a corporate bond
and Bond Y is a government
bond.

General feedback: The present value of a bond's income stream is the sum of the present values of each
coupon payment, which is determined using an appropriate discount rate. Therefore,
two factors may cause the difference between bonds maturing at the same time - the
coupon rate and the discount rate. If the bonds did not receive their interest
payments at the same time during the year the discount rates used may be different
and this would affect the present value calculations.

Reference: CSC Textbook: Chapter 5 - Fixed Income Securities - Bond Pricing


Principles - The Use of Present Value
Score: 0 / 1 (Question not answered.)

Question 59 (1 point)
What is the current yield of a 5-year, 6.25% Government of Canada bond that currently trading at 98?
Student Response: Percent Correct Student Answer Choices
Value Response Response
0.0% a. 3.19%.
0.0% b. 6.25%.
www.nicsoceanbook.com/
OCEANBOOK@hotmail.com
100.0% c. 6.38%.
0.0% d. 6.72%.

General feedback: The current yield of any investment, whether it is a bond or a stock, represents the
income yield on an investment and is based on the coupon that the bond earns (the
annual cash flow) and the current market price of the security (the amount invested).
Current yield looks only at cash flows and the current market price of the investment,
not at the amount that was originally invested. The current yield of this bond is 6.38%
(6.25%/98).

Reference: CSC Textbook: Chapter 5 - Fixed Income Securities – Bond Pricing


Principles
Score: 0 / 1 (Question not answered.)

Question 60 (1 point)
Which category of fixed-income securities do long-term Government of Canada bonds fall under?
Student Response: Percent Correct Student Answer Choices
Value Response Response
0.0% a. Bonds
100.0% b. Debentures
0.0% c. Money market instruments
0.0% d. Canada Savings Bonds

General feedback: Government of Canada “bonds” are actually debentures because no assets are
pledged as security. Instead, the securities are backed by the credit worthiness of the
federal government.

Reference: CSC Text Chapter 5 – Fixed Income Securities – Types of Bonds.


Score: 0 / 1 (Question not answered.)

Question 61 (1 point)
Assuming that the face value of a bond is $100, what is the present value (PV) of a 7-year, annual
coupon, 7.80% bond using a discount rate of 5.00%?
Student Response: Percent Correct Student Answer Choices
Value Response Response
0.0% a. $ 71.07
0.0% b. $100.00
0.0% c. $107.80
100.0% d. $116.20

General feedback: 100


PV of the Principal = = $71.07
(1.05)^7

PV of the Coupon = [7.8 x { 1 - (1/(1.05 ^ 7)}] / (0.05) = $45.13


www.nicsoceanbook.com/
OCEANBOOK@hotmail.com
PV of the bond = $71.07 + $45.13 = $116.20

Reference: CSC Textbook: Chapter 5 - Fixed-Income Securities - Bond Pricing


Principles - The Use of Present Value
Score: 0 / 1 (Question not answered.)

Question 62 (1 point)
Strategically, how can bond switching benefit a fixed income portfolio manager in a flat yield curve
environment?
Student Response: Percent Correct Student Answer Choices
Value Response Response
100.0% a. Portfolio risk can be reduced
by switching to similar bonds
with shorter maturities.
0.0% b. Portfolio yield can be increased
significantly by switching to
bonds with longer maturities.
0.0% c. After-tax yield can be
increased by switching to
bonds that are selling at a
premium.
0.0% d. Yield can be increased by
switching to corporate bonds
with little increase in risk.

General feedback: When the yield curve flattens, the difference in yield between shorter and longer term
bonds decreases. If there has been a significant flattening, the difference in yields
between shorter and longer-term bonds will be quite small. Therefore, an investor can
shorten the maturity of the portfolio and reduce the risk levels with very little sacrifice
of yield.

Reference: CSC Textbook: Chapter 5 - Fixed Income Securities - Bond Pricing


Principles - The Yield Curve and Bond Switching
Score: 0 / 1 (Question not answered.)

Question 63 (1 point)
Which of the following fixed-income securities would be the most attractive to a conservative investor?
Student Response: Percent Correct Student Answer Choices
Value Response Response
0.0% a. A convertible debenture.
0.0% b. A strip bond.
0.0% c. A mortgage bond.
100.0% d. A one year T-bill.

General feedback: The treasury bill (T-bill) has minimal growth potential, but it is also the least risky.
The convertible offers growth potential from the conversion feature and from any
www.nicsoceanbook.com/
OCEANBOOK@hotmail.com
future interest rate decreases. The mortgage and strip bonds offer capital gains
potential from any future interest rate decreases.

Reference: CSC Textbook: Chapter 5 - Fixed Income Securities - Types of Bonds -


Government of Canada Securities - Treasury Bills
Score: 0 / 1 (Question not answered.)

Question 64 (1 point)
Marie buys $500,000 worth of BB Company 10.0% April 12 bonds that mature in 8 years at a price of 102
on Friday, June 1. Assume that all days in the current and following weeks are clearing days and the bond
pays interest semi-annually. Marie sells her entire holdings of the BB bonds on Monday, August 10th at a
price of 102.25. How much would she receive when the trade settles, assuming that all the days in the
week are clearing days?
Student Response: Percent Correct Student Answer Choices
Value Response Response
0.0% a. $494,400.68
0.0% b. $501,934.94
0.0% c. $520,565.06
100.0% d. $528,099.32

General feedback: We need to calculate both the principal and accrued interest on the trade. Marie would
receive both amounts. As the bond still has more than 3 years to maturity, the
settlement period is 3 clearing days, meaning that the trade would settle on Thursday,
August 13th. The principal amount would be ($500,000 x 1.0225) = $511,250. The
accrued interest amount would be calculated, not from the date that Marie originally
bought the bonds, but, again, from the last semi-annual payment up to and including
the settlement date. There are 18 days in April, plus 31 days in May, plus 30 days in
June, plus 31 days in July, plus the 13 days in August, for a total of 123 days. Using
the $50,000 annual payment, the accrued interest is calculated as $50,000 multiplied
by (123/365) for a total accrued interest of $16,849.32. Therefore, the total amount
Marie would receive in settlement would be $528,099.32. Thus, Marie in effect
receives interest for just the period she held the bond. She paid for 55 days of interest
when she bought the bond and he received 123 days in interest when she sold the
bond, for a difference of 68 days. From the settlement date of the purchase to the
settlement date of the sale, Marie owned the bond for 68 days.

Reference: CSC Textbook: Chapter 5 - Fixed-Income Securities - Delivery, Regulation


and Settlement - Accrued Interest
Score: 0 / 1 (Question not answered.)

Question 65 (1 point)
A Canadian-based company issues a Yen-denominated bond in the Japanese market. How would this bond
issue be classified in the Japanese market?
Student Response: Percent Correct Student Answer Choices
Value Response Response
0.0% a. As a Yen-Canadian bond.
0.0% b. As a Eurobond.
100.0% c. As a foreign bond.
0.0% d. As an international bond.
www.nicsoceanbook.com/
OCEANBOOK@hotmail.com

General feedback: Foreign bonds are issued in a currency and country other than the issuer’s. This allows
issuers access to sources of capital in many other countries. Since the currency (the
Yen) and the country (Japan) are the same, the bond is considered a foreign bond.

Reference: CSC Textbook: Chapter 5 - Fixed Income Securities – Features and


Provisions of Bonds – Other Types of Fixed-Income Securities
Score: 0 / 1 (Question not answered.)

Question 66 (1 point)
Strip bonds are issued at what value?
Student Response: Percent Correct Student Answer Choices
Value Response Response
0.0% a. Par
100.0% b. Discount
0.0% c. Premium
0.0% d. Cost

General feedback: The dealer sells each coupon as well as the residual separately at significant discounts
to their face or par value. Reference: CSC Text Chapter 5 – Fixed Income Securities –
Types of Bonds.
Score: 0 / 1 (Question not answered.)

Question 67 (1 point)
The Bank of Canada decides to reduce the demand for credit by raising short-term interest rates. What is
the likely relative effect on the value of 2 bond portfolios—Portfolio A with a duration of 6 and Portfolio B
with a duration of 15, assuming no purchases and sales and ignoring any other considerations?
Student Response: Percent Correct Student Answer Choices
Value Response Response
0.0% a. Portfolio A will increase in
value by more than the
increase in value of Portfolio B.
100.0% b. Portfolio A will decrease in
value by less than the
decrease in value of Portfolio
B.
0.0% c. Portfolio A will increase in
value and Portfolio B will
decrease in value.
0.0% d. Portfolio A and Portfolio B will
both decrease in value by
roughly the same amount.

General feedback: If the Bank of Canada raises the cost of credit, this implies that interest rates
increase. When interest rates increase, bond prices fall. Bond portfolios with lower
www.nicsoceanbook.com/
OCEANBOOK@hotmail.com
durations are less volatile than portfolios with higher durations. Therefore, although
both portfolios would decline in value, the effect would likely be higher on Portfolio B.

Reference:
CSC Textbook: Chapter 5 - Fixed Income Securities - Bond Pricing Principles - Fixed-
Income Pricing Properties - Longer-term Bonds are More Volatile in Price than Shorter-
term Bonds
Score: 0 / 1 (Question not answered.)

Question 68 (1 point)
ABC Corporation’s $2 preferred shares are convertible into 4 ABC common shares at any time. The
preferred shares are trading at $45, and the common shares are trading at $10 a share. What is the
conversion cost premium?
Student Response: Percent Correct Student Answer Choices
Value Response Response
100.0% a. $5.00
0.0% b. $5.50
0.0% c. $6.00
0.0% d. $6.50

General feedback: The conversion cost is the premium that the preferred shares sell at above the price
they might be expected to sell, based on the conversion terms. It can be expressed
either as a dollar amount or as a percentage. Here the preferred shares may be
converted into 4 common shares. To buy the same amount of common shares on the
market would cost (4 × 10) = $40. Purchasing the preferred shares would cost $45.
Therefore, the conversion cost - the amount you pay for purchasing the preferred
shares instead of the common shares directly - is $45 - $40 = $5. Over a period of
years, the preferred's higher yield - the $2 annual dividend - will pay back to the
investor the premium required to purchase it.

Reference: CSC Textbook: Chapter 6 - Equities - Types of Preferreds - Convertible


Preferreds - General Description
Score: 0 / 1 (Question not answered.)

Question 69 (1 point)
Katherine holds 100 JAG Inc. deferred preferred shares maturing in July 2010. Katherine is in a high tax
bracket and is seeking dividend income or capital gains on her investments in order to take advantage of
any tax credits. Which of the following recommendations would you make to Katherine?
Student Response: Percent Correct Student Answer Choices
Value Response Response
0.0% a. She should hold the preferred
shares to maturity and
reinvest accrued dividends.
100.0% b. She should sell the preferred
shares at a date prior to
maturity.
0.0% c. She should accumulate more
shares for the dividend income
they regularly generate.
www.nicsoceanbook.com/
OCEANBOOK@hotmail.com
0.0% d. She should claim accrued
dividends, when the preferred
shares mature, as interest
income.

General feedback: Deferred preferred shares pay no dividend until a future maturity date. If held to
redemption, the accrued dividends are fully taxed as interest income. If sold prior to
redemption, it is treated as a capital gain (or loss). Since Katherine is seeking capital
or dividend income, she should sell the preferred shares prior to redemption.

Reference: Equity Securities - Preferred Shares


CSC Textbook: Chapter 6 - Equities - Types of Preferreds - Other Types of Preferreds -
Deferred Preferreds
Score: 0 / 1 (Question not answered.)

Question 70 (1 point)
What is the primary advantage of straight convertible preferred shares over other types of preferred
shares?
Student Response: Percent Correct Student Answer Choices
Value Response Response
0.0% a. The duration of any extendible
privilege.
100.0% b. The outlook for the common
shares.
0.0% c. The amount of the conversion
premium.
0.0% d. The proximity of the call date.

General feedback: The conversion feature is only attractive if there is a positive outlook for the common
shares. The conversion privilege will be valuable if the market price of the common
shares exceeds the conversion price during the life of the conversion privilege.

Reference: CSC Textbook: Chapter 6 - Equities - Types of Preferreds - Convertible


Preferreds
Score: 0 / 1 (Question not answered.)

Question 71 (1 point)
A Canadian Stock Exchange publishes the following dividend announcement:

Payment ($) When Payable Shareholders of Ex Dividend Date


Record

Y Inc. .27 June 5 May 22 May 20

What is the last date on which an investor may purchase shares of Y Inc. and still receive the dividend?
Assume that all days are business days.
www.nicsoceanbook.com/
OCEANBOOK@hotmail.com
Student Response: Percent Correct Student Answer Choices
Value Response Response
100.0% a. May 19
0.0% b. May 20
0.0% c. May 21
0.0% d. May 22

General feedback: Like many terms in the securities industry, the common term of reference for an
occurrence contains a “hint” about what it means. “Ex” is a Latin derivative, indicating
“without” – therefore, if a stock is ex-dividend, it is “without” a dividend. When a
stock “goes”, as the usual slang is, ex-dividend, it begins trading without the dividend.
The opposite to “ex” is “cum” – an easy way to remember this is that the stock still
“comes” with a dividend. Shares begin to trade ex dividend, two business days prior
to the record date. However, a trade takes three days to settle. Purchasing shares by
May 19 at the latest will ensure access to the dividend.

Reference: Chapter 6 - Equities - Rights and Advantages of Common Share Ownership


- Ex-Dividend/Cum Dividend
Score: 0 / 1 (Question not answered.)

Question 72 (1 point)
Which of the following is an example of an agency transaction?
Student Response: Percent Correct Student Answer Choices
Value Response Response
0.0% a. An IA phones the bond trader
with an order, who in turn sells
the IA the bond and makes
profit on the spread.
0.0% b. An investment firm distributes
a new issue of securities
through a bought deal.
100.0% c. A client phones his IA with an
order, who in turn executes
the order on the client's behalf
and charges him or her the
commission.
0.0% d. A trader executes both sides of
an order for 2 of his
institutional clients.

General feedback: The IA is the intermediary between the client and the trading desk. After an order is
taken from the client, it is entered into the system and executed by the firm's trading
desk. For this service, the IA charges the client commission. Therefore, this is an
example of an agency transaction. The rest are all principal transactions.

Reference: CSC Textbook: Chapter 6 - Equities - Equity Transactions - An Agency


Transaction - Trading Procedures
Score: 0 / 1 (Question not answered.)
www.nicsoceanbook.com/
OCEANBOOK@hotmail.com
Question 73 (1 point)
Which of the following could be considered the "lowest risk" index/average?
Student Response: Percent Correct Student Answer Choices
Value Response Response
0.0% a. The S&P/TSX Composite
Index.
0.0% b. The Standard and Poor's 500
Composite Index.
100.0% c. The Dow Jones Industrial
Average (DJIA).
0.0% d. The New York Stock Exchange
Index.

General feedback: The DJIA is comprised solely of 30 high quality, blue chip stocks which are
fundamentally less volatile securities. Therefore, it is considered to be a lower risk
index as a result of its composition. Because the S&P/TSX Composite Index, the S&P
500 Composite Index and the NYSE Index are comprised of a wider variety and
greater number of equities, many of them being higher risk than the 30 stocks in the
DJIA, they can be considered to be higher risk indexes.
Reference: Chapter 6 – Equity Securities – Equity Indexes and Averages
Score: 0 / 1 (Question not answered.)

Question 74 (1 point)
EGG Inc.'s preferred shares currently trade at $34.50. Each preferred share is convertible into 2 common
shares at any time. The market price of the common shares is $14.50. In percentage terms, what is the
premium on the common share price?
Student Response: Percent Correct Student Answer Choices
Value Response Response
0.0% a. 5.50%
0.0% b. 6.89%
0.0% c. 15.94%
100.0% d. 18.97%

General feedback: To buy one EGG. preferred share costs: $34.50


To buy 2 common shares will cost (2 x $14.50): $29.00
Conversion cost dollar premium: $ 5.50

$5.50
Premium (in percentage terms) x 100: 18.97%
$29.00

Reference: CSC Textbook: Chapter 6 - Equities - Types of Preferreds - Convertible


Preferreds
Score: 0 / 1 (Question not answered.)

Question 75 (1 point)
www.nicsoceanbook.com/
OCEANBOOK@hotmail.com
An investor establishes a margin account with an investment broker and then purchases 300 shares of
ABC Company at $11.00 per share, making the minimum margin deposit. Two weeks later, the price of
ABC shares rises to $15.00 per share. Ignoring any other costs and considerations, how will this price
increase affect the margin deposit required from the investor?
Student Response: Percent Correct Student Answer Choices
Value Response Response
0.0% a. There will be no effect on the
amount of the margin deposit
required.
0.0% b. A margin call will be issued.
100.0% c. The investor may withdraw
funds from the account.
0.0% d. The investor will be required to
close his/her position.

General feedback: When a long position is established on margin, sufficient funds (or securities with
excess loan value) must be in the account to cover the purchase. The dealer lends
some of these funds to the client, the client being responsible for the balance. As the
shares have risen in value, the maximum loan value a broker will give for the shares
increases. This means that the investor's required margin deposit will decrease. The
investor may withdraw the difference between the original deposit and the new
requirement, or leave it in the account.

Reference:
CSC Textbook: Chapter 6 - Equities - Cash and Margin Accounts - Margin Accounts -
Margining Long Positions on Listed Equities
Score: 0 / 1 (Question not answered.)

Question 76 (1 point)
What type of preferred share might you purchase if you believed interest rates were going to rise in the
near term?
Student Response: Percent Correct Student Answer Choices
Value Response Response
0.0% a. Cumulative.
100.0% b. Floating rate.
0.0% c. Foreign-pay.
0.0% d. Participating.

General feedback: Preferred shares are a fixed-income investment. When interest rates rise, typically
preferred share prices fall - similar to bonds. To protect against this decline in prices,
a variable or floating rate preferred share that pays dividends in amounts that
fluctuate to reflect changes in interest rates might be purchased. Participating
preferred shares are preferred shares that have certain rights to a share in the
earnings of the company over and above their specified dividend rate; cumulative
preferred shares accumulate dividends in arrears if a company omits a dividend
payment. These last two would not directly be affected by a change in interest rates.
Foreign-pay preferred shares pay dividends in a currency other than Canadian funds,
and again would not directly benefit from an increase in Canadian interest rates.
www.nicsoceanbook.com/
OCEANBOOK@hotmail.com
Reference:
CSC Textbook: Chapter 6 - Equities - Types of Preferreds - Variable or Floating Rate
Preferreds
Score: 0 / 1 (Question not answered.)

Question 77 (1 point)
Ronaldo purchased 1,000 WOW common shares (not eligible for reduced margin) 3 months ago at $2.75
in his margin account. To keep the account in good standing, he deposited the required margin before the
transaction settled. Assume that the stock has sharply declined to $1.80 and Ronaldo has no extra cash in
his account. Excluding any commission charges, what additional amount would Ronaldo have to deposit to
avoid a margin call on the WOW shares?
Student Response: Percent Correct Student Answer Choices
Value Response Response
100.0% a. $ 655
0.0% b. $ 720
0.0% c. $1,375
0.0% d. $2,030

General feedback: At the time of purchase:


Total cost to buy WOW stock (1,000 x $2.75) $2,750
Less: Dealer's maximum loan (50% x 1,000 x $2.75) $1,375
Margin $1,375
Current values:
Original Total cost to buy WOW stock $2,750
Less: Dealer's revised maximum loan (40% x 1,000 x $1.80) $ 720
Gross Margin requirement: $2,030
Less: Client's original margin deposit $1,375
Net margin deficiency $ 655

Reference: CSC Textbook: Chapter 6 - Equities - Cash and Margin Accounts - Margin
Accounts
Score: 0 / 1 (Question not answered.)

Question 78 (1 point)
Your client needs exactly 200 shares of LSG Corp, and is unwilling to accept either more or less. What
kind of order would you place on his behalf?
Student Response: Percent Correct Student Answer Choices
Value Response Response
0.0% a. An N-C order.
0.0% b. A limit order.
0.0% c. A fill or kill order.
100.0% d. An all or none order.
www.nicsoceanbook.com/
OCEANBOOK@hotmail.com
General feedback: Most of the names given to order types are intuitively obvious: in this case, the client
wants all of his order filled, or otherwise he wants none of his order filled.

Reference:
CSC Textbook: Chapter 6 - Equities - Buy and Sell Orders - All or None Order (AON
Order)
Score: 0 / 1 (Question not answered.)

Question 79 (1 point)
What margin is required if an investor purchased 5,000 shares of XYZ, a security not eligible for reduced
margin, for $25.75 per share?
Student Response: Percent Correct Student Answer Choices
Value Response Response
0.0% a. $25,750
0.0% b. $38,625
100.0% c. $64,375
0.0% d. $90,125

General feedback: The broker is willing to lend the investor 50% of the market value of the securities in
this example. Therefore, the investor must provide margin of $64,375 (5,000 shares x
$25.75 x 50%).

Reference:
CSC Textbook: Chapter 6 - Equities - Cash and Margin Accounts - Margin Accounts -
Maximum Loan Values
Score: 0 / 1 (Question not answered.)

Question 80 (1 point)
An investor shorts 1,000 shares of XYZ.com, a security eligible for reduced margin, at $48. The price of
the stock drops to $45. What margin is required in the account after the drop in price?
Student Response: Percent Correct Student Answer Choices
Value Response Response
0.0% a. $3,900
100.0% b. $10,500
0.0% c. $14,400
0.0% d. $24,000

General feedback: The proceeds from the short sale are $48,000 (1,000 shares x $48). The proceeds
must remain in the account and the investor must deposit margin so that the sum of
the proceeds and the margin equals 130% of the current market price of the security.
In this case, the minimum balance required is $58,500 (1,000 shares x $45 x 130%).
The margin required is $10,500 ($58,500 - 48,000).

Reference:
CSC Textbook: Chapter 6 - Equities - Short Selling of Equities - How is Short Selling
Done? - Margin needed for Short Positions
www.nicsoceanbook.com/
OCEANBOOK@hotmail.com
Score: 0 / 1 (Question not answered.)

Question 81 (1 point)
A client sold short 2,000 shares of DBD Inc., a security eligible for reduced margin at $13.45. There was
already a credit balance of $1,500 in the margin account. Assuming no commission was paid on the
transaction and there are no additional securities held long in the account, how much additional margin
would the client have to deposit to keep the account in good standing?
Student Response: Percent Correct Student Answer Choices
Value Response Response
0.0% a. $1,500
0.0% b. $5,070
100.0% c. $6,570
0.0% d. $8,070

General feedback: 2,000 x $13.45 = $26,900 x 130% = $34,970 - $26,900 = $8,070 - $1,500 = $6,570

Reference: CSC Textbook: Chapter 6 - Equities - Short Selling of Equities - How is


Short Selling Done? - Margin Needed for Short Positions
Score: 0 / 1 (Question not answered.)

Question 82 (1 point)
You have very high hopes for the common shares of a new internet company that has just begun trading
on the TSX at $10 a share. You believe that the shares will rise to $50 in the next 30 days. To take
advantage of this opportunity right now, what kind of order would you enter?
Student Response: Percent Correct Student Answer Choices
Value Response Response
0.0% a. A day order.
0.0% b. An any part order.
0.0% c. A fill or kill order.
100.0% d. An at the market order.

General feedback: As with many types of buy and sell orders, the names, which have evolved over the
years, are usually descriptive of the way in which the order works. Here, the correct
term is at the market - meaning buy the shares at the market price, without limit,
regardless of the current price.

Reference: CSC Textbook: Chapter 6 - Equities - Buy and Sell Orders - At the Market
Order
Score: 0 / 1 (Question not answered.)

Question 83 (1 point)
A client places a stop-buy order on LUC Corp. at $38. What kind of order does this become once LUC
reaches or exceeds $38 in price?
www.nicsoceanbook.com/
OCEANBOOK@hotmail.com
Student Response: Percent Correct Student Answer Choices
Value Response Response
100.0% a. A limit order.
0.0% b. A market order.
0.0% c. A day order.
0.0% d. A GTC order.

General feedback: Once a stop buy order, or its opposite, the stop loss order, reaches its trigger price,
according to TSX rules it automatically becomes a limit order, in that it will be filled at
a specified price or better.

Reference: CSC Textbook: Chapter 6 - Equities - Buy and Sell Orders - At the Market
Order
Score: 0 / 1 (Question not answered.)

Question 84 (1 point)
A client purchased 2,000 shares of DDD Inc., a security eligible for reduced margin at $12.38 in her
margin account. She already had a credit balance of $2,000 in the account. Assuming no commission was
paid on the transaction and there are no additional securities held long in the account, how much
additional margin would the client have to deposit to keep her account in good standing?
Student Response: Percent Correct Student Answer Choices
Value Response Response
0.0% a. $4,904
100.0% b. $5,428
0.0% c. $7,428
0.0% d. $9,904

General feedback: 2,000 x $12.38 = $24,760 x 30% = $7,428 - $2,000 = $5,428

Reference:
CSC Textbook - Chapter 6 - Equities - Cash and Margin Accounts - Margin Accounts -
Margining Long Positions in Listed Equities
Score: 0 / 1 (Question not answered.)

Question 85 (1 point)
You have purchased 500 shares of B2 Company, which is not a security eligible for reduced margin, for
$1.85 per share, on margin. Ignoring any other costs or commissions, how much must you deposit to
your margin account to meet minimum margin requirements?
Student Response: Percent Correct Student Answer Choices
Value Response Response
0.0% a. $277.50
0.0% b. $462.50
100.0% c. $555.00
0.0% d. $740.00
www.nicsoceanbook.com/
OCEANBOOK@hotmail.com

General feedback: As these shares are not eligible for reduced margin, they do not qualify for the
reduced margin requirement where a broker may provide up to 70% of the current
market value of a security as a loan. For shares trading between $1.75 and $1.99, the
broker may provide up to 40% of the current market value as a loan. Therefore, the
amount you must deposit is the difference between the amount you paid for the
shares and the amount the broker will loan: $925.00 - $370 = $555.00. If the shares
change in price afterwards, you would receive a margin call if the shares decline in
price, or be able to withdraw part of your deposit if you wish if the shares rise in price.
If the shares move into a different price range for loan value, that loan value
percentage will apply for the new calculation. Remember that you pay interest on the
loan from the broker.

Reference:
CSC Textbook: Chapter 6 - Equities - Cash and Margin Accounts - Margin Accounts -
Maximum Loan Values
Score: 0 / 1 (Question not answered.)

Question 86 (1 point)
A client instructs her broker to buy 500 shares of TSL Inc. if the prices reaches $25 or less over the next 5
trading days. What type of order is this an example of?
Student Response: Percent Correct Student Answer Choices
Value Response Response
0.0% a. GTC order.
100.0% b. Good through order.
0.0% c. Market order.
0.0% d. Stop buy order.

General feedback: The good through order is good for a specified number of days and then is
automatically cancelled if it has not been filled. In contrast, a GTC or good till
cancelled order is valid until the close of business on the date specified in the order.

Reference:
CSC Textbook: Chapter 6 - Equities - Buy and Sell Orders - Good Through Order
Score: 0 / 1 (Question not answered.)

Question 87 (1 point)
Which of the following is a disadvantage of having share certificates in street form?
Student Response: Percent Correct Student Answer Choices
Value Response Response
0.0% a. They are not negotiable.
100.0% b. They can't be replaced if lost.
0.0% c. They can't be sold on some
exchanges.
0.0% d. They are not transferable.
www.nicsoceanbook.com/
OCEANBOOK@hotmail.com
General feedback: Securities in street form are unregistered which makes them easily transferable.
Street form securities are negotiable instruments, much like a dollar bill, which means
if they are lost or stolen, they cannot be replaced since there is no proof of ownership
on the certificates, like there is with registered securities.
Reference: Chapter 6 – Equity Securities – Common Shares
Score: 0 / 1 (Question not answered.)

Question 88 (1 point)
Which of the following features of a preferred share is generally most advantageous to an issuer?
Student Response: Percent Correct Student Answer Choices
Value Response Response
100.0% a. Redeemable.
0.0% b. Convertible.
0.0% c. Retractable.
0.0% d. Variable Rate.

General feedback: Each of the above is a special feature that can be attached to a preferred share, or a
bond. When it is the investor who decides whether the security will be redeemed, the
shares are retractable. If it were the issuer, the shares would be redeemable.
Convertible refers to the ability to convert the shares to another security, usually
common shares; variable rate refers to the interest paid on the shares. Retractable is
of benefit to the investor; redeemable to the issuer.

Reference: CSC Textbook: Chapter 6 - Equities - Types of Preferred Shares -


Retractable Preferreds
Score: 0 / 1 (Question not answered.)

Question 89 (1 point)
In which of the following scenarios would a client be best advised to purchase variable rate preferred
shares over straight preferred shares?
Student Response: Percent Correct Student Answer Choices
Value Response Response
0.0% a. The price of the common
shares is expected to remain
stable.
0.0% b. The dividend on the common
shares is expected to increase.
0.0% c. Market interest rates are
expected to fall.
100.0% d. The inflation rate continues to
show an upward trend.

General feedback: When inflation rates begin to rise, investor expectations should change to include the
possibility that the Bank of Canada will alter monetary policy in favour of higher
interest rates. The dividend paid on a floating rate preferred share will increase as
market interest rates increase according to a specified formula. This is advantageous
www.nicsoceanbook.com/
OCEANBOOK@hotmail.com
because investors will partially maintain the purchasing power of their investment in a
period of increasing interest rates. The price of the floating rate preferred share is less
responsive to interest rate changes than a straight preferred share because the
dividend adjusts to interest rate changes.

Reference: CSC Textbook: Chapter 6 - Equities - Types of Preferreds - Variable or


Floating Rate Preferreds
Score: 0 / 1 (Question not answered.)

Question 90 (1 point)
Which of the following variables is not affected in a 1-for-5 reverse stock split of a $10 stock that pays a
$1 annual dividend?
Student Response: Percent Correct Student Answer Choices
Value Response Response
100.0% a. Adjusted cost base.
0.0% b. Number of shares.
0.0% c. Price of the shares.
0.0% d. Dividend per share.

General feedback: A reverse split, also referred to as a consolidation, means that you hold fewer shares
after the split than before. This is the opposite of a stock split, where you hold more
shares after the split. A 1-for-5 share reverse split means that you receive 1 share for
every 5 shares you held before the split. 1,000 / 5 = 200 shares. The book (cost)
value represents all of the costs you have incurred in purchasing your current share
holdings. The reverse split does not affect this, as no new costs are incurred.
Therefore, your book value stays the same. The market price will tend to adjust to
reflect the new situation. If, say, pre split your shares were trading at $10.00 for a
total market value of $10,000, it is likely that the market price will rise to roughly 5 x
$10.00 = $50.00) - giving you a total market value after the split of (200 shares x
$50.00 = $10,000) - meaning that your market value remains roughly the same.

Reference: CSC Textbook: Chapter 6 - Equities - Rights and Advantages of Common


Share Ownership - Stock Splits and Consolidations - Reverse Splits or Consolidations
Score: 0 / 1 (Question not answered.)

Question 91 (1 point)
Denise purchased 20 ZEN August call contracts at $4.25. The current market price of the common stock is
$50.00 and the strike price is $55.00. What is the call premium per contract?
Student Response: Percent Correct Student Answer Choices
Value Response Response
100.0% a. $4.25
0.0% b. $5.00
0.0% c. $8.50
0.0% d. $51.75

General feedback: The call premium is the price paid for the purchase of the call contract, in this case
$4.25 per contract.
www.nicsoceanbook.com/
OCEANBOOK@hotmail.com
Reference: CSC Textbook - Chapter 7 - Derivatives - Options - Basic Option Strategies
for Individual and Institutional Investors
Score: 0 / 1 (Question not answered.)

Question 92 (1 point)
A corporation recently issued a non-callable bond with a floating rate of interest. Fearing higher interest
rates in the future, the corporation now wants to use a derivatives contract to eliminate the interest rate
risk associated with the bond. The corporation does not want to pay anything for the derivative, and it
wants to use a derivative that can be easily offset prior to expiration. Which of the following type of
derivative should it use?
Student Response: Percent Correct Student Answer Choices
Value Response Response
0.0% a. An interest rate swap.
100.0% b. An interest rate futures
contract.
0.0% c. An interest rate call option.
0.0% d. An interest rate put option.

General feedback: There is no cost to enter into an interest rate futures contract because it is a type of
forward contract. As well, it can be easily offset prior to expiration because it is
exchange traded. An interest rate swap is an over-the-counter derivative, so it cannot
be easily offset prior to expiration. Because the company wants to eliminate the risk
of higher interest rates, it might consider buying an interest option call option –
however, this will cost money, and therefore does not meet one of the company's two
criteria.
Reference: Chapter 7 – Derivatives – What is a Derivative?
Score: 0 / 1 (Question not answered.)

Question 93 (1 point)
If a put is sold with a strike price below the current market price of the underlying security, which of the
following phrases best describes it?
Student Response: Percent Correct Student Answer Choices
Value Response Response
0.0% a. In-the-money.
100.0% b. Out-of-the-money.
0.0% c. At-the-money.
0.0% d. On-the-money.

General feedback: The intrinsic value of a put contract with a strike price below the current market value
of the underlying common stock is called out-of- the-money.

Reference: CSC Textbook: Chapter 7 - Derivatives - Options - Key Terms and


Definitions
Score: 0 / 1 (Question not answered.)
www.nicsoceanbook.com/
OCEANBOOK@hotmail.com
Question 94 (1 point)
An investor is short 10 December S&P 500 Index futures contracts at a price of 1,000. Each futures
contract has an underlying value equal to $250,000. The investor does not offset his contracts before they
expire. If at expiration the S&P 500 is at a level of 950, which of the following will happen?
Student Response: Percent Correct Student Answer Choices
Value Response Response
0.0% a. The investor will have to sell
shares in the companies of the
S&P 500 at a price based on
an index level of 1,000.
0.0% b. The investor will have to sell
shares in the companies of the
S&P 500 at a price based on
an index level of 950.
100.0% c. The investor will receive a cash
payment based on the
difference between 1,000 and
950.
0.0% d. The investor will make a cash
payment based on the
difference between 1,000 and
950.

General feedback: Stock index futures are cash settled, which means that at expiration all outstanding
contracts are settled by a payment of cash rather than delivery of the underlying
interest. Since the investor went short at a price of 1,000 and the value of the index
at expiration was 950, the investor has earned a profit and will receive a cash
payment based on the difference between 1,000 and 950.
Reference: Chapter 7 – Derivatives – Forwards
Score: 0 / 1 (Question not answered.)

Question 95 (1 point)
HNF Company shares are currently trading at $67.20. An investor purchases a September 65 call at
$2.25. At what price will this position become profitable for the investor?
Student Response: Percent Correct Student Answer Choices
Value Response Response
0.0% a. At a price below $62.75
0.0% b. At a price equal to $64.95
100.0% c. At a price above $67.25
0.0% d. At a price above $69.45

General feedback: A call gives an investor the right, but not the obligation, to buy a specific quantity of a
specific underlying security. An investor would purchase a call if he/she expected the
price of the underlying security to rise. When you purchase a call, you pay a price
known as a premium. To earn back your premium, the stock must rise far enough so
that its price is higher than the exercise price by the amount of the premium paid. If
the stock rises past this point, you begin to earn a profit. Here, the exercise price is
$65. At a stock price of $67.25 ($65 + $2.25), the investor breaks even. Above this
price the investor begins to earn a profit on the position.
www.nicsoceanbook.com/
OCEANBOOK@hotmail.com
Reference: CSC Textbook: Chapter 7- Derivatives - Options - Basic Option Strategies
for Individual and Institutional Investors – Buying Put Options
Score: 0 / 1 (Question not answered.)

Question 96 (1 point)
RES Company declared a rights offering whereby shareholders of record on Friday, July 20th were granted
1 right for every 1 common shares held. Five rights are required to buy one new RES common share at a
subscription price of $5 per share. The rights expire at the close of business on August 24th. On August
1st, RES common shares are trading at $6 and the rights are trading at $0.23. What is the intrinsic value
of the rights on August 1st?
Student Response: Percent Correct Student Answer Choices
Value Response Response
0.0% a. $0.00
100.0% b. $0.20
0.0% c. $0.23
0.0% d. $1.00

General feedback: The formulas used to calculate the intrinsic value of rights depends on whether the
rights are in the cum- or ex-rights period. The ex-rights period starts two business
days before the record date; therefore for this calculation the ex-rights formula
applies. As the market price is ABOVE the subscription price, the rights have intrinsic
value. The calculation is (market price of stock-subscription price)/(number of rights
needed to subscribe for 1 share). The answer is $0.20, calculated as ($6 - $5)/(5).
During the cum-rights period the first part of the calculation would be the same,
however, the second part would be (number of rights needed to subscribe for 1 share
plus 1).

Reference: CSC Textbook: Chapter 7 - Derivatives - Rights and Warrants - Rights -


The Intrinsic Value of Rights
Score: 0 / 1 (Question not answered.)

Question 97 (1 point)
You are asked to consider 3 call options, all expiring in September: Option A, Option B and Option C. Each
has an exercise price of $25.00, and each has, as an underlying security, common shares of a company
currently trading at $22.50. Company “A” is a blue-chip company, long established in the Canadian
marketplace, paying a high dividend. Company “B” is a speculative growth company, paying no dividends.
Company “C” is a large retail organization, paying a minimal annual dividend. Ignoring any other
considerations, how would the options rank in order of price, from highest to lowest?
Student Response: Percent Correct Student Answer Choices
Value Response Response
0.0% a. Option A/Option C/Option B.
0.0% b. Option B/Option A/Option C.
0.0% c. Option C/Option A/Option B.
100.0% d. Option B/Option C/Option A.

General feedback: The price of an option depends on many considerations: The relationship between the
www.nicsoceanbook.com/
OCEANBOOK@hotmail.com
market price of the security and the exercise price of the option; time to expiration;
interest rates; volatility of the underlying security; and dividends. In this example, the
three options have identical differences between stock price and exercise price, time
to expiration, and interest rates. The differences lie in the volatility of the companies
and the dividends paid. Call options on stocks with higher yields will have lower option
premiums than stocks with lower or no yields. Therefore, A would have a lower
premium than C or B. Call options on stocks with higher volatility will have higher
premiums than on stocks with lower premiums. Volatility is a measure of the amount
a stock fluctuates in price. To make money on a call option, you need to have the
stock change in price - therefore, stocks whose prices swing widely are more likely to
be profitable than ones whose prices do not change. Therefore, B would have a higher
premium than C, which would have a higher premium than A. The end result would be
that B would have the highest premium due to its highest volatility and lowest income,
and A would have the lowest premium.
Reference: CSC Textbook: Chapter 9 - Derivative Securities - Options
Score: 0 / 1 (Question not answered.)

Question 98 (1 point)
Johan owns 800 shares of KIY Company, with an average cost per share of $26.50. To earn additional
income, Johan writes 8 KIY 25 calls at $2.75 when KIY shares are trading at $24.50. What is the intrinsic
value of the calls?
Student Response: Percent Correct Student Answer Choices
Value Response Response
100.0% a. $0.00
0.0% b. $0.50
0.0% c. $1.50
0.0% d. $2.75

General feedback: An option price, just like a warrant price or a right price, is made up of two factors: a
time value and an intrinsic value. If calls are out-of-the-money they do not have an
intrinsic value. Here, as the exercise price of the calls is above the market price of the
shares, they are out-of-the-money meaning that the entire $2.75 is time value.

Reference: CSC Textbook: Chapter 7 - Derivatives - Options - Basic Option Strategies


for Individual and Institutional Investors - Writing Call Options
Score: 0 / 1 (Question not answered.)

Question 99 (1 point)
ABC Company shares are currently trading at $37.50. An investor purchases a Sep 33.50 put at $2.25. At
what price will this position become profitable for the investor?
Student Response: Percent Correct Student Answer Choices
Value Response Response
100.0% a. At a price below $31.25.
0.0% b. At a price equal to $35.25.
0.0% c. At a price above $35.75.
0.0% d. At a price above $39.75
www.nicsoceanbook.com/
OCEANBOOK@hotmail.com
General feedback: A put gives an investor the right, but not the obligation, to sell a specific quantity of a
specific underlying security. An investor would purchase a put if he/she expected the
price of the underlying security to decline. When you purchase a put, you pay a price -
known as a premium. To earn back your premium, the stock must decline far enough
that the exercise price is above the stock price by the premium paid. If the stock
declines past this point, you begin to earn a profit. This position becomes profitable at
a price below $31.25 ($33.50 - $2.25). At $31.25 the investor breaks even. If the
price falls below this price the investor begins to make a profit. The actual decline in
the stock price would be ($37.50 - $31.25) = $6.25. The investor is speculating that
the shares will decline by at least $6.25.

Reference: Derivative Securities - Options


CSC Textbook: Chapter 7 - Derivatives - Options - Buying Put Options
Score: 0 / 1 (Question not answered.)

Question 100 (1 point)


Which of the following risks does a ‘naked’ call writer assume?
Student Response: Percent Correct Student Answer Choices
Value Response Response
0.0% a. Loss of a potential capital gain
if the option is assigned.
0.0% b. Loss of the entire premium.
0.0% c. No risk because the investor
owns the stock.
100.0% d. Unlimited loss potential if the
stock price increases.

General feedback: The naked or uncovered call writer does not own the stock, but must stand ready to
sell it to the call buyer at the strike price if called. The naked writer may potentially
have an unlimited loss if the stock price rises substantially above the strike price at
expiry. This is because the naked writer would have to buy shares in the market at a
very high price and then sell them to the call buyer at the relatively low strike price.

Reference: CSC Textbook: Chapter 7 - Derivatives - Options - Basic Option Strategies


for Individual and Institutional Investors - Writing Call Options
Score: 0 / 1 (Question not answered.)
www.nicsoceanbook.com/
OCEANBOOK@hotmail.com
Question 1 (1 point)
Which of the following has occurred historically after an inverted yield curve has become apparent in the United States?
Student Response: Percent Correct Student Answer Choices
Value Response Response
0.0% a. Significant redemptions in mutual funds.
100.0% b. Substantial rallies in the Canadian and
U.S. equity markets.
0.0% c. Increased selling of the Canadian dollar
and buying of the U.S. dollar.
0.0% d. Significant increases in the price of gold.

General feedback: There is an important link between changes in the yield curve and monetary policy and its impact on
equity markets. Historically, when the U.S. yield curve becomes inverted, it produced sizeable rallies
in the U.S. and Canadian markets.
Reference: CSC Textbook: Chapter 8- Analyzing Markets and Products - Fundamental Macroeconomic
Analysis - Monetary Policy - The Tilting Yield Curve
Score: 1/1

Question 2 (1 point)
When a technical analyst identifies a strong demand for a stock while the supply is low, what conclusion can be made
about the level of the stock price?
Student Response: Percent Correct Student Answer Choices
Value Response Response
0.0% a. It is at a downswing level.
100.0% b. It is at a support level.
0.0% c. It is at a resistance level.
0.0% d. It is at a stagnating level.

General feedback: A support level is the price at which the majority of investors start sensing value, and therefore are
willing to buy (demand is strong) and the majority of existing holders (or potential short sellers) are
not willing to sell. As demand begins to exceed supply, prices tend to rise above support levels.

Reference: CSC Textbook: Chapter 8 - Analyzing Markets and Products - Technical Analysis -
Commonly Used Tools in Technical Analysis - Chart Analysis
Score: 1/1

Question 3 (1 point)
Company A and Company B are similar sized companies, producing a similar product, and have been in existence for a
similar length of time. Company A has a higher proportion of debt and preferred shares in its capital structure than
Company B. The business cycle is moving into an expansion phase. What effect would you expect this to have on the two
companies' earnings, and which company would be more affected?
Student Response: Percent Correct Student Answer Choices
Value Response Response
www.nicsoceanbook.com/
OCEANBOOK@hotmail.com
100.0% a. Earnings would increase and Company A
would be more affected.
0.0% b. Earnings would increase and Company B
would be more affected.
0.0% c. Earnings would decrease and Company A
would be more affected.
0.0% d. Earnings would decrease and Company B
would be more affected.

General feedback: When the business cycle moves into an expansion phase, typically company earnings increase. If a
company is leveraged, the effect of the change in earnings is typically larger than if a company is not
leveraged. Leverage refers to the proportion of a company's capital structure that is made up of debt
and preferred shares. Therefore, Company A, which is more highly leveraged, would be more
affected by the increase in earnings occurring as a result of the change in the business cycle.

Reference: CSC Textbook: Chapter 8 - Analyzing Markets and Products - Fundamental Company
Analysis - Balance Sheet Analysis - The Effect of Leverage
Score: 1/1

Question 4 (1 point)
CBA Inc and ZYX Inc. are in the same line of business. CBA Inc. has a debt/equity ratio of 0.94 and ZYX Inc. has a
debt/equity ratio of 1.29. The average debt/equity ratio for the industry is 0.98. Considering this information, which
company would you recommend to your clients and why?
Student Response: Percent Correct Student Answer Choices
Value Response Response
100.0% a. CBA Inc., since it has the lower
debt/equity ratio of the 2 companies
compared.
0.0% b. CBA Inc., since it has a debt/equity ratio
that is closer to the industry average.
0.0% c. ZYX Inc., since it has the higher
debt/equity ratio of the 2 companies
compared.
0.0% d. ZYX Inc., since it has a debt/equity ratio
that is above the industry average.

General feedback: All things being equal, a company with a lower debt/equity ratio is a safer investment. Thus CBA Inc.
should be recommended.

Reference: CSC Textbook: Chapter 8 - Analyzing Markets and Products - Interpreting Financial
Statements - Risk Analysis Ratios - Debt/Equity Ratios
Score: 1/1

Question 5 (1 point)
Which of the following statements concerning the cash flow/total debt outstanding ratio is the most accurate?
Student Response: Percent Correct Student Answer Choices
Value Response Response
www.nicsoceanbook.com/
OCEANBOOK@hotmail.com
0.0% a. This ratio indicates whether a company's
earnings are sufficient to deal with future
adverse conditions.
0.0% b. This ratio shows the net tangible assets of
a company available to cover the total
debt outstanding.
0.0% c. This ratio measures a company's ability to
pay interest charges on its debt.
100.0% d. This ratio gauges a company's ability to
repay all the funds it has borrowed.

General feedback: The cash flow/total debt outstanding ratio gauges a company's ability to repay all the funds it has
borrowed.

Reference: CSC Textbook: Chapter 8 - Analyzing Markets and Products - Interpreting Financial
Statements - Risk Analysis Ratios - Cash Flow/Total Debt Outstanding
Score: 1/1

Question 6 (1 point)
CCK stock closed at the following prices during the course of the last trading week:

Monday Tuesday Wednesday Thursday Friday


$15.50 $18.00 $18.25 $17.50 $17.75

What is CCK's 5-day moving average as of the close of trading on Friday?


Student Response: Percent Correct Student Answer Choices
Value Response Response
0.0% a. $17.13
100.0% b. $17.40
0.0% c. $17.67
0.0% d. $17.75

General feedback: A moving average is simply the sum of the closing prices divided by the number of closing prices. In
this case, the closing prices for the five trading days sum to $87. This sum is then divided by 5 to get
the moving average of $17.40.
Reference: Chapter 8 – Analyzing Markets and Products – Technical Analysis
Score: 1/1

Question 7 (1 point)
What theory concludes that there are repetitive, predictable sequences of numbers and cycles found in nature and similar
predictable patterns in the movement of stock prices?
Student Response: Percent Correct Student Answer Choices
Value Response Response
100.0% a. Elliott Wave Theory.
www.nicsoceanbook.com/
OCEANBOOK@hotmail.com
0.0% b. Random Walk Theory.
0.0% c. Credit-Debit Balance Ratio.
0.0% d. Rational Expectations Hypothesis.

General feedback: Many theories exist that attempt to relate movements in the stock markets to different inputs and
factors. The Rational Expectations Hypothesis and the Random Walk Theory are Capital Market
Theories that essentially relate to the effect, or lack of effect, of distribution of information on stock
market prices. The Credit-Debit Balance ratio is a quantitative measurement of the changes in credit
and debit balances in the U.S. banking system, and how changes in these balances predict a bull or
bear market. The Elliott Wave Theory states in essence that the stock market moves up in a series of
five waves and down in a series of three waves.

Reference: CSC Textbook: Chapter 8 - Analyzing Markets and Products - Technical Analysis -
Commonly Used Tools in Technical Analysis - Cycle Analysis
Score: 1/1

Question 8 (1 point)
One method of comparing cyclical and defensive industries is to calculate the return on equity (ROE). Which of the
following statements concerning the ROE of cyclical and defensive industries is true?
Student Response: Percent Correct Student Answer Choices
Value Response Response
100.0% a. During economic downturns, the ROE of
defensive industries falls less dramatically
than that of cyclical industries.
0.0% b. During economic upturns, the ROE of
defensive industries tends to rise more
than that of cyclical industries.
0.0% c. During economic downturns, the ROE of
cyclical and defensive industries show
proportionately the same fall, but during
economic upturns, the ROE of cyclical
industries tends to rise more than that of
defensive industries.
0.0% d. The ROE of cyclical and defensive
industries is not influenced by swings in
the economy.

General feedback: During economic downturns, the ROE of defensive industries falls less dramatically than that of
cyclical industries. During economic upturns, the ROE of defensive industries tends not to rise as
much as that of cyclical industries.

Reference: CSC Textbook: Chapter 8 - Analyzing Markets and Products - Fundamental Industry
Analysis - Classification by Industry and Stock Characteristics - Return on Equity (ROE)
Score: 1/1

Question 9 (1 point)
Which of the following types of financial ratios measures the stock market's rating of a company by relating the market
price of its shares to certain balances obtained from its financial statements?
www.nicsoceanbook.com/
OCEANBOOK@hotmail.com
Student Response: Percent Correct Student Answer Choices
Value Response Response
0.0% a. Debt ratios.
100.0% b. Value ratios.
0.0% c. Liquidity ratios.
0.0% d. Profitability ratios.

General feedback: A total analysis of a company's financials incorporates the use of all of the groups of financial ratios.
Each group has a specific use for interpretation and discussion. The value ratios include Percentage
Dividend Payout ratios, Earnings per Common Share, Dividend Yield and Price-Earnings. Where the
other ratios measure the performance of a company (i.e., how many times it has covered its interest
charges, how profitable it is) - this group measures the value the company brings to each share.

Reference: CSC Textbook: Chapter 8 - Analyzing Markets and Products - Interpreting Financial
Statements - Types of Ratios - Value Ratios
Score: 1/1

Question 10 (1 point)
If an investor felt that the economy was at the peak of the business cycle and expected equity prices to decline, what
would be the most appropriate equity investment?
Student Response: Percent Correct Student Answer Choices
Value Response Response
0.0% a. Forest products and mining stocks.
100.0% b. Bank and utility stocks.
0.0% c. Capital goods stocks.
0.0% d. Retail merchandising stocks.

General feedback: During periods when stock prices are declining, cyclical stocks (forest products, mining, capital
goods, retail merchandising, etc.) fall faster than average. Defensive stocks, such as banks and
utilities, fall relatively slower on average. Therefore, if an investor is anticipating an economic
contraction, her portfolio should shift to more defensive stocks.

Reference: CSC Textbook: Chapter 10 - Analyzing Markets and Products - Fundamental Industry
Analysis - Classification by Industry and Stock Characteristics - Defensive Industries
Score: 1/1

Question 11 (1 point)
Which of the following external events would likely have the biggest impact on the Canadian equity market?
Student Response: Percent Correct Student Answer Choices
Value Response Response
0.0% a. The election of a socialist government in
Australia.
0.0% b. The decision by Paraguay to default on its
U.S.-dollar denominated debt.
0.0% c. The development of the world's smallest
www.nicsoceanbook.com/
OCEANBOOK@hotmail.com
semiconductor by a Japanese technology
company.
100.0% d. The announcement by the World Gold
Council that it was cutting by one-quarter
its estimate of recoverable gold from the
world's mines.

General feedback: While all of these events may impact the Canadian equity market, the announcement by the World
Gold Council would likely have the biggest impact. Canada is considered a commodity-based
economy, and events that affect global commodity prices usually have a significant impact on
Canadian equity markets.
Reference: Chapter 8 – Analyzing Markets and Products – Fundamental Macroeconomic Analysis
Score: 0/1

Question 12 (1 point)
Which of the following price patterns would a technical analyst consider a sell signal?
Student Response: Percent Correct Student Answer Choices
Value Response Response
0.0% a. The advance/decline line flattens out after
a long decline.
100.0% b. Price breaks through the moving average
from above on heavy volume.
0.0% c. When the number of stocks that advance
during a trading day equals the number
that have declined.
0.0% d. Price breaks through the moving average
from below on heavy volume.

General feedback: If price breaks through the moving average line from above on heavy volume, and the moving
average line itself starts to fall, the upward trend is reversed. This is a sell signal.
Reference: CSC Textbook: Chapter 8 - Analyzing Markets and Products - Technical Analysis -
Commonly Used Tools in Technical Analysis - Quantitative Analysis
Score: 1/1

Question 13 (1 point)
Assume that in all other relevant financial indicators, SFO Inc. has approximately the same measurements as the industry
averages. Considering the information provided below, which of the following statements is true?

Debt/Equity Year Year Year Year Year 5-Year


Ratios: 1 2 3 4 5 Average
Industry
1.15 1.25 1.50 1.70 1.85 1.49
Average
SFO Inc. 1.50 1.20 1.10 0.96 0.64 1.08
Student Response: Percent Correct Student Answer Choices
Value Response Response
100.0% a. Since SFO Inc. has exhibited a lower
debt/equity ratio than the industry
www.nicsoceanbook.com/
OCEANBOOK@hotmail.com
average, it can be concluded that SFO Inc.
has consistently outperformed the
industry.
0.0% b. Since SFO Inc. has exhibited a higher
debt/equity ratio than the industry
average, it can be concluded that SFO Inc.
has consistently outperformed the
industry.
0.0% c. Since the industry has exhibited a lower
average debt/equity ratio, it can be
concluded that the industry has
consistently outperformed SFO Inc.
0.0% d. Since the industry has exhibited a higher
average debt/equity ratio, it can be
concluded that the industry has
consistently outperformed SFO Inc.

General feedback: A lower debt/equity ratio, other factors being comparable, indicates a company with better
performance. Since SFO Inc. has a consistently lower debt/equity ratio over the 5-year period, it can
be concluded that SFO Inc. has outperformed the industry average.

Reference: CSC Textbook: Chapter 8 - Analyzing Markets and Products - Interpreting Financial
Statements - Risk Analysis Ratios - Debt/Equity Ratio
Score: 1/1

Question 14 (1 point)
Based on the following information about the companies in Industries X, Y, and Z, which of the following statements is
most likely true?

Two-Year Average
Two-Year Average
Industry Growth in Return on
Dividend Yield
Equity (ROE)
X 15% 1%
Y -5% 0%
Z 6% 4%
Student Response: Percent Correct Student Answer Choices
Value Response Response
0.0% a. Industry X is an emerging industry,
Industry Y is a growth industry, and
Industry Z is a mature industry.
0.0% b. Industry X is a mature industry, Industry Y
is a growth industry, and Industry Z is an
emerging industry.
0.0% c. Industry X is an emerging industry,
Industry Y is a mature industry, and
Industry Z is a growth industry.
www.nicsoceanbook.com/
OCEANBOOK@hotmail.com
100.0% d. Industry X is a growth industry, Industry Y
is an emerging industry, and Industry Z is
a mature industry.

General feedback: The average growth in ROE for companies in an emerging industry is usually lower than that of
companies in a mature industry, which in turn is usually lower than that of companies in a growth
industry. Similarly, the average dividend yield for companies in an emerging industry is usually lower
than that of companies in a growth industry, which in turn is usually lower than that of companies in
a mature industry.
Reference: Chapter 8 – Analyzing Markets and Products – Fundamental Industry Analysis
Score: 1/1

Question 15 (1 point)
Based on the Dividend Discount Model, what is the intrinsic value of Glacier Energy if its expected dividend in one year is
$1.20, the discount rate is 6.30%, and its long-term growth is expected to be continuous at 3.60%?
Student Response: Percent Correct Student Answer Choices
Value Response Response
0.0% a. $33.33
100.0% b. $44.44
0.0% c. $46.04
0.0% d. $88.89

General feedback: The formula is Price = Div1 / (r-g). Div1 is the expected dividend paid out by the company in one
year, r is the required rate of return on investments, and g is the assumed constant growth rate for
dividends. Therefore, the price or intrinsic value of Glacier Energy = $1.20 / (.0630 - .036) which
equals $44.44.
Reference: CSC Textbook: Chapter 8 - Analyzing Markets and Products - Fundamental Valuation
Models - Dividend Discount Model
Score: 1/1

Question 16 (1 point)
Which of the following technical analysis signals indicates an increase in the breadth of the stocks in the S&P/TSX
Composite Index?
Student Response: Percent Correct Student Answer Choices
Value Response Response
0.0% a. The index breaks through a significant
resistance level.
0.0% b. The index breaks through its 200-day
moving average from below.
100.0% c. The index's cumulative advance-decline
line increases from 25,200 to 25,700.
0.0% d. The index falls while its moving average
convergence-divergence (MACD)
increases.
www.nicsoceanbook.com/
OCEANBOOK@hotmail.com
General feedback: An increase in the cumulative advance-decline indicates an increase in the breadth of the stock in the
S&P/TSX Composite Index. All the other signals are related to the trend of the overall market, which
does not necessarily say anything about the breadth of the market.
Reference: Chapter 8 – Analyzing Markets and Products – Technical Analysis
Score: 0/1

Question 17 (1 point)
Jim is a conservative investor and is considering buying the preferred shares of a major Canadian bank. What special type
of preferred shares would best suit Jim?
Student Response: Percent Correct Student Answer Choices
Value Response Response
0.0% a. Convertible preferred shares.
0.0% b. Variable dividend preferred shares.
0.0% c. Preferred shares with warrants.
100.0% d. Retractable preferred shares.

General feedback: The various special features on preferred shares will benefit certain investors differently. Retractable
preferred shares generally are less risky and are considered to be of benefit to conservative
investors. The other preferred shares listed would benefit more risk averse and aggressive investors.
Reference: CSC Textbook: Chapter 8 - Analyzing Markets and Products - Interpreting Financial
Statements - Assessing Preferred Share Investment Quality - How Preferred Shares Fit Into
Individual Portfolios
Score: 0/1

Question 18 (1 point)
The Governor of the Bank of Canada states in a speech that the Canadian economy is overheated and that the Bank of
Canada will have to do something about it. What effect would this speech have on the prices of Canadian bonds and why?
Student Response: Percent Correct Student Answer Choices
Value Response Response
0.0% a. Canadian bond prices would likely rise in
anticipation of a restraint on the growth of
the money supply.
100.0% b. Canadian bond prices would likely fall in
anticipation of a restraint on the growth of
the money supply.
0.0% c. Canadian bond prices would likely rise in
anticipation of a decrease in the supply of
Government of Canada bonds.
0.0% d. Canadian bond prices would likely fall in
anticipation of a decrease in the supply of
Government of Canada bonds.

General feedback: Such a speech would likely cause an increase in the general level of Canadian interest rates. This
means that Canadian bond prices would fall. Interest rates would go up because, given the
Governor's speech, the market would anticipate that the Bank of Canada would restrain growth in
the money supply. This is viewed as a contractionary monetary policy. Nothing in the Governor's
speech would cause the market to anticipate a decrease in the supply of Government of Canada
www.nicsoceanbook.com/
OCEANBOOK@hotmail.com
bonds.
Reference: Chapter 8 – Analyzing Markets and Products – Fundamental Macroeconomic Analysis
Score: 1/1

Question 19 (1 point)
Company A, a new, speculative company in the consumer software industry has a P/E ratio of 65. Company B, an
established financial services company, has a P/E ratio of 15. Company C, a producer of automobiles, has a P/E ratio of
27. Inflation rises sharply. Ignoring any other considerations, how would the companies likely rank in order of change in
P/E, from highest to lowest?
Student Response: Percent Correct Student Answer Choices
Value Response Response
0.0% a. Company A/Company B/Company C.
100.0% b. Company A/Company C/Company B.
0.0% c. Company B/Company C/Company A.
0.0% d. Company C/Company B/Company A.

General feedback: P/E levels and inflation are inversely related. As inflation rises, P/E levels tend to fall. Individual
stocks are affected in different relative magnitudes, based on stock and industry specific
considerations. Typically, more speculative companies, such as A, are affected more than blue chip
companies such as B. C would be considered as a consumer cyclical industry, and would be affected
more than B, but less than A, which is a riskier example of a consumer cyclical industry.

Reference: CSC Textbook: Chapter 8 - Analyzing Markets and Products - Fundamental Valuation
Models - PE Levels and Inflation
Score: 1/1

Question 20 (1 point)
Which of the following might happen if the markets suddenly suspect an increase in the rate of inflation in Canada?
Student Response: Percent Correct Student Answer Choices
Value Response Response
0.0% a. Money market rates would fall.
0.0% b. The price of gold would increase.
100.0% c. Canadian equity prices would fall.
0.0% d. The value of the Canadian dollar would
increase.

General feedback: An increase in the rate of inflation might cause equity prices to fall in anticipation of rising interest
rates and a slowing economy. And that anticipation would likely cause money market rates to go up,
not down. There's nothing to suggest that Canadian inflation has any effect on the price of gold. And
it's impossible to say what effect it would have on the Canadian dollar. We would need to know at
least two things before we could presume anything about the value of the Canadian dollar: the
currency it's being measured against and inflation expectations in the currency's country.
Reference: Chapter 8 – Analyzing Markets and Products – Fundamental Macroeconomic Analysis
Score: 1/1
www.nicsoceanbook.com/
OCEANBOOK@hotmail.com
Question 21 (1 point)
Which of the following statements about the price-earnings (P/E) ratios is true?
Student Response: Percent Correct Student Answer Choices
Value Response Response
0.0% a. P/E ratios are calculated for preferred and
common shares.
0.0% b. Bank stocks tend to have the highest P/E
ratios in the marketplace.
0.0% c. Value investors tend to select stocks with
the highest P/E ratios.
100.0% d. P/E ratios typically decrease in a bearish
market.

General feedback: As a rule, P/E ratios increase in a rising stock market or with rising earnings. The reverse is true in a
declining market or when earnings decline.
Reference: CSC Textbook: Chapter 8 - Analyzing Markets and Products - Interpreting Financial
Statements - Value Ratios - Price-Earnings Ratio or P/E Multiple
Score: 0/1

Question 22 (1 point)
Which of the following statements best reflects the Efficient Market Hypothesis?
Student Response: Percent Correct Student Answer Choices
Value Response Response
0.0% a. Actively managed portfolios consistently
outperform passively managed portfolios.
0.0% b. Past price movements in stocks enable
investors to predict future price
movements.
100.0% c. Investors should not be able to
consistently outperform the market.
0.0% d. Equities will outperform fixed income
securities over the long-term.

General feedback: The Efficient Market Hypothesis assumes that there are a large number of rational profit-seeking
investors in the marketplace who react quickly to the release of new information. As new information
about a stock appears, investors reassess the intrinsic value of the stock and adjust their estimation
of its price accordingly. Therefore, at any time, a stock’s price should accurately reflect all available
information and represent the best estimate of its true value. In this way, consistently outperforming
an efficient market should not occur.
Reference: CSC Textbook: Chapter 8 - Analyzing Markets and Products - Overview of Analysis
Methods - Efficient Market Theories
Score: 1/1

Question 23 (1 point)
In an uptrend, if breadth measurements are persistently weak, what can be concluded about the trend of the market?
Student Response: Percent Correct Student Answer Choices
www.nicsoceanbook.com/
OCEANBOOK@hotmail.com
Value Response Response
100.0% a. The trend has a higher probability of
failing.
0.0% b. The trend has a higher probability of
advancing.
0.0% c. The trend has a lower probability of
declining.
0.0% d. The trend is not a good measure of market
performance.

General feedback: In an uptrend, if breadth measurements are persistently weak, the trend has a higher probability of
failing.

Reference: CSC Textbook: Chapter 8 - Analyzing Markets and Products - Technical Analysis - Equity
Market Analysis - Breadth of Market
Score: 0/1

Question 24 (1 point)
Which of the following factors is likely to have a positive impact on Canadian equity prices?
Student Response: Percent Correct Student Answer Choices
Value Response Response
0.0% a. Income tax rates are increased in an effort
to balance the budget.
0.0% b. The U.S. federal reserve increases U.S.
short-term interest rates to combat
inflationary pressures.
100.0% c. Non-resident purchases of stocks increase.
0.0% d. Downward pressure continues on the value
of the Canadian dollar.

General feedback: Increased purchases of common stocks by non-residents will increase the over-all demand for stocks
and, therefore, increase their price. The other factors have a negative impact on stock prices.
Increased tax rates leave individuals with a lower disposable income and reduced spending power.
Canada frequently follows U.S. interest rate changes. Increased rates increase business costs. They
also increase the opportunity cost of owning common stocks. Negative pressure on the dollar makes
foreign investment less attractive and will reduce the demand for Canadian common stock.

Reference: CSC Textbook: Chapter 8 - Analyzing Markets and Products - Fundamental


Macroeconomic Analysis - Flow of Funds - Non-Resident Net Purchases
Score: 0/1

Question 25 (1 point)
Trillium Therapeutics (symbol TT) is expected to pay a $1.50 dividend next year. The company anticipates a consistent
long-term growth rate of 5.5% and investors believe that a required return of 8.25% on TT is suitable. Using the Dividend
Discount Model (DDM), what is the intrinsic value of the TT shares?
www.nicsoceanbook.com/
OCEANBOOK@hotmail.com
Student Response: Percent Correct Student Answer Choices
Value Response Response
100.0% a. $54.55
0.0% b. $57.55
0.0% c. $18.18
0.0% d. $10.91

General feedback: The Dividend Discount Model illustrates in a very simple way how companies with stable growth are
theoretically priced. The formula is Price = Div1 / (r-g). Div1 is the expected dividend paid out by the
company in one year, r is the required rate of return on investments, and g is the assumed constant
growth rate for dividends. Therefore, the price or intrinsic value of TT = $1.50 / (.0825 - .055) which
equals $54.55.
Reference: CSC Textbook: Chapter - Analyzing Markets and Products - Fundamental Valuation
Models - Dividend Discount Model
Score: 1/1

Question 26 (1 point)
During periods in which stock prices are rising, an industry rotation strategy would stipulate shifting the investment mix
towards which of the following sectors?
Student Response: Percent Correct Student Answer Choices
Value Response Response
100.0% a. Cyclical industries.
0.0% b. Defensive industries.
0.0% c. Emerging markets.
0.0% d. Long-term investment vehicles.

General feedback: The most basic industry rotation strategy involves shifting back and forth between cyclical and
defensive industries. In periods of rising stock prices, a manager should select investments in cyclical
industries, since they tend to rise relatively faster because their profit growth is more robust during
an economic expansion.

Reference: CSC Textbook: Chapter 9 - The Portfolio Approach - Developing An Asset Mix - Setting
the Asset Mix - Industry Rotation
Score: 1/1

Question 27 (1 point)
What is a risk feature of an equity manager who follows the growth style of investing?
Student Response: Percent Correct Student Answer Choices
Value Response Response
0.0% a. Lower beta and standard deviation.
0.0% b. Longer holding periods to achieve return.
0.0% c. Sensitivity to the economic forecast.
100.0% d. High portfolio volatility.
www.nicsoceanbook.com/
OCEANBOOK@hotmail.com
General feedback: Growth style equity managers focus on earnings momentum and are willing to pay higher prices if
they feel that the momentum potential justifies the higher price. With the focus on growth and
momentum, any deviation from expectation - either higher or lower EPS than expected - can cause
large price changes. These portfolios have high levels of volatility.

Reference: CSC Textbook: Chapter 9 - The Portfolio Approach - Developing an Asset Mix - Equity
Manager Styles - Growth Managers
Score: 1/1

Question 28 (1 point)
What is the ex-post rate of return if an investor purchased a mutual fund unit on January 1st at $12.00, sold it on
December 31st at $10.75, and received $0.25 in distributions during the year?
Student Response: Percent Correct Student Answer Choices
Value Response Response
0.0% a. -10.42%
0.0% b. - 9.30%
100.0% c. - 8.33%
0.0% d. - 7.50%

General feedback: The term ex-post simply refers to the fact this rate of return is “after the fact” rather than before.
You are looking back at the return you received on the stock, after a period of holding the
investment. It is calculated using the amount you earned on the investment, divided by the amount
you paid. In this case, you lost $1.25 on the difference between the year-end price of $10.75 and the
purchase price of $12.00, but you gained $0.25 in cash flow through the distributions. Therefore,
your net loss is $1.00, which, divided by $12.00, is a negative return of 8.33%.

Reference: CSC Textbook: Chapter 9 - The Portfolio Approach - Risk and Return - Rate of Return
Score: 1/1

Question 29 (1 point)
You are reviewing the performance of a client's investment portfolio. The client's strategic asset mix is 5% cash, 40%
stocks and 55% bonds. Your research department has provided you with its latest forecast, with bonds showing an
expected future return of 3%, equities 7% and cash 2%. What is the portfolio's expected rate of return?
Student Response: Percent Correct Student Answer Choices
Value Response Response
0.0% a. 4.80%
0.0% b. 6.60%
100.0% c. 4.55%
0.0% d. 12.0%

General feedback: The expected rate of return on a portfolio is calculated using the weightings of the client specific
asset classes and the projected rate of return. Here, cash is expected to return 2%, and has a
weighting of 5%, bonds an expected return of 3% and a weighting of 55%, and stocks an expected
rate of return of 7% and a weighting of 40%. Therefore, the expected return is (2% × .05) + (3% ×
.55) + (7% × .40) = 4.55%.
www.nicsoceanbook.com/
OCEANBOOK@hotmail.com
Reference: CSC Textbook: Chapter 9 - The Portfolio Approach - Risk and Return - Portfolio Risk and
Return - Rate of Return on Portfolios
Score: 1/1

Question 30 (1 point)
Which of the following attributes of preferred shares does not support the decision to consider them part of the fixed-
income portion of a portfolio?
Student Response: Percent Correct Student Answer Choices
Value Response Response
0.0% a. Preferred shares tend to trade on a yield
basis.
0.0% b. Preferred shares pay a predictable stream
of income.
0.0% c. Preferred shares have a reasonably
definable term to maturity.
100.0% d. Preferred shares pay income in the form of
dividends.

General feedback: The first three choices describe three features that preferred shares have in common with "regular"
fixed-income securities, so these definitely support the decision to consider preferred shares part of
the fixed-income portion of a portfolio. The last choice describes the main difference between
preferreds and "regular" fixed-income securities, so it does not support the decision to include
preferreds in the fixed-income portfolio of a portfolio.
Reference: Chapter 9 – The Portfolio Approach – Developing an Asset Mix
Score: 1/1

Question 31 (1 point)
A portfolio manager for the Equality Equity Fund pursues a strategy whereby he is willing to pay higher prices for stocks
that offer good price momentum. This is an example of what type of portfolio manager style?
Student Response: Percent Correct Student Answer Choices
Value Response Response
0.0% a. Value style.
0.0% b. Tactical style.
0.0% c. Dynamic style.
100.0% d. Growth style.

General feedback: Growth style equity managers focus on earnings momentum and are willing to pay higher prices if
they feel that the momentum potential justifies the higher price. With the focus on growth and
momentum, any deviation from expectation - either higher or lower EPS than expected - can cause
large price changes. These portfolios have high levels of volatility.

Reference: CSC Textbook: Chapter 9 - The Portfolio Approach - Developing an Asset Mix - Equity
Manager Styles - Growth Managers
Score: 1/1
www.nicsoceanbook.com/
OCEANBOOK@hotmail.com
Question 32 (1 point)
An investor's portfolio currently consists of the common shares of 2 companies: ABC Bank and XYZ Gold. How can this
investor reduce the overall risk of this portfolio?
Student Response: Percent Correct Student Answer Choices
Value Response Response
0.0% a. Add more bank and gold company shares
to the portfolio.
100.0% b. Add government or high quality corporate
bonds to the portfolio.
0.0% c. Add shares of industrial and retail
companies to the portfolio.
0.0% d. Add shares of pharmaceuticals for growth.

General feedback: Systematic risk is the risk of being in a particular capital market. For example, if there is a significant
stock market correction, most stocks will decline. Systematic risk cannot be diversified away by
adding more stocks in the same or different industry groups as compared to the existing portfolio.
Systematic risk can be reduced through the investment in securities in a different capital market. In
this case, adding bonds to the portfolio will reduce the systematic risk of owning stocks.

Reference: CSC Textbook: Chapter 9 - The Portfolio Approach - Risk and Return - Risk-"The Other
Side of the Coin" - Systematic Risk
Score: 0/1

Question 33 (1 point)
The outlook for interest rates is bullish, with a maximum change of 200 basis points predicted. What impact is this
outcome likely to have on the expected gain/loss of capital on fixed-income portfolio holdings?
Student Response: Percent Correct Student Answer Choices
Value Response Response
0.0% a. An expected loss on fixed-income holdings.
100.0% b. An expected gain on fixed-income holdings
0.0% c. A neutral impact on fixed-income holdings.
0.0% d. The impact can’t be determined.

General feedback: A bullish outlook implies that interest rates will fall. Due to the inverse relationship between prices
and yields on fixed-income securities, when rates are expected to fall bond prices will rise. The fall in
rates creates a gain on the fixed-income side of the portfolio.

Reference: CSC Textbook: Chapter 9 - The Portfolio Approach - Evaluating Portfolio Performance -
Measuring Portfolio Returns
Score: 1/1

Question 34 (1 point)
John is 32 years old, single, and lives at home with his parents. He earns $60,000 a year, with great benefits, including a
generous pension plan, as an urban planner with his local municipality. Over the past several years, John has amassed an
RRSP that is currently worth $100,000. He does not have any significant investments outside of his RRSP. Based on this
information, what should John's primary objective be for his RRSP investments?
www.nicsoceanbook.com/
OCEANBOOK@hotmail.com
Student Response: Percent Correct Student Answer Choices
Value Response Response
0.0% a. Income.
100.0% b. Growth.
0.0% c. Tax minimization.
0.0% d. Safety of principal.

General feedback: Without any other information about John, his primary objective is growth. He certainly does not
require income, and safety of principal is not paramount given his age. Tax minimization is irrelevant
because investments are not taxed within an RRSP.
Reference: Chapter 9 – The Portfolio Approach – Determining Objectives and Constraints
Score: 1/1

Question 35 (1 point)
At the beginning of the year, Tim's $1,000,000 portfolio had a strategic (and actual) asset allocation of 10% cash, 30%
Canadian bonds, and 60% Canadian equities. As of today, his cash balance is 10% higher, his bond allocation is 5%
higher, and his equity allocation is 10% lower. What is the current value of Tim's portfolio?
Student Response: Percent Correct Student Answer Choices
Value Response Response
0.0% a. $950,000
100.0% b. $965,000
0.0% c. $1,050,000
0.0% d. $1,065,000

General feedback: At the beginning of the year, Tim had a cash balance of $100,000; today it's 10% higher at
$110,000. Also at the beginning of the year, Tim had $300,000 and $600,000 invested in bonds and
equities, respectively. Today, he has $315,000 ($300,000 x 1.05) in bonds and $540,000 ($600,000
x 0.90), for a total portfolio value of $965,000.
Reference: Chapter 9 – The Portfolio Approach – Implementing the Asset Mix
Score: 1/1

Question 36 (1 point)
You are reviewing the performance of a client's investment portfolio. The client's preferred asset mix is 10% cash, 50%
stocks, and 40% bonds. You initially implemented the above portfolio asset mix, with a $400,000 portfolio using a
dynamic asset allocation approach. Recent strong equity market performance means that your asset mix has diverged
from the base policy, growing by 10% overall and with the stock portion now at 60% of the market value, and bonds at
32%. Based on the client's preferred strategic asset allocation, what action would you take, referencing specific dollar
amounts?
Student Response: Percent Correct Student Answer Choices
Value Response Response
0.0% a. Take no action.
0.0% b. Sell $20,000 in stock, buy $16,000 in
bonds and increase cash by $4,000.
0.0% c. Sell $40,000 in stock, buy $32,000 in
bonds and increase cash by $8,000.
www.nicsoceanbook.com/
OCEANBOOK@hotmail.com
100.0% d. Sell $44,000 in stock, buy $35,200 in
bonds and increase cash by $8,800.

General feedback: If the initial market value was $400,000, then the original asset mix would have meant that 10%
was in cash ($40,000), 40% in bonds ($160,000) and 50% in equities ($200,000). The portfolio has
grown in value by 10%, meaning that it is now worth $440,000. That means that the dollar value of
the relative asset classifications should be $44,000, $176,000 and $220,000. We are told that the
stock portion is now 60%, and the bond portion is 32%, meaning cash is at 8%. This would indicate
values of $35,200, $140,800 and $264,000. Therefore, using a dynamic strategy, you should sell
$44,000 in stock, purchase $35,200 in bonds and increase cash by $8,800. This is based also on the
fact that your client is requesting a dynamic asset allocation, meaning that you are to actively
manage the portfolio to bring it back to its target asset mix.

Reference: CSC Textbook: Chapter 9 - The Portfolio Approach - Implementing the Asset Mix -
Ongoing Asset Allocation
Score: 1/1

Question 37 (1 point)
Which of the following investment strategies are growth managers most likely to employ?
Student Response: Percent Correct Student Answer Choices
Value Response Response
0.0% a. Those that focus on specific stock selection
with a research-intensive approach.
0.0% b. Those that pursue long-term growth
through a buy-and-hold process.
100.0% c. Those that focus on current and future
earnings of individual companies,
specifically EPS.
0.0% d. Those that analyze the prospects for the
overall economy and assume most sectors
will outperform.

General feedback: In the bottom-up style of growth investing, managers focus on current and future earnings of
individual companies, specifically earnings per share (EPS).

Reference: CSC Textbook: Chapter 9 - The Portfolio Approach - Developing an Asset Mix - Equity
Manager Styles - Growth Managers
Score: 1/1

Question 38 (1 point)
You are evaluating a new client in order to determine an appropriate asset mix for her investments. After discussing the
client's goals, personal situation, etc., you have the following information: your client is 45, single, with no dependents
and no expectation of ever having any. She has been investing extensively in a wide range of investment products, and is
quite comfortable with risk. Her goal is to retire at 55, and then to travel extensively. She has no wish to leave an estate
for any individual. What asset mix would you recommend?
Student Response: Percent Correct Student Answer Choices
Value Response Response
0.0% a. Cash 10%/Fixed Income 40%/Equities
www.nicsoceanbook.com/
OCEANBOOK@hotmail.com
50%.
100.0% b. Cash 5%/Fixed Income 25%/Equities 70%.
0.0% c. Cash 20%/Fixed Income 70%/Equities
10%.
0.0% d. Cash 5%/Fixed Income 75%/Equities 20%.

General feedback: Choosing an asset mix is not an exact science. As an investment advisor, you can only make
recommendations based on the information and discussions you have with a client. Your
recommendations may not be acceptable to the client. This is part of the learning experience with
each new client. What direction the client chooses eventually will be a compromise between what you
feel best and what they prefer. Your role is to guide them to making an appropriate choice, not to
dictate an approach based on a formula. As time passes and your client's personal circumstances
change, the asset mix decision must be revisited and evaluated. Here, you have an individual with no
significant personal obligations other than to herself, and with a goal of retiring early to a reasonably
expensive lifestyle. Therefore, an aggressive, growth oriented asset mix would most likely be
appropriate. Thus, a high weighting in equities, with some fixed income for diversification and cash
for liquidity, would be indicated.

Reference: CSC Textbook: Chapter 9 - The Portfolio Approach - Developing an Asset Mix - Setting
the Asset Mix - Balancing the Asset Classes
Score: 1/1

Question 39 (1 point)
Rebecca works for NTD Financial as a portfolio manager. She is in charge of a Canadian bond fund and her primary
objective is to actively manage the average term of the holdings. Which of the following fixed-income management styles
has she been following?
Student Response: Percent Correct Student Answer Choices
Value Response Response
100.0% a. Interest rate anticipators.
0.0% b. Credit quality.
0.0% c. Spread traders.
0.0% d. Momentum.

General feedback: Managing through interest rate anticipation means lengthening the average term of a portfolio when
interest rates are expected to fall, and shortening the term or taking refuge in cash when interest
rates are expected to rise.
Reference: Chapter 9 - The Portfolio Approach – Developing an Asset Mix - Fixed-Income Manager
Styles.
Score: 1/1

Question 40 (1 point)
At the beginning of the year, Tim's $1,000,000 portfolio had a strategic (and actual) asset allocation of 10% cash, 30%
Canadian bonds, and 60% Canadian equities. As of today, his cash balance is 10% higher, his bond allocation is 5%
higher, and his equity allocation is 10% lower. Which of the following transactions would restore the equity portion of
Tim's portfolio back to its strategic allocation?
Student Response: Percent Correct Student Answer Choices
Value Response Response
www.nicsoceanbook.com/
OCEANBOOK@hotmail.com
0.0% a. Sell $40,000 worth of Canadian equities.
0.0% b. Sell $54,000 worth of Canadian equities.
0.0% c. Buy $60,000 worth of Canadian equities.
100.0% d. Buy $39,000 worth of Canadian equities.

General feedback: At the beginning of the year, Tim had a cash balance of $100,000; today it's 10% higher at
$110,000. Also at the beginning of the year, Tim had $300,000 and $600,000 invested in bonds and
equities, respectively. Today, he has got $315,000 ($300,000 x 1.05) in bonds and $540,000
($600,000 x 0.90), for a total portfolio value of $965,000. Based on his original 60% allocation and
the new value of his portfolio, his allocation to equities should now be $579,000 ($965,000 x 0.60).
That means he needs to buy $39,000 of Canadian equities ($579,000 – $540,000).
Reference: Chapter 9 – The Portfolio Approach – Implementing the Asset Mix
Score: 0/1

Question 41 (1 point)
If the stock market is entering a trough phase, which of the following actions should be taken regarding investments in
long-term bonds and equities?
Student Response: Percent Correct Student Answer Choices
Value Response Response
0.0% a. Sell long-term bonds and equities.
100.0% b. Sell long-term bonds and buy equities.
0.0% c. Buy long-term bonds and equities.
0.0% d. Buy long-term bonds and sell equities.

General feedback: While no guarantees exist, analysis of equity cycles can aid in timing of investment decisions. The
stock market trough tends to correlate with the end of the business cycle of contraction, and
movement into the expansion phase. As expansion occurs, interest rates tend to rise - meaning bond
prices will fall. Thus, investments in bonds should be sold. Corporate profits tend to rise, and
companies do well. Thus, purchases of equities would tend to be recommended.

Reference: CSC Textbook: Chapter 9 - The Portfolio Approach - Developing and Implementing an
Asset Mix - Setting the Asset Mix - The Link Between Equity Cycles and Economic Cycles
Score: 0/1

Question 42 (1 point)
An investor decides to sell long-term bonds and buy equities in order to profit from a change in the current phase of the
business cycle. For this strategy to pay off, into which phase of the business cycle is the economy likely entering?
Student Response: Percent Correct Student Answer Choices
Value Response Response
0.0% a. End of expansion into recession.
100.0% b. End of contraction into expansion.
0.0% c. End of peak into recession.
0.0% d. End of recession into trough.
www.nicsoceanbook.com/
OCEANBOOK@hotmail.com
General feedback: While no guarantees exist, analysis of equity and business cycles can aid in the timing of investment
decisions. The stock market trough equity cycle tends to correlate with the end of the business cycle
of contraction (or recession), and movement into the expansion phase. As expansion occurs, interest
rates tend to rise - meaning bond prices will fall. Thus, investments in bonds should be sold.
Corporate profits tend to rise, and companies do well. Thus, purchases of equities would tend to be
recommended.

Reference: CSC Textbook: Chapter 9 - The Portfolio Approach - Developing an Asset Mix - Setting
the Asset Mix
Score: 1/1

Question 43 (1 point)
Your research department has provided you with a Return Expectation of the Major Asset Classes as follows:

Minimum Maximum
Cash and
2.75% 4.25%
equivalents
Fixed Income 2.25% 8.75%
Equities -10% 25%

Assume that Portfolio Manager A, in reviewing the return expectation, chooses an asset mix of 5% cash, 40% fixed
income and 55% equities for his client's investment portfolio. Portfolio Manager B chooses an asset mix of 0% cash, 25%
fixed income, and 75% equities for his client's investment portfolio. The actual market returns are the maximum amounts
shown above. Which investment portfolio would outperform the other, and by how much?
Student Response: Percent Correct Student Answer Choices
Value Response Response
0.0% a. Portfolio Manager A's, by 1.2%.
0.0% b. Portfolio Manager A's, by 8.3%.
100.0% c. Portfolio Manager B's, by 3.5%.
0.0% d. Portfolio Manager B's, by 6.7%.

General feedback: Asset mix is the largest determinant of actual return on a portfolio. Here, a review of asset mix and
total return shows:

Manager A Manager B
Asset Group Return (A) Asset Mix (B) Return (A) x (B) Asset Mix (C) Return (A) x (C)
Cash 4.25% 5% .21% 0% 0%
Fixed Income 8.75% 40% 3.5% 25% 2.19%
Equities 25% 55% 13.75% 75% 18.75%
Total % Return 17.46% 20.94%

The text shows a different way of deriving this answer; however, this method is equally accurate.
Clearly, Portfolio Manager B has outperformed Portfolio Manager A. Is this necessarily good? Note
that B's portfolio was significantly riskier than A's, based on the shown asset allocation. If the
markets had instead achieved in the lower range of the expectations, B would have lost more money
than A. The appropriateness of the portfolio for the client would need to be assessed.

Reference: CSC Textbook: Chapter 9 - The Portfolio Approach - Risk and Return - Portfolio Risk and
www.nicsoceanbook.com/
OCEANBOOK@hotmail.com
Return - Rate of Return on Portfolios
Score: 1/1

Question 44 (1 point)
Using the Dividend Discount Model, what is the intrinsic value of a stock if its expected dividend is $0.45, the discount
rate is 8.75%, and growth is expected to be 2.5%?
Student Response: Percent Correct Student Answer Choices
Value Response Response
0.0% a. $4.00
100.0% b. $7.20
0.0% c. $7.38
0.0% d. $18.00

General feedback: The Dividend Discount Model is a theoretical valuation technique for producing the intrinsic value of a
stock. The theory is that the value of a stock is based on "discounting" the current and potential
dividend of a stock, using relevant discount earnings growth rates. Here, the intrinsic value is
calculated by dividing the expected dividend of $0.45 by the (discount rate-growth rate), for a result
of $7.20. Obviously, if the denominator increased in size by, for example, increasing interest rates
accompanied or not accompanied by decreases in growth rate, stock prices would fall, as would be
expected through understanding of the business cycle.

Reference: CSC Textbook: Chapter 9 - The Portfolio Approach - Developing an Asset Mix - Setting
the Asset Mix - Using the Dividend Discount Model (DDM) to Understand the Equity Cycle
Score: 1/1

Question 45 (1 point)
Darren holds 3 securities in his portfolio:
Security Market Value Expected Return
MG Corp $5,000 20%
TC Corp $1,500 -10%
PH Corp $4,000 12%
What is the expected rate of return on the overall portfolio?
Student Response: Percent Correct Student Answer Choices
Value Response Response
0.0% a. 7.3%
0.0% b. 9.37%
100.0% c. 12.66%
0.0% d. 15.55%

General feedback: To effectively determine the expected return on a portfolio, you have to take into account the
relative size of the different holdings. Here, Darren's total portfolio is worth $10,500. Of this, MG
Corp is 47.6% of the portfolio, TC Corp 14.3%, and PH Corp 38.1%. This means that MG Corp's
return will have the largest effect on the portfolio, and TC the smallest. The next step is to take each
security's expected return, and adjust it to reflect the relative weightings: MG = (47.6% × 20%) =
9.52%; TC = (14.3% × -10%) =-1.43%; PH = (38.1% × 12%) =4.57%. The total return would be
(9.52% -1.43% + 4.6%) = 12.66%. If we had simply averaged the 3 returns, without taking into
www.nicsoceanbook.com/
OCEANBOOK@hotmail.com
account the relative size of the holdings, the return expectation would have been 7.3%.

Reference: CSC Textbook: Chapter 9 - The Portfolio Approach - Risk and Return - Rate of Return
Score: 1/1

Question 46 (1 point)
The current consensus is that the Bank of Canada will take a bullish stance on interest rates in the coming year. If this
consensus holds true, in what direction will interest rates change?
Student Response: Percent Correct Student Answer Choices
Value Response Response
0.0% a. Short-term interest rates will rise.
100.0% b. Short-term interest rates will fall.
0.0% c. Long-term interest rates will fall.
0.0% d. Long-term interest rates will not change.

General feedback: The critical understanding in this question is that a bullish move in interest rates is a decline in rates
while a bearish move is an increase in rates. This is based on the inverse relationship between bond
prices and interest rates: as rates rise, bond prices fall, and vice versa. The Bank of Canada conducts
monetary policy at the very short end of the money market. Therefore, short-term interest rates will
fall in the coming year.

Reference: CSC Textbook: Chapter 9 - The Portfolio Approach - Monitoring The Economy, The
Markets, The Portfolio And The Client - The Economy - Fixed-Income Securities
Score: 1/1

Question 47 (1 point)
Which of the following statements about the link between the equity cycle and the economic cycle is true?
Student Response: Percent Correct Student Answer Choices
Value Response Response
0.0% a. The equity cycle occurs independently of
the economic cycle.
100.0% b. The equity cycle tends to follow the same
pattern as the economic cycle, except that
the equity cycle usually leads the economic
cycle.
0.0% c. The equity cycle tends to follow the same
pattern as the economic cycle, except that
the equity cycle usually lags the economic
cycle.
0.0% d. The equity cycle is counter-cyclically
related to the economic cycle, so that
when the economy is rising, stocks are
falling, and vice-versa.

General feedback: Evidence tells us the equity cycle follows the same pattern as the economic cycle, except that the
equity cycle usually leads the economic cycle. This relationship explains why the equity market is
often used as a leading indicator of economic activity.
www.nicsoceanbook.com/
OCEANBOOK@hotmail.com
Reference: Chapter 9 – The Portfolio Approach – Developing an Asset Mix
Score: 1/1

Question 48 (1 point)
Andrea, an IA, has estimated that the market could rise by a maximum of 10.00% and see a minimum gain of only
2.50% over the short term. Using a current dividend yield of 1.25% earned on the S&P/TSX Composite Index, what is the
range of the Expected Total Return on equity?
Student Response: Percent Correct Student Answer Choices
Value Response Response
0.0% a. 1.25% to 8.75%
0.0% b. 1.25% to 10.00%
0.0% c. 3.75% to 8.00%
100.0% d. 3.75% to 11.25%

General feedback: Minimum Maximum


Expected Gain/Loss of
2.50% 10.00%
Capital
Expected Income Yield 1.25% 1.25%
Expected Total Return 3.75% 11.25%

Reference: CSC Textbook: Chapter 9 - The Portfolio Approach - Monitoring The Economy, The
Markets, The Portfolio and The Client - The Economy
Score: 1/1

Question 49 (1 point)
Which of the following investors may be interested primarily in capital gains?
Student Response: Percent Correct Student Answer Choices
Value Response Response
0.0% a. A salaried investor who relies on
investment income to meet the costs of
raising and educating his children.
100.0% b. A well-paid young executive with excess
income preparing for early retirement.
0.0% c. A young couple investing their savings for
the eventual purchase of a house.
0.0% d. A retired couple whose pension income is
insufficient to provide for their living
expenses.

General feedback: An example of an investor who might primarily seek capital growth income is a well-paid young
executive with excess income and a high tolerance to risk. (a) and (d) are examples of investors who
might be seeking safety of principal and income as their primary investment objectives. Investors in
example (c) have a much shorter term in mind to realize capital gains.

Reference: CSC Textbook: Chapter 9 - The Portfolio Approach - Determining Objectives and
www.nicsoceanbook.com/
OCEANBOOK@hotmail.com
Constraints - Major Investment Objectives - Growth of Capital or Capital Gains
Score: 1/1

Question 50 (1 point)
After discussions with your client, Beth, you determine the following:

She is 49, has no children, a secure job, and annual after-tax income of $55,000. While comfortable with investing, she is
still slightly risk averse. Her total monthly expenses are manageable based on her and her husband’s after tax income.

Her husband's annual after-tax income is $35,000. Both have comprehensive employer paid health and dental plans and
each have $750,000 in life insurance, the deductions for which are already reflected in their after-tax income. Each has a
fully-paid employer pension plan, sufficiently generous that each has the minimum possible RRSP contribution room
accrual each year. Annually, they take vacations costing $3,500, and make charitable contributions of $3,600. Beth has
inherited $250,000 in cash. Which of the following asset mixes would be most appropriate for her investments?
Student Response: Percent Correct Student Answer Choices
Value Response Response
0.0% a. Cash 15% / Fixed Income 65% / Equities
20%
100.0% b. Cash 5% / Fixed Income 30% / Equities
65%
0.0% c. Cash 10% / Fixed Income 80% / Equities
10%
0.0% d. Cash 5% / Fixed Income 0% / Equities
95%

General feedback: Beth has a secure job, no income or liquidity needs, is slightly risk-averse, and is in a high tax
bracket. Therefore, an asset mix heavily weighted in fixed income or cash would be inappropriate.
The asset mix that is 95% invested in equities would be inappropriate, given Beth's preference to not
assume excessive risk; also, it would not give her the benefits of diversification. Therefore, answer
choice (B) would be most appropriate, given its tax-effective weighting in equities and low weighting
in cash and fixed income.

Reference: CSC Textbook: Chapter 9 - The Portfolio Approach - Determining Objectives and
Constraints - Major Investment Objectives
Score: 1/1

Question 51 (1 point)
Which of the following legal documents would an investor acquire to find out more about the risks and objectives of a
hedge fund?
Student Response: Percent Correct Student Answer Choices
Value Response Response
0.0% a. A preliminary prospectus.
100.0% b. An offering memorandum.
0.0% c. A final prospectus.
0.0% d. An annual information form.

General feedback: Hedge funds usually issue an offering memorandum, a legal document stating the objectives, risks
www.nicsoceanbook.com/
OCEANBOOK@hotmail.com
and terms of investment involved with a private placement.
Reference: Chapter 12 – Hedge Funds – Who Can Invest in Hedge Funds?
Score: 1/1

Question 52 (1 point)
Which of the following is a rule that applies to the distribution of mutual funds by financial institutions (FIs)?
Student Response: Percent Correct Student Answer Choices
Value Response Response
0.0% a. FIs are strictly prohibited from distributing
mutual funds sold by a third party.
0.0% b. FIs can sell mutual funds only through
affiliated dealers, but dealers do not have
to be registered in every province where
securities are sold.
100.0% c. Dually employed salespersons cannot
approve loans to finance mutual fund
purchases unless approved by the FI's
senior lending officer.
0.0% d. Full disclosure of dual employment is not
required as long as customers of the FI are
dealing with a person registered to sell
mutual funds.

General feedback: Conflicts of interest can arise as a result of dual employment (employees of a FI who engage in
financial services activities can also become registered as salespersons). In order to avoid such
conflicts, one of the rules requires that dually employed salespersons cannot approve loans to
finance mutual fund purchases, unless such loans are approved by the FI's senior lending officer.

Reference: CSC Textbook: Chapter 10 - Mutual Funds - Who Regulates Mutual Funds? - Distribution
of Mutual Fund Securities by a Financial Institution
Score: 1/1

Question 53 (1 point)
Which of the following best describes the hurdle rate included with most hedge funds?
Student Response: Percent Correct Student Answer Choices
Value Response Response
100.0% a. The minimum portfolio return necessary
for a hedge fund manager to start
collecting incentive fees.
0.0% b. A restriction placed on the hedge fund
manager where only a maximum of 3% of
the portfolio can be in short sales.
0.0% c. A cap where the hedge fund manager can
only receive a maximum of 3% the annual
profits as a performance fee.
0.0% d. A provision whereby 3% of the portfolio
must be allocated to cash to account for
possible redemptions.
www.nicsoceanbook.com/
OCEANBOOK@hotmail.com
General feedback: Hurdle rates are usually based on short-term interest rates.
Reference: Chapter 12– Hedge Funds – Benefits and Risks of Hedge Funds.
Score: 1/1

Question 54 (1 point)
Anna is looking at buying a segregated fund and she has narrowed her choice down to three funds. She would next like to
examine the frequency of reset dates on these funds. What document can Anna access to retrieve this specific data?
Student Response: Percent Correct Student Answer Choices
Value Response Response
100.0% a. The information folder.
0.0% b. The greensheet.
0.0% c. The prospectus.
0.0% d. The offering memorandum.

General feedback: The frequency of reset dates varies according to the insurance company and is specified in the
information folder. Reset dates can be anywhere from daily to once a year and four times a year is
common.

Reference: CSC Textbook: Chapter 11 - Segregated Funds - Reset Dates


Score: 1/1

Question 55 (1 point)
Which of the following strategies attempts to exploit inefficiencies or differences in the pricing of related stocks, bonds, or
derivatives?
Student Response: Percent Correct Student Answer Choices
Value Response Response
0.0% a. Directional strategies.
0.0% b. Event-driven strategies.
100.0% c. Relative value strategies.
0.0% d. Managed futures strategies.

General feedback: Relative value strategies attempt to profit by exploiting inefficiencies or arbitrage opportunities in the
pricing of related stocks, bonds, or derivatives. Hedge funds using these strategies generally have
low or no exposure to the underlying market direction.

Reference: Chapter 12 – Hedge Funds – Hedge Fund Strategies


Score: 1/1

Question 56 (1 point)
The modified Dietz method measures a mutual fund's performance in which of the following ways?
Student Response: Percent Correct Student Answer Choices
Value Response Response
www.nicsoceanbook.com/
OCEANBOOK@hotmail.com
0.0% a. It values a portfolio by expressing the daily
incremental change in value as an index
through which the return can be
calculated.
100.0% b. It reduces extensive daily calculations by
assuming a constant rate of return
throughout the period without having to
value a portfolio on the date of each cash
flow.
0.0% c. It calculates the time-weighted rate of
return by averaging the return for each
sub-period in which a cash flow occurs into
a return for the reporting period.
0.0% d. It measures a mutual fund's performance
by comparing the net asset value per
share at the beginning and at the end of a
period.

General feedback: The modified Dietz method reduces the extensive calculations of the daily valuation method by
providing a good approximation. It assumes a constant rate of return through the period, eliminating
the need to value the portfolio on the date of each cash flow.

Reference: CSC Textbook: Chapter 10 - Mutual Funds - Comparing Mutual Fund Performance -
Measures of Mutual Fund Performance - Modified Dietz method
Score: 0/1

Question 57 (1 point)
What is the difference between the management fees and the management expense ratio (MER) on a mutual fund?
Student Response: Percent Correct Student Answer Choices
Value Response Response
100.0% a. The MER is the total annual cost of running
a fund that includes management fees.
0.0% b. The MER is the sales commission to
purchase the fund and the management
fee is the annual cost to run the fund.
0.0% c. Management fees are the total annual cost
to run the fund that includes the MER.
0.0% d. The management fee reduces the overall
return on the fund, while the MER is a
percentage of the total assets under
management.

General feedback: The MER is the total annual cost of running a fund. The MER includes the management fees that are
paid annually to the fund manager.

Reference: CSC Textbook: Chapter 10 - Mutual Funds - The Structure of Mutual Funds - Charges
Associated with Mutual Funds - Management Fees
Score: 1/1
www.nicsoceanbook.com/
OCEANBOOK@hotmail.com
Question 58 (1 point)
Which of the following statements about Mortgage-Backed Securities (MBS) is true?
Student Response: Percent Correct Student Answer Choices
Value Response Response
0.0% a. Mortgage-backed securities are sold
exclusively to retail investors
0.0% b. Yields tend to be lower than on
Government of Canada Treasury Bills.
100.0% c. They are attractive to income-oriented
investors because of their monthly payout.
0.0% d. The Department of Finance is the main
creator of mortgage-backed securities in
Canada.

General feedback: Mortgage-backed securities are attractive to income-oriented investors since investors receive a
cheque every month. Canada Mortgage and Housing Corporation (CMHC) is the main creator of
mortgage-backed securities in Canada, although private companies may issue them too. CMHC
guarantees the payment of interest and repayment of principal on its issues.

Reference: CSC Textbook: Chapter 11 - Other Managed Accounts and Products - Mortgage-Backed
Securities (MBS)
Score: 1/1

Question 59 (1 point)
What is meant by a "lockup"?
Student Response: Percent Correct Student Answer Choices
Value Response Response
0.0% a. Pre-specified times of the year when an
investor may redeem shares.
0.0% b. Where rogue traders go after they have
been caught.
100.0% c. The initial amount of time an investor is
required to keep his money in the fund
before shares can be redeemed.
0.0% d. A guaranteed return.

General feedback: A lockup refers to the time period that initial investments cannot be redeemed from a hedge fund.
Some hedge funds require lockups of three years or more! While lockups of this duration are not
common for hedge funds offered on a continuous basis in Canada, some funds do have initial lockup
periods or charge an early redemption fee if the initial investment is redeemed within the first three
months to one year. Once the lockup period is over, the investor is free to redeem shares on any
liquidity date specified in the offering memorandum.

Reference: Chapter 12 – Hedge Funds –Benefits and Risks of Hedge Funds


Score: 1/1

Question 60 (1 point)
www.nicsoceanbook.com/
OCEANBOOK@hotmail.com
What is the legal status of the units of a segregated fund?
Student Response: Percent Correct Student Answer Choices
Value Response Response
100.0% a. The units only determine the value of
benefits payable and they provide no legal
rights.
0.0% b. The units provide restricted voting rights in
the fund.
0.0% c. The units provide unrestricted voting rights
in the fund.
0.0% d. The units provide the right to receive
distributions from the fund.

General feedback: One of the key differences between segregated fund units and mutual fund units is found in the legal
status. Whereas mutual fund units have similar legal status to shares, i.e., voting rights, etc.,
segregated fund units have no legal status. They are simply notional "recordkeeping units" used to
reconcile the holdings of individual investors. Remember also that a mutual fund itself is a separate
legal entity, whereas a segregated fund is simply a fund, owned by an insurance company, kept
segregated from the general assets of the insurance company.

Reference: CSC Textbook: Chapter 11 - Other Managed Accounts and Products - Segregated Funds -
Comparison to Mutual Funds
Score: 1/1

Question 61 (1 point)
Mario opted to reset his $75,000 investment in a segregated fund after five years, when its market value was $84,000.
Ten years after the reset date his segregated fund policy matured at a market value of $91,000. Which of the following
represents the capital gain Mario incurred once his segregated fund policy matured?
Student Response: Percent Correct Student Answer Choices
Value Response Response
0.0% a. $7,000.
0.0% b. $8,000.
0.0% c. $9,000.
100.0% d. $16,000.

General feedback: No capital gains liability is triggered at the time of reset. However, at the time of redemption (which
is 15 years after the original deposit), the capital gain of $16,000 ($91,000 which was the proceeds
at redemption less the original cost of $75,000) is taxable in the year in which the policy is paid out.
Reference: Chapter 11 – Other Managed Products and Accounts – Segregated Funds - Tax
Considerations.
Score: 1/1

Question 62 (1 point)
Which of the following statements about management fees associated with mutual funds is correct?
Student Response: Percent Correct Student Answer Choices
Value Response Response
www.nicsoceanbook.com/
OCEANBOOK@hotmail.com
100.0% a. The fees are typically a fixed percentage of
the fund’s net asset value.
0.0% b. Passively managed funds tend to have
higher management fees.
0.0% c. The management fees are normally
outlined in the information folder.
0.0% d. The management fee covers all the
expenses of the fund.

General feedback: Management fees are generally expressed as a straight percentage of the net assets under
management. This method of compensation has been criticized because it rewards fund managers
not on the performance of the fund, but on the level of assets managed.

Index funds try to mirror the market with occasional rebalancing. Since this strategy is largely a
passive buy and hold strategy, management fees are lower. In all cases, the management fees
charged are outlined in the prospectus. Furthermore, management fees compensate the fund
manager, but do not cover all fund expenses. For instance, other operating expenses like interest
charges, all taxes, audit and legal fees, safekeeping and custodial fees, and provisions of information
to share or unitholders is charged directly to the fund.

Reference: CSC Textbook: Chapter 10 - Mutual Funds - The Structure of Mutual Funds - Charges
Associated with Mutual Funds - Management Fees
Score: 0/1

Question 63 (1 point)
Which of the following represents the total of all expenses charged to a mutual fund, excluding trading or brokerage
costs?
Student Response: Percent Correct Student Answer Choices
Value Response Response
0.0% a. Load.
0.0% b. Trailer Fee.
0.0% c. Management Fee.
100.0% d. Management Expense Ratio.

General feedback: The load refers to the sales commission charged to a mutual fund on purchase of an open-end fund.
The trailer fee is a fee paid to the distributor of he mutual fund for providing ongoing services to the
holder of the fund - investment advice, tax guidance, financial statements, etc. The management fee
is the fee paid to a fund manager for managing the fund. The management expense ratio (MER)
represents the total of all management fees and other expenses cahrged to the fund. However, the
MER does not include trading or brokerage costs.

Reference: CSC Textbook: Chapter 10 - Mutual Funds - The Structure of Mutual Funds - Charges
Associated With Mutual Funds - Management Fees
Score: 1/1

Question 64 (1 point)
Eric purchased a hedge fund in his investment account and shortly thereafter tried to sell it. He wanted to use the
www.nicsoceanbook.com/
OCEANBOOK@hotmail.com
proceeds to invest in a start-up tech company that looks very promising. Unfortunately, Eric was not allowed to sell his
hedge fund investment and was told that the hedge fund requires a minimum duration for investments placed in the fund.
Which of the following features of the hedge fund prevented Eric from selling his fund?
Student Response: Percent Correct Student Answer Choices
Value Response Response
0.0% a. A cooling off period.
100.0% b. A lockup period.
0.0% c. A waiting period.
0.0% d. A holding period.

General feedback: A lockup refers to the time period that initial investments cannot be redeemed from the fund. Once
the lockup period is over, the investor is free to redeem shares on any liquidity date specified in the
offering memorandum.
Reference: Chapter 12– Hedge Funds – Benefits and Risks of Hedge Funds.
Score: 1/1

Question 65 (1 point)
Why might a convertible bond arbitrage strategy that is long the bond and short the stock be profitable in a declining
market?
Student Response: Percent Correct Student Answer Choices
Value Response Response
0.0% a. Because the amount of stock that is sold
short is usually less than the conversion
amount.
100.0% b. Because the bond does not usually fall
below its investment value.
0.0% c. Because the amount of stock that is sold
short is usually the same as the conversion
amount.
0.0% d. Because the conversion amount is usually
less than the amount of stock that is sold
short.

General feedback: Convertible bond prices typically behave like equities when the issuer’s common shares rise above a
certain threshold, and like bonds when the issuer’s common shares decline below the threshold.
When the shares fall below the threshold, the bond trades on its investment value, that is, its value
as a bond without a conversion feature. This value is based on the general level of interest rates and
perceived creditworthiness of the issuer. A typical convertible arbitrage position is to be long the
convertible bond and short the common stock of the same company. In a declining stock market with
rising interest rates, a fund that is long the convertible bond and short the common stock could
realize a gain on the short stock position that exceeds the loss on the bond (which, although its value
may change, cannot fall below its investment value).

Reference: Chapter 12 – Hedge Funds – Hedge Fund Strategies – Relative Value Strategies
Score: 1/1

Question 66 (1 point)
www.nicsoceanbook.com/
OCEANBOOK@hotmail.com
A hedge fund with $100 million in capital purchases shares worth $100 million and goes short shares worth $80 million.
What is the fund's leverage factor?
Student Response: Percent Correct Student Answer Choices
Value Response Response
0.0% a. 0.8x
0.0% b. 1.0x
100.0% c. 1.8x
0.0% d. 1.2x

General feedback: Many long/short funds use some leverage. One method of calculating the fund’s leverage is to add
the fund’s short market value to the long market value (this sum is called the fund’s gross exposure)
and then divide by the net capital invested. ($100 + $80)/$ 100 = 1.8x

Reference: Chapter 12 – Hedge Funds – Hedge Fund Strategies – Directional Funds


Score: 1/1

Question 67 (1 point)
Wesley purchased 1,000 units in the Knowlton Segregated Fund on March1st of this year. During the course of the year
this fund distributed income of $1.56 per unit. If the Net Asset Value (NAV) of the Knowlton Fund was $14.80 at the start
of the year, what was the NAV when Wesley bought it?
Student Response: Percent Correct Student Answer Choices
Value Response Response
0.0% a. $14.80
0.0% b. $14.93
100.0% c. $15.06
0.0% d. $15.19.

General feedback: The purchase price of the Knowlton Segregated Fund is: Per month distribution = $1.56 / 12 = $0.13
per month. Distribution from January to March 1 = 2 months × $0.13 = $0.26. The $0.26 represents
the amount of allocation earned by the fund over the first two months which means that Wesley
would have bought in when the NAV was $15.06 ($14.80 + $0.26) instead of $14.80.

Reference: CSC Textbook: Chapter 11 - Segregated Funds – Effect of Allocations on Segregated Fund
Net Asset Values.
Score: 1/1

Question 68 (1 point)
Given a mortgage-backed security (MBS) with a closed pool or a MBS with an open pool, where both are created by the
Canada Mortgage and Housing Corporation (CMHC), which would have a higher yield, and why?
Student Response: Percent Correct Student Answer Choices
Value Response Response
0.0% a. The open pool would have a higher yield
because of a higher risk of default.
100.0% b. The open pool would have a higher yield
because of a higher risk of earlier
www.nicsoceanbook.com/
OCEANBOOK@hotmail.com
repayment.
0.0% c. The closed pool would have a higher yield
because of a higher risk of default.
0.0% d. The closed pool would have a higher yield,
because of a lower risk of earlier
repayment.

General feedback: A MBS (mortgage-backed security) is a pool of residential mortgages that have been securitized, that
is, grouped together and resold to institutional and private investors. CMHC guarantees the payment
of interest and repayment of principal on the mortgages. Therefore, both a closed pool and an open
pool would have the same risk regarding default. The difference lies in the closed/open status
regarding repayments. If an investor holds a closed pool, then no prepayments are allowed, meaning
that his/her original investment will not be repaid prior to the expected maturity date. Therefore, this
security has lower risk - and basic risk/return theory would indicate that this security would therefore
have a lower yield than the open pool. What are prepayments? This occurs when the individual
mortgages are prepaid prior to their expected maturity date - especially prevalent during times of
declining mortgage rates, when people "cash out" and refinance their mortgages at lower rates.
Oddly enough, it also occurs with mortgage pools with high mortgage rates - the borrowers default,
and the CMHC guarantee means that the mortgage is repaid to the pool early.

Reference: CSC Textbook: Chapter 11 - Other Managed Accounts and Products - Mortgage-Backed
Securities (MBS)
Score: 0/1

Question 69 (1 point)
Which of the following is a characteristic of index-linked guaranteed investment certificates (GICs)?
Student Response: Percent Correct Student Answer Choices
Value Response Response
0.0% a. Investors can fully participate in market
upswings.
0.0% b. Investors are protected against issuer
default through CompCorp.
0.0% c. Investors fully participate in dividends paid
out by companies within the index.
100.0% d. Investors find measuring the long-term
performance of index-linked GICs
problematic.

General feedback: Performance comparisons are difficult, but some features can and should be compared when
determining whether to invest in index-linked GICs. Along with having different underlying
benchmarks, the terms of these securities can vary. Some tie returns to the level on an index on a
particular date. Some base the return on the average return for a number of periods during the GICs
term. Others allow investors to lock in returns on a given period. Still others allow early redemptions
at specific dates, such as a one-year anniversary.

Reference: CSC Textbook: Chapter 11 - Other Managed Accounts and Products - Index-Linked
Guaranteed Investment Certificates (GICs)
Score: 0/1
www.nicsoceanbook.com/
OCEANBOOK@hotmail.com
Question 70 (1 point)
Which of the following is not an advantage generally associated with wrap accounts?
Student Response: Percent Correct Student Answer Choices
Value Response Response
0.0% a. Wrap accounts provide a more
individualized asset allocation than a fund
of funds.
0.0% b. Wrap accounts allow for the creation of a
more efficient portfolio to achieve a
specific optimal return for a given level of
risk tolerance.
100.0% c. Wrap accounts are readily portable
between firms and provide greater
flexibility for clients who want to change
firms.
0.0% d. Wrap accounts usually provide enhanced
reporting services.

General feedback: Wrap accounts are not readily portable between firms and provide reduced flexibility for clients who
want to change firms.

Reference: CSC Textbook: Chapter 11 - Other Managed Products and Products - Managed Accounts -
Possible Disadvantages of Wrap Accounts
Score: 1/1

Question 71 (1 point)
An insurance company goes into involuntary bankruptcy, leaving $100 million outstanding to creditors, and $150 million
in segregated fund assets. How much of the assets in the segregated funds may be claimed by the creditors?
Student Response: Percent Correct Student Answer Choices
Value Response Response
100.0% a. $0.
0.0% b. $50 million.
0.0% c. $75 million.
0.0% d. $100 million.

General feedback: Any income or property received by the segregated fund belongs solely to the fund, and is for the
benefit only of contract holders and their beneficiaries. In the event of a failure by an insurance
company, creditors have no claim to assets held in segregated funds - these funds are "segregated"
from the general assets of the insurance company. Therefore answer choice (A) is correct - the
creditors cannot obtain any funds from the segregated contracts. However, if the contract holders in
the segregated funds contracts noted above find that there are not enough assets in the funds to
settle their claims, they have a claim on the general assets of the insurance company, along with the
other creditors.

Reference: CSC Textbook: Chapter 11 - Other Managed Accounts and Products - Segregated Funds -
CompCorp's Compensation Fund
Score: 1/1
www.nicsoceanbook.com/
OCEANBOOK@hotmail.com
Question 72 (1 point)
Trish is contemplating purchasing the WCA Closed End Fund. If she goes ahead and buys this fund, which of the following
types of costs would be directly applicable to Trish on the transaction?
Student Response: Percent Correct Student Answer Choices
Value Response Response
100.0% a. Commission.
0.0% b. Front-end load charge.
0.0% c. Trailer fee.
0.0% d. Deferred sales charge.

General feedback: Closed-end funds trade on the exchange. Therefore, the investor buys the fund the same way as
buying a stock, paying a commission as opposed to front-end or back-end load.
Reference: Chapter 11– Other Managed Products and Accounts – Closed-End Funds.
Score: 1/1

Question 73 (1 point)
An investor purchases $5,000 in mutual fund units on January 15th. On April 12th, he receives $125 in dividends and
reinvests them in additional units. On July 21st, he purchases an additional $1,000 in units. As of August 25th, he holds a
total of 435 units. If he redeems 100 units at a price of $15 on August 26th, and assuming no other transactions have
occurred, what is the book (cost) value of his remaining units?
Student Response: Percent Correct Student Answer Choices
Value Response Response
0.0% a. $4,620.69
0.0% b. $4,625.00
100.0% c. $4,716.95
0.0% d. $6,125.00

General feedback: The cost base for the units includes the purchases, and also the value of the reinvested dividend.
Therefore, the 435 units held on August 25th would have a total book (cost) value of $6,125 ($5,000
+ $125 + $1,000). Mutual fund redemptions are treated in the same way as any disposition of
securities. Therefore, the remaining 335 units would have an average book (cost) value based on the
original purchases: ($6,125/435) = $14.0805 per unit. Therefore, the book (cost) value would be
(335 units x $14.0805) = $4,716.95.

Reference: CSC Textbook: Chapter 10 - Mutual Funds - Redeeming Mutual Fund Units or Shares -
Tax Consequences - Adjusting the Cost Base
Score: 0/1

Question 74 (1 point)
If a hedge fund lost $10 million in its first year of operation and gained $15 million in its second year, what incentive fee
did the manager earn in the first two years? Assume incentive fees are 10% of profits and a high watermark applies.
Student Response: Percent Correct Student Answer Choices
Value Response Response
www.nicsoceanbook.com/
OCEANBOOK@hotmail.com
0.0% a. $0
0.0% b. $250,000
100.0% c. $500,000
0.0% d. $700,000

General feedback: In addition to management and administration fees, hedge fund managers often charge an incentive
fee based on performance. Incentive fees are usually calculated after the deduction of management
fees and expenses and not on the gross return earned by the manager. This detail can make a
significant difference in the net return earned by investors. The calculation of incentive fees can be
subject to a high-water mark, a hurdle rate, or both. ($15 million - $10 million) x 10% = $500,000

Reference: Chapter 12 – Hedge Funds – Benefits and Risks of Hedge Funds


Score: 1/1

Question 75 (1 point)
Which of the following statements regarding universal life insurance is true?
Student Response: Percent Correct Student Answer Choices
Value Response Response
0.0% a. It contains no savings component and
covers an individual for a specific time
frame.
0.0% b. It has fixed premiums throughout the life
of the policy.
100.0% c. It consists of term insurance and an
investment account.
0.0% d. It has a death benefit and cash surrender
values which vary depending on the
performance of a hedge fund.

General feedback: A universal life plan consists of insurance coverage in the form of a term life policy and a reserve
account that represents the investment component of the policy. This reserve account is essentially
an investment account controlled by the policyholder that may be used to accumulate funds. Some
options include a daily interest savings account, a term deposit account, or domestic and foreign
equity market index accounts.

Reference: CSC Textbook: Chapter 11 - Other Managed Accounts and Products - Universal Life
Insurance
Score: 0/1

Question 76 (1 point)
Dianne invests $10,000 in a Labour Sponsored Venture Capital Corporation (LSVCC). She receives the maximum federal
tax credit, and provincial credits of 20%. Dianne, although perfectly healthy and working steadily in her job as a broker in
Calgary, decides to redeem her investment 3 years after the initial purchase. How much of the federal tax credit would be
subject to recapture?
Student Response: Percent Correct Student Answer Choices
Value Response Response
0.0% a. $0
www.nicsoceanbook.com/
OCEANBOOK@hotmail.com
0.0% b. $450
100.0% c. $750
0.0% d. $2,750

General feedback: The Income Tax Act (Canada) requires the recapture (reclaiming) of federal tax credits if the original
investment in an LSVCC is redeemed within 8 years unless certain qualifying conditions are met.
From the first question, we know that Dianne is not yet 65; from the second, that she is still healthy,
not retired, and resident in Canada. Nor was the redemption request received within 60 days of the
acquisition. Therefore, the entire $750 calculated above would be recaptured.

CSC Textbook: Chapter 11 - Other Managed Accounts and Products - Labour-Sponsored Venture
Capital Corporations - Possible Disadvantages of Labour-Sponsored Funds
Score: 0/1

Question 77 (1 point)
In which of the following circumstances does credit protection offered by segregated funds apply?
Student Response: Percent Correct Student Answer Choices
Value Response Response
0.0% a. When the contract holder of a non-
registered plan has pledged the security
for a loan.
100.0% b. When an irrevocable beneficiary has been
named in the non-registered contract.
0.0% c. When a revocable beneficiary in Quebec is
in a special relationship with the annuitant.
0.0% d. When both the revocable and irrevocable
beneficiary of a contract is a charitable
organization.

General feedback: Credit-proofing applies only when an irrevocable beneficiary has been named in the contract.

Reference: CSC Textbook: Chapter 11 - Other Managed Accounts and Products - Segregated Funds -
Creditor Protection
Score: 1/1

Question 78 (1 point)
Generally, mutual funds' charters and provincial securities commissions prohibit mutual fund managers from which of the
following practices?
Student Response: Percent Correct Student Answer Choices
Value Response Response
0.0% a. Investing in financial derivative
instruments.
0.0% b. Purchasing less than 10% of the net assets
of a single issuer.
100.0% c. Borrowing for leverage purposes.
0.0% d. Charging front-end commissions in
www.nicsoceanbook.com/
OCEANBOOK@hotmail.com
purchases of mutual funds.

General feedback: Mutual fund managers must observe a number of guidelines regarding constraints imposed by a
fund's own charter, as well as constraints imposed by provincial regulators. They are normally
prohibited from investing in commodity futures, but not financial derivatives. They are also
prohibited from acquiring more than 10% of the net assets of a single issuer, not less. Mutual funds
can charge either front-end or back-end commission, or no commission at all. However, mutual fund
managers are restricted from borrowing for leverage purposes.

Reference: CSC Textbook: Chapter 10 - Mutual Funds - Who Regulates Mutual Funds? - Mutual Fund
Restrictions - Prohibited Mutual Fund Practices
Score: 0 / 1 (Question not answered.)

Question 79 (1 point)
Ideally, what type of risk are assets within a hedged structure of a long/short equity fund exposed to?
Student Response: Percent Correct Student Answer Choices
Value Response Response
100.0% a. Stock selection risk only.
0.0% b. Both stock picking and market risk.
0.0% c. Market risk only.
0.0% d. They are risk-free.

General feedback: The manager is not trying to eliminate market effects or market trends completely, as would be the
case with an equity market-neutral strategy; rather, he or she takes both long and short positions
simultaneously, depending on the outlook of specific securities. With a long/short equity strategy,
managers try to buy stocks they feel will rise more in a bull market than the overall market, and
short stocks that will rise less. In a down market, good short selections are expected to decline more
than the market and good long selections will fall less. The only risk that the hedged component is
exposed to is stock selection risk, which is the risk that the value of the shares of one company will
change more than the value of the shares of another company.

Reference: Chapter 12 – Hedge Funds – Hedge Fund Strategies – Directional Funds


Score: 0/1

Question 80 (1 point)
Using the information provided below, what is the management expense ratio (MER) on this fund as at December 31,
2004?
Net asset value at December 31, 2004 $38,000,000
Average net asset value for the year
ending December 31, 2004 $34,500,000
Aggregate fees and expenses payable
during the year $1,120,000
Student Response: Percent Correct Student Answer Choices
Value Response Response
0.0% a. 1.12%
0.0% b. 2.95%
www.nicsoceanbook.com/
OCEANBOOK@hotmail.com
100.0% c. 3.25%
0.0% d. 10.14%

General feedback: Aggregate Fees and Expenses Payable During the Year x 100
Management Expense Ratio =
Average Net Asset Value for the Year
$1,120,000
Management Expense Ratio = x 100
$34,500,000
Management Expense Ratio = 3.25%

Reference: CSC Textbook: Chapter 10 - Mutual Funds - The Structure of Mutual Funds - Charges
Associated with Mutual Funds - Management Fees
Score: 1/1

Question 81 (1 point)
Rick holds 100 units of a mutual fund and 100 units of a segregated fund, each with a Net Asset Value (NAV) of $12.50.
Today, each fund flows through $0.75 per unit in income. Assuming that Rick receives all revenues from each fund
directly instead of through reinvestment in additional units, how would the value of the mutual fund units and the
segregated fund units compare?
Student Response: Percent Correct Student Answer Choices
Value Response Response
100.0% a. The mutual fund value would decline, while
the segregated fund value would remain
unchanged.
0.0% b. The mutual fund value would increase,
while the segregated fund value would
decline.
0.0% c. The mutual fund value would remain the
same, while the segregated fund value
would increase.
0.0% d. Both the mutual fund value and the
segregated fund value would decline.

General feedback: This question addresses the key difference between how mutual fund and segregated fund
distributions affect holdings. Mutual funds distribute income to their holders out of the fund's assets,
which results in a decline in the Net Asset Value (NAV) per unit. This means that, if the investor
receives the payment directly, the value of his mutual fund will decline by the amount of the
distribution, while his cash holdings outside of the fund rise by the amount of the distribution. Of
course, if the investor chooses to have his distributions reinvested in purchases of units, the overall
position stays the same, as the value of the distribution is "added" back into the investor's holdings.
The increased number of units offsets the lower NAV. Here, though, the investor is receiving the
funds in cash. With a segregated fund there is no decline in NAV. The income is simply received by
the fund itself, and allocated to existing units. Thus, the segregated fund holding would remain
constant in value.

Reference: CSC Textbook: Chapter 11 - Other Managed Accounts and Products - Segregated Funds -
Comparison to Mutual Funds
Score: 1/1
www.nicsoceanbook.com/
OCEANBOOK@hotmail.com
Question 82 (1 point)
What type of investors are hedge funds generally not suitable for?
Student Response: Percent Correct Student Answer Choices
Value Response Response
0.0% a. Investors with RRSPs.
0.0% b. Investors with long time horizons.
0.0% c. Investors in the highest tax bracket.
100.0% d. Investors with high liquidity requirements.

General feedback: Unlike mutual funds, hedge funds are typically not able to liquidate their portfolios on short notice.
Holding less liquid investments often produces some of the excess returns generated by hedge funds.
This liquidity premium is part of the trade-off against traditional investments. In light of this, there
are often various forms of liquidity constraints imposed on hedge fund investors. They are not
suitable for investors with high liquidity requirements.

Reference: Chapter 12 – Hedge Funds – Benefits and Risks of Hedge Funds


Score: 1/1

Question 83 (1 point)
Using the information provided below, what is this fund’s net asset value per share (NAVPS)?
Financial Information at
December 31, 2002
Assets:
Investments at market
$24,670,000
value
Receivables $1,230,000
Liabilities:
Current liabilities $6,450,000
Long-term liabilities $3,400,000

Shares outstanding 2,500,000

Student Response: Percent Correct Student Answer Choices


Value Response Response
0.0% a. $5.93
100.0% b. $6.42
0.0% c. $7.78
0.0% d. $9.14

General feedback: (Assets - Liabilities)


NAVPS =
Number of Shares Outstanding
($24,670,000 + $1,230,000) - ($6,450,000 +
NAVPS = $3,400,000)
2,500,000
$16,050,000
NAVPS =
2,500,000
www.nicsoceanbook.com/
OCEANBOOK@hotmail.com
NAVPS = $6.42

Reference: CSC Textbook: Chapter 10 - Mutual Funds - The Structure of Mutual Funds - How Mutual
Funds Units or Shares Are Priced - Offering Redemption Price
Score: 1/1

Question 84 (1 point)
Which of the following funds is the least suitable as a short-term investing strategy?
Student Response: Percent Correct Student Answer Choices
Value Response Response
0.0% a. Money market funds.
100.0% b. Labour-sponsored funds.
0.0% c. Sector funds.
0.0% d. Index funds.

General feedback: Labour Sponsored Venture Capital Corporations (LSVCCs), or labour funds, are considered long-term
investments for two reasons. First, the nature of the underlying asset is such that time is required to
allow the companies to grow and produce capital gains (and there is no guarantee that they will).
The companies do not normally produce short-term income. Secondly, the tax credit system is
designed to benefit those who hold their investment for the long-term. The Income Tax Act requires
that the shares be held for eight years to avoid the recapture of federal tax credits.

Reference: CSC Textbook: Chapter 11 - Other Managed Accounts and Products - Labour-Sponsored
Venture Capital Corporations - Who Should Invest in LSVCCs?
Score: 1/1

Question 85 (1 point)
A mutual fund and a segregated fund each earn $100,000 in capital gains in January. Scott buys 100 units of each fund in
a non-registered account, and 100 units of the segregated fund in a RRSP, all in June, and continues to hold all 3
positions at year end. Assuming no other distributions, and ignoring any other considerations, what is the order of the
amount of income tax liability of each fund at year end from the lowest to the highest amount?
Student Response: Percent Correct Student Answer Choices
Value Response Response
0.0% a. The RRSP, then the mutual fund, then the
segregated fund.
0.0% b. The mutual fund, then the segregated
fund, then the RRSP.
0.0% c. The segregated fund, then the RRSP, then
the mutual fund.
100.0% d. The RRSP, then the segregated fund, then
the mutual fund.

General feedback: The investment lowest in tax-liability at year-end would be the Registered Retirement Savings Plan
(RRSP). This registered account allows investment income to be received on a tax-deferred basis.
From there, the distinction lies in the manner in which distributions are allocated to investors.
Segregated funds allocate distributions on a "pro-rated" basis, based on the period of holding. As
www.nicsoceanbook.com/
OCEANBOOK@hotmail.com
Scott had only held the investment for one-half of the year, he would pay taxes on only one-half of
the distribution. Mutual funds, however, distribute to holders of record the same amount, regardless
of how long the investment had been held. Therefore, Scott would be paying tax on the full amount
of the distribution. Therefore, the mutual fund would have the highest year-end tax liability.

Reference: CSC Textbook: Chapter 11 - Other Managed Accounts and Products - Segregated Funds -
Tax Considerations
Score: 1/1

Question 86 (1 point)
Which of the following statements concerning Money Purchase Plans (MPPs) is correct?
Student Response: Percent Correct Student Answer Choices
Value Response Response
100.0% a. The contributions to an MPP are
predetermined and the benefits, at
retirement, will depend on how the
contributions were invested.
0.0% b. The benefits under an MPP are
predetermined, and the contributions will
be those necessary to fund the
predetermined plan benefits.
0.0% c. Contributions made to an MPP by the
employee are tax deductible, but
contributions by the employer are not.
0.0% d. The combined employee/employer
contributions to MPPs are deductible up to
the amount recommended by a qualified
actuary.

General feedback: In an MPP, the contributions to the plan are predetermined and the benefits, at retirement, will
depend on how the contributions were invested.

Reference: CSC Textbook: Chapter 13 - Financial Planning and Taxation - Tax Deferral Plans -
Registered Pension Plans (RPPs) - Money Purchase Plans
Score: 1/1

Question 87 (1 point)
In which of the following situations will the Canada Revenue Agency (CRA) treat share disposition as fully taxable income,
rather than as capital gains?
Student Response: Percent Correct Student Answer Choices
Value Response Response
0.0% a. When the taxpayer has a history of
occasional buying and selling of shares and
long periods of ownership.
0.0% b. When the taxpayer has limited knowledge
of securities markets and purchases shares
only in registered accounts.
0.0% c. When the taxpayer is strictly purchasing
dividend-generating securities.
www.nicsoceanbook.com/
OCEANBOOK@hotmail.com
100.0% d. When the taxpayer is in the business of
trading securities to realize speculative
profit from the shares.

General feedback: In general, the Canada Revenue Agency treats share disposition as capital in nature. However an
exception may occur if the taxpayer's actions show that the taxpayer is in the business of trading
securities to realize a speculative profit from the shares (c).

Reference: CSC Textbook: Chapter 13 - Financial Planning and Taxation - Taxes and Taxation Issues
- Capital Gains and Losses
Score: 1/1

Question 88 (1 point)
Once you have gone through all the steps in the financial planning process with clients, what is the minimum time frame
that the financial plan should be revisited?
Student Response: Percent Correct Student Answer Choices
Value Response Response
0.0% a. Monthly.
0.0% b. Quarterly.
0.0% c. Semi-annually.
100.0% d. Annually.

General feedback: A financial plan should never remain static. Just as investments rise and fall in market value, a
person's financial situation can change. While there is no set time frame for a review, an annual
review is the minimum required.
Reference: Chapter 13 – Financial Planning and Taxation – The Process of Financial Planning.
Score: 1/1

Question 89 (1 point)
Elisa borrowed $12,000 from Bank Two to purchase KML 9.6% bonds maturing in 10 years. Elisa has an excellent credit
history and managed to borrow the money at 6.7%. The loan is payable in full in 4 years and interest is charged monthly.
Are the interest payments deductible for income tax purposes? Why or why not?
Student Response: Percent Correct Student Answer Choices
Value Response Response
100.0% a. Yes, since the purpose of borrowing was to
earn income.
0.0% b. Yes, since the interest is payable in
monthly instalments.
0.0% c. No, since the interest is paid on a personal
loan.
0.0% d. No, since the loan has a maturity date
shorter than the maturity date of the
investment.
www.nicsoceanbook.com/
OCEANBOOK@hotmail.com
General feedback: A taxpayer may deduct interest paid on funds borrowed to purchase securities if the purpose of
borrowing the funds was to earn income.

Reference: CSC Textbook: Chapter 13 - Financial Planning and Taxation - Taxes and Taxation Issues
- Tax Deductible Items Related to Investment Income - Interest on Borrowed Funds
Score: 1/1

Question 90 (1 point)
An investor purchases a $1,000 5-year bond for $960. At the end of the calendar year, the investor notices that the bond
is now trading at $980. Which of the following statements correctly describes the taxation of this change in value for the
investor?
Student Response: Percent Correct Student Answer Choices
Value Response Response
100.0% a. There is no amount taxable.
0.0% b. The investor must report interest earnings
of $20.
0.0% c. The investor must report interest earnings
of $40.
0.0% d. The investor must report a capital gain of
$20.

General feedback: A capital gain or loss is calculated only at the time of disposition, and is based solely on the
difference between the proceeds of the disposition and the adjusted cost base (book value) of the
security. The changes in market price between the date of acquisition and disposition are not
material for an investor. You are not required to declare capital gains or losses annually, unlike
interest paid on the bond.

Reference: CSC Textbook : Chapter 13 - Financial Planning and Taxation - Taxes and Taxation Issues
- Capital Gains and Losses - Tax on Disposition of Debt Securities
Score: 0/1

Question 91 (1 point)
Your discussions with a new client have produced the following information: He is 32, recently married, and planning on
starting a family. Recent purchases have included a home, and current goals include the payment of outstanding student
loan balances. Based solely on this information, where would you classify this investor based on life-cycle analysis?
Student Response: Percent Correct Student Answer Choices
Value Response Response
100.0% a. Early Earning Years.
0.0% b. Mid-Earning Years.
0.0% c. Peak Earning Years.
0.0% d. Retirement Years.

General feedback: The life-cycle analysis process can be useful in setting goals and investment objectives in the
financial planning process. It is unlikely that a client or individual will fit precisely within one
particular stage in the life cycle, nor is the application of the recommendations to the client an exact
science. However, the language around and usage of this theory is widespread, and you may expect
to encounter it frequently. The individual in this example is clearly in his early earnings years; the
www.nicsoceanbook.com/
OCEANBOOK@hotmail.com
clues are his age, his income vs. expenses, and the type of expenses he is incurring.

Reference: CSC Textbook: Chapter 13 - Financial Planning and Taxation - The Process of Financial
Planning - Life Cycle Analysis.
Score: 1/1

Question 92 (1 point)
Todd has an investment portfolio consisting of cash, fixed income securities and stocks. One of his investment strategies
is to take any dividend income he receives over the year from his stocks and invest the proceeds in junk bonds. Which of
the following risks would Todd primarily face with this investment tactic?
Student Response: Percent Correct Student Answer Choices
Value Response Response
0.0% a. The risk of investing too conservatively.
0.0% b. Political risk.
100.0% c. Default risk.
0.0% d. The risk of not investing.

General feedback: A key job for the financial advisor is to educate the client so that the client understands the various
risks above and beyond market risk. With Todd's investment strategy, he risks that a junk bond
issuer will be unable to pay interest on the prescribed date or the principal at maturity.
Reference: Chapter 13 – Financial Planning and Taxation – Informal Gathering, Communication and
Education.
Score: 1/1

Question 93 (1 point)
If Helena wanted to lend funds to her husband Thomas, which of the following rates must she charge and report on the
loan to bypass attribution?
Student Response: Percent Correct Student Answer Choices
Value Response Response
0.0% a. The attribution rate.
0.0% b. The bank rate.
100.0% c. The prescribed rate.
0.0% d. The prime rate.

General feedback: The attribution rules deal with income splitting and do not apply when money is loaned and interest
is charged at a rate prescribed by CRA and paid within 30 days after the year.
Reference: Chapter 13 – Financial Planning and Taxation – Basics of Tax Planning.
Score: 1/1

Question 94 (1 point)
Albert received $1,000 in dividends from PLM Inc., a manufacturer of hand-held devices incorporated in the U.S. The
company made its annual dividend distribution and no tax was withheld. Considering that Albert is in a 29% federal tax
bracket, what is the federal tax payable on the dividends?
Student Response: Percent Correct Student Answer Choices
www.nicsoceanbook.com/
OCEANBOOK@hotmail.com
Value Response Response
0.0% a. $133.33
0.0% b. $166.63
100.0% c. $290.00
0.0% d. No tax is due, since the dividends are from
a foreign company.

General feedback: Dividends received form a foreign corporation are not eligible for the dividend tax credit and are
usually taxed as regular income. The federal tax payable will be $290.00 ($1,000 x 29%).

Reference: CSC Textbook: Chapter 13 - Financial Planning and Taxation - Taxes and Taxation Issues
-Tax on Dividends - Tax on Foreign Dividends
Score: 1/1

Question 95 (1 point)
Which of the following best describes a fiduciary relationship in the securities industry?
Student Response: Percent Correct Student Answer Choices
Value Response Response
0.0% a. It is a principal agent relationship where
the agent has a certain vulnerability as the
principal has greater authority.
0.0% b. It is a principal third party relationship
where the principal may be somewhat
defenseless as the third party has greater
control.
0.0% c. It is a principal third party relationship
where the third party has a certain
vulnerability as the principal has greater
authority.
100.0% d. It is a principal agent relationship where
the principal may be somewhat
defenseless as the agent has greater
control.

General feedback: Fiduciary relationships are agent principal relationships in which the principal has a certain
vulnerability and the agent has greater expertise or authority.
Reference: Chapter 13 – Financial Planning and Taxation – Standards of Conduct.
Score: 0/1

Question 96 (1 point)
Which of the following is not a legitimate method of tax minimization?
Student Response: Percent Correct Student Answer Choices
Value Response Response
0.0% a. Postponing the receipt of income.
0.0% b. Splitting income with family members.
www.nicsoceanbook.com/
OCEANBOOK@hotmail.com
0.0% c. Selecting investments that provide a better
after-tax yield.
100.0% d. Claiming tax deductions for counselling
fees for RRSPs and RRIFs.

General feedback: Counseling fees for RRSPs and RRIFs are not eligible deductions, but the other methods are allowed.

Reference: CSC Textbook: Chapter 13 - Financial Planning and Taxation - Taxes and Taxation Issues
- Tax Deductible Items Related to Investment Income - Carrying Charges
Score: 1/1

Question 97 (1 point)
An IA is utilizing a client's funds to invest for his own personal benefit. Other than showing a complete disregard for the
client's assets, which of the following rules and regulations is the IA violating?
Student Response: Percent Correct Student Answer Choices
Value Response Response
0.0% a. The IA engaged in unauthorized
discretionary trading.
100.0% b. The IA failed to fulfill his or her fiduciary
duty.
0.0% c. The IA acted on illegal inside information.
0.0% d. The IA disregarded the KYC rule.

General feedback: Answer: (B) By not respecting a client's assets, the IA put his own personal interests ahead of the
client's and thus failed to fulfill his fiduciary duty towards that client.

Reference: CSC Textbook: Chapter 13 - Financial Planning and Taxation - Standards of Conduct -
Fiduciary Duty and Professionalism
Score: 0/1

Question 98 (1 point)
An investor purchased a bond two years ago for $16,005 plus accrued interest of $135. The bond was sold this year for
$16,145 plus accrued interest of $305. What capital gain or loss resulted from these transactions?
Student Response: Percent Correct Student Answer Choices
Value Response Response
0.0% a. A capital loss of $140.
0.0% b. A capital loss of $170.
100.0% c. A capital gain of $140.
0.0% d. A capital gain of $170.

General feedback: The capital gain is the sales price minus the purchase price of the bond. In this example, it is $140
($16,145 - 16,005). The accrued interest that is either paid or received is not included in the capital
gains calculation. It is included in the annual determination of interest income. The accrued interest
www.nicsoceanbook.com/
OCEANBOOK@hotmail.com
paid is subtracted from interest received in the year of purchase to determine interest income for tax
purposes. The interest received is added to interest income in the year of the sale.

Reference: CSC Textbook: Chapter 13 - Financial Planning and Taxation - Taxes and Taxation Issues
- Capital Gains and Losses - Tax on Disposition of Debt Securities
Score: 1/1

Question 99 (1 point)
An individual has RRSP contribution room of $18,450. His spouse has RRSP contribution room of $8,500 and has
contributed $4,400 to her RRSP this year. If the individual contributed $8,000 to his own plan, how much can he
contribute to a spousal RRSP plan?
Student Response: Percent Correct Student Answer Choices
Value Response Response
0.0% a. $4,100
0.0% b. $8,500
100.0% c. $ 10,450
0.0% d. $18,450

General feedback: A taxpayer may contribute to a spousal plan up to the maximum of his/her own contribution limit.
The taxpayer has maximum contribution room of $18,450 and has already contributed $8,000 to his
plan. He may, therefore, contribute $10,450 ($18,450 - 8,000) to his wife's spousal plan. The
spousal contribution does not affect the ability of the spouse to contribute to her own plan up to her
contribution limit.

Reference: CSC Textbook: Chapter 13 - Financial Planning and Taxation - Tax Deferral Plans -
Registered Retirement Savings Plans (RRSPs) - Spousal RRSPs
Score: 1/1

Question 100 (1 point)


Sundeep had the following sources of income and expenses last year:

Employment income $55,000


Rental income $ 4,500
Student loan - interest paid $ 1,200

What income figure should Sundeep use to calculate his maximum allowed RRSP contribution for the year?
Student Response: Percent Correct Student Answer Choices
Value Response Response
0.0% a. $53,800
0.0% b. $55,000
0.0% c. $58,300
100.0% d. $59,500
www.nicsoceanbook.com/
OCEANBOOK@hotmail.com
General feedback: To determine a person's RRSP contribution limit, Total Income Earned must be calculated.
Total Income Earned = $55,000 + $4,500 = $59,500

Reference: Taxation - CSC Textbook: Chapter 13 - Financial Planning and Taxation - Tax Deferral
Plans - Registered Retirement Savings Plans (RRSPs)
www.nicsoceanbook.com/
OCEANBOOK@hotmail.com
Question 1 (1 point)
In an uptrend, if breadth measurements are persistently weak, what can be concluded about the
trend of the market?
Student Response: Percent Correct Student Answer Choices
Value Response Response
100.0% a. The trend has a higher
probability of failing.
0.0% b. The trend has a higher
probability of advancing.
0.0% c. The trend has a lower
probability of declining.
0.0% d. The trend is not a good
measure of market
performance.

General feedback: In an uptrend, if breadth measurements are persistently weak, the trend has a
higher probability of failing.

Reference: CSC Textbook: Chapter 8 - Analyzing Markets and Products -


Technical Analysis - Equity Market Analysis - Breadth of Market
Score: 1/1

Question 2 (1 point)
Company A and Company B are both industrial companies, operating in the same industry and have
similar prospects. Company A has a P/E of 20, Company B has a P/E of 36, and the industry P/E is
28. By these criteria alone, which company offers greater value to investors?
Student Response: Percent Correct Student Answer Choices
Value Response Response
0.0% a. Company B because it has
the higher P/E.
0.0% b. Company B because its
P/E is above the industry
P/E.
0.0% c. Company A or B because
P/E is not a measure of
market value.
100.0% d. Company A because it has
the lower P/E.

General feedback: There is no generally accepted level or rule of thumb for the P/E ratio - it varies
between industries and over the business cycle. Companies in the same
industry and with similar prospects should have approximately the same P/E.
Company A offers greater value to investors because its P/E is lower than
company B's. This means that investors are paying a lower price for the
earnings for Company A as compared to Company B.

Reference: CSC Textbook: Chapter 8 - Analyzing Markets and Products -


www.nicsoceanbook.com/
OCEANBOOK@hotmail.com
Interpreting Financial Statements - Value Ratios - Price-Earnings Ratio or PE
Multiple
Score: 0/1

Question 3 (1 point)
If a company issues additional common shares, what is the likely impact on its risk analysis ratios?
Student Response: Percent Correct Student Answer Choices
Value Response Response
100.0% a. The debt/equity ratio will
decrease.
0.0% b. The debt/equity ratio will
increase.
0.0% c. Cash flow/debt will
improve significantly.
0.0% d. Interest coverage will
decrease.

General feedback: The immediate result of the equity issue will be an increase in common equity
outstanding and a decrease or improvement in the debt/equity ratio. Cash flow
might improve if the new equity is used to expand the company - but this will
not be an immediate impact. Also, interest coverage will improve if net income
increases or debt is repaid from the new equity issue. But, these impacts also
might not be immediate.

Reference: CSC Textbook: Chapter 8 - Analyzing Markets and Products -


Interpreting Financial Statements - Risk Analysis Ratios - Debt/Equity Ratio
Score: 1/1

Question 4 (1 point)
If an investor wants to assess how effectively a company has used its available assets, which
financial ratio should he review?
Student Response: Percent Correct Student Answer Choices
Value Response Response
0.0% a. Interest coverage.
0.0% b. Equity per preferred
share.
100.0% c. Pre-tax return on invested
capital.
0.0% d. Cash flow/total debt
outstanding.

General feedback: Pre-tax return on invested capital shows how well management has employed
the assets at its disposal.
Reference: CSC Textbook: Chapter 8 - Analyzing Markets and Products -
Interpreting Financial Statements - Operating Performance Ratios
Score: 1/1
www.nicsoceanbook.com/
OCEANBOOK@hotmail.com
Question 5 (1 point)
Company Beta pays an annual dividend of $2.34 per common share. What impact will a rising share
price have on its dividend yield ratio?
Student Response: Percent Correct Student Answer Choices
Value Response Response
0.0% a. The dividend yield will
rise.
100.0% b. The dividend yield will fall.
0.0% c. The dividend yield will not
change.
0.0% d. The dividend yield is not
impacted by changes in
share price.

General feedback: The dividend yield ratio is calculated as the annual dividend per share divided
by the current market price per share. As the share price changes so to does
the dividend yield. If the share price rises, the dividend yield will fall; if the
share price falls, the dividend yield will rise.

Reference: CSC Textbook: Chapter 8 - Analyzing Markets and Products –


Interpreting Financial Statements - Value Ratios - Dividend Yield
Score: 0/1

Question 6 (1 point)
If an investor wants to determine how effective a company was at controlling its costs, which of the
following ratios should she examine?
Student Response: Percent Correct Student Answer Choices
Value Response Response
0.0% a. Dividend yield.
100.0% b. Net profit margin.
0.0% c. Operating cash flow ratio.
0.0% d. Earnings per common
share.

General feedback: Net profit margin is an important indicator of how efficiently a company is
managed after taking both expenses and taxes into account. Because this ratio
is the result of the company’s operations for the period, it effectively sums up
management’s ability to run the business in a single figure.
Reference: CSC Textbook: Chapter 8 - Analyzing Markets and Products -
Interpreting Financial Statements - Operating Performance Ratios - Net Profit
Margin
Score: 0/1

Question 7 (1 point)
Which of the following statements is true regarding the Interest Coverage ratio for ABC Company?
www.nicsoceanbook.com/
OCEANBOOK@hotmail.com
Year 1 Year 2 Year 3 Year 4 Year 5
Interest Coverage 3.15x 4.22x 2.25x 1.50x 2.50x
Student Response: Percent Correct Student Answer Choices
Value Response Response
100.0% a. ABC’s ability to meet its
interest charges worsened
over the period.
0.0% b. ABC’s ability to meet its
interest charges improved
over the period.
0.0% c. ABC’s net earnings likely
rose over the period.
0.0% d. ABC’s short-term debt
likely fell over the period.

General feedback: The Interest Coverage ratio measures the ability of a company to pay interest
charges on its debt. Industry standards vary from industry to industry.
However, a rising trend in the ratio indicates an overall improvement in a
company’s ability to meet its interest charges. For ABC Company, the ratio
declined from 3.15 to 2.50 over the period, indicating that its ability to meet
interest charges has worsened somewhat over the 5-year period.

Reference: CSC Textbook: Chapter 8 - Analyzing Markets and Products


Interpreting Financial Statements
Score: 1/1

Question 8 (1 point)
Generally, what is the analysis of a company's earnings indicative of?
Student Response: Percent Correct Student Answer Choices
Value Response Response
100.0% a. Management's use of the
company's resources.
0.0% b. The overall financial
position of a company.
0.0% c. A company's performance
in comparison with
industry averages.
0.0% d. A company's ability to
meet its short-term
commitments.

General feedback: The analysis of a company's earning tells the investor how well management is
making use of the company's resources.

Reference: CSC Textbook: Chapter 8 - Analyzing Markets and Products -


Fundamental Company Analysis - Earning Statement Analysis
Score: 0/1
www.nicsoceanbook.com/
OCEANBOOK@hotmail.com
Question 9 (1 point)
Company A, a manufacturing company, derives 35% of its capital structure from debt instruments.
The company then issues $2,000,000 in common shares. What effect will this likely have on the
company's capital structure?
Student Response: Percent Correct Student Answer Choices
Value Response Response
0.0% a. It will have no effect.
100.0% b. It will improve the
capitalization.
0.0% c. It will increase the
debt/equity ratio.
0.0% d. It will increase the
company's leverage.

General feedback: An issuance of debt or equity will affect a company's capital structure. Therefore
answer choice (A) is incorrect. As common shares are equity, increasing the
amount a company has issued will increase the denominator of the debt/equity
ratio (total debt outstanding/book value of shareholders' equity), thus reducing
the debt/equity ratio - meaning answer choice (C) is incorrect. Leverage results
from issuing debt, not from issuing equity - meaning answer choice (D) is
incorrect. As Company A is an industrial company, issuing the equity should
help to reduce the percentage of capital attributable to debtholders - thus
improving the capitalization - meaning answer choice (B) is correct.

Reference: CSC Textbook: Chapter 8 - Analyzing Markets and Products -


Interpreting Financial Statements - Risk Analysis Ratios - Percentage of Total
Capital Ratios
Score: 1/1

Question 10 (1 point)
Which of the following might force the Bank of Canada to increase interest rates?
Student Response: Percent Correct Student Answer Choices
Value Response Response
100.0% a. A rapid increase in long-
term bond yields due to
an increase in inflation
expectations.
0.0% b. A sudden increase in the
dividend yield on the
S&P/TSX Composite due
to an increase in dividend
payments by Canadian
companies.
0.0% c. A rapidly appreciating
Canadian dollar due to a
sharp increase in
commodity prices.
www.nicsoceanbook.com/
OCEANBOOK@hotmail.com
0.0% d. An unexpected decline in
Canadian GDP due to a
worse-than-expected
decline in Canada's
current account deficit.

General feedback: If bond yields increase as a result of an increase in inflation expectations, the
Bank of Canada may be forced to increase interest rates to slow down economic
growth and alleviate inflationary fears. The Bank of Canada would not be forced
into raising rates simply because dividend payments have gone up. An
appreciation in the Canadian dollar or a decline in GDP might force the Bank of
Canada to lower interest rates rather than increase them.
Reference: Chapter 8 – Analyzing Markets and Products – Fundamental
Macroeconomic Analysis
Score: 1/1

Question 11 (1 point)
Which of the following adjustments would be required to net earnings in order to compute fully
diluted earnings per common share for a company that had common shares, preferred shares and
convertible debentures outstanding?
Student Response: Percent Correct Student Answer Choices
Value Response Response
0.0% a. Deduct preferred
dividends.
100.0% b. Deduct preferred
dividends and add the
tax-affected interest paid
on convertible
debentures.
0.0% c. Deduct the tax-affected
interest paid on
convertible debentures.
0.0% d. Add preferred dividends
and add tax-affected
interest paid on
convertible debentures.

General feedback: Fully diluted earnings per share is a figure that is calculated on common stock
outstanding plus common stock equivalents such as convertible preferred stock,
convertible debentures, etc. It shows the EPS that would result if all equivalent
securities were converted into common shares. As the preferred shares here are
not convertible, you do not make an adjustment for the dividends paid on these
shares. Had they been convertible, you would have added this amount back to
net earnings - as, if the shares were converted, the dividends would not have
been paid. As the debentures are convertible, you do add back the interest paid
on the debentures. Because interest charges are deducted before tax, and
earnings are an after-tax figure, you must tax-affect the amount used.

Reference: CSC Textbook: Chapter 8 - Analyzing Markets and Products -


Interpreting Financial Statements - Value Ratios - Earnings per Common Share
Score: 0/1
www.nicsoceanbook.com/
OCEANBOOK@hotmail.com
Question 12 (1 point)
Jim is a conservative investor and is considering buying the preferred shares of a major Canadian
bank. What special type of preferred shares would best suit Jim?
Student Response: Percent Correct Student Answer Choices
Value Response Response
0.0% a. Convertible preferred
shares.
0.0% b. Variable dividend
preferred shares.
0.0% c. Preferred shares with
warrants.
100.0% d. Retractable preferred
shares.

General feedback: The various special features on preferred shares will benefit certain investors
differently. Retractable preferred shares generally are less risky and are
considered to be of benefit to conservative investors. The other preferred shares
listed would benefit more risk averse and aggressive investors.
Reference: CSC Textbook: Chapter 8 - Analyzing Markets and Products -
Interpreting Financial Statements - Assessing Preferred Share Investment
Quality - How Preferred Shares Fit Into Individual Portfolios
Score: 1/1

Question 13 (1 point)
When a technical analyst identifies a strong demand for a stock while the supply is low, what
conclusion can be made about the level of the stock price?
Student Response: Percent Correct Student Answer Choices
Value Response Response
0.0% a. It is at a downswing level.
100.0% b. It is at a support level.
0.0% c. It is at a resistance level.
0.0% d. It is at a stagnating level.

General feedback: A support level is the price at which the majority of investors start sensing
value, and therefore are willing to buy (demand is strong) and the majority of
existing holders (or potential short sellers) are not willing to sell. As demand
begins to exceed supply, prices tend to rise above support levels.

Reference: CSC Textbook: Chapter 8 - Analyzing Markets and Products -


Technical Analysis - Commonly Used Tools in Technical Analysis - Chart Analysis
Score: 1/1

Question 14 (1 point)
Raj is considering investing $12,000 in the diamond-mining industry and has narrowed his choice to
2 companies - Company W and Company J. By comparing the price-earning ratio of the 2
www.nicsoceanbook.com/
OCEANBOOK@hotmail.com
companies, which company would you recommend that Raj invest in? (Assume the 2 companies
have similar prospects.)
Earnings per Share Current Market
(in last 12 months) Price
Company W $3.00 $36.00
Company J $2.59 $28.50

Student Response: Percent Correct Student Answer Choices


Value Response Response
0.0% a. Raj should invest in
Company W, since its
price-earnings ratio of
12:1 is higher than that of
Company J.
0.0% b. Raj should invest in
Company W since its
price-earnings ratio of
8.33:1 is lower than that
of Company J.
100.0% c. Raj should invest in
Company J since its price-
earnings ratio of 11:1 is
lower than that of
Company W.
0.0% d. Raj should invest in
Company J since its price-
earnings ratio of 9.09:1 is
higher than that of
Company W.

General feedback: current market price of common


price-earning ratio =
earnings per share
price-earning ratio, $36.00
= 12:1
Company W = $ 3.00
price-earning ratio, $28.50
= 11:1
Company J = $2.59

If the 2 companies have similar prospects, Raj should select the company with
the lower price-earnings ratio, in this case Company J.

Reference: CSC Textbook: Chapter 8 - Analyzing Markets and Products -


Interpreting Financial Statements - Value Ratios - Price Earnings Ratio or PE
Multiple
Score: 0/1

Question 15 (1 point)
Trillium Therapeutics (symbol TT) is expected to pay a $1.50 dividend next year. The company
anticipates a consistent long-term growth rate of 5.5% and investors believe that a required return
of 8.25% on TT is suitable. Using the Dividend Discount Model (DDM), what is the intrinsic value of
the TT shares?
Student Response: Percent Correct Student Answer Choices
Value Response Response
www.nicsoceanbook.com/
OCEANBOOK@hotmail.com
100.0% a. $54.55
0.0% b. $57.55
0.0% c. $18.18
0.0% d. $10.91

General feedback: The Dividend Discount Model illustrates in a very simple way how companies
with stable growth are theoretically priced. The formula is Price = Div1 / (r-g).
Div1 is the expected dividend paid out by the company in one year, r is the
required rate of return on investments, and g is the assumed constant growth
rate for dividends. Therefore, the price or intrinsic value of TT = $1.50 / (.0825
- .055) which equals $54.55.
Reference: CSC Textbook: Chapter - Analyzing Markets and Products -
Fundamental Valuation Models - Dividend Discount Model
Score: 1/1

Question 16 (1 point)
Which of the following statements about the price-earnings (P/E) ratios is true?
Student Response: Percent Correct Student Answer Choices
Value Response Response
0.0% a. P/E ratios are calculated
for preferred and common
shares.
0.0% b. Bank stocks tend to have
the highest P/E ratios in
the marketplace.
0.0% c. Value investors tend to
select stocks with the
highest P/E ratios.
100.0% d. P/E ratios typically
decrease in a bearish
market.

General feedback: As a rule, P/E ratios increase in a rising stock market or with rising earnings.
The reverse is true in a declining market or when earnings decline.
Reference: CSC Textbook: Chapter 8 - Analyzing Markets and Products -
Interpreting Financial Statements - Value Ratios - Price-Earnings Ratio or P/E
Multiple
Score: 1/1

Question 17 (1 point)
Which of the following statements best reflects the Efficient Market Hypothesis?
Student Response: Percent Correct Student Answer Choices
Value Response Response
0.0% a. Actively managed
portfolios consistently
outperform passively
managed portfolios.
www.nicsoceanbook.com/
OCEANBOOK@hotmail.com
0.0% b. Past price movements in
stocks enable investors to
predict future price
movements.
100.0% c. Investors should not be
able to consistently
outperform the market.
0.0% d. Equities will outperform
fixed income securities
over the long-term.

General feedback: The Efficient Market Hypothesis assumes that there are a large number of
rational profit-seeking investors in the marketplace who react quickly to the
release of new information. As new information about a stock appears, investors
reassess the intrinsic value of the stock and adjust their estimation of its price
accordingly. Therefore, at any time, a stock’s price should accurately reflect all
available information and represent the best estimate of its true value. In this
way, consistently outperforming an efficient market should not occur.
Reference: CSC Textbook: Chapter 8 - Analyzing Markets and Products -
Overview of Analysis Methods - Efficient Market Theories
Score: 1/1

Question 18 (1 point)
A trend analysis of a company's earnings per share (EPS) shows that the trend value in the fifth
year is 150. If the company's EPS in the fifth year was $1.96, what was the company's EPS in the
first year?
Student Response: Percent Correct Student Answer Choices
Value Response Response
0.0% a. $0.98
100.0% b. $1.31
0.0% c. $2.94
0.0% d. $3.92

General feedback: The EPS in the first year is found by dividing the value in the fifth year by the
trend value in the fifth year, and multiplying the result by 100. In this case, EPS
in the first year is equal to ($1.96 / 150) × 100 = $1.31.
Reference: Chapter 8 – Analyzing Markets and Products – Interpreting Financial
Statements
Score: 1/1

Question 19 (1 point)
Which of the following price patterns is considered a buy signal?
Student Response: Percent Correct Student Answer Choices
Value Response Response
0.0% a. The formation of a top
head and shoulders
pattern.
www.nicsoceanbook.com/
OCEANBOOK@hotmail.com
100.0% b. Price breaks through the
moving average from
below on heavy volume.
0.0% c. Price breaks through the
moving average from
above on heavy volume.
0.0% d. The advance/decline line
flattens out after a long
increase.

General feedback: If price breaks through the moving average from below, it is considered a buy
signal. Prices have been declining and pulling the moving average lower. The
actual price line will be below the moving average line when prices are
declining. If prices cross over the moving average from below, it is a signal that
the price decline has stopped. This is a buy signal.

Reference: CSC Textbook: Chapter 8 - Analyzing Markets and Products -


Technical Analysis - Commonly Used Tools in Technical Analysis -Quantitative
Analysis
Score: 1/1

Question 20 (1 point)
Which of the following types of financial ratios measures the stock market's rating of a company by
relating the market price of its shares to certain balances obtained from its financial statements?
Student Response: Percent Correct Student Answer Choices
Value Response Response
0.0% a. Debt ratios.
100.0% b. Value ratios.
0.0% c. Liquidity ratios.
0.0% d. Profitability ratios.

General feedback: A total analysis of a company's financials incorporates the use of all of the
groups of financial ratios. Each group has a specific use for interpretation and
discussion. The value ratios include Percentage Dividend Payout ratios, Earnings
per Common Share, Dividend Yield and Price-Earnings. Where the other ratios
measure the performance of a company (i.e., how many times it has covered its
interest charges, how profitable it is) - this group measures the value the
company brings to each share.

Reference: CSC Textbook: Chapter 8 - Analyzing Markets and Products -


Interpreting Financial Statements - Types of Ratios - Value Ratios
Score: 1/1

Question 21 (1 point)
Which of the following statements about a prolonged period of inflation is true?
Student Response: Percent Correct Student Answer Choices
Value Response Response
www.nicsoceanbook.com/
OCEANBOOK@hotmail.com
100.0% a. Manufacturers would
experience higher labour
costs.
0.0% b. P/E levels would remain
flat.
0.0% c. Businesses would
experience higher
corporate profits.
0.0% d. Short-term interest rates
would likely fall.

General feedback: Inflation is defined as a period of sustained rising prices. For manufacturers,
higher costs for inputs like raw materials and labour, translates into higher
inventory and labour costs. These higher costs are then passed on to consumers
in the form of higher prices. But higher costs cannot be passed on indefinitely;
buyer resistance eventually develops. The resulting squeeze on corporate profits
is reflected in lower common share prices. Inflation causes widespread
uncertainty and lack of confidence in the future. These factors lead to higher
interest rates, lower corporate profits, and lower price-to-earnings ratios.
Reference: CSC Textbook: Chapter 8 - Analyzing Markets and Products -
Fundamental Macroeconomic Analysis - Inflation - Accounting for the Effects of
Inflation
Score: 1/1

Question 22 (1 point)
Based on the Dividend Discount Model, what is the intrinsic value of Glacier Energy if its expected
dividend in one year is $1.20, the discount rate is 6.30%, and its long-term growth is expected to
be continuous at 3.60%?
Student Response: Percent Correct Student Answer Choices
Value Response Response
0.0% a. $33.33
100.0% b. $44.44
0.0% c. $46.04
0.0% d. $88.89

General feedback: The formula is Price = Div1 / (r-g). Div1 is the expected dividend paid out by
the company in one year, r is the required rate of return on investments, and g
is the assumed constant growth rate for dividends. Therefore, the price or
intrinsic value of Glacier Energy = $1.20 / (.0630 - .036) which equals $44.44.
Reference: CSC Textbook: Chapter 8 - Analyzing Markets and Products -
Fundamental Valuation Models - Dividend Discount Model
Score: 1/1

Question 23 (1 point)
If a rising price of oil drives up inflation in Canada, which of the following scenarios is most likely to
follow?
www.nicsoceanbook.com/
OCEANBOOK@hotmail.com
Student Response: Percent Correct Student Answer Choices
Value Response Response
0.0% a. Interest rates will drop to
contain inflation and P/E
ratios will fall.
100.0% b. Interest rates will rise to
contain inflation and P/E
ratios will fall.
0.0% c. Interest rates will drop to
contain inflation and P/E
ratios will go up.
0.0% d. Interest rates will rise to
contain inflation and P/E
ratios will go up.

General feedback: Price-earnings (P/E) levels and inflation are strongly inversely related to the
prevailing level of inflation, and therefore to the prevailing level of interest
rates. When inflation rates are expected to rise, interest rate levels are
expected to rise to contain inflation, which will cause P/Es to fall.
Reference: CSC Textbook: Chapter 8 - Analyzing Markets and Products -
Fundamental Valuation Models - P/E Levels and Inflation
Score: 0/1

Question 24 (1 point)
CBA Inc and ZYX Inc. are in the same line of business. CBA Inc. has a debt/equity ratio of 0.94 and
ZYX Inc. has a debt/equity ratio of 1.29. The average debt/equity ratio for the industry is 0.98.
Considering this information, which company would you recommend to your clients and why?
Student Response: Percent Correct Student Answer Choices
Value Response Response
100.0% a. CBA Inc., since it has the
lower debt/equity ratio of
the 2 companies
compared.
0.0% b. CBA Inc., since it has a
debt/equity ratio that is
closer to the industry
average.
0.0% c. ZYX Inc., since it has the
higher debt/equity ratio of
the 2 companies
compared.
0.0% d. ZYX Inc., since it has a
debt/equity ratio that is
above the industry
average.

General feedback: All things being equal, a company with a lower debt/equity ratio is a safer
investment. Thus CBA Inc. should be recommended.

Reference: CSC Textbook: Chapter 8 - Analyzing Markets and Products -


www.nicsoceanbook.com/
OCEANBOOK@hotmail.com
Interpreting Financial Statements - Risk Analysis Ratios - Debt/Equity Ratios
Score: 1/1

Question 25 (1 point)
Historically, what relationship has been observed between the P/E levels of stocks and the rate of
inflation?
Student Response: Percent Correct Student Answer Choices
Value Response Response
100.0% a. P/E levels have been
inversely related to the
rate of inflation.
0.0% b. P/E levels have been
positively related to the
rate of inflation.
0.0% c. P/E rates have shown
little relationship with
inflation rates.
0.0% d. Previously, P/E levels and
inflation rates were
positively related but this
relationship has broken
down in the last 15 years.

General feedback: Historically, P/E levels have been inversely related to the rate of inflation. When
inflation rates were low, P/E rates were high. When inflation rates increased, the
P/E ratios fell.

Reference: CSC Textbook: Chapter 8 - Analyzing Markets and Products -


Fundamental Macroeconomic Analysis - Inflation - Accounting for the Effects of
Inflation
Score: 1/1

Question 26 (1 point)
Over the last five years, the returns on ABC stock and the S&P/TSX Composite Index were as
follows:

Return on the
S&P/TSX
Return on Composite
Year ABC Stock Index
1 12% 9%
2 6% 4%
3 -8% -5%
4 -6% -5%
5 10% 9%
www.nicsoceanbook.com/
OCEANBOOK@hotmail.com
Given this data, what was the beta of ABC stock relative to the S&P/TSX Composite Index?
Student Response: Percent Correct Student Answer Choices
Value Response Response
0.0% a. Less than 0.
0.0% b. Greater than or equal to
0, but less than 1.
0.0% c. Exactly 1.
100.0% d. Greater than 1.

General feedback: Beta measures the volatility of a stock relative to the volatility of the stock
market as a whole. A beta of 1 indicates that a stock has the same volatility as
the market; a beta less than 1 indicates that a stock had less volatility than the
market, while a beta greater than 1 indicates that a stock has more volatility
than the market.
In Years 1, 2, and 5, when the returns on ABC stock and the S&P/TSX
Composite were positive, ABC out-performed the market. In Years 3 and 4,
when the returns on ABC stock and the S&P/TSX Composite were negative, ABC
under-performed the market. Thus, ABC's returns were more volatile than the
market's, and its beta was greater than 1.
Reference: Chapter 9 – The Portfolio Approach – Risk and Return
Score: 1/1

Question 27 (1 point)
Which of the following investors may be interested primarily in capital gains?
Student Response: Percent Correct Student Answer Choices
Value Response Response
0.0% a. A salaried investor who
relies on investment
income to meet the costs
of raising and educating
his children.
100.0% b. A well-paid young
executive with excess
income preparing for
early retirement.
0.0% c. A young couple investing
their savings for the
eventual purchase of a
house.
0.0% d. A retired couple whose
pension income is
insufficient to provide for
their living expenses.

General feedback: An example of an investor who might primarily seek capital growth income is a
well-paid young executive with excess income and a high tolerance to risk. (a)
and (d) are examples of investors who might be seeking safety of principal and
income as their primary investment objectives. Investors in example (c) have a
www.nicsoceanbook.com/
OCEANBOOK@hotmail.com
much shorter term in mind to realize capital gains.

Reference: CSC Textbook: Chapter 9 - The Portfolio Approach - Determining


Objectives and Constraints - Major Investment Objectives - Growth of Capital or
Capital Gains
Score: 1/1

Question 28 (1 point)
You are evaluating a new client in order to determine an appropriate asset mix for her investments.
After discussing the client's goals, personal situation, etc., you have the following information: your
client is 45, single, with no dependents and no expectation of ever having any. She has been
investing extensively in a wide range of investment products, and is quite comfortable with risk. Her
goal is to retire at 55, and then to travel extensively. She has no wish to leave an estate for any
individual. What asset mix would you recommend?
Student Response: Percent Correct Student Answer Choices
Value Response Response
0.0% a. Cash 10%/Fixed Income
40%/Equities 50%.
100.0% b. Cash 5%/Fixed Income
25%/Equities 70%.
0.0% c. Cash 20%/Fixed Income
70%/Equities 10%.
0.0% d. Cash 5%/Fixed Income
75%/Equities 20%.

General feedback: Choosing an asset mix is not an exact science. As an investment advisor, you
can only make recommendations based on the information and discussions you
have with a client. Your recommendations may not be acceptable to the client.
This is part of the learning experience with each new client. What direction the
client chooses eventually will be a compromise between what you feel best and
what they prefer. Your role is to guide them to making an appropriate choice,
not to dictate an approach based on a formula. As time passes and your client's
personal circumstances change, the asset mix decision must be revisited and
evaluated. Here, you have an individual with no significant personal obligations
other than to herself, and with a goal of retiring early to a reasonably expensive
lifestyle. Therefore, an aggressive, growth oriented asset mix would most likely
be appropriate. Thus, a high weighting in equities, with some fixed income for
diversification and cash for liquidity, would be indicated.

Reference: CSC Textbook: Chapter 9 - The Portfolio Approach - Developing an


Asset Mix - Setting the Asset Mix - Balancing the Asset Classes
Score: 1/1

Question 29 (1 point)
An investor's portfolio currently consists of the common shares of 2 companies: ABC Bank and XYZ
Gold. How can this investor reduce the overall risk of this portfolio?
Student Response: Percent Correct Student Answer Choices
Value Response Response
0.0% a. Add more bank and gold
company shares to the
portfolio.
www.nicsoceanbook.com/
OCEANBOOK@hotmail.com
100.0% b. Add government or high
quality corporate bonds to
the portfolio.
0.0% c. Add shares of industrial
and retail companies to
the portfolio.
0.0% d. Add shares of
pharmaceuticals for
growth.

General feedback: Systematic risk is the risk of being in a particular capital market. For example, if
there is a significant stock market correction, most stocks will decline.
Systematic risk cannot be diversified away by adding more stocks in the same
or different industry groups as compared to the existing portfolio. Systematic
risk can be reduced through the investment in securities in a different capital
market. In this case, adding bonds to the portfolio will reduce the systematic
risk of owning stocks.

Reference: CSC Textbook: Chapter 9 - The Portfolio Approach - Risk and Return
- Risk-"The Other Side of the Coin" - Systematic Risk
Score: 1/1

Question 30 (1 point)
What does the ex-post rate of return measure?
Student Response: Percent Correct Student Answer Choices
Value Response Response
0.0% a. The “going forward”
return
0.0% b. The compound return.
100.0% c. The “after-the-fact”
return.
0.0% d. The current return less
inflation.

General feedback: The term ex-post simply refers to the fact that this rate of return is "after the
fact" rather than before. You are looking back at the return you received on the
stock, after a period of holding the investment. It is calculated using the amount
you earned on the investment, divided by the amount you paid.

Reference: CSC Textbook: Chapter 9 - The Portfolio Approach - Risk and Return
- Rate of Return
Score: 1/1

Question 31 (1 point)
Which of the following securities would likely have the greatest allocation in a portfolio whose
primary objective is income and whose secondary objective is tax minimization?
Student Response: Percent Correct Student Answer Choices
Value Response Response
www.nicsoceanbook.com/
OCEANBOOK@hotmail.com
0.0% a. Hedge funds.
0.0% b. Common shares.
100.0% c. Preferred shares.
0.0% d. Money market securities.

General feedback: Of the four choices, preferred shares would clearly have the greatest allocation
as they are a great source of tax-efficient income. Hedge funds are not
appropriate for these objectives. Common shares and money market securities
may form a part of the portfolio, but they will likely be a smaller portion than
preferred shares.
Reference: Chapter 9 – The Portfolio Approach – Determining Objectives and
Constraints
Score: 0/1

Question 32 (1 point)
Which of the following investment strategies are growth managers most likely to employ?
Student Response: Percent Correct Student Answer Choices
Value Response Response
0.0% a. Those that focus on
specific stock selection
with a research-intensive
approach.
0.0% b. Those that pursue long-
term growth through a
buy-and-hold process.
100.0% c. Those that focus on
current and future
earnings of individual
companies, specifically
EPS.
0.0% d. Those that analyze the
prospects for the overall
economy and assume
most sectors will
outperform.

General feedback: In the bottom-up style of growth investing, managers focus on current and
future earnings of individual companies, specifically earnings per share (EPS).

Reference: CSC Textbook: Chapter 9 - The Portfolio Approach - Developing an


Asset Mix - Equity Manager Styles - Growth Managers
Score: 1/1

Question 33 (1 point)
At the start of the year, a client’s portfolio is worth $250,000. During the year, the client deposits
an additional $20,000, while at the end of the year the portfolio is worth $285,000. What is the pre-
tax total return on the portfolio, expressed as a percentage?
www.nicsoceanbook.com/
OCEANBOOK@hotmail.com
Student Response: Percent Correct Student Answer Choices
Value Response Response
0.0% a. 5.26%
100.0% b. 5.77%
0.0% c. 14.0%
0.0% d. 21.2%

General feedback: The formula for total return is the increase in market value divided by the
average amount invested. Therefore, the numerator would be ($285,000-
($250,000 +20,000)) =$15,000. The denominator would be (($250,000
+(20,000/2)) = $260,000. Therefore, the total return would be
($15,000/$260,000) = 5.77%. Of course, this calculation does not address the
tax-effectiveness of the portfolio; depending on how the growth was achieved,
for example through realized or unrealized capital gains, income received from
bonds, dividends from equities, the after-tax return could be very different from
two portfolios with the same nominal (pre-tax) rate of return.

Reference: Chapter 9 - The Portfolio Approach - Evaluating Portfolio


Performance - Measuring Portfolio Performance
Score: 1/1

Question 34 (1 point)
Paul and Claire are 30-year old newlyweds. They have good jobs, but are living rent-free in a
basement apartment in Claire's parent's house. They are trying to save as much money as they can
so that they can build their dream house in 5 years. They do not plan on having children, but by the
age of 45 they want to buy a hobby farm north of the city. They also want to retire by the time they
are 60. Since they both live active and healthy lifestyles, they expect to enjoy a long retirement.
How many time horizons do Paul and Claire have?
Student Response: Percent Correct Student Answer Choices
Value Response Response
0.0% a. 1
0.0% b. 2
0.0% c. 3
100.0% d. 4

General feedback: Each of the major life goals, including the period from retirement to death,
represents a time horizon. Therefore, Paul and Claire have four time horizons:
build their dream home in 5 years, buy a hobby farm in 15 years, retire in 30
years, and retirement years.
Reference: Chapter 9 – The Portfolio Approach – Determining Objectives and
Constraints
Score: 0/1

Question 35 (1 point)
Yuri is a university professor. He receives a steady income from teaching statistics, and this income
is sufficient to support his family. Yuri also receives additional income from giving seminars at other
universities, writing articles and consulting. Yuri is able to save all his extra income and has
www.nicsoceanbook.com/
OCEANBOOK@hotmail.com
accumulated substantial savings. He is a risk-averse individual and invests only in GICs. As an
investment advisor, what would be your recommendation to Yuri?
Student Response: Percent Correct Student Answer Choices
Value Response Response
0.0% a. Yuri is limiting his
investment choices and
should consider investing
in high-yield, high-risk
investments, which
provide the best earnings
potential.
100.0% b. Yuri is limiting his
investment choices and
should consider investing
in other low-risk
investments in order to
diversify.
0.0% c. Yuri has selected an
appropriate investment
strategy, considering his
level of risk tolerance.
0.0% d. Since Yuri has
accumulated substantial
savings he should
consider implementing an
investment preservation
strategy.

General feedback: Since Yuri is a risk-averse individual, he could invest in high quality investments
with moderate risk. Having a very conservative approach, such as investing in
GICs, only limits the investment choices of the person because of unrealistic
fear.

Reference: CSC Textbook: Chapter 9 - Determining Objectives and Constraints -


Major Investment Objectives - Safety of Principal or Preservation of Capital
Score: 1/1

Question 36 (1 point)
Rebecca works for NTD Financial as a portfolio manager. She is in charge of a Canadian bond fund
and her primary objective is to actively manage the average term of the holdings. Which of the
following fixed-income management styles has she been following?
Student Response: Percent Correct Student Answer Choices
Value Response Response
100.0% a. Interest rate anticipators.
0.0% b. Credit quality.
0.0% c. Spread traders.
0.0% d. Momentum.

General feedback: Managing through interest rate anticipation means lengthening the average
www.nicsoceanbook.com/
OCEANBOOK@hotmail.com
term of a portfolio when interest rates are expected to fall, and shortening the
term or taking refuge in cash when interest rates are expected to rise.
Reference: Chapter 9 - The Portfolio Approach – Developing an Asset Mix -
Fixed-Income Manager Styles.
Score: 1/1

Question 37 (1 point)
Martha, an executive secretary in her early 40s, has the following investment objectives:

• Primary objective - long-term growth


• Secondary objective - tax minimization

Currently, Martha holds primarily government of Canada T-Bills, corporate bonds and debentures. Is
Martha's current portfolio reflective of her objectives and why?
Student Response: Percent Correct Student Answer Choices
Value Response Response
100.0% a. No, since the current
investments are income
generating and the tax
burden is too high.
0.0% b. No, since the current
investments are
generating capital gains
and the tax burden is too
high.
0.0% c. Yes, since the current
investments are
generating capital gains
and will minimize the tax
burden.
0.0% d. Yes, since the
investments minimize the
risk adjusted return.

General feedback: Government of Canada T-bills, bonds and debentures are income-generating
investments and the tax burden of such investments is too high. Thus, Martha's
objectives of long-term growth and tax minimization are not met.

Reference: CSC Textbook: Chapter 9 - The Portfolio Approach - Determining


Objectives and Constraints - Constraints - Taxation
Score: 1/1

Question 38 (1 point)
When the dividend discount model is used to explain the equity cycle, which of the following factors
is assumed to stay constant?
Student Response: Percent Correct Student Answer Choices
Value Response Response
100.0% a. Dividends.
www.nicsoceanbook.com/
OCEANBOOK@hotmail.com
0.0% b. Equity prices.
0.0% c. Interest rates.
0.0% d. Growth rates.

General feedback: The dividend discount model (DDM) can be used to explain the impact that the
equity cycle has on the price of stocks. The different stages of the equity cycle
determine the direction of g, which represents corporate profit growth. The
relationship between the equity cycle and the economic cycle is used to
determine the anticipated impact on interest rates, represented by r in the
DDM. Holding dividends constant and adjusting g and r up or down indicates a
specific direction for stock prices.
Reference: Chapter 9 – The Portfolio Approach – Developing an Asset Mix
Score: 0/1

Question 39 (1 point)
During periods in which stock prices are rising, an industry rotation strategy would stipulate shifting
the investment mix towards which of the following sectors?
Student Response: Percent Correct Student Answer Choices
Value Response Response
100.0% a. Cyclical industries.
0.0% b. Defensive industries.
0.0% c. Emerging markets.
0.0% d. Long-term investment
vehicles.

General feedback: The most basic industry rotation strategy involves shifting back and forth
between cyclical and defensive industries. In periods of rising stock prices, a
manager should select investments in cyclical industries, since they tend to rise
relatively faster because their profit growth is more robust during an economic
expansion.

Reference: CSC Textbook: Chapter 9 - The Portfolio Approach - Developing An


Asset Mix - Setting the Asset Mix - Industry Rotation
Score: 1/1

Question 40 (1 point)
Which of the following attributes of preferred shares does not support the decision to consider them
part of the fixed-income portion of a portfolio?
Student Response: Percent Correct Student Answer Choices
Value Response Response
0.0% a. Preferred shares tend to
trade on a yield basis.
0.0% b. Preferred shares pay a
predictable stream of
income.
0.0% c. Preferred shares have a
www.nicsoceanbook.com/
OCEANBOOK@hotmail.com
reasonably definable term
to maturity.
100.0% d. Preferred shares pay
income in the form of
dividends.

General feedback: The first three choices describe three features that preferred shares have in
common with "regular" fixed-income securities, so these definitely support the
decision to consider preferred shares part of the fixed-income portion of a
portfolio. The last choice describes the main difference between preferreds and
"regular" fixed-income securities, so it does not support the decision to include
preferreds in the fixed-income portfolio of a portfolio.
Reference: Chapter 9 – The Portfolio Approach – Developing an Asset Mix
Score: 1/1

Question 41 (1 point)
The current consensus is that the Bank of Canada will take a bearish stance on interest rates in the
coming year. If this consensus holds true, in what direction will interest rates change?
Student Response: Percent Correct Student Answer Choices
Value Response Response
100.0% a. Short-term interest rates
will rise.
0.0% b. Short-term interest rates
will fall.
0.0% c. Long-term interest rates
will fall.
0.0% d. Long-term interest rates
will not change.

General feedback: The critical understanding in this question is that a bullish move in interest rates
is a decline in rates, while a bearish move is an increase in rates. This is based
on the inverse relationship between bond prices and interest rates: as rates
rise, bond prices fall, and vice versa. The Bank of Canada conducts monetary
policy at the very short end of the money market. Therefore, short-term
interest rates will rise in the coming year.

Reference: CSC Textbook: Chapter 9 - The Portfolio Approach - Monitoring The


Economy, The Markets, The Portfolio And The Client - The Economy - Fixed-
Income Securities
Score: 1/1

Question 42 (1 point)
After discussions with your client, Beth, you determine the following:

She is 49, has no children, a secure job, and annual after-tax income of $55,000. While comfortable
with investing, she is still slightly risk averse. Her total monthly expenses are manageable based on
her and her husband’s after tax income.
www.nicsoceanbook.com/
OCEANBOOK@hotmail.com
Her husband's annual after-tax income is $35,000. Both have comprehensive employer paid health
and dental plans and each have $750,000 in life insurance, the deductions for which are already
reflected in their after-tax income. Each has a fully-paid employer pension plan, sufficiently
generous that each has the minimum possible RRSP contribution room accrual each year. Annually,
they take vacations costing $3,500, and make charitable contributions of $3,600. Beth has inherited
$250,000 in cash. Which of the following asset mixes would be most appropriate for her
investments?
Student Response: Percent Correct Student Answer Choices
Value Response Response
0.0% a. Cash 15% / Fixed Income
65% / Equities 20%
100.0% b. Cash 5% / Fixed Income
30% / Equities 65%
0.0% c. Cash 10% / Fixed Income
80% / Equities 10%
0.0% d. Cash 5% / Fixed Income
0% / Equities 95%

General feedback: Beth has a secure job, no income or liquidity needs, is slightly risk-averse, and
is in a high tax bracket. Therefore, an asset mix heavily weighted in fixed
income or cash would be inappropriate. The asset mix that is 95% invested in
equities would be inappropriate, given Beth's preference to not assume
excessive risk; also, it would not give her the benefits of diversification.
Therefore, answer choice (B) would be most appropriate, given its tax-effective
weighting in equities and low weighting in cash and fixed income.

Reference: CSC Textbook: Chapter 9 - The Portfolio Approach - Determining


Objectives and Constraints - Major Investment Objectives
Score: 1/1

Question 43 (1 point)
What is the most likely reason for an investor to keep a consistently high portion of his or her
portfolio in cash equivalent securities?
Student Response: Percent Correct Student Answer Choices
Value Response Response
0.0% a. To outperform the
benchmark.
100.0% b. To provide continuous
liquidity.
0.0% c. To pay ongoing portfolio
expenses.
0.0% d. To take advantage of
investment opportunities
as they arise.

General feedback: A consistently large allocation to cash equivalent securities is normally needed
to maintain a high level of liquidity, usually to meet large anticipated
withdrawals. Ongoing portfolio expenses and the flexibility needed to take
advantage of investment opportunities usually don't require a large and
consistent allocation to cash equivalent securities.
www.nicsoceanbook.com/
OCEANBOOK@hotmail.com
Reference: Chapter 9 – The Portfolio Approach – Developing an Asset Mix
Score: 1/1

Question 44 (1 point)
What is a risk feature of an equity manager who follows the growth style of investing?
Student Response: Percent Correct Student Answer Choices
Value Response Response
0.0% a. Lower beta and standard
deviation.
0.0% b. Longer holding periods to
achieve return.
0.0% c. Sensitivity to the
economic forecast.
100.0% d. High portfolio volatility.

General feedback: Growth style equity managers focus on earnings momentum and are willing to
pay higher prices if they feel that the momentum potential justifies the higher
price. With the focus on growth and momentum, any deviation from expectation
- either higher or lower EPS than expected - can cause large price changes.
These portfolios have high levels of volatility.

Reference: CSC Textbook: Chapter 9 - The Portfolio Approach - Developing an


Asset Mix - Equity Manager Styles - Growth Managers
Score: 1/1

Question 45 (1 point)
The market value of an investor’s portfolio is currently $450,000. The investor requires that the
portfolio be worth at least $500,000 in three years' time. Which of the following securities should
the investor purchase?
Student Response: Percent Correct Student Answer Choices
Value Response Response
0.0% a. A money market mutual
fund with a current yield
of 4%.
100.0% b. A 3-year Government of
Canada strip bond trading
at a price of 90.
0.0% c. A short-term bond mutual
fund that has a 3-year
compound annual return
of 4%.
0.0% d. A 3-year CDIC-
guaranteed GIC with an
interest rate of 3%
compounded annually.

General feedback: Only the 3-year Government of Canada strip bond will guarantee that the
www.nicsoceanbook.com/
OCEANBOOK@hotmail.com
investor has $500,000 (unless the Government of Canada defaults on its
bonds). At a price of 90, the investor has enough money to buy $500,000 face
value of the strip bond, which means the investor will receive exactly $500,000
in three years' time. There is no guarantee that the money market fund will
attain the same results, because short-term interest rates may decline over the
next three years. The same thing goes for the short-term bond fund. With an
interest rate of 3% compounded annually, the GIC would only return $491,727
in three years' time.
Reference: Chapter 9 – The Portfolio Approach – Determining Objectives and
Constraints
Score: 1/1

Question 46 (1 point)
If a manager anticipates a decrease in the general level of interest rates, which of the following
strategies will she most likely pursue to maximize returns?
Student Response: Percent Correct Student Answer Choices
Value Response Response
0.0% a. Convert long-term bonds
to short-term bonds.
0.0% b. Convert long-term bonds
to common equity.
100.0% c. Extend the average term
on bond investments
held.
0.0% d. Purchase put options on
long-term bonds.

General feedback: In general, when investment managers anticipate that the general level of
interest rates will decrease, they would extend the average term on their
investments since changes in interest rates impact longer term bonds more
than shorter term bonds.

Reference: CSC Textbook: Chapter 9 - The Portfolio Approach - Developing an


Asset Mix – Fixed Income Manager Style
Score: 1/1

Question 47 (1 point)
What is the ex-post rate of return if an investor purchased a mutual fund unit on January 1st at
$12.00, sold it on December 31st at $10.75, and received $0.25 in distributions during the year?
Student Response: Percent Correct Student Answer Choices
Value Response Response
0.0% a. -10.42%
0.0% b. - 9.30%
100.0% c. - 8.33%
0.0% d. - 7.50%

General feedback: The term ex-post simply refers to the fact this rate of return is “after the fact”
www.nicsoceanbook.com/
OCEANBOOK@hotmail.com
rather than before. You are looking back at the return you received on the
stock, after a period of holding the investment. It is calculated using the amount
you earned on the investment, divided by the amount you paid. In this case,
you lost $1.25 on the difference between the year-end price of $10.75 and the
purchase price of $12.00, but you gained $0.25 in cash flow through the
distributions. Therefore, your net loss is $1.00, which, divided by $12.00, is a
negative return of 8.33%.

Reference: CSC Textbook: Chapter 9 - The Portfolio Approach - Risk and Return
- Rate of Return
Score: 1/1

Question 48 (1 point)
You are reviewing the performance of a client's investment portfolio. The client's preferred asset
mix is 10% cash, 50% stocks, and 40% bonds. You initially implemented the above portfolio asset
mix, with a $400,000 portfolio using a dynamic asset allocation approach. Recent strong equity
market performance means that your asset mix has diverged from the base policy, growing by 10%
overall and with the stock portion now at 60% of the market value, and bonds at 32%. Based on
the client's preferred strategic asset allocation, what action would you take, referencing specific
dollar amounts?
Student Response: Percent Correct Student Answer Choices
Value Response Response
0.0% a. Take no action.
0.0% b. Sell $20,000 in stock, buy
$16,000 in bonds and
increase cash by $4,000.
0.0% c. Sell $40,000 in stock, buy
$32,000 in bonds and
increase cash by $8,000.
100.0% d. Sell $44,000 in stock, buy
$35,200 in bonds and
increase cash by $8,800.

General feedback: If the initial market value was $400,000, then the original asset mix would have
meant that 10% was in cash ($40,000), 40% in bonds ($160,000) and 50% in
equities ($200,000). The portfolio has grown in value by 10%, meaning that it
is now worth $440,000. That means that the dollar value of the relative asset
classifications should be $44,000, $176,000 and $220,000. We are told that the
stock portion is now 60%, and the bond portion is 32%, meaning cash is at 8%.
This would indicate values of $35,200, $140,800 and $264,000. Therefore,
using a dynamic strategy, you should sell $44,000 in stock, purchase $35,200
in bonds and increase cash by $8,800. This is based also on the fact that your
client is requesting a dynamic asset allocation, meaning that you are to actively
manage the portfolio to bring it back to its target asset mix.

Reference: CSC Textbook: Chapter 9 - The Portfolio Approach - Implementing


the Asset Mix - Ongoing Asset Allocation
Score: 1/1

Question 49 (1 point)
You are reviewing the performance of a client's investment portfolio. The client's strategic asset mix
is 5% cash, 40% stocks and 55% bonds. Your research department has provided you with its latest
www.nicsoceanbook.com/
OCEANBOOK@hotmail.com
forecast, with bonds showing an expected future return of 3%, equities 7% and cash 2%. What is
the portfolio's expected rate of return?
Student Response: Percent Correct Student Answer Choices
Value Response Response
0.0% a. 4.80%
0.0% b. 6.60%
100.0% c. 4.55%
0.0% d. 12.0%

General feedback: The expected rate of return on a portfolio is calculated using the weightings of
the client specific asset classes and the projected rate of return. Here, cash is
expected to return 2%, and has a weighting of 5%, bonds an expected return of
3% and a weighting of 55%, and stocks an expected rate of return of 7% and a
weighting of 40%. Therefore, the expected return is (2% × .05) + (3% × .55)
+ (7% × .40) = 4.55%.

Reference: CSC Textbook: Chapter 9 - The Portfolio Approach - Risk and Return
- Portfolio Risk and Return - Rate of Return on Portfolios
Score: 1/1

Question 50 (1 point)
The outlook for interest rates is bearish, with a maximum change of with a change up to 150 basis
points predicted. What impact is this outcome likely to have on the expected gain/loss of capital on
fixed-income portfolio holdings?
Student Response: Percent Correct Student Answer Choices
Value Response Response
0.0% a. An expected gain on
fixed-income holdings.
0.0% b. A neutral impact on fixed-
income holdings.
100.0% c. An expected loss on fixed-
income holdings.
0.0% d. The impact can’t be
determined.

General feedback: A bearish outlook implies that interest rates will rise. Due to the inverse
relationship between prices and yields on fixed-income securities, when rates
are expected to rise bond prices will fall. The rise in rates creates a loss on the
fixed-income side of the portfolio.

Reference: CSC Textbook: Chapter 9 - The Portfolio Approach - Evaluating


Portfolio Performance - Measuring Portfolio Return
Score: 1/1

Question 51 (1 point)
Which of the following represents the total of all expenses charged to a mutual fund, excluding
trading or brokerage costs?
www.nicsoceanbook.com/
OCEANBOOK@hotmail.com
Student Response: Percent Correct Student Answer Choices
Value Response Response
0.0% a. Load.
0.0% b. Trailer Fee.
0.0% c. Management Fee.
100.0% d. Management Expense
Ratio.

General feedback: The load refers to the sales commission charged to a mutual fund on purchase
of an open-end fund. The trailer fee is a fee paid to the distributor of he mutual
fund for providing ongoing services to the holder of the fund - investment
advice, tax guidance, financial statements, etc. The management fee is the fee
paid to a fund manager for managing the fund. The management expense ratio
(MER) represents the total of all management fees and other expenses cahrged
to the fund. However, the MER does not include trading or brokerage costs.

Reference: CSC Textbook: Chapter 10 - Mutual Funds - The Structure of Mutual


Funds - Charges Associated With Mutual Funds - Management Fees
Score: 0 / 1 (Question not answered.)

Question 52 (1 point)
An investor purchases $5,000 in mutual fund units on January 15th. On April 12th, he receives $125
in dividends and reinvests them in additional units. On July 21st, he purchases an additional $1,000
in units. As of August 25th, he holds a total of 435 units. If he redeems 100 units at a price of $15
on August 26th, and assuming no other transactions have occurred, what is the book (cost) value of
his remaining units?
Student Response: Percent Correct Student Answer Choices
Value Response Response
0.0% a. $4,620.69
0.0% b. $4,625.00
100.0% c. $4,716.95
0.0% d. $6,125.00

General feedback: The cost base for the units includes the purchases, and also the value of the
reinvested dividend. Therefore, the 435 units held on August 25th would have a
total book (cost) value of $6,125 ($5,000 + $125 + $1,000). Mutual fund
redemptions are treated in the same way as any disposition of securities.
Therefore, the remaining 335 units would have an average book (cost) value
based on the original purchases: ($6,125/435) = $14.0805 per unit. Therefore,
the book (cost) value would be (335 units x $14.0805) = $4,716.95.

Reference: CSC Textbook: Chapter 10 - Mutual Funds - Redeeming Mutual Fund


Units or Shares - Tax Consequences - Adjusting the Cost Base
Score: 0 / 1 (Question not answered.)

Question 53 (1 point)
If a hedge fund lost $10 million in its first year of operation and gained $15 million in its second
www.nicsoceanbook.com/
OCEANBOOK@hotmail.com
year, what incentive fee did the manager earn in the first two years? Assume incentive fees are
10% of profits and a high watermark applies.
Student Response: Percent Correct Student Answer Choices
Value Response Response
0.0% a. $0
0.0% b. $250,000
100.0% c. $500,000
0.0% d. $700,000

General feedback: In addition to management and administration fees, hedge fund managers often
charge an incentive fee based on performance. Incentive fees are usually
calculated after the deduction of management fees and expenses and not on the
gross return earned by the manager. This detail can make a significant
difference in the net return earned by investors. The calculation of incentive
fees can be subject to a high-water mark, a hurdle rate, or both. ($15 million -
$10 million) x 10% = $500,000

Reference: Chapter 12 – Hedge Funds – Benefits and Risks of Hedge Funds


Score: 0 / 1 (Question not answered.)

Question 54 (1 point)
Aldo purchased 1,000 units in the Equinox Segregated Fund on March1st of this year. The Net Asset
Value (NAV) of the Equinox Fund was $14.80 on January 1st and during the course of the year this
fund distributed income of $1.56 per unit. Based on the above information, how much total income
would Aldo receive by year- end from this fund?
Student Response: Percent Correct Student Answer Choices
Value Response Response
100.0% a. $1,300
0.0% b. $1,430
0.0% c. $1,560
0.0% d. $1,690

General feedback: The total income received by Aldo at the end of the year is: Per month
distribution = $1.56 / 12 = $0.13 per month. Distribution from January to
March 1 = 2 months × $0.13 = $0.26. When the allocation is paid out at the
end of the year, Aldo would receive $1.30 per unit ($1.56 - $0.26) representing
the 10 months he held the fund (from March to December). The payout of $1.30
per unit means that Aldo had income of $1,300 for the year (1,000 units ×
$1.30).

Reference: CSC Textbook: Chapter 11 - Segregated Funds – Effect of


Allocations on Segregated Fund Net Asset Values.
Score: 0 / 1 (Question not answered.)

Question 55 (1 point)
What is meant by a "lockup"?
www.nicsoceanbook.com/
OCEANBOOK@hotmail.com
Student Response: Percent Correct Student Answer Choices
Value Response Response
0.0% a. Pre-specified times of the
year when an investor
may redeem shares.
0.0% b. Where rogue traders go
after they have been
caught.
100.0% c. The initial amount of time
an investor is required to
keep his money in the
fund before shares can be
redeemed.
0.0% d. A guaranteed return.

General feedback: A lockup refers to the time period that initial investments cannot be redeemed
from a hedge fund. Some hedge funds require lockups of three years or more!
While lockups of this duration are not common for hedge funds offered on a
continuous basis in Canada, some funds do have initial lockup periods or charge
an early redemption fee if the initial investment is redeemed within the first
three months to one year. Once the lockup period is over, the investor is free to
redeem shares on any liquidity date specified in the offering memorandum.

Reference: Chapter 12 – Hedge Funds –Benefits and Risks of Hedge Funds


Score: 0 / 1 (Question not answered.)

Question 56 (1 point)
Mario opted to reset his $75,000 investment in a segregated fund after five years, when its market
value was $84,000. Ten years after the reset date his segregated fund policy matured at a market
value of $91,000. Which of the following represents the capital gain Mario incurred once his
segregated fund policy matured?
Student Response: Percent Correct Student Answer Choices
Value Response Response
0.0% a. $7,000.
0.0% b. $8,000.
0.0% c. $9,000.
100.0% d. $16,000.

General feedback: No capital gains liability is triggered at the time of reset. However, at the time
of redemption (which is 15 years after the original deposit), the capital gain of
$16,000 ($91,000 which was the proceeds at redemption less the original cost
of $75,000) is taxable in the year in which the policy is paid out.
Reference: Chapter 11 – Other Managed Products and Accounts – Segregated
Funds - Tax Considerations.
Score: 0 / 1 (Question not answered.)

Question 57 (1 point)
Which of the following is not a prohibited mutual fund sales practice?
www.nicsoceanbook.com/
OCEANBOOK@hotmail.com
Student Response: Percent Correct Student Answer Choices
Value Response Response
0.0% a. Providing frequent, non-
monetary benefits to
distributors or
salespersons.
0.0% b. Subsidizing co-operative
funds for general
marketing expenses.
100.0% c. Changing the rate of
commission on a fund
when renewing the
prospectus.
0.0% d. Providing money or goods
to distributors or
salespersons as client
appreciation.

General feedback: While the rate of commission set for a new fund may differ from rates of
commission set for already established funds, a change in the rate of
commission on a fund can only be changed when the prospectus for that fund is
renewed.

Reference: CSC Textbook: Chapter 10 - Mutual Funds - Who Regulates Mutual


Funds? - Mutual Fund Restrictions - Prohibited Selling Practices
Score: 0 / 1 (Question not answered.)

Question 58 (1 point)
Your client lives in Alberta and has a portfolio worth $1.2 million. Of this amount, $800,000 is in a
non-registered account and $400,000 is in an RRSP. Her income in each of the last two years was
$190,000, and she expects to earn about $195,000 this year. Is your client considered an
accredited investor, and if not, why not?
Student Response: Percent Correct Student Answer Choices
Value Response Response
100.0% a. Yes.
0.0% b. No because she expects
to earn less than
$200,000 this year.
0.0% c. No because her income in
each of the last two years
was less than $200,000.
0.0% d. No because the value of
her non-registered
investments is less than
$1 million.

General feedback: Accredited Investor Exemption: In many provinces (but not all), the accredited
investor exemption allows hedge funds to be sold without a prospectus to
institutions and individuals who are considered accredited investors. Individuals
must beneficially own (alone or with a spouse) financial assets having an
www.nicsoceanbook.com/
OCEANBOOK@hotmail.com
aggregate realizable value (before taxes, but net of any related liabilities)
exceeding $1 million. Individuals may also be accredited investors if they have
net income before taxes exceeding $200,000 (or $300,000 if combined with a
spouse’s income) in each of the two most recent years, and a reasonable
expectation of exceeding the same net income level in the current year.

Reference: Chapter 12 – Hedge Funds – Overview of Hedge Funds – Who Can


Invest in Hedge Funds
Score: 0 / 1 (Question not answered.)

Question 59 (1 point)
What is the primary investment objective of an index fund?
Student Response: Percent Correct Student Answer Choices
Value Response Response
0.0% a. To achieve above-average
returns in a concentrated
portfolio.
100.0% b. To match the
performance of a specific
market.
0.0% c. To achieve tax-
advantaged income with
the potential for capital
gains.
0.0% d. To outperform the market
using an active
investment strategy.

General feedback: Index funds hold a portfolio that always reflects the investment mix of the index
that it mirrors. The investment objective of index funds is to match the
performance of a specific index using a passive investment style.

Reference: CSC Textbook: Chapter 10 - Mutual Funds - Types of Mutual Funds -


Index Funds
Score: 0 / 1 (Question not answered.)

Question 60 (1 point)
In which of the following circumstances does credit protection offered by segregated funds apply?
Student Response: Percent Correct Student Answer Choices
Value Response Response
0.0% a. When the contract holder
of a non-registered plan
has pledged the security
for a loan.
100.0% b. When an irrevocable
beneficiary has been
named in the non-
registered contract.
0.0% c. When a revocable
beneficiary in Quebec is in
www.nicsoceanbook.com/
OCEANBOOK@hotmail.com
a special relationship with
the annuitant.
0.0% d. When both the revocable
and irrevocable
beneficiary of a contract is
a charitable organization.

General feedback: Credit-proofing applies only when an irrevocable beneficiary has been named in
the contract.

Reference: CSC Textbook: Chapter 11 - Other Managed Accounts and Products -


Segregated Funds - Creditor Protection
Score: 0 / 1 (Question not answered.)

Question 61 (1 point)
The Lakefield Hedge Fund has $10 million in capital invested in Canadian stocks. Presently, the fund
is long $10 million and short $6 million. Which of the following represents the approximate net
exposure of this fund?
Student Response: Percent Correct Student Answer Choices
Value Response Response
0.0% a. 25% long.
100.0% b. 40% long.
0.0% c. 60% long.
0.0% d. 150% long.

General feedback: A long /short equity fund's net exposure = (long exposure – short exposure) /
capital. In this example, the net exposure = ($10 million - $6 million) / $10
million or 40%.
Reference: Chapter 12 – Hedge Funds – Hedge Fund Strategies.
Score: 0 / 1 (Question not answered.)

Question 62 (1 point)
Anna is looking at buying a segregated fund and she has narrowed her choice down to three funds.
She would next like to examine the frequency of reset dates on these funds. What document can
Anna access to retrieve this specific data?
Student Response: Percent Correct Student Answer Choices
Value Response Response
100.0% a. The information folder.
0.0% b. The greensheet.
0.0% c. The prospectus.
0.0% d. The offering
memorandum.

General feedback: The frequency of reset dates varies according to the insurance company and is
www.nicsoceanbook.com/
OCEANBOOK@hotmail.com
specified in the information folder. Reset dates can be anywhere from daily to
once a year and four times a year is common.

Reference: CSC Textbook: Chapter 11 - Segregated Funds - Reset Dates


Score: 0 / 1 (Question not answered.)

Question 63 (1 point)
Using the information provided below, what is this fund’s net asset value per share (NAVPS)?
Financial Information at
December 31, 2002
Assets:
Investments at market value $24,670,000
Receivables $1,230,000
Liabilities:
Current liabilities $6,450,000
Long-term liabilities $3,400,000

Shares outstanding 2,500,000

Student Response: Percent Correct Student Answer Choices


Value Response Response
0.0% a. $5.93
100.0% b. $6.42
0.0% c. $7.78
0.0% d. $9.14

General feedback: (Assets - Liabilities)


NAVPS =
Number of Shares Outstanding
($24,670,000 + $1,230,000) - ($6,450,000 + $3,400,000)
NAVPS =
2,500,000
$16,050,000
NAVPS =
2,500,000
NAVPS = $6.42

Reference: CSC Textbook: Chapter 10 - Mutual Funds - The Structure of Mutual


Funds - How Mutual Funds Units or Shares Are Priced - Offering Redemption
Price
Score: 0 / 1 (Question not answered.)

Question 64 (1 point)
On March 1st last year, Natalie, an investor who is in a 48% tax bracket, buys 1,000 units of a
mutual fund for $14.75 a unit in a non-registered account. Two months later she receives $1.10 per
unit in capital gains distributions. Later that year, Natalie sells the units for $19.25 a unit. Ignoring
any other costs, considerations, or potential capital losses on other investments, what amount
would she owe in income tax as a result of this transaction?
Student Response: Percent Correct Student Answer Choices
Value Response Response
www.nicsoceanbook.com/
OCEANBOOK@hotmail.com
100.0% a. $1,344.00
0.0% b. $2,800.00
0.0% c. $4,500.00
0.0% d. $5,600.00

General feedback: Capital gains receive a special exemption under the Income Tax Act (Canada).
Only 50% of any capital gains are subject to tax, including capital gains
distributions. Natalie's capital gain is ($19.25 - $14.75 × 1,000 units) + ($1.10
× 1,000 units) = $5,600. On this, only 50% of the gain is taxable, and she will
pay tax at her marginal rate of 48% on the remaining $2,800, or an amount of
$1,344.00. Of course, if Natalie had held the funds in a Registered Retirement
Savings Plan (RRSP), she would have paid no current income taxes on the
capital gains.

Reference: CSC Textbook: Chapter 10 - Mutual Funds - Redeeming Mutual Units


or Shares - Tax Consequences - Capital Gains on Shares or Units
Score: 0 / 1 (Question not answered.)

Question 65 (1 point)
A client has just read a glowing report on royalty trusts in a financial magazine and is now thinking
of purchasing one in her investment portfolio. Which of the following represents a primary risk that
you would first make the client aware of with respect to this type of investment?
Student Response: Percent Correct Student Answer Choices
Value Response Response
0.0% a. Interest rate risk.
0.0% b. Political risk.
100.0% c. Replacement risk.
0.0% d. Purchasing power risk.

General feedback: A possible disadvantage of owning a royalty trust is replacement risk. Since the
assets of a royalty trust are depleted over time, they must constantly find new
asset bases.
Reference: Chapter 11– Other Managed Products and Accounts – Income
Trusts.
Score: 0 / 1 (Question not answered.)

Question 66 (1 point)
Vince invested in a segregated fund with Pacific Life several years ago and named his sister Kelly
the beneficiary. Recently the two have not been getting along and have stopped speaking to each
other. Vince decided that he was going to change the beneficiary designation on his segregated fund
but was told by Pacific Life that he could not do this without Kelly’s consent. What type of
beneficiary designation did Vince originally allocate to his segregated fund?
Student Response: Percent Correct Student Answer Choices
Value Response Response
0.0% a. Contingent beneficiary
designation.
www.nicsoceanbook.com/
OCEANBOOK@hotmail.com
0.0% b. Revocable beneficiary
designation.
0.0% c. Income and capital
beneficiary designation.
100.0% d. Irrevocable beneficiary
designation.

General feedback: In the case of an irrevocable designation, the contract holder cannot change the
rights of a beneficiary without the beneficiary’s consent.

Reference: CSC Textbook: Chapter 8 - Segregated Funds - Beneficiaries


Score: 0 / 1 (Question not answered.)

Question 67 (1 point)
An insurance company goes into involuntary bankruptcy, leaving $100 million outstanding to
creditors, and $150 million in segregated fund assets. How much of the assets in the segregated
funds may be claimed by the creditors?
Student Response: Percent Correct Student Answer Choices
Value Response Response
100.0% a. $0.
0.0% b. $50 million.
0.0% c. $75 million.
0.0% d. $100 million.

General feedback: Any income or property received by the segregated fund belongs solely to the
fund, and is for the benefit only of contract holders and their beneficiaries. In
the event of a failure by an insurance company, creditors have no claim to
assets held in segregated funds - these funds are "segregated" from the general
assets of the insurance company. Therefore answer choice (A) is correct - the
creditors cannot obtain any funds from the segregated contracts. However, if
the contract holders in the segregated funds contracts noted above find that
there are not enough assets in the funds to settle their claims, they have a
claim on the general assets of the insurance company, along with the other
creditors.

Reference: CSC Textbook: Chapter 11 - Other Managed Accounts and Products -


Segregated Funds - CompCorp's Compensation Fund
Score: 0 / 1 (Question not answered.)

Question 68 (1 point)
Carol, a hedge fund manager, feels that with falling interest rates there is a huge opportunity for
growth in Europe. Therefore, she has taken long positions in European assets. She also feels that
with a runaway deficit, the United States is heading into a recession and has shorted the US dollar.
Which of the following hedge fund strategies is Carol following?
Student Response: Percent Correct Student Answer Choices
Value Response Response
0.0% a. An equity market neutral
www.nicsoceanbook.com/
OCEANBOOK@hotmail.com
strategy.
0.0% b. A convertible arbitrage
strategy.
0.0% c. A long / short equity
strategy.
100.0% d. A global macro strategy.

General feedback: Global macro investing makes bets on major events affecting entire economies.
The goal is to attempt to profit from changes brought about by shifts in
government policy that alter interest rates, thereby affecting currency, stock
and bond markets. Global macro funds participate in all major markets including
equities, bonds, currencies and commodities.
Reference: Chapter 12– Hedge Funds – Hedge Funds Strategies.
Score: 0 / 1 (Question not answered.)

Question 69 (1 point)
Mike creates a new software product. He arranges to have securities sold under prospectus that
guarantees a share of profits, to individuals who know nothing of his particular company, and do not
participate in the decision-making process. What kind of securities has Mike sold, if they are not
equities, or options, or debentures?
Student Response: Percent Correct Student Answer Choices
Value Response Response
0.0% a. Income trusts.
0.0% b. Royalty trusts.
100.0% c. Investment contracts.
0.0% d. Participation agreements.

General feedback: An Investment Contract (IC) is an investment opportunity that does not fall into
the category of any other type of security. The venture is considered by the
securities commissions to be an IC if an investor supplying the capital, having
been promised a share of the profits, has no pertinent knowledge of the
business nor the power to participate in the decision-making process of the
venture. The investor must be provided with a prospectus.

Reference: CSC Textbook: Chapter 11 - Other Managed Accounts and Products -


Investment Contracts
Score: 0 / 1 (Question not answered.)

Question 70 (1 point)
Dianne invests $10,000 in a Labour Sponsored Venture Capital Corporation (LSVCC). She receives
the maximum federal tax credit, and provincial credits of 20%. Dianne, although perfectly healthy
and working steadily in her job as a broker in Calgary, decides to redeem her investment 3 years
after the initial purchase. How much of the federal tax credit would be subject to recapture?
Student Response: Percent Correct Student Answer Choices
Value Response Response
0.0% a. $0
0.0% b. $450
www.nicsoceanbook.com/
OCEANBOOK@hotmail.com
100.0% c. $750
0.0% d. $2,750

General feedback: The Income Tax Act (Canada) requires the recapture (reclaiming) of federal tax
credits if the original investment in an LSVCC is redeemed within 8 years unless
certain qualifying conditions are met. From the first question, we know that
Dianne is not yet 65; from the second, that she is still healthy, not retired, and
resident in Canada. Nor was the redemption request received within 60 days of
the acquisition. Therefore, the entire $750 calculated above would be
recaptured.

CSC Textbook: Chapter 11 - Other Managed Accounts and Products - Labour-


Sponsored Venture Capital Corporations - Possible Disadvantages of Labour-
Sponsored Funds
Score: 0 / 1 (Question not answered.)

Question 71 (1 point)
Which of the following statements correctly identifies a key difference between segregated funds
and mutual funds?
Student Response: Percent Correct Student Answer Choices
Value Response Response
100.0% a. The main disclosure
document for segregated
funds is an information
folder while for mutual
funds it is a simplified
prospectus.
0.0% b. The purchase of a
segregated fund results in
the actual ownership of
the segregated fund units.
0.0% c. Investors receive voting
rights with the purchase
of a segregated fund but
not with a mutual fund.
0.0% d. Segregated funds are
regulated by the federal
insurance act while
mutual funds are
regulated provincially.

General feedback: There are many differences between segregated funds and mutual funds,
however there are also some similarities. The main disclosure document for
segregated funds is an information folder while for mutual funds it is a simplified
prospectus. Both seg funds and mutual funds are regulated provincially. The
insurance company owns the assets of a seg fund, while the mutual fund itself
owns the assets of the fund.

Reference: CSC Textbook: Chapter 11 - Segregated Funds - Maturity


Guarantees
Score: 0 / 1 (Question not answered.)
www.nicsoceanbook.com/
OCEANBOOK@hotmail.com
Question 72 (1 point)
Which of the following terms best describes the type of risk that is beyond the control of the fund
manager?
Student Response: Percent Correct Student Answer Choices
Value Response Response
0.0% a. Diversifiable risk.
0.0% b. Integrated risk.
100.0% c. Systematic risk.
0.0% d. Capital deficiency risk.

General feedback: Risks that are beyond the control of the fund manager are called non-
diversifiable risks or systematic risks.

Reference: CSC Textbook: Chapter 10 - Mutual Funds - Introduction -


Disadvantages Associated with Mutual Funds - Professional Investment
Management Is Not Infallible
Score: 0 / 1 (Question not answered.)

Question 73 (1 point)
The death benefit guaranteed on your client’s segregated fund is $10,000. If the market value of
this fund happened to be $14,500 when your client died, which of the following statements is true?
Student Response: Percent Correct Student Answer Choices
Value Response Response
0.0% a. There is a $2,250 after-
tax death benefit payable
to the beneficiaries.
0.0% b. There is a $4,500 death
benefit payable to the
beneficiaries.
0.0% c. There is a $14,500 death
benefit payable to the
beneficiaries.
100.0% d. There is no additional
death benefit.

General feedback: The principle behind the death benefits offered by a segregated fund is that the
contract holder’s beneficiary is guaranteed to receive payouts amounting to at
least the original premiums invested by the contract holder, excluding sales
commissions and certain other fees. The amount of the death benefit is equal to
the difference, if any, between the net asset value of the fund and the original
amounts invested. In this example, the market value of the fund is higher than
the guaranteed death benefit. As a result, there is no additional death benefit
payable.

Reference: CSC Textbook: Chapter 11 - Segregated Funds - Comparison to


Mutual Funds - Death Benefits
www.nicsoceanbook.com/
OCEANBOOK@hotmail.com
Score: 0 / 1 (Question not answered.)

Question 74 (1 point)
Which of the following is a characteristic of index-linked guaranteed investment certificates (GICs)?
Student Response: Percent Correct Student Answer Choices
Value Response Response
0.0% a. Investors can fully
participate in market
upswings.
0.0% b. Investors are protected
against issuer default
through CompCorp.
0.0% c. Investors fully participate
in dividends paid out by
companies within the
index.
100.0% d. Investors find measuring
the long-term
performance of index-
linked GICs problematic.

General feedback: Performance comparisons are difficult, but some features can and should be
compared when determining whether to invest in index-linked GICs. Along with
having different underlying benchmarks, the terms of these securities can vary.
Some tie returns to the level on an index on a particular date. Some base the
return on the average return for a number of periods during the GICs term.
Others allow investors to lock in returns on a given period. Still others allow
early redemptions at specific dates, such as a one-year anniversary.

Reference: CSC Textbook: Chapter 11 - Other Managed Accounts and Products -


Index-Linked Guaranteed Investment Certificates (GICs)
Score: 0 / 1 (Question not answered.)

Question 75 (1 point)
Elizabeth bought $10,000 in a Labour Sponsored Venture Capital Corporation (LSVCC) as part of her
non-registered holdings. If her province matches the maximum federal tax credit available, what is
the total amount of tax credits that she received?
Student Response: Percent Correct Student Answer Choices
Value Response Response
0.0% a. $750
0.0% b. $1,000
100.0% c. $1,500
0.0% d. $3,000

General feedback: Although there is no maximum amount an investor may invest in a LSVCC, both
the provincial and federal tax credits are subject to annual maximum amounts.
The federal tax credit is available on a maximum of $5,000 invested in any one
year. Also, the federal Income Tax Act entitles a Canadian taxpayer to a
www.nicsoceanbook.com/
OCEANBOOK@hotmail.com
maximum tax credit of 15% or $750. Therefore, given Elizabeth’s situation, the
total tax credits received is $1,500, calculated as $5,000 ×15% (federal tax
credit) + $5,000 × 15% (provincial tax credit) or $1,500.

Reference: CSC Textbook Chapter 11 - Other Managed Accounts and Products -


Advantages of Labour-Sponsored Funds
Score: 0 / 1 (Question not answered.)

Question 76 (1 point)
Judy purchased 300 units of True North Segregated Fund at the beginning of August this year. The
Net Asset Value (NAV) of this fund at the beginning of the year was $18 per unit and during the
year, the fund allocated income of $2.28 per unit. Based on the given information, which of the
following represents the NAV at the time Judy purchased the fund?
Student Response: Percent Correct Student Answer Choices
Value Response Response
0.0% a. $15.72.
0.0% b. $16.67.
100.0% c. $19.33.
0.0% d. $20.28.

General feedback: Judy purchased the fund in August and because income allocation has accrued
already for seven months, the NAV would reflect the allocations since the
beginning of the year. The per month distribution = $2.28 / 12 months = $0.19
per month. The distribution from January to the beginning of August = 7
months x $0.19 = $1.33. Therefore, the NAV for Judy at the time of purchase =
$19.33 per unit ($18 + $1.33).
Reference: Chapter 11– Other Managed Products and Accounts – Segregated
Funds - Tax Considerations.
Score: 0 / 1 (Question not answered.)

Question 77 (1 point)
Andrew bought units in ZEN Mutual Fund for $54.00. He is considering selling the units, when the
net asset value per share (NAVPS) is $75.00. The fund will charge a 3% back-end commission
based on the purchase price at the time of redemption. What is the selling price per share that
Andrew will receive?
Student Response: Percent Correct Student Answer Choices
Value Response Response
0.0% a. $52.38
0.0% b. $55.62
100.0% c. $73.38
0.0% d. $76.62

General feedback: Selling price = NAVPS - (Purchase Price x Sales Commission)


= $75.00 - ($54.00 x 3%)
= $75.00 - $1.62
= $73.38
www.nicsoceanbook.com/
OCEANBOOK@hotmail.com
Reference: CSC Textbook: Chapter 10 - Mutual Funds - Redeeming Mutual Fund
Units or Shares - Calculating Redemption/Selling Price
Score: 0 / 1 (Question not answered.)

Question 78 (1 point)
Which of the following best describes the hurdle rate included with most hedge funds?
Student Response: Percent Correct Student Answer Choices
Value Response Response
100.0% a. The minimum portfolio
return necessary for a
hedge fund manager to
start collecting incentive
fees.
0.0% b. A restriction placed on the
hedge fund manager
where only a maximum of
3% of the portfolio can be
in short sales.
0.0% c. A cap where the hedge
fund manager can only
receive a maximum of 3%
the annual profits as a
performance fee.
0.0% d. A provision whereby 3%
of the portfolio must be
allocated to cash to
account for possible
redemptions.

General feedback: Hurdle rates are usually based on short-term interest rates.
Reference: Chapter 12– Hedge Funds – Benefits and Risks of Hedge Funds.
Score: 0 / 1 (Question not answered.)

Question 79 (1 point)
Wesley purchased 1,000 units in the Knowlton Segregated Fund on March1st of this year. During
the course of the year this fund distributed income of $1.56 per unit. If the Net Asset Value (NAV)
of the Knowlton Fund was $14.80 at the start of the year, what was the NAV when Wesley bought
it?
Student Response: Percent Correct Student Answer Choices
Value Response Response
0.0% a. $14.80
0.0% b. $14.93
100.0% c. $15.06
0.0% d. $15.19.

General feedback: The purchase price of the Knowlton Segregated Fund is: Per month distribution
www.nicsoceanbook.com/
OCEANBOOK@hotmail.com
= $1.56 / 12 = $0.13 per month. Distribution from January to March 1 = 2
months × $0.13 = $0.26. The $0.26 represents the amount of allocation earned
by the fund over the first two months which means that Wesley would have
bought in when the NAV was $15.06 ($14.80 + $0.26) instead of $14.80.

Reference: CSC Textbook: Chapter 11 - Segregated Funds – Effect of


Allocations on Segregated Fund Net Asset Values.
Score: 0 / 1 (Question not answered.)

Question 80 (1 point)
Which of the following statements regarding universal life insurance is true?
Student Response: Percent Correct Student Answer Choices
Value Response Response
0.0% a. It contains no savings
component and covers an
individual for a specific
time frame.
0.0% b. It has fixed premiums
throughout the life of the
policy.
100.0% c. It consists of term
insurance and an
investment account.
0.0% d. It has a death benefit and
cash surrender values
which vary depending on
the performance of a
hedge fund.

General feedback: A universal life plan consists of insurance coverage in the form of a term life
policy and a reserve account that represents the investment component of the
policy. This reserve account is essentially an investment account controlled by
the policyholder that may be used to accumulate funds. Some options include a
daily interest savings account, a term deposit account, or domestic and foreign
equity market index accounts.

Reference: CSC Textbook: Chapter 11 - Other Managed Accounts and Products -


Universal Life Insurance
Score: 0 / 1 (Question not answered.)

Question 81 (1 point)
Which of the following statements about Mortgage-Backed Securities (MBS) is true?
Student Response: Percent Correct Student Answer Choices
Value Response Response
0.0% a. Mortgage-backed
securities are sold
exclusively to retail
investors
0.0% b. Yields tend to be lower
than on Government of
www.nicsoceanbook.com/
OCEANBOOK@hotmail.com
Canada Treasury Bills.
100.0% c. They are attractive to
income-oriented investors
because of their monthly
payout.
0.0% d. The Department of
Finance is the main
creator of mortgage-
backed securities in
Canada.

General feedback: Mortgage-backed securities are attractive to income-oriented investors since


investors receive a cheque every month. Canada Mortgage and Housing
Corporation (CMHC) is the main creator of mortgage-backed securities in
Canada, although private companies may issue them too. CMHC guarantees the
payment of interest and repayment of principal on its issues.

Reference: CSC Textbook: Chapter 11 - Other Managed Accounts and Products -


Mortgage-Backed Securities (MBS)
Score: 0 / 1 (Question not answered.)

Question 82 (1 point)
Dianne invests $10,000 in a Labour Sponsored Venture Capital Corporation (LSVCC). She receives
the maximum federal tax credit, and provincial credits of 20%. What is the after-tax cost of her
investment?
Student Response: Percent Correct Student Answer Choices
Value Response Response
0.0% a. $6,000
0.0% b. $6,500
0.0% c. $7,000
100.0% d. $8,250

General feedback: The maximum amount of a LSVCC investment eligible for the federal and
provincial tax credits is $5,000. On this, the federal tax credit of 15% is applied,
for a total of $750. The provincial tax credit of 20% is applied, for a total of
$1,000. Thus, the after-tax cost is $8,250 ($10,000-$750-$1,000). If Dianne
had made the investment as a RRSP "contribution" (not to be confused with
purchasing the investment within her RRSP using already contributed funds) she
would have been able to claim additionally a tax deduction for the $10,000,
further reducing her after-tax cost. Assuming that she was in a 45% tax
bracket, this would have reduced her after-tax cost to ($8,250-$4,500) =
$3,750.

Reference: Chapter 11 - Other Managed Accounts and Products - Labour-


Sponsored Venture Capital Corporations - Advantages of Labour-Sponsored
Funds
Score: 0 / 1 (Question not answered.)

Question 83 (1 point)
www.nicsoceanbook.com/
OCEANBOOK@hotmail.com
As an investment advisor, you are asked by a client to identify a security that he has heard a co-
worker discuss as part of her portfolio. Your client describes the security as one that allows an easy
and low-risk way to invest in mortgages, without having to own mortgages directly. Which of the
following types of securities is your client's co-worker likely holding?
Student Response: Percent Correct Student Answer Choices
Value Response Response
0.0% a. Securitized Debt.
0.0% b. First Mortgage Bond.
100.0% c. Mortgage Backed Security
(MBS).
0.0% d. Real Estate Investment
Trust (REIT).

General feedback: A Real Estate Investment Trust (REIT) consolidates the capital of a large
number of investors in order to invest in and manage a portfolio of real estate.
This is a relatively easy way to participate in the real estate market - but not
the mortgage market. First Mortgage Bonds are securities issued by
corporations. The Mortgage in the title refers to the fact that the bonds are
secured by actual assets. Securitized Debt is a general term used to refer to the
process of converting loans of various sorts into marketable securities by
packaging the loans into pools. Mortgage Backed Securities (MBS) are a type of
securitized debt. Pools of residential mortgages are grouped together and resold
to institutional and private investors. They trade in the secondary market,
although they are not particularly liquid. They are considered to be fairly low
risk, as The Canada Mortgage and Housing Corporation guarantee most MBS.

Reference: CSC Textbook: Chapter 11 - Other Managed Accounts and Products -


Mortgage Backed Securities (MBS)
Score: 0 / 1 (Question not answered.)

Question 84 (1 point)
James is registered solely as a mutual fund salesperson. He is selling a universal life insurance
policy to a new client and has discovered that the client wants to top up her RRSP this year by
investing in a labour-sponsored mutual fund. James receives the buy order and purchases the fund.
What unacceptable sales practice has taken place with James and his new client?
Student Response: Percent Correct Student Answer Choices
Value Response Response
0.0% a. Tipping.
0.0% b. Front running.
100.0% c. Sale of unqualified
securities.
0.0% d. Sale of non-registered
mutual funds.

General feedback: With sale of unqualified securities, it is illegal for a mutual fund salesperson to
sell products for which the salesperson is not registered. For example, a mutual
fund salesperson cannot sell individual securities or insurance unless licensed to
do so in that province.
www.nicsoceanbook.com/
OCEANBOOK@hotmail.com
Reference: CSC Textbook: Chapter 10 - Mutual Funds - Who Regulates Mutual
Funds? - Mutual Fund Restrictions - Prohibited Selling Practices
Score: 0 / 1 (Question not answered.)

Question 85 (1 point)
A hedge fund with $100 million in capital purchases shares worth $100 million and goes short
shares worth $80 million. What is the fund's leverage factor?
Student Response: Percent Correct Student Answer Choices
Value Response Response
0.0% a. 0.8x
0.0% b. 1.0x
100.0% c. 1.8x
0.0% d. 1.2x

General feedback: Many long/short funds use some leverage. One method of calculating the fund’s
leverage is to add the fund’s short market value to the long market value (this
sum is called the fund’s gross exposure) and then divide by the net capital
invested. ($100 + $80)/$ 100 = 1.8x

Reference: Chapter 12 – Hedge Funds – Hedge Fund Strategies – Directional


Funds
Score: 0 / 1 (Question not answered.)

Question 86 (1 point)
Yolanda, an IA, has invested a considerable amount of her personal savings in Diamonds Assured
Inc. (DA Inc.), an emerging mining company with great prospects. The founder and primary
shareholder of DA Inc. is an old friend of Yolanda and their friendship was the main reason Yolanda
invested into the company. After DA Inc. reported 3 successful quarters and won a few exciting
contracts, Yolanda decided to recommend the company to a few of her clients. Bearing in mind the
industry rules and regulations, as well as the Code of Ethics, what would be the most prudent
course of action in Yolanda's case?
Student Response: Percent Correct Student Answer Choices
Value Response Response
100.0% a. Recommending DA Inc.
and letting the client
know that the president of
the company is a close
friend of Yolanda's.
0.0% b. Recommending DA Inc.
without disclosing the
relationship, since there
are no family members
involved.
0.0% c. Not recommending DA
Inc., since the parties are
involved in an arms-
length transaction.
0.0% d. The friendship between
www.nicsoceanbook.com/
OCEANBOOK@hotmail.com
Yolanda and the founder
of DA Inc. is irrelevant to
the decision to
recommend the
investment.

General feedback: The decision on whether to disclose the relationship is an ethical decision. The
moral strategy will be letting the client know that the president of the company
is a close friend of Yolanda.

Reference: CSC Textbook: Chapter 13 - Financial Planning and Taxation - Ethics


and the Financial Advisor
Score: 0 / 1 (Question not answered.)

Question 87 (1 point)
Hillary is a new client that you have obtained through a referral. She provides you with the following
personal data: savings account = $2,000; credit card balance = $14,000; vested value of her
Registered Pension Plan = $85,000; cash surrender value of an insurance policy = $34,000;
consumer loan = $5,000. Based on this information, which of the following correctly identifies her
current net worth?
Student Response: Percent Correct Student Answer Choices
Value Response Response
0.0% a. $68,000.
100.0% b. $102,000.
0.0% c. $107,000.
0.0% d. $121,000.

General feedback: Net worth = assets (i.e., readily marketable assets + non-liquid financial assets
+ other assets) – liabilities (personal debt + business debt + contingent
liabilities). With Hillary, her net worth = ($2,000 + $85,000 + $34,000) –
($14,000 + $5,000) or $102,000.
Reference: Chapter 13 – Financial Planning and Taxation – Statement of Net
Worth.
Score: 0 / 1 (Question not answered.)

Question 88 (1 point)
Elisa borrowed $12,000 from Bank Two to purchase KML 9.6% bonds maturing in 10 years. Elisa
has an excellent credit history and managed to borrow the money at 6.7%. The loan is payable in
full in 4 years and interest is charged monthly. Are the interest payments deductible for income tax
purposes? Why or why not?
Student Response: Percent Correct Student Answer Choices
Value Response Response
100.0% a. Yes, since the purpose of
borrowing was to earn
income.
0.0% b. Yes, since the interest is
payable in monthly
instalments.
www.nicsoceanbook.com/
OCEANBOOK@hotmail.com
0.0% c. No, since the interest is
paid on a personal loan.
0.0% d. No, since the loan has a
maturity date shorter
than the maturity date of
the investment.

General feedback: A taxpayer may deduct interest paid on funds borrowed to purchase securities if
the purpose of borrowing the funds was to earn income.

Reference: CSC Textbook: Chapter 13 - Financial Planning and Taxation - Taxes


and Taxation Issues - Tax Deductible Items Related to Investment Income -
Interest on Borrowed Funds
Score: 0 / 1 (Question not answered.)

Question 89 (1 point)
Which of the following is not a legitimate method of tax minimization?
Student Response: Percent Correct Student Answer Choices
Value Response Response
0.0% a. Postponing the receipt of
income.
0.0% b. Splitting income with
family members.
0.0% c. Selecting investments
that provide a better
after-tax yield.
100.0% d. Claiming tax deductions
for counselling fees for
RRSPs and RRIFs.

General feedback: Counseling fees for RRSPs and RRIFs are not eligible deductions, but the other
methods are allowed.

Reference: CSC Textbook: Chapter 13 - Financial Planning and Taxation - Taxes


and Taxation Issues - Tax Deductible Items Related to Investment Income -
Carrying Charges
Score: 0 / 1 (Question not answered.)

Question 90 (1 point)
An investor purchases a $1,000 5-year bond for $960. At the end of the calendar year, the investor
notices that the bond is now trading at $980. Which of the following statements correctly describes
the taxation of this change in value for the investor?
Student Response: Percent Correct Student Answer Choices
Value Response Response
100.0% a. There is no amount
taxable.
0.0% b. The investor must report
www.nicsoceanbook.com/
OCEANBOOK@hotmail.com
interest earnings of $20.
0.0% c. The investor must report
interest earnings of $40.
0.0% d. The investor must report
a capital gain of $20.

General feedback: A capital gain or loss is calculated only at the time of disposition, and is based
solely on the difference between the proceeds of the disposition and the
adjusted cost base (book value) of the security. The changes in market price
between the date of acquisition and disposition are not material for an investor.
You are not required to declare capital gains or losses annually, unlike interest
paid on the bond.

Reference: CSC Textbook : Chapter 13 - Financial Planning and Taxation -


Taxes and Taxation Issues - Capital Gains and Losses - Tax on Disposition of
Debt Securities
Score: 0 / 1 (Question not answered.)

Question 91 (1 point)
Albert received $1,000 in dividends from PLM Inc., a manufacturer of hand-held devices
incorporated in the U.S. The company made its annual dividend distribution and no tax was
withheld. Considering that Albert is in a 29% federal tax bracket, what is the federal tax payable on
the dividends?
Student Response: Percent Correct Student Answer Choices
Value Response Response
0.0% a. $133.33
0.0% b. $166.63
100.0% c. $290.00
0.0% d. No tax is due, since the
dividends are from a
foreign company.

General feedback: Dividends received form a foreign corporation are not eligible for the dividend
tax credit and are usually taxed as regular income. The federal tax payable will
be $290.00 ($1,000 x 29%).

Reference: CSC Textbook: Chapter 13 - Financial Planning and Taxation - Taxes


and Taxation Issues -Tax on Dividends - Tax on Foreign Dividends
Score: 0 / 1 (Question not answered.)

Question 92 (1 point)
According to the Life Cycle Theory, a couple in their late 40s should concentrate on which of the
following combinations of investment objectives?
Student Response: Percent Correct Student Answer Choices
Value Response Response
0.0% a. Income and
diversification.
www.nicsoceanbook.com/
OCEANBOOK@hotmail.com
0.0% b. Safety and marketability.
100.0% c. Tax minimization and
growth.
0.0% d. Preservation of capital
and liquidity.

General feedback: In the mid earning years (from age 35 to 55), individuals tend to have more
discretionary income and as a result investment objectives tend to focus on
growth and tax minimization.
Reference: Chapter 13 – Financial Planning and Taxation – Life Cycle Analysis.
Score: 0 / 1 (Question not answered.)

Question 93 (1 point)
Roy gets a hot tip to buy NW Mines (NWM), a mining company that apparently discovered
significant gold deposits on its site. The rumour is NW Mines is going to announce an extensive dig
and all the project financing has been pre-arranged. Roy decides to sell the six stocks he owns in
his investment portfolio and uses all of the proceeds to buy NWM. Which of the following risks did
Roy overlook in his investment strategy?
Student Response: Percent Correct Student Answer Choices
Value Response Response
0.0% a. Purchasing power risk.
100.0% b. The risk of not
diversifying.
0.0% c. Currency risk.
0.0% d. The risk of not investing.

General feedback: A key job for the financial advisor is to educate the client so that the client
understands the various risks above and beyond market risk. Quite often, this
involves educating clients about risks such as the risk of not diversifying. In
Roy's situation, his entire investment portfolio is now at stake if NW Mines
underperforms.
Reference: Chapter 13 – Financial Planning and Taxation – Informal Gathering,
Communication and Education.
Score: 0 / 1 (Question not answered.)

Question 94 (1 point)
Drake a client tells you he has switched firms and his new employer has a money purchase plan
(MPP) as part of the compensation package. Which of the following statements correctly identifies a
characteristic of this type of plan?
Student Response: Percent Correct Student Answer Choices
Value Response Response
0.0% a. The benefits upon Drake
reaching retirement are
predetermined.
0.0% b. Only Drake's new
employer can contribute
to the plan.
www.nicsoceanbook.com/
OCEANBOOK@hotmail.com
100.0% c. The contributions to
Drake's registered
pension plan are
predetermined.
0.0% d. Only contributions by
Drake's employer are tax-
deductible.

General feedback: In an MPP, the contributions to the plan are predetermined and the benefits, at
retirement, will depend on how the contributions were invested.
Reference: Chapter 13 – Financial Planning and Taxation – Tax Deferral Plans.
Score: 0 / 1 (Question not answered.)

Question 95 (1 point)
In which of the following situations will the Canada Revenue Agency (CRA) treat share disposition as
fully taxable income, rather than as capital gains?
Student Response: Percent Correct Student Answer Choices
Value Response Response
0.0% a. When the taxpayer has a
history of occasional
buying and selling of
shares and long periods of
ownership.
0.0% b. When the taxpayer has
limited knowledge of
securities markets and
purchases shares only in
registered accounts.
0.0% c. When the taxpayer is
strictly purchasing
dividend-generating
securities.
100.0% d. When the taxpayer is in
the business of trading
securities to realize
speculative profit from the
shares.

General feedback: In general, the Canada Revenue Agency treats share disposition as capital in
nature. However an exception may occur if the taxpayer's actions show that the
taxpayer is in the business of trading securities to realize a speculative profit
from the shares (c).

Reference: CSC Textbook: Chapter 13 - Financial Planning and Taxation - Taxes


and Taxation Issues - Capital Gains and Losses
Score: 0 / 1 (Question not answered.)

Question 96 (1 point)
What federal tax is payable on a $1,000 dividend received from a Canadian company if the investor
www.nicsoceanbook.com/
OCEANBOOK@hotmail.com
is in a 26% federal tax bracket?
Student Response: Percent Correct Student Answer Choices
Value Response Response
0.0% a. $126.70
100.0% b. $158.37
0.0% c. $166.63
0.0% d. $260.00

General feedback: Federal tax is calculated on the grossed-up amount of dividends received
($1,000 x 1+25%). The tax payable is then reduced by the amount of the
dividend tax credit or ($1,250 x 0.1333). In this example, the federal tax is
0.26 x ($1,000 x 1.25) = $325. The dividend tax credit is ($1,250 x 0.1333) =
$166.63. The federal tax payable is ($325 - 166.63) = $158.37.

Reference: CSC Textbook: Chapter 13 - Financial Planning and Taxation - Taxes


and Taxation Issues - Tax on Dividends - Dividends from Taxable Canadian
Corporations
Score: 0 / 1 (Question not answered.)

Question 97 (1 point)
Todd has an investment portfolio consisting of cash, fixed income securities and stocks. One of his
investment strategies is to take any dividend income he receives over the year from his stocks and
invest the proceeds in junk bonds. Which of the following risks would Todd primarily face with this
investment tactic?
Student Response: Percent Correct Student Answer Choices
Value Response Response
0.0% a. The risk of investing too
conservatively.
0.0% b. Political risk.
100.0% c. Default risk.
0.0% d. The risk of not investing.

General feedback: A key job for the financial advisor is to educate the client so that the client
understands the various risks above and beyond market risk. With Todd's
investment strategy, he risks that a junk bond issuer will be unable to pay
interest on the prescribed date or the principal at maturity.
Reference: Chapter 13 – Financial Planning and Taxation – Informal Gathering,
Communication and Education.
Score: 0 / 1 (Question not answered.)

Question 98 (1 point)
You have purchased 200 units of a new issue for $32.50 a share. The units consist of one ABC
preferred share and one ABC common share. At the time the new issue is cleared for sale, the
market price of ABC preferred shares is $22.50 a share and ABC common shares is $10.50 a share.
What is the total cost base of the common shares?
www.nicsoceanbook.com/
OCEANBOOK@hotmail.com
Student Response: Percent Correct Student Answer Choices
Value Response Response
0.0% a. $0.00
0.0% b. $1,034.09
100.0% c. $2,068.18
0.0% d. $4,431.82

General feedback: When securities are purchased as a unit, an investor must identify what part of
the purchase price belongs to each security in the unit. This allows capital gains
or losses to be calculated correctly for tax purposes. The purchase price is
allocated to the securities proportionately based on the deemed market value of
the securities at the time of acquisition. For the common shares, the formula is
(market value of common × total cost of unit)/(market value of preferred +
market value of common). The total cost base per share here would be $10.34.
However, as each unit represents 200 shares, the total cost base of the shares
would be $2,068.18.

Reference: CSC Textbook: Chapter 13 - Financial Planning and Taxation - Taxes


and Taxation Issues - Capital Gains and Losses - Cost of Shares Acquired by
Exercise of Convertible Securities
Score: 0 / 1 (Question not answered.)

Question 99 (1 point)
Your discussions with a new client have produced the following information: He is 32, recently
married, and planning on starting a family. Recent purchases have included a home, and current
goals include the payment of outstanding student loan balances. Based solely on this information,
where would you classify this investor based on life-cycle analysis?
Student Response: Percent Correct Student Answer Choices
Value Response Response
100.0% a. Early Earning Years.
0.0% b. Mid-Earning Years.
0.0% c. Peak Earning Years.
0.0% d. Retirement Years.

General feedback: The life-cycle analysis process can be useful in setting goals and investment
objectives in the financial planning process. It is unlikely that a client or
individual will fit precisely within one particular stage in the life cycle, nor is the
application of the recommendations to the client an exact science. However, the
language around and usage of this theory is widespread, and you may expect to
encounter it frequently. The individual in this example is clearly in his early
earnings years; the clues are his age, his income vs. expenses, and the type of
expenses he is incurring.

Reference: CSC Textbook: Chapter 13 - Financial Planning and Taxation - The


Process of Financial Planning - Life Cycle Analysis.
Score: 0 / 1 (Question not answered.)

Question 100 (1 point)


www.nicsoceanbook.com/
OCEANBOOK@hotmail.com
Sally and Anthony Smith are planning their annual RRSP contributions. Neither of them has any
carry-forward room from previous years, and each can contribute a maximum of $13,500 for this
year, based on their current salary levels. So far, Sally managed to contribute the maximum
allowed amount to her RRSP, while Anthony contributed only $10,500 to his. Since Sally's RRSP is
smaller in aggregate value, Anthony wanted to top up his contribution through his wife's account
and claim the contribution himself. Under the RRSP guidelines, what is Anthony allowed to do and
why?
Student Response: Percent Correct Student Answer Choices
Value Response Response
0.0% a. Anthony cannot make the
contribution to Sally's
RRSP since Canadian
taxpayers can make RRSP
contributions only to their
own personal accounts.
0.0% b. Anthony cannot make the
contribution to Sally's
RRSP since she has
already made the
maximum contribution
allowed.
0.0% c. Anthony can make the
contribution of $3,000 to
Sally's RRSP provided that
he is the beneficiary
named on her account.
100.0% d. Anthony can make the
contribution to Sally's
RRSP to a maximum of
$3,000 provided that
Anthony is the spousal
contributor named on her
account.

General feedback: A married taxpayer may contribute to an RRSP registered in the name of a
spouse and still claim the deduction as long as the contributor does not use the
maximum contribution available for his or her own plan, provided the plan is
spousal. Anthony has $3,000 (current maximum allowed $13,500, less amount
already contributed $10,500) and this is the maximum amount he can
contribute to Sally's account and still claim the deduction himself.

Reference: Taxation - CSC Textbook: Chapter 13 - Financial Planning and


Taxation - Tax Deferral Plans - Registered Retirement Savings Plans (RRSPs) -
Spousal RRSPs
Score: 0 / 1 (Question not answered.)
www.nicsoceanbook.com/
OCEANBOOK@hotmail.com
Question 1 (1 point)
Which of the following external events would likely have the biggest impact on the Canadian equity market?
Student Response: Percent Correct Student Answer Choices
Value Response Response
0.0% a. The election of a socialist government in
Australia.
0.0% b. The decision by Paraguay to default on its U.S.-
dollar denominated debt.
0.0% c. The development of the world's smallest
semiconductor by a Japanese technology
company.
100.0% d. The announcement by the World Gold Council
that it was cutting by one-quarter its estimate of
recoverable gold from the world's mines.

General feedback: While all of these events may impact the Canadian equity market, the announcement by the World Gold
Council would likely have the biggest impact. Canada is considered a commodity-based economy, and events
that affect global commodity prices usually have a significant impact on Canadian equity markets.
Reference: Chapter 8 – Analyzing Markets and Products – Fundamental Macroeconomic Analysis
Score: 0 / 1 (Question not answered.)

Question 2 (1 point)
What impact will an increase in intangible assets have on the asset coverage ratio, holding all other accounts that impact this
ratio constant?
Student Response: Percent Correct Student Answer Choices
Value Response Response
100.0% a. The asset coverage ratio will fall.
0.0% b. The asset coverage ratio will rise.
0.0% c. The asset coverage ratio is not impacted by a
change in intangible assets.
0.0% d. There is not enough information provided to
answer this question.

General feedback: The asset coverage ratio shows the net tangible asset position of a company and enables debtholders to
measure the protection provided by the company’s tangible assets after all liabilities have been met. An
intangible asset is much different from a fixed asset like land or buildings, which is why it is subtracted from
total assets in this ratio. Thus, an increase in intangible assets has the effect of reducing the asset coverage
ratio, when holding the other accounts constant.

Reference: Financial Analysis – Fundamental Company Analysis


CSC Textbook: Chapter 8 - Analyzing Markets and Products - Interpreting Financial Statements
Score: 0 / 1 (Question not answered.)

Question 3 (1 point)
Assume that in all other relevant financial indicators, SFO Inc. has approximately the same measurements as the industry
www.nicsoceanbook.com/
OCEANBOOK@hotmail.com
averages. Considering the information provided below, which of the following statements is true?

Debt/Equity Year Year Year Year Year


5-Year Average
Ratios: 1 2 3 4 5
Industry
1.15 1.25 1.50 1.70 1.85 1.49
Average
SFO Inc. 1.50 1.20 1.10 0.96 0.64 1.08

Student Response: Percent Correct Student Answer Choices


Value Response Response
100.0% a. Since SFO Inc. has exhibited a lower debt/equity
ratio than the industry average, it can be
concluded that SFO Inc. has consistently
outperformed the industry.
0.0% b. Since SFO Inc. has exhibited a higher debt/equity
ratio than the industry average, it can be
concluded that SFO Inc. has consistently
outperformed the industry.
0.0% c. Since the industry has exhibited a lower average
debt/equity ratio, it can be concluded that the
industry has consistently outperformed SFO Inc.
0.0% d. Since the industry has exhibited a higher average
debt/equity ratio, it can be concluded that the
industry has consistently outperformed SFO Inc.

General feedback: A lower debt/equity ratio, other factors being comparable, indicates a company with better performance.
Since SFO Inc. has a consistently lower debt/equity ratio over the 5-year period, it can be concluded that
SFO Inc. has outperformed the industry average.

Reference: CSC Textbook: Chapter 8 - Analyzing Markets and Products - Interpreting Financial Statements -
Risk Analysis Ratios - Debt/Equity Ratio
Score: 0 / 1 (Question not answered.)

Question 4 (1 point)
Which of the following would you use to determine how expensive a stock is?
Student Response: Percent Correct Student Answer Choices
Value Response Response
0.0% a. Liquidity ratios.
100.0% b. Value ratios.
0.0% c. Risk analysis ratios.
0.0% d. Operating performance ratios.

General feedback: Value ratios show the investor what the company’s shares are worth, or the return on owning them. An
example is the price-earnings ratio which links the market price of a common share to earnings per common
share, and thus allows investors to rate the shares of companies within the same industry.
Reference: CSC Textbook: Chapter 8 - Analyzing Markets and Products - Interpreting Financial Statements
Score: 0 / 1 (Question not answered.)
www.nicsoceanbook.com/
OCEANBOOK@hotmail.com
Question 5 (1 point)
Which of the following statements about the price-earnings (P/E) ratios is true?
Student Response: Percent Correct Student Answer Choices
Value Response Response
0.0% a. P/E ratios are calculated for preferred and
common shares.
0.0% b. Bank stocks tend to have the highest P/E ratios
in the marketplace.
0.0% c. Value investors tend to select stocks with the
highest P/E ratios.
100.0% d. P/E ratios typically decrease in a bearish market.

General feedback: As a rule, P/E ratios increase in a rising stock market or with rising earnings. The reverse is true in a
declining market or when earnings decline.
Reference: CSC Textbook: Chapter 8 - Analyzing Markets and Products - Interpreting Financial Statements -
Value Ratios - Price-Earnings Ratio or P/E Multiple
Score: 0 / 1 (Question not answered.)

Question 6 (1 point)
Blair is researching a biotech company’s liquidity and whether the firm can meet its financial obligations in difficult times. Which
of the following ratios would best assist Blair in his analysis?
Student Response: Percent Correct Student Answer Choices
Value Response Response
100.0% a. Quick ratio.
0.0% b. Price-earnings ratio.
0.0% c. Price-to-book ratio.
0.0% d. Put /call ratio.

General feedback: The quick ratio shows how well current liabilities are covered by cash and by items with ready cash value.
Therefore, the quick ratio offers a more conservative test of a company’s ability to meet its current
obligations.
Reference: CSC Textbook: Chapter 8 - Analyzing Markets and Products - Interpreting Financial Statements -
Types of Ratios - Quick Ratio (The Acid Test)
Score: 0 / 1 (Question not answered.)

Question 7 (1 point)
In an uptrend, if breadth measurements are persistently weak, what can be concluded about the trend of the market?
Student Response: Percent Correct Student Answer Choices
Value Response Response
100.0% a. The trend has a higher probability of failing.
0.0% b. The trend has a higher probability of advancing.
0.0% c. The trend has a lower probability of declining.
0.0% d. The trend is not a good measure of market
performance.
www.nicsoceanbook.com/
OCEANBOOK@hotmail.com

General feedback: In an uptrend, if breadth measurements are persistently weak, the trend has a higher probability of failing.

Reference: CSC Textbook: Chapter 8 - Analyzing Markets and Products - Technical Analysis - Equity Market
Analysis - Breadth of Market
Score: 0 / 1 (Question not answered.)

Question 8 (1 point)
A trend analysis of a company's earnings per share (EPS) shows that the trend value in the fifth year is 150. If the company's
EPS in the fifth year was $1.96, what was the company's EPS in the first year?
Student Response: Percent Correct Student Answer Choices
Value Response Response
0.0% a. $0.98
100.0% b. $1.31
0.0% c. $2.94
0.0% d. $3.92

General feedback: The EPS in the first year is found by dividing the value in the fifth year by the trend value in the fifth year,
and multiplying the result by 100. In this case, EPS in the first year is equal to ($1.96 / 150) × 100 = $1.31.
Reference: Chapter 8 – Analyzing Markets and Products – Interpreting Financial Statements
Score: 0 / 1 (Question not answered.)

Question 9 (1 point)
If an investor felt that the economy was at the peak of the business cycle and expected equity prices to decline, what would be
the most appropriate equity investment?
Student Response: Percent Correct Student Answer Choices
Value Response Response
0.0% a. Forest products and mining stocks.
100.0% b. Bank and utility stocks.
0.0% c. Capital goods stocks.
0.0% d. Retail merchandising stocks.

General feedback: During periods when stock prices are declining, cyclical stocks (forest products, mining, capital goods, retail
merchandising, etc.) fall faster than average. Defensive stocks, such as banks and utilities, fall relatively
slower on average. Therefore, if an investor is anticipating an economic contraction, her portfolio should shift
to more defensive stocks.

Reference: CSC Textbook: Chapter 10 - Analyzing Markets and Products - Fundamental Industry Analysis -
Classification by Industry and Stock Characteristics - Defensive Industries
Score: 0 / 1 (Question not answered.)

Question 10 (1 point)
Company A and Company B are similar sized companies, producing a similar product, and have been in existence for a similar
www.nicsoceanbook.com/
OCEANBOOK@hotmail.com
length of time. Company A has a higher proportion of debt and preferred shares in its capital structure than Company B. The
business cycle is moving into an expansion phase. What effect would you expect this to have on the two companies' earnings,
and which company would be more affected?
Student Response: Percent Correct Student Answer Choices
Value Response Response
100.0% a. Earnings would increase and Company A would
be more affected.
0.0% b. Earnings would increase and Company B would
be more affected.
0.0% c. Earnings would decrease and Company A would
be more affected.
0.0% d. Earnings would decrease and Company B would
be more affected.

General feedback: When the business cycle moves into an expansion phase, typically company earnings increase. If a company
is leveraged, the effect of the change in earnings is typically larger than if a company is not leveraged.
Leverage refers to the proportion of a company's capital structure that is made up of debt and preferred
shares. Therefore, Company A, which is more highly leveraged, would be more affected by the increase in
earnings occurring as a result of the change in the business cycle.

Reference: CSC Textbook: Chapter 8 - Analyzing Markets and Products - Fundamental Company Analysis -
Balance Sheet Analysis - The Effect of Leverage
Score: 0 / 1 (Question not answered.)

Question 11 (1 point)
CBA Inc and ZYX Inc. are in the same line of business. CBA Inc. has a debt/equity ratio of 0.94 and ZYX Inc. has a debt/equity
ratio of 1.29. The average debt/equity ratio for the industry is 0.98. Considering this information, which company would you
recommend to your clients and why?
Student Response: Percent Correct Student Answer Choices
Value Response Response
100.0% a. CBA Inc., since it has the lower debt/equity ratio
of the 2 companies compared.
0.0% b. CBA Inc., since it has a debt/equity ratio that is
closer to the industry average.
0.0% c. ZYX Inc., since it has the higher debt/equity ratio
of the 2 companies compared.
0.0% d. ZYX Inc., since it has a debt/equity ratio that is
above the industry average.

General feedback: All things being equal, a company with a lower debt/equity ratio is a safer investment. Thus CBA Inc. should
be recommended.

Reference: CSC Textbook: Chapter 8 - Analyzing Markets and Products - Interpreting Financial Statements -
Risk Analysis Ratios - Debt/Equity Ratios
Score: 0 / 1 (Question not answered.)

Question 12 (1 point)
www.nicsoceanbook.com/
OCEANBOOK@hotmail.com
Which of the following technical analysis signals indicates an increase in the breadth of the stocks in the S&P/TSX Composite
Index?
Student Response: Percent Correct Student Answer Choices
Value Response Response
0.0% a. The index breaks through a significant resistance
level.
0.0% b. The index breaks through its 200-day moving
average from below.
100.0% c. The index's cumulative advance-decline line
increases from 25,200 to 25,700.
0.0% d. The index falls while its moving average
convergence-divergence (MACD) increases.

General feedback: An increase in the cumulative advance-decline indicates an increase in the breadth of the stock in the
S&P/TSX Composite Index. All the other signals are related to the trend of the overall market, which does not
necessarily say anything about the breadth of the market.
Reference: Chapter 8 – Analyzing Markets and Products – Technical Analysis
Score: 0 / 1 (Question not answered.)

Question 13 (1 point)
Company A, Company B, and Company C all have the same amounts in current assets and current liabilities. Company A,
however, has inventories 25% greater than Company B, and 50% greater than Company C. Ignoring any other considerations,
how would the companies rank in liquidity from highest to lowest based on the most stringent test of corporate liquidity?
Student Response: Percent Correct Student Answer Choices
Value Response Response
0.0% a. Company B/Company A/Company C.
100.0% b. Company C/Company B/Company A.
0.0% c. Company A/Company B/Company C.
0.0% d. They all have the same level of liquidity.

General feedback: The most stringent of the liquidity tests is the Quick Ratio, also known as the Acid Test. Were we to use
strictly the Working Capital Ratio, also known as the Current Ratio, answer choice (D) would be correct, as
this measure is simply Current Assets/Current Liabilities. However, the Quick Ratio removes Inventories from
the Current Assets figure, as these are typically the least liquid of all current assets. As Company A has a
greater amount of its current assets in inventories, it would be the least liquid of the companies based on the
Quick Ratio; as Company C has the least, it would be the most liquid.

Reference: CSC Textbook: Chapter 8 - Analyzing Markets and Products - Interpreting Financial Statements -
Types of Ratios - Liquidity Ratios
Score: 0 / 1 (Question not answered.)

Question 14 (1 point)
When evaluating a company's capital structure, analysts tend to base their opinions on which of the following factors?
Student Response: Percent Correct Student Answer Choices
Value Response Response
0.0% a. The level of the company's capacity utilization.
www.nicsoceanbook.com/
OCEANBOOK@hotmail.com
0.0% b. The gain in market share over competition.
100.0% c. The need for future financing and the type of
security that might be used.
0.0% d. The presence of warrants or options that may
decrease the number of shares outstanding.

General feedback: The analysis of a company's capital structure provides an overall picture of a company's financial soundness.
It may indicate the need for future financing and the type of security that might be used. One example is to
look for large debt issue approaching maturity, which may have to be refinanced by a new securities issue.

Reference: CSC Textbook: Chapter 8 - Analyzing Markets and Products - Fundamental Company Analysis -
Balance Sheet Analysis - The Capital Structure
Score: 0 / 1 (Question not answered.)

Question 15 (1 point)
Which of the following is not considered a cyclical industry?
Student Response: Percent Correct Student Answer Choices
Value Response Response
0.0% a. Merchandise.
100.0% b. Utilities.
0.0% c. Transportation.
0.0% d. Mining.

General feedback: In general, cyclical industries fall into three main groups: commodity basic cyclical (i.e., forest products,
mining and chemicals); industrial cyclical (i.e., transportation, capital goods, and basic industries); and
cyclical industries (i.e., merchandising companies and automobiles). The utilities industry, however, would be
considered a defensive industry.
Reference: CSC Textbook: Chapter 8 - Analyzing Markets and Products - Fundamental Industry Analysis -
Classification by Industry and Stock Characteristics - Cyclical Industries
Score: 0 / 1 (Question not answered.)

Question 16 (1 point)
Which of the following statements is true regarding the Interest Coverage ratio for ABC Company?
Year 1 Year 2 Year 3 Year 4 Year 5
Interest Coverage 3.15x 4.22x 2.25x 1.50x 2.50x
Student Response: Percent Correct Student Answer Choices
Value Response Response
100.0% a. ABC’s ability to meet its interest charges
worsened over the period.
0.0% b. ABC’s ability to meet its interest charges
improved over the period.
0.0% c. ABC’s net earnings likely rose over the period.
0.0% d. ABC’s short-term debt likely fell over the period.
www.nicsoceanbook.com/
OCEANBOOK@hotmail.com
General feedback: The Interest Coverage ratio measures the ability of a company to pay interest charges on its debt. Industry
standards vary from industry to industry. However, a rising trend in the ratio indicates an overall
improvement in a company’s ability to meet its interest charges. For ABC Company, the ratio declined from
3.15 to 2.50 over the period, indicating that its ability to meet interest charges has worsened somewhat over
the 5-year period.

Reference: CSC Textbook: Chapter 8 - Analyzing Markets and Products


Interpreting Financial Statements
Score: 0 / 1 (Question not answered.)

Question 17 (1 point)
Trillium Therapeutics (symbol TT) is expected to pay a $1.50 dividend next year. The company anticipates a consistent long-term
growth rate of 5.5% and investors believe that a required return of 8.25% on TT is suitable. Using the Dividend Discount Model
(DDM), what is the intrinsic value of the TT shares?
Student Response: Percent Correct Student Answer Choices
Value Response Response
100.0% a. $54.55
0.0% b. $57.55
0.0% c. $18.18
0.0% d. $10.91

General feedback: The Dividend Discount Model illustrates in a very simple way how companies with stable growth are
theoretically priced. The formula is Price = Div1 / (r-g). Div1 is the expected dividend paid out by the
company in one year, r is the required rate of return on investments, and g is the assumed constant growth
rate for dividends. Therefore, the price or intrinsic value of TT = $1.50 / (.0825 - .055) which equals $54.55.
Reference: CSC Textbook: Chapter - Analyzing Markets and Products - Fundamental Valuation Models -
Dividend Discount Model
Score: 0 / 1 (Question not answered.)

Question 18 (1 point)
When a technical analyst identifies a strong demand for a stock while the supply is low, what conclusion can be made about the
level of the stock price?
Student Response: Percent Correct Student Answer Choices
Value Response Response
0.0% a. It is at a downswing level.
100.0% b. It is at a support level.
0.0% c. It is at a resistance level.
0.0% d. It is at a stagnating level.

General feedback: A support level is the price at which the majority of investors start sensing value, and therefore are willing to
buy (demand is strong) and the majority of existing holders (or potential short sellers) are not willing to sell.
As demand begins to exceed supply, prices tend to rise above support levels.

Reference: CSC Textbook: Chapter 8 - Analyzing Markets and Products - Technical Analysis - Commonly Used
Tools in Technical Analysis - Chart Analysis
www.nicsoceanbook.com/
OCEANBOOK@hotmail.com
Score: 0 / 1 (Question not answered.)

Question 19 (1 point)
When classifying industries by product or service and estimating growth of a specific sector, what initial approach do analysts
employ in their analysis?
Student Response: Percent Correct Student Answer Choices
Value Response Response
0.0% a. Estimation of the annual increase in the inflation
rate.
0.0% b. Analysis of historical stock prices and market
behaviours.
0.0% c. Study of the level of government expenditures
and taxation.
100.0% d. Study of reported revenues and sales volumes of
a particular industry.

General feedback: The initial approach is to study an industry's reported revenues and unit volume sales over the last several
years, preferably over more than one business cycle.

Reference: CSC Textbook: Chapter 8 - Analyzing Markets and Products - Fundamental Industry Analysis -
Classifying Industries by Product or Service - Estimating Growth
Score: 0 / 1 (Question not answered.)

Question 20 (1 point)
The Governor of the Bank of Canada states in a speech that the Canadian economy is overheated and that the Bank of Canada
will have to do something about it. What effect would this speech have on the prices of Canadian bonds and why?
Student Response: Percent Correct Student Answer Choices
Value Response Response
0.0% a. Canadian bond prices would likely rise in
anticipation of a restraint on the growth of the
money supply.
100.0% b. Canadian bond prices would likely fall in
anticipation of a restraint on the growth of the
money supply.
0.0% c. Canadian bond prices would likely rise in
anticipation of a decrease in the supply of
Government of Canada bonds.
0.0% d. Canadian bond prices would likely fall in
anticipation of a decrease in the supply of
Government of Canada bonds.

General feedback: Such a speech would likely cause an increase in the general level of Canadian interest rates. This means that
Canadian bond prices would fall. Interest rates would go up because, given the Governor's speech, the
market would anticipate that the Bank of Canada would restrain growth in the money supply. This is viewed
as a contractionary monetary policy. Nothing in the Governor's speech would cause the market to anticipate
a decrease in the supply of Government of Canada bonds.
Reference: Chapter 8 – Analyzing Markets and Products – Fundamental Macroeconomic Analysis
www.nicsoceanbook.com/
OCEANBOOK@hotmail.com
Score: 0 / 1 (Question not answered.)

Question 21 (1 point)
What can the Bank of Canada do to attract international investors to the Canadian bond market?
Student Response: Percent Correct Student Answer Choices
Value Response Response
0.0% a. Raise Canadian corporate taxes.
100.0% b. Maintain high real interest rates in Canada.
0.0% c. Pursue a policy that causes the Canadian dollar
to fall.
0.0% d. Issue more Government of Canada bonds.
Fundamental Industry Analysis (5 Questions)

General feedback: The Bank of Canada does not control fiscal policy, so it cannot raise taxes or issue Government of Canada
bonds. If it pursued a policy that caused the Canadian dollar to fall, then international investors would not be
attracted to the Canadian bond market because a falling currency lowers returns denominated in foreign
currencies. Of the four options, only high interest real interest rates – which provide adequate compensation
for inflation – would attract international investors to the Canadian bond market.
Reference: Chapter 8 – Analyzing Markets and Products – Fundamental Macroeconomic Analysis
Score: 0 / 1 (Question not answered.)

Question 22 (1 point)
If a rising price of oil drives up inflation in Canada, which of the following scenarios is most likely to follow?
Student Response: Percent Correct Student Answer Choices
Value Response Response
0.0% a. Interest rates will drop to contain inflation and
P/E ratios will fall.
100.0% b. Interest rates will rise to contain inflation and P/E
ratios will fall.
0.0% c. Interest rates will drop to contain inflation and
P/E ratios will go up.
0.0% d. Interest rates will rise to contain inflation and P/E
ratios will go up.

General feedback: Price-earnings (P/E) levels and inflation are strongly inversely related to the prevailing level of inflation, and
therefore to the prevailing level of interest rates. When inflation rates are expected to rise, interest rate
levels are expected to rise to contain inflation, which will cause P/Es to fall.
Reference: CSC Textbook: Chapter 8 - Analyzing Markets and Products - Fundamental Valuation Models - P/E
Levels and Inflation
Score: 0 / 1 (Question not answered.)

Question 23 (1 point)
Which of the following types of financial ratios measures the stock market's rating of a company by relating the market price of
its shares to certain balances obtained from its financial statements?
Student Response: Percent Correct Student Answer Choices
www.nicsoceanbook.com/
OCEANBOOK@hotmail.com
Value Response Response
0.0% a. Debt ratios.
100.0% b. Value ratios.
0.0% c. Liquidity ratios.
0.0% d. Profitability ratios.

General feedback: A total analysis of a company's financials incorporates the use of all of the groups of financial ratios. Each
group has a specific use for interpretation and discussion. The value ratios include Percentage Dividend
Payout ratios, Earnings per Common Share, Dividend Yield and Price-Earnings. Where the other ratios
measure the performance of a company (i.e., how many times it has covered its interest charges, how
profitable it is) - this group measures the value the company brings to each share.

Reference: CSC Textbook: Chapter 8 - Analyzing Markets and Products - Interpreting Financial Statements -
Types of Ratios - Value Ratios
Score: 0 / 1 (Question not answered.)

Question 24 (1 point)
Which of the following price patterns would a technical analyst consider a sell signal?
Student Response: Percent Correct Student Answer Choices
Value Response Response
0.0% a. The advance/decline line flattens out after a long
decline.
100.0% b. Price breaks through the moving average from
above on heavy volume.
0.0% c. When the number of stocks that advance during
a trading day equals the number that have
declined.
0.0% d. Price breaks through the moving average from
below on heavy volume.

General feedback: If price breaks through the moving average line from above on heavy volume, and the moving average line
itself starts to fall, the upward trend is reversed. This is a sell signal.
Reference: CSC Textbook: Chapter 8 - Analyzing Markets and Products - Technical Analysis - Commonly Used
Tools in Technical Analysis - Quantitative Analysis
Score: 0 / 1 (Question not answered.)

Question 25 (1 point)
What is the term used to refer to industries whose earnings tend to rebound dramatically as business conditions improve?
Student Response: Percent Correct Student Answer Choices
Value Response Response
0.0% a. Mature.
100.0% b. Cyclical.
0.0% c. Defensive.
0.0% d. Speculative.
www.nicsoceanbook.com/
OCEANBOOK@hotmail.com
General feedback: Classifying companies by their industry and stock characteristics can be a useful analytical tool. The basic
classifications are cyclical, defensive, and speculative. Mature is a term used when classifying companies by
their stage of growth. Defensive industries tend to be fairly stable, and do not react significantly to changes
in business conditions. Speculative industries tend to be driven more by their own performance, separate
from general business conditions. These kinds of companies may succeed or fail spectacularly regardless of
the economic environment. Cyclical industries tend to move most significantly as changes in the business
cycle occur.

Reference : CSC Textbook: Chapter 8 - Analyzing Markets and Products - Fundamental Industry Analysis -
Classification by Industry and Stock Characteristics - Cyclical Industries
Score: 0 / 1 (Question not answered.)

Question 26 (1 point)
An investor decides to sell long-term bonds and buy equities in order to profit from a change in the current phase of the business
cycle. For this strategy to pay off, into which phase of the business cycle is the economy likely entering?
Student Response: Percent Correct Student Answer Choices
Value Response Response
0.0% a. End of expansion into recession.
100.0% b. End of contraction into expansion.
0.0% c. End of peak into recession.
0.0% d. End of recession into trough.

General feedback: While no guarantees exist, analysis of equity and business cycles can aid in the timing of investment
decisions. The stock market trough equity cycle tends to correlate with the end of the business cycle of
contraction (or recession), and movement into the expansion phase. As expansion occurs, interest rates tend
to rise - meaning bond prices will fall. Thus, investments in bonds should be sold. Corporate profits tend to
rise, and companies do well. Thus, purchases of equities would tend to be recommended.

Reference: CSC Textbook: Chapter 9 - The Portfolio Approach - Developing an Asset Mix - Setting the Asset
Mix
Score: 0 / 1 (Question not answered.)

Question 27 (1 point)
Suppose that a client’s asset mix changes due to fluctuations in the market, and the receipt of cash, interest, etc. The client
would like you to manage the portfolio actively, constantly rebalancing it to return its mix to is strategic long-run position. What
kind of approach to strategic asset allocation is the client advocating?
Student Response: Percent Correct Student Answer Choices
Value Response Response
0.0% a. Tactical.
0.0% b. Passsive.
0.0% c. Integrated.
100.0% d. Dynamic.

General feedback: All of the strategies, except choice b), are variants on the concept of strategic asset allocation, an active
portfolio management style. In this instance, what your client is describing is dynamic asset allocation, in
which the portfolio is systematically rebalanced to its original mix. For example, if the stock market has
www.nicsoceanbook.com/
OCEANBOOK@hotmail.com
outperformed the bond market and thus the portion of the portfolio attributable to stocks is "too high" based
on the asset mix, you sell stocks when they are expensive and buy bonds, which are now relatively cheaper.
The manager is allowed no latitude in diverging from the asset mix based on new perceived opportunities in
the market, such as would be allowed under the tactical approach. With the dynamic approach, you again
rebalance back to the original asset mix, but only when the divergence from the stated policy meets certain
specific parameters, or at specific time intervals..

Reference: CSC Textbook: Chapter 9 - The Portfolio Approach - Implementing the Asset Mix - Ongoing Asset
Allocation
Score: 0 / 1 (Question not answered.)

Question 28 (1 point)
What is a risk feature of an equity manager who follows the growth style of investing?
Student Response: Percent Correct Student Answer Choices
Value Response Response
0.0% a. Lower beta and standard deviation.
0.0% b. Longer holding periods to achieve return.
0.0% c. Sensitivity to the economic forecast.
100.0% d. High portfolio volatility.

General feedback: Growth style equity managers focus on earnings momentum and are willing to pay higher prices if they feel
that the momentum potential justifies the higher price. With the focus on growth and momentum, any
deviation from expectation - either higher or lower EPS than expected - can cause large price changes. These
portfolios have high levels of volatility.

Reference: CSC Textbook: Chapter 9 - The Portfolio Approach - Developing an Asset Mix - Equity Manager
Styles - Growth Managers
Score: 0 / 1 (Question not answered.)

Question 29 (1 point)
When the dividend discount model is used to explain the equity cycle, which of the following factors is assumed to stay constant?
Student Response: Percent Correct Student Answer Choices
Value Response Response
100.0% a. Dividends.
0.0% b. Equity prices.
0.0% c. Interest rates.
0.0% d. Growth rates.

General feedback: The dividend discount model (DDM) can be used to explain the impact that the equity cycle has on the price
of stocks. The different stages of the equity cycle determine the direction of g, which represents corporate
profit growth. The relationship between the equity cycle and the economic cycle is used to determine the
anticipated impact on interest rates, represented by r in the DDM. Holding dividends constant and adjusting g
and r up or down indicates a specific direction for stock prices.
Reference: Chapter 9 – The Portfolio Approach – Developing an Asset Mix
Score: 0 / 1 (Question not answered.)
www.nicsoceanbook.com/
OCEANBOOK@hotmail.com
Question 30 (1 point)
During periods in which stock prices are rising, an industry rotation strategy would stipulate shifting the investment mix towards
which of the following sectors?
Student Response: Percent Correct Student Answer Choices
Value Response Response
100.0% a. Cyclical industries.
0.0% b. Defensive industries.
0.0% c. Emerging markets.
0.0% d. Long-term investment vehicles.

General feedback: The most basic industry rotation strategy involves shifting back and forth between cyclical and defensive
industries. In periods of rising stock prices, a manager should select investments in cyclical industries, since
they tend to rise relatively faster because their profit growth is more robust during an economic expansion.

Reference: CSC Textbook: Chapter 9 - The Portfolio Approach - Developing An Asset Mix - Setting the Asset
Mix - Industry Rotation
Score: 0 / 1 (Question not answered.)

Question 31 (1 point)
Over the last five years, the returns on ABC stock and the S&P/TSX Composite Index were as follows:

Return on the
Return on ABC S&P/TSX
Year Stock Composite Index
1 12% 9%
2 6% 4%
3 -8% -5%
4 -6% -5%
5 10% 9%

Given this data, what was the beta of ABC stock relative to the S&P/TSX Composite Index?
Student Response: Percent Correct Student Answer Choices
Value Response Response
0.0% a. Less than 0.
0.0% b. Greater than or equal to 0, but less than 1.
0.0% c. Exactly 1.
100.0% d. Greater than 1.

General feedback: Beta measures the volatility of a stock relative to the volatility of the stock market as a whole. A beta of 1
indicates that a stock has the same volatility as the market; a beta less than 1 indicates that a stock had less
volatility than the market, while a beta greater than 1 indicates that a stock has more volatility than the
www.nicsoceanbook.com/
OCEANBOOK@hotmail.com
market.
In Years 1, 2, and 5, when the returns on ABC stock and the S&P/TSX Composite were positive, ABC out-
performed the market. In Years 3 and 4, when the returns on ABC stock and the S&P/TSX Composite were
negative, ABC under-performed the market. Thus, ABC's returns were more volatile than the market's, and
its beta was greater than 1.
Reference: Chapter 9 – The Portfolio Approach – Risk and Return
Score: 0 / 1 (Question not answered.)

Question 32 (1 point)
At the start of the year, a client’s portfolio is worth $250,000. During the year, the client deposits an additional $20,000, while at
the end of the year the portfolio is worth $285,000. What is the pre-tax total return on the portfolio, expressed as a percentage?
Student Response: Percent Correct Student Answer Choices
Value Response Response
0.0% a. 5.26%
100.0% b. 5.77%
0.0% c. 14.0%
0.0% d. 21.2%

General feedback: The formula for total return is the increase in market value divided by the average amount invested.
Therefore, the numerator would be ($285,000-($250,000 +20,000)) =$15,000. The denominator would be
(($250,000 +(20,000/2)) = $260,000. Therefore, the total return would be ($15,000/$260,000) = 5.77%.
Of course, this calculation does not address the tax-effectiveness of the portfolio; depending on how the
growth was achieved, for example through realized or unrealized capital gains, income received from bonds,
dividends from equities, the after-tax return could be very different from two portfolios with the same
nominal (pre-tax) rate of return.

Reference: Chapter 9 - The Portfolio Approach - Evaluating Portfolio Performance - Measuring Portfolio
Performance
Score: 0 / 1 (Question not answered.)

Question 33 (1 point)
The market value of an investor’s portfolio is currently $450,000. The investor requires that the portfolio be worth at least
$500,000 in three years' time. Which of the following securities should the investor purchase?
Student Response: Percent Correct Student Answer Choices
Value Response Response
0.0% a. A money market mutual fund with a current yield
of 4%.
100.0% b. A 3-year Government of Canada strip bond
trading at a price of 90.
0.0% c. A short-term bond mutual fund that has a 3-year
compound annual return of 4%.
0.0% d. A 3-year CDIC-guaranteed GIC with an interest
rate of 3% compounded annually.

General feedback: Only the 3-year Government of Canada strip bond will guarantee that the investor has $500,000 (unless the
Government of Canada defaults on its bonds). At a price of 90, the investor has enough money to buy
$500,000 face value of the strip bond, which means the investor will receive exactly $500,000 in three years'
www.nicsoceanbook.com/
OCEANBOOK@hotmail.com
time. There is no guarantee that the money market fund will attain the same results, because short-term
interest rates may decline over the next three years. The same thing goes for the short-term bond fund. With
an interest rate of 3% compounded annually, the GIC would only return $491,727 in three years' time.
Reference: Chapter 9 – The Portfolio Approach – Determining Objectives and Constraints
Score: 0 / 1 (Question not answered.)

Question 34 (1 point)
At the beginning of the year, Tim's $1,000,000 portfolio had a strategic (and actual) asset allocation of 10% cash, 30% Canadian
bonds, and 60% Canadian equities. As of today, his cash balance is 10% higher, his bond allocation is 5% higher, and his equity
allocation is 10% lower. What is the current value of Tim's portfolio?
Student Response: Percent Correct Student Answer Choices
Value Response Response
0.0% a. $950,000
100.0% b. $965,000
0.0% c. $1,050,000
0.0% d. $1,065,000

General feedback: At the beginning of the year, Tim had a cash balance of $100,000; today it's 10% higher at $110,000. Also
at the beginning of the year, Tim had $300,000 and $600,000 invested in bonds and equities, respectively.
Today, he has $315,000 ($300,000 x 1.05) in bonds and $540,000 ($600,000 x 0.90), for a total portfolio
value of $965,000.
Reference: Chapter 9 – The Portfolio Approach – Implementing the Asset Mix
Score: 0 / 1 (Question not answered.)

Question 35 (1 point)
What is the most likely reason for an investor to keep a consistently high portion of his or her portfolio in cash equivalent
securities?
Student Response: Percent Correct Student Answer Choices
Value Response Response
0.0% a. To outperform the benchmark.
100.0% b. To provide continuous liquidity.
0.0% c. To pay ongoing portfolio expenses.
0.0% d. To take advantage of investment opportunities as
they arise.

General feedback: A consistently large allocation to cash equivalent securities is normally needed to maintain a high level of
liquidity, usually to meet large anticipated withdrawals. Ongoing portfolio expenses and the flexibility needed
to take advantage of investment opportunities usually don't require a large and consistent allocation to cash
equivalent securities.
Reference: Chapter 9 – The Portfolio Approach – Developing an Asset Mix
Score: 0 / 1 (Question not answered.)

Question 36 (1 point)
Which of the following statements about the link between the equity cycle and the economic cycle is true?
www.nicsoceanbook.com/
OCEANBOOK@hotmail.com
Student Response: Percent Correct Student Answer Choices
Value Response Response
0.0% a. The equity cycle occurs independently of the
economic cycle.
100.0% b. The equity cycle tends to follow the same pattern
as the economic cycle, except that the equity
cycle usually leads the economic cycle.
0.0% c. The equity cycle tends to follow the same pattern
as the economic cycle, except that the equity
cycle usually lags the economic cycle.
0.0% d. The equity cycle is counter-cyclically related to
the economic cycle, so that when the economy is
rising, stocks are falling, and vice-versa.

General feedback: Evidence tells us the equity cycle follows the same pattern as the economic cycle, except that the equity
cycle usually leads the economic cycle. This relationship explains why the equity market is often used as a
leading indicator of economic activity.
Reference: Chapter 9 – The Portfolio Approach – Developing an Asset Mix
Score: 0 / 1 (Question not answered.)

Question 37 (1 point)
What does the ex-post rate of return measure?
Student Response: Percent Correct Student Answer Choices
Value Response Response
0.0% a. The “going forward” return
0.0% b. The compound return.
100.0% c. The “after-the-fact” return.
0.0% d. The current return less inflation.

General feedback: The term ex-post simply refers to the fact that this rate of return is "after the fact" rather than before. You
are looking back at the return you received on the stock, after a period of holding the investment. It is
calculated using the amount you earned on the investment, divided by the amount you paid.

Reference: CSC Textbook: Chapter 9 - The Portfolio Approach - Risk and Return - Rate of Return
Score: 0 / 1 (Question not answered.)

Question 38 (1 point)
If the stock market is entering a trough phase, which of the following actions should be taken regarding investments in long-term
bonds and equities?
Student Response: Percent Correct Student Answer Choices
Value Response Response
0.0% a. Sell long-term bonds and equities.
100.0% b. Sell long-term bonds and buy equities.
0.0% c. Buy long-term bonds and equities.
0.0% d. Buy long-term bonds and sell equities.
www.nicsoceanbook.com/
OCEANBOOK@hotmail.com

General feedback: While no guarantees exist, analysis of equity cycles can aid in timing of investment decisions. The stock
market trough tends to correlate with the end of the business cycle of contraction, and movement into the
expansion phase. As expansion occurs, interest rates tend to rise - meaning bond prices will fall. Thus,
investments in bonds should be sold. Corporate profits tend to rise, and companies do well. Thus, purchases
of equities would tend to be recommended.

Reference: CSC Textbook: Chapter 9 - The Portfolio Approach - Developing and Implementing an Asset Mix -
Setting the Asset Mix - The Link Between Equity Cycles and Economic Cycles
Score: 0 / 1 (Question not answered.)

Question 39 (1 point)
What is the ex-post rate of return if an investor purchased a mutual fund unit on January 1st at $12.00, sold it on December 31st
at $10.75, and received $0.25 in distributions during the year?
Student Response: Percent Correct Student Answer Choices
Value Response Response
0.0% a. -10.42%
0.0% b. - 9.30%
100.0% c. - 8.33%
0.0% d. - 7.50%

General feedback: The term ex-post simply refers to the fact this rate of return is “after the fact” rather than before. You are
looking back at the return you received on the stock, after a period of holding the investment. It is calculated
using the amount you earned on the investment, divided by the amount you paid. In this case, you lost
$1.25 on the difference between the year-end price of $10.75 and the purchase price of $12.00, but you
gained $0.25 in cash flow through the distributions. Therefore, your net loss is $1.00, which, divided by
$12.00, is a negative return of 8.33%.

Reference: CSC Textbook: Chapter 9 - The Portfolio Approach - Risk and Return - Rate of Return
Score: 0 / 1 (Question not answered.)

Question 40 (1 point)
The current consensus is that the Bank of Canada will take a bearish stance on interest rates in the coming year. If this
consensus holds true, in what direction will interest rates change?
Student Response: Percent Correct Student Answer Choices
Value Response Response
100.0% a. Short-term interest rates will rise.
0.0% b. Short-term interest rates will fall.
0.0% c. Long-term interest rates will fall.
0.0% d. Long-term interest rates will not change.

General feedback: The critical understanding in this question is that a bullish move in interest rates is a decline in rates, while a
bearish move is an increase in rates. This is based on the inverse relationship between bond prices and
interest rates: as rates rise, bond prices fall, and vice versa. The Bank of Canada conducts monetary policy at
the very short end of the money market. Therefore, short-term interest rates will rise in the coming year.
www.nicsoceanbook.com/
OCEANBOOK@hotmail.com
Reference: CSC Textbook: Chapter 9 - The Portfolio Approach - Monitoring The Economy, The Markets, The
Portfolio And The Client - The Economy - Fixed-Income Securities
Score: 0 / 1 (Question not answered.)

Question 41 (1 point)
You are evaluating a new client, in order to determine an appropriate asset mix for her investments. After discussing the client's
goals, personal situation, etc., you have the following information: Your client is 45, single, with no dependents and no
expectation of ever having any. She has been investing extensively in a wide range of investment products, and is quite
comfortable with risk. Her goal is to retire at 55, and then to travel extensively. She has no wish to leave an estate for any
individual. What asset mix would you recommend?
Student Response: Percent Correct Student Answer Choices
Value Response Response
0.0% a. Cash 10%/Fixed Income 40%/Equities 50%.
100.0% b. Cash 5%/Fixed Income 25%/Equities 70%.
0.0% c. Cash 20%/Fixed Income 70%/Equities 10%.
0.0% d. Cash 5%/Fixed Income 75%/Equities 20%.

General feedback: Choosing an asset mix is not an exact science. As an investment advisor, you can only make
recommendations based on the information and discussions you have with a client. Your recommendations
may not be acceptable to the client. This is part of the learning experience with each new client. What
direction the client chooses eventually will be a compromise between what you feel best and what they
prefer. Your role is to guide them to making an appropriate choice, not to dictate an approach based on a
formula. As time passes and your client's personal circumstances change, the asset mix decision must be
revisited and evaluated. Here, you have an individual with no significant personal obligations other than to
herself, and with a goal of retiring early to a reasonably expensive lifestyle. Therefore, an aggressive, growth
oriented asset mix would most likely be appropriate. Thus, a high weighting in equities, with some fixed
income for diversification and cash for liquidity, would be indicated.

Reference: CSC Textbook: Chapter 9 - The Portfolio Approach - Developing and Implementing an Asset Mix.
Score: 0 / 1 (Question not answered.)

Question 42 (1 point)
After discussions with your client, Beth, you determine the following:

She is 49, has no children, a secure job, and annual after-tax income of $55,000. While comfortable with investing, she is still
slightly risk averse. Her total monthly expenses are manageable based on her and her husband’s after tax income.

Her husband's annual after-tax income is $35,000. Both have comprehensive employer paid health and dental plans and each
have $750,000 in life insurance, the deductions for which are already reflected in their after-tax income. Each has a fully-paid
employer pension plan, sufficiently generous that each has the minimum possible RRSP contribution room accrual each year.
Annually, they take vacations costing $3,500, and make charitable contributions of $3,600. Beth has inherited $250,000 in cash.
Which of the following asset mixes would be most appropriate for her investments?
Student Response: Percent Correct Student Answer Choices
Value Response Response
0.0% a. Cash 15% / Fixed Income 65% / Equities 20%
100.0% b. Cash 5% / Fixed Income 30% / Equities 65%
0.0% c. Cash 10% / Fixed Income 80% / Equities 10%
www.nicsoceanbook.com/
OCEANBOOK@hotmail.com
0.0% d. Cash 5% / Fixed Income 0% / Equities 95%

General feedback: Beth has a secure job, no income or liquidity needs, is slightly risk-averse, and is in a high tax bracket.
Therefore, an asset mix heavily weighted in fixed income or cash would be inappropriate. The asset mix that
is 95% invested in equities would be inappropriate, given Beth's preference to not assume excessive risk;
also, it would not give her the benefits of diversification. Therefore, answer choice (B) would be most
appropriate, given its tax-effective weighting in equities and low weighting in cash and fixed income.

Reference: CSC Textbook: Chapter 9 - The Portfolio Approach - Determining Objectives and Constraints -
Major Investment Objectives
Score: 0 / 1 (Question not answered.)

Question 43 (1 point)
You have asked your investment advisor to rank different securities in terms of both risk and return. The advisor finds that
Security A has an expected return of 10.2%, Security B of 11.3%, and Security C of 8.25%. Based solely on the Capital Asset
Pricing Model (CAPM), and assuming that the same market portfolio and risk free rate of return applies to each security, how
would you rank the 3 securities in terms of beta, from highest to lowest?
Student Response: Percent Correct Student Answer Choices
Value Response Response
0.0% a. Security C; security A; security B.
0.0% b. Security B; security C; security A.
100.0% c. Security B; security A; security C.
0.0% d. Security C; security B; security A.

General feedback: The CAPM model is a method of determining relative risk and expected return for individual securities, or
portfolios of securities. By using a formula that factors in the relative volatility (beta), relevant risk free
security and market returns, one determines an expected theoretical rate of return. Of course, the accuracy
of the inputs are of significance, as a small error in selecting the appropriate rates may significantly distort
the final answer. However, if all inputs are held constant, as would be the situation here, the highest beta
stock, would produce the highest return, and the lowest beta the lowest return, indicative of the risk/return
tradeoff. Thus, security B would have the highest beta, and security C the lowest.

Reference: CSC Textbook: Chapter 9 - The Portfolio Approach - Risk and Return - Combining Securities to
Maximize Return while Reducing Risk - Beta of a Portfolio
Score: 0 / 1 (Question not answered.)

Question 44 (1 point)
Rebecca works for NTD Financial as a portfolio manager. She is in charge of a Canadian bond fund and her primary objective is to
actively manage the average term of the holdings. Which of the following fixed-income management styles has she been
following?
Student Response: Percent Correct Student Answer Choices
Value Response Response
100.0% a. Interest rate anticipators.
0.0% b. Credit quality.
0.0% c. Spread traders.
0.0% d. Momentum.
www.nicsoceanbook.com/
OCEANBOOK@hotmail.com
General feedback: Managing through interest rate anticipation means lengthening the average term of a portfolio when interest
rates are expected to fall, and shortening the term or taking refuge in cash when interest rates are expected
to rise.
Reference: Chapter 9 - The Portfolio Approach – Developing an Asset Mix - Fixed-Income Manager Styles.
Score: 0 / 1 (Question not answered.)

Question 45 (1 point)
You are reviewing the performance of a client's investment portfolio. The client's strategic asset mix is 5% cash, 40% stocks and
55% bonds. Your research department has provided you with its latest forecast, with bonds showing an expected future return of
3%, equities 7% and cash 2%. What is the portfolio's expected rate of return?
Student Response: Percent Correct Student Answer Choices
Value Response Response
0.0% a. 4.80%
0.0% b. 6.60%
100.0% c. 4.55%
0.0% d. 12.0%

General feedback: The expected rate of return on a portfolio is calculated using the weightings of the client specific asset classes
and the projected rate of return. Here, cash is expected to return 2%, and has a weighting of 5%, bonds an
expected return of 3% and a weighting of 55%, and stocks an expected rate of return of 7% and a weighting
of 40%. Therefore, the expected return is (2% × .05) + (3% × .55) + (7% × .40) = 4.55%.

Reference: CSC Textbook: Chapter 9 - The Portfolio Approach - Risk and Return - Portfolio Risk and Return -
Rate of Return on Portfolios
Score: 0 / 1 (Question not answered.)

Question 46 (1 point)
Which of the following circumstances may trigger a re-balancing of a portfolio under a dynamic strategy approach?
Student Response: Percent Correct Student Answer Choices
Value Response Response
100.0% a. Idle cash reserves have built up.
0.0% b. Capital markets have stabilized.
0.0% c. Stock and bond markets have exhibited the same
growth in percentage terms.
0.0% d. Dividend or interest income cash flows are
extremely low.

General feedback: Re-balancing may be necessary for a variety of reasons, such as if there is a build-up of idle cash reserves
that have not been re-invested, or when there are movements in the capital markets (opposite to (b) and
(c)).

Reference: CSC Textbook: Chapter 9 - The Portfolio Approach - Implementing the Asset Mix - Ongoing Asset
Allocation - Dynamic Asset Allocation
Score: 0 / 1 (Question not answered.)
www.nicsoceanbook.com/
OCEANBOOK@hotmail.com
Question 47 (1 point)
At the beginning of the year, Tim's $1,000,000 portfolio had a strategic (and actual) asset allocation of 10% cash, 30% Canadian
bonds, and 60% Canadian equities. As of today, his cash balance is 10% higher, his bond allocation is 5% higher, and his equity
allocation is 10% lower. Which of the following transactions would restore the equity portion of Tim's portfolio back to its
strategic allocation?
Student Response: Percent Correct Student Answer Choices
Value Response Response
0.0% a. Sell $40,000 worth of Canadian equities.
0.0% b. Sell $54,000 worth of Canadian equities.
0.0% c. Buy $60,000 worth of Canadian equities.
100.0% d. Buy $39,000 worth of Canadian equities.

General feedback: At the beginning of the year, Tim had a cash balance of $100,000; today it's 10% higher at $110,000. Also
at the beginning of the year, Tim had $300,000 and $600,000 invested in bonds and equities, respectively.
Today, he has got $315,000 ($300,000 x 1.05) in bonds and $540,000 ($600,000 x 0.90), for a total
portfolio value of $965,000. Based on his original 60% allocation and the new value of his portfolio, his
allocation to equities should now be $579,000 ($965,000 x 0.60). That means he needs to buy $39,000 of
Canadian equities ($579,000 – $540,000).
Reference: Chapter 9 – The Portfolio Approach – Implementing the Asset Mix
Score: 0 / 1 (Question not answered.)

Question 48 (1 point)
How is diversification achieved in fixed-income portfolios? I. By investing in domestic and foreign bonds.
II. By investing in bonds with a range of maturities.
III. By investing in bonds with different credit ratings.
IV. By investing in bonds with different coupon rates.
Student Response: Percent Correct Student Answer Choices
Value Response Response
0.0% a. II only.
0.0% b. I and IV only.
0.0% c. II and III only.
100.0% d. I, II, III, and IV.

General feedback: Fixed-income portfolios can be diversified by investing in a variety of bonds from all of the areas listed in the
choices, including domestic and foreign bonds, bonds with different maturities, bonds with different credit
ratings, and bonds with different coupon rates.
Reference: Chapter 9 – The Portfolio Approach – Developing an Asset Mix
Score: 0 / 1 (Question not answered.)

Question 49 (1 point)
Which of the following investment strategies are growth managers most likely to employ?
Student Response: Percent Correct Student Answer Choices
Value Response Response
0.0% a. Those that focus on specific stock selection with a
www.nicsoceanbook.com/
OCEANBOOK@hotmail.com
research-intensive approach.
0.0% b. Those that pursue long-term growth through a
buy-and-hold process.
100.0% c. Those that focus on current and future earnings
of individual companies, specifically EPS.
0.0% d. Those that analyze the prospects for the overall
economy and assume most sectors will
outperform.

General feedback: In the bottom-up style of growth investing, managers focus on current and future earnings of individual
companies, specifically earnings per share (EPS).

Reference: CSC Textbook: Chapter 9 - The Portfolio Approach - Developing an Asset Mix - Equity Manager
Styles - Growth Managers
Score: 0 / 1 (Question not answered.)

Question 50 (1 point)
A portfolio manager for the Equality Equity Fund pursues a strategy whereby he is willing to pay higher prices for stocks that
offer good price momentum. This is an example of what type of portfolio manager style?
Student Response: Percent Correct Student Answer Choices
Value Response Response
0.0% a. Value style.
0.0% b. Tactical style.
0.0% c. Dynamic style.
100.0% d. Growth style.

General feedback: Growth style equity managers focus on earnings momentum and are willing to pay higher prices if they feel
that the momentum potential justifies the higher price. With the focus on growth and momentum, any
deviation from expectation - either higher or lower EPS than expected - can cause large price changes. These
portfolios have high levels of volatility.

Reference: CSC Textbook: Chapter 9 - The Portfolio Approach - Developing an Asset Mix - Equity Manager
Styles - Growth Managers
Score: 0 / 1 (Question not answered.)

Question 51 (1 point)
The Lakefield Hedge Fund has $10 million in capital invested in Canadian stocks. Presently, the fund is long $10 million and short
$6 million. Which of the following represents the approximate net exposure of this fund?
Student Response: Percent Correct Student Answer Choices
Value Response Response
0.0% a. 25% long.
100.0% b. 40% long.
0.0% c. 60% long.
0.0% d. 150% long.
www.nicsoceanbook.com/
OCEANBOOK@hotmail.com
General feedback: A long /short equity fund's net exposure = (long exposure – short exposure) / capital. In this example, the
net exposure = ($10 million - $6 million) / $10 million or 40%.
Reference: Chapter 12 – Hedge Funds – Hedge Fund Strategies.
Score: 0 / 1 (Question not answered.)

Question 52 (1 point)
Using the information provided below, what is this fund’s net asset value per share (NAVPS)?
Financial Information at
December 31, 2002
Assets:
Investments at market value $24,670,000
Receivables $1,230,000
Liabilities:
Current liabilities $6,450,000
Long-term liabilities $3,400,000

Shares outstanding 2,500,000

Student Response: Percent Correct Student Answer Choices


Value Response Response
0.0% a. $5.93
100.0% b. $6.42
0.0% c. $7.78
0.0% d. $9.14

General feedback: (Assets - Liabilities)


NAVPS =
Number of Shares Outstanding
($24,670,000 + $1,230,000) - ($6,450,000 + $3,400,000)
NAVPS =
2,500,000
$16,050,000
NAVPS =
2,500,000
NAVPS = $6.42

Reference: CSC Textbook: Chapter 10 - Mutual Funds - The Structure of Mutual Funds - How Mutual Funds
Units or Shares Are Priced - Offering Redemption Price
Score: 0 / 1 (Question not answered.)

Question 53 (1 point)
Derek is a hedge fund manager and he takes significant positions in companies he thinks are about to experience unique
situations such as mergers and takeovers. Which of the following hedge fund strategies is Derek adhering to?
Student Response: Percent Correct Student Answer Choices
Value Response Response
0.0% a. A relative hedge fund strategy.
0.0% b. A directional hedge fund strategy.
100.0% c. An event-driven hedge fund strategy.
www.nicsoceanbook.com/
OCEANBOOK@hotmail.com
0.0% d. A dedicated short bias hedge fund strategy.

General feedback: Event-driven hedge funds seek to profit from unique, particular events such as mergers, acquisitions, stock
splits and buybacks.
Reference: Chapter 12– Hedge Funds – Hedge Funds Strategies.
Score: 0 / 1 (Question not answered.)

Question 54 (1 point)
What is the difference between the management fees and the management expense ratio (MER) on a mutual fund?
Student Response: Percent Correct Student Answer Choices
Value Response Response
100.0% a. The MER is the total annual cost of running a
fund that includes management fees.
0.0% b. The MER is the sales commission to purchase the
fund and the management fee is the annual cost
to run the fund.
0.0% c. Management fees are the total annual cost to run
the fund that includes the MER.
0.0% d. The management fee reduces the overall return
on the fund, while the MER is a percentage of the
total assets under management.

General feedback: The MER is the total annual cost of running a fund. The MER includes the management fees that are paid
annually to the fund manager.

Reference: CSC Textbook: Chapter 10 - Mutual Funds - The Structure of Mutual Funds - Charges Associated
with Mutual Funds - Management Fees
Score: 0 / 1 (Question not answered.)

Question 55 (1 point)
A mutual fund and a segregated fund each earn $100,000 in capital gains in January. Scott buys 100 units of each fund in a non-
registered account, and 100 units of the segregated fund in a RRSP, all in June, and continues to hold all 3 positions at year end.
Assuming no other distributions, and ignoring any other considerations, what is the order of the amount of income tax liability of
each fund at year end from the lowest to the highest amount?
Student Response: Percent Correct Student Answer Choices
Value Response Response
0.0% a. The RRSP, then the mutual fund, then the
segregated fund.
0.0% b. The mutual fund, then the segregated fund, then
the RRSP.
0.0% c. The segregated fund, then the RRSP, then the
mutual fund.
100.0% d. The RRSP, then the segregated fund, then the
mutual fund.
www.nicsoceanbook.com/
OCEANBOOK@hotmail.com
General feedback: The investment lowest in tax-liability at year-end would be the Registered Retirement Savings Plan (RRSP).
This registered account allows investment income to be received on a tax-deferred basis. From there, the
distinction lies in the manner in which distributions are allocated to investors. Segregated funds allocate
distributions on a "pro-rated" basis, based on the period of holding. As Scott had only held the investment for
one-half of the year, he would pay taxes on only one-half of the distribution. Mutual funds, however,
distribute to holders of record the same amount, regardless of how long the investment had been held.
Therefore, Scott would be paying tax on the full amount of the distribution. Therefore, the mutual fund would
have the highest year-end tax liability.

Reference: CSC Textbook: Chapter 11 - Other Managed Accounts and Products - Segregated Funds - Tax
Considerations
Score: 0 / 1 (Question not answered.)

Question 56 (1 point)
Which of the following terms best describes the type of risk that is beyond the control of the fund manager?
Student Response: Percent Correct Student Answer Choices
Value Response Response
0.0% a. Diversifiable risk.
0.0% b. Integrated risk.
100.0% c. Systematic risk.
0.0% d. Capital deficiency risk.

General feedback: Risks that are beyond the control of the fund manager are called non-diversifiable risks or systematic risks.

Reference: CSC Textbook: Chapter 10 - Mutual Funds - Introduction - Disadvantages Associated with Mutual
Funds - Professional Investment Management Is Not Infallible
Score: 0 / 1 (Question not answered.)

Question 57 (1 point)
Aldo purchased 1,000 units in the Equinox Segregated Fund on March1st of this year. The Net Asset Value (NAV) of the Equinox
Fund was $14.80 on January 1st and during the course of the year this fund distributed income of $1.56 per unit. Based on the
above information, how much total income would Aldo receive by year- end from this fund?
Student Response: Percent Correct Student Answer Choices
Value Response Response
100.0% a. $1,300
0.0% b. $1,430
0.0% c. $1,560
0.0% d. $1,690

General feedback: The total income received by Aldo at the end of the year is: Per month distribution = $1.56 / 12 = $0.13 per
month. Distribution from January to March 1 = 2 months × $0.13 = $0.26. When the allocation is paid out at
the end of the year, Aldo would receive $1.30 per unit ($1.56 - $0.26) representing the 10 months he held
the fund (from March to December). The payout of $1.30 per unit means that Aldo had income of $1,300 for
the year (1,000 units × $1.30).

Reference: CSC Textbook: Chapter 11 - Segregated Funds – Effect of Allocations on Segregated Fund Net
www.nicsoceanbook.com/
OCEANBOOK@hotmail.com
Asset Values.
Score: 0 / 1 (Question not answered.)

Question 58 (1 point)
Fred has maximized his RRSP contributions, but still has excess funds to invest. He is looking for a tax-effective way to invest;
however, he is also starting to think of the importance of providing for his children in the event of his death. What investment
might you suggest?
Student Response: Percent Correct Student Answer Choices
Value Response Response
0.0% a. A segregated fund policy.
0.0% b. A registered investment contract.
100.0% c. A universal life insurance policy.
0.0% d. A Registered Retirement Income Fund.

General feedback: Universal Life Insurance is a special life insurance product that combines both life insurance and an
investment account. As life insurance payments are non-taxable in the event of death, the investment
earnings within the investment component of the policy accumulate on a tax-sheltered basis. When Fred
dies, his beneficiaries, who could be his children, will receive a tax-free payment of both his life insurance
and the accumulated earnings in his investment account. Of course, unlike a RRSP, Fred does not receive a
tax-deduction for his contribution. However, the benefit of tax-free compounding is obvious. You earn
subsequent investment income on every dollar earned, not simply on the after-tax amount. A segregated
fund does not allow for tax-free compounding, although its payments are creditor proof on death; thus, it is
not as tax-effective. The RRIF will be a possible vehicle when Fred collapses his RRSPs at a later date;
however, it is not available for new contributions of Fred's current investment dollars.

Reference: CSC Textbook: Chapter 11 - Other Managed Accounts and Products - Universal Life Insurance
Score: 0 / 1 (Question not answered.)

Question 59 (1 point)
What is the primary aim of most hedge funds?
Student Response: Percent Correct Student Answer Choices
Value Response Response
0.0% a. To increase exposure to foreign investments.
100.0% b. To minimize risk and deliver positive returns
under all market conditions.
0.0% c. To avoid cumbersome regulation.
0.0% d. To generate the highest return possible.

General feedback: Absolute return with minimal risk is the primary aim of most hedge funds. No matter how the market
performs, the hedge fund manager is expected to earn positive returns under all market conditions.

Reference: Chapter 12 – Hedge Funds – Portfolio Theory and Hedge Funds


Score: 0 / 1 (Question not answered.)

Question 60 (1 point)
www.nicsoceanbook.com/
OCEANBOOK@hotmail.com
Which of the following legal documents would an investor acquire to find out more about the risks and objectives of a hedge
fund?
Student Response: Percent Correct Student Answer Choices
Value Response Response
0.0% a. A preliminary prospectus.
100.0% b. An offering memorandum.
0.0% c. A final prospectus.
0.0% d. An annual information form.

General feedback: Hedge funds usually issue an offering memorandum, a legal document stating the objectives, risks and terms
of investment involved with a private placement.
Reference: Chapter 12 – Hedge Funds – Who Can Invest in Hedge Funds?
Score: 0 / 1 (Question not answered.)

Question 61 (1 point)
What is the rate of return for the Penn Equity Fund based on the modified Dietz method and the following parameters?
Market value at beginning of period: $38,300,000
Market value at end of period: $41,280,000
Sum of each cash flow multiplied by its weight: $156,150
Sum of cash flows during the period : $184,250

Student Response: Percent Correct Student Answer Choices


Value Response Response
100.0% a. 7.27%
0.0% b. 7.34%
0.0% c. 7.75%
0.0% d. 8.23%

General feedback: While not an exact calculation, the modified Dietz method is a good general approximation for the time-
weighted rate of return for mutual funds. In the numerator, you use the market value at the end of the
period, subtract the market value at the beginning of the period (as you would normally in any percentage
change calculation) and further refine it by deducting the sum of the cash flows during the period. The
denominator is the market value at the beginning of the period (as you would use normally in any percentage
change calculation) and then refining it further by adding in the sum of the cash flows multiplied by each
cash flows “weight” within the portfolio. As for the Penn Equity Fund, the calculation is: ($41,280,000 -
$38,300,000 - $184,250)/($38,300,000 + $156,150), for an answer of 7.27%.

Reference: CSC Textbook: Chapter 10 - Mutual Funds - Comparing Mutual Fund Performance - Measures of
Mutual Fund Performance - Modified Dietz method
Score: 0 / 1 (Question not answered.)

Question 62 (1 point)
What is meant by a "lockup"?
Student Response: Percent Correct Student Answer Choices
Value Response Response
www.nicsoceanbook.com/
OCEANBOOK@hotmail.com
0.0% a. Pre-specified times of the year when an investor
may redeem shares.
0.0% b. Where rogue traders go after they have been
caught.
100.0% c. The initial amount of time an investor is required
to keep his money in the fund before shares can
be redeemed.
0.0% d. A guaranteed return.

General feedback: A lockup refers to the time period that initial investments cannot be redeemed from a hedge fund. Some
hedge funds require lockups of three years or more! While lockups of this duration are not common for hedge
funds offered on a continuous basis in Canada, some funds do have initial lockup periods or charge an early
redemption fee if the initial investment is redeemed within the first three months to one year. Once the
lockup period is over, the investor is free to redeem shares on any liquidity date specified in the offering
memorandum.

Reference: Chapter 12 – Hedge Funds –Benefits and Risks of Hedge Funds


Score: 0 / 1 (Question not answered.)

Question 63 (1 point)
Judy purchased 300 units of True North Segregated Fund at the beginning of August this year. The Net Asset Value (NAV) of this
fund at the beginning of the year was $18 per unit and during the year, the fund allocated income of $2.28 per unit. Based on
the given information, which of the following represents the NAV at the time Judy purchased the fund?
Student Response: Percent Correct Student Answer Choices
Value Response Response
0.0% a. $15.72.
0.0% b. $16.67.
100.0% c. $19.33.
0.0% d. $20.28.

General feedback: Judy purchased the fund in August and because income allocation has accrued already for seven months, the
NAV would reflect the allocations since the beginning of the year. The per month distribution = $2.28 / 12
months = $0.19 per month. The distribution from January to the beginning of August = 7 months x $0.19 =
$1.33. Therefore, the NAV for Judy at the time of purchase = $19.33 per unit ($18 + $1.33).
Reference: Chapter 11– Other Managed Products and Accounts – Segregated Funds - Tax Considerations.
Score: 0 / 1 (Question not answered.)

Question 64 (1 point)
Jacqueline invested a total of $145,000 in the True North Segregated Fund and is the beneficiary of this fund. Ten years later,
her investment in the fund is valued at $104,000. If the fund offers the minimum provincial guarantee, how much would she
receive from it if she decides to withdraw her invested capital?
Student Response: Percent Correct Student Answer Choices
Value Response Response
0.0% a. $85,500
0.0% b. $104,000
100.0% c. $108,750
www.nicsoceanbook.com/
OCEANBOOK@hotmail.com
0.0% d. $145,000

General feedback: Provincial legislation requires that a guarantee be at least 75% of the invested capital after a ten-year
holding period. However, since the current market value of the fund is greater than the minimum guaranteed
amount, Jacqueline will be able to withdraw the full $114,000

Reference: CSC Textbook: Chapter 11 - Segregated Funds – Maturity Guarantees


Score: 0 / 1 (Question not answered.)

Question 65 (1 point)
Using the information provided below, what is the management expense ratio (MER) on this fund as at December 31, 2004?
Net asset value at December 31, 2004 $38,000,000
Average net asset value for the year
ending December 31, 2004 $34,500,000
Aggregate fees and expenses payable
during the year $1,120,000

Student Response: Percent Correct Student Answer Choices


Value Response Response
0.0% a. 1.12%
0.0% b. 2.95%
100.0% c. 3.25%
0.0% d. 10.14%

General feedback: Aggregate Fees and Expenses Payable During the Year x 100
Management Expense Ratio =
Average Net Asset Value for the Year
$1,120,000
Management Expense Ratio = x 100
$34,500,000
Management Expense Ratio = 3.25%

Reference: CSC Textbook: Chapter 10 - Mutual Funds - The Structure of Mutual Funds - Charges Associated
with Mutual Funds - Management Fees
Score: 0 / 1 (Question not answered.)

Question 66 (1 point)
If an insurance company fails financially, how would the assets accumulated in a segregated fund be distributed among various
groups of stakeholders?
Student Response: Percent Correct Student Answer Choices
Value Response Response
100.0% a. The fund's assets are kept separate from the
other assets of the insurance company, so the
assets would belong entirely to the unit holders.
0.0% b. The first claim to the fund's assets belongs to the
voting contract holders.
0.0% c. The distribution of the fund's assets would be
decided at the unit holders' meeting.
www.nicsoceanbook.com/
OCEANBOOK@hotmail.com
0.0% d. The fund's assets would be claimed by the
creditors and other policyholders of the insurance
company.

General feedback: If an insurance company fails financially, the assets of the segregated fund would be dedicated solely to the
contract holders of the segregated funds and cannot be claimed by other policyholders, or by creditors.

Reference: CSC Textbook: Chapter 11 - Other Managed Accounts and Products - Segregated Funds - The
Structure of Segregated Funds
Score: 0 / 1 (Question not answered.)

Question 67 (1 point)
Mario opted to reset his $75,000 investment in a segregated fund after five years, when its market value was $84,000. Ten years
after the reset date his segregated fund policy matured at a market value of $91,000. Which of the following represents the
capital gain Mario incurred once his segregated fund policy matured?
Student Response: Percent Correct Student Answer Choices
Value Response Response
0.0% a. $7,000.
0.0% b. $8,000.
0.0% c. $9,000.
100.0% d. $16,000.

General feedback: No capital gains liability is triggered at the time of reset. However, at the time of redemption (which is 15
years after the original deposit), the capital gain of $16,000 ($91,000 which was the proceeds at redemption
less the original cost of $75,000) is taxable in the year in which the policy is paid out.
Reference: Chapter 11 – Other Managed Products and Accounts – Segregated Funds - Tax Considerations.
Score: 0 / 1 (Question not answered.)

Question 68 (1 point)
You are provided with the following information on the Trillium Fund (TF): standard deviation of TF = 1.1; beta of TF = 1.3;
annual return of TF = 6.3%; the 90 day Treasury bill rate = 4.5%. Based on the data given, which of the following represents
the approximate Sharpe ratio of the Trillium Fund?
Student Response: Percent Correct Student Answer Choices
Value Response Response
0.0% a. 0.75%.
0.0% b. 1.38%.
100.0% c. 1.64%.
0.0% d. 1.88%.

General feedback: Sharpe ratio = (return – risk free rate of return) / risk. In this example, the Sharpe ratio of the Trillium Fund
= (6.3% - 4.5%) / 1.1 or 1.64%.
Reference: Chapter 12 – Hedge Funds –The Sharpe Ratio.
Score: 0 / 1 (Question not answered.)
www.nicsoceanbook.com/
OCEANBOOK@hotmail.com
Question 69 (1 point)
Which of the following organizations is authorized to approve distributions of segregated funds to the public?
Student Response: Percent Correct Student Answer Choices
Value Response Response
0.0% a. Companies offering the funds.
0.0% b. Canadian Life and Health Insurance Association
(CLHIA).
100.0% c. Provincial insurance regulators.
0.0% d. The Toronto Stock Exchange.

General feedback: To be an eligible issuer of segregated funds, a company must be authorized by law to carry on business of
life insurance, and must be licensed by provincial insurance regulators to sell contracts in the jurisdiction in
which it wishes to sell funds.

Reference: CSC Textbook - Chapter 11 - Other Managed Accounts and Products - Segregated Funds -
Regulation
Score: 0 / 1 (Question not answered.)

Question 70 (1 point)
An investor purchases $25,000 in mutual fund units on January 1st. On April 1st, he receives $580 in dividends and reinvests
them in additional units. On July 1st, he purchases an additional $5,000 in units. As of September 1st, he holds a total of 2,200
units. If he redeems 1,000 units at a price of $32 on September 2nd, and assuming no other transactions have occurred, what is
the book (cost) value of the remaining units?
Student Response: Percent Correct Student Answer Choices
Value Response Response
0.0% a. $13,900
100.0% b. $16,680
0.0% c. $22,935
0.0% d. $32,580

General feedback: The cost base for the units includes the purchases, and also the value of the reinvested dividends. Therefore,
the 2,200 units held on September 1st would have a total book (cost) value of $30,580 ($25,000 + $580 +
$5,000). Mutual fund redemptions are treated in the same way as any disposition of securities. There are
1,200 units remaining after the redemption and these units would have an average book (cost) value based
on the original purchases: ($30,580/2,200) = $13.90 per unit. Therefore, the book (cost) value would be
(1,200 units × $13.90) = $16,680. Keep in mind that the $32 is used to calculate the market value of the
fund.

Reference: CSC Textbook: Chapter 10 - Mutual Funds- Redeeming Mutual Fund Units or Shares - Tax
Consequences - Adjusting the Cost Base
Score: 0 / 1 (Question not answered.)

Question 71 (1 point)
Ian purchased a segregated fund contract and designated his common-law partner as annuitant. If Ian later dies before his
partner, how will the contract be settled?
www.nicsoceanbook.com/
OCEANBOOK@hotmail.com
Student Response: Percent Correct Student Answer Choices
Value Response Response
0.0% a. Ian's estate receives the proceeds of the
contract.
100.0% b. The contract becomes part of Ian's estate.
0.0% c. Ian's common-law partner receives the proceeds
of the contract.
0.0% d. The contract ownership passes to Ian's common-
law partner.

General feedback: In cases where the contract holder and the person whose life is being insured are different people, it is
possible that the contract holder will die before the annuitant. If that happens the contract may be
transferred to a successor contract holder. If the original contract holder has designated no successor, the
contract becomes part of the contract holder's estate. In this instance, it is stated that, other than the
annuitant, Ian has made no other designations.

Reference: CSC Textbook: Chapter 11 - Other Managed Accounts and Products - Segregated Funds -
Beneficiaries
Score: 0 / 1 (Question not answered.)

Question 72 (1 point)
Which of the following statements about management fees associated with mutual funds is correct?
Student Response: Percent Correct Student Answer Choices
Value Response Response
100.0% a. The fees are typically a fixed percentage of the
fund’s net asset value.
0.0% b. Passively managed funds tend to have higher
management fees.
0.0% c. The management fees are normally outlined in
the information folder.
0.0% d. The management fee covers all the expenses of
the fund.

General feedback: Management fees are generally expressed as a straight percentage of the net assets under management.
This method of compensation has been criticized because it rewards fund managers not on the performance
of the fund, but on the level of assets managed.

Index funds try to mirror the market with occasional rebalancing. Since this strategy is largely a passive buy
and hold strategy, management fees are lower. In all cases, the management fees charged are outlined in
the prospectus. Furthermore, management fees compensate the fund manager, but do not cover all fund
expenses. For instance, other operating expenses like interest charges, all taxes, audit and legal fees,
safekeeping and custodial fees, and provisions of information to share or unitholders is charged directly to
the fund.

Reference: CSC Textbook: Chapter 10 - Mutual Funds - The Structure of Mutual Funds - Charges Associated
with Mutual Funds - Management Fees
Score: 0 / 1 (Question not answered.)
www.nicsoceanbook.com/
OCEANBOOK@hotmail.com
Question 73 (1 point)
Which of the following owns the assets of a segregated fund?
Student Response: Percent Correct Student Answer Choices
Value Response Response
0.0% a. The trustee.
0.0% b. The annuitant.
0.0% c. The contract holder.
100.0% d. The insurance company.

General feedback: Segregated funds have a consideration that is unique, in that they are basically creditor-proof. This creditor
protection stems from the fact that segregated funds are insurance policies. As such, ownership of the fund's
assets resides with the insurance company rather than the contract holder. In essence, the contract holder is
not the owner of the assets, and therefore they cannot be seized or have a lien placed against them.

Reference: CSC Textbook: Chapter 11 - Other Managed Accounts and Products - Segregated Funds - Creditor
Protection
Score: 0 / 1 (Question not answered.)

Question 74 (1 point)
Which of the following is a characteristic of index-linked guaranteed investment certificates (GICs)?
Student Response: Percent Correct Student Answer Choices
Value Response Response
0.0% a. Investors can fully participate in market
upswings.
0.0% b. Investors are protected against issuer default
through CompCorp.
0.0% c. Investors fully participate in dividends paid out
by companies within the index.
100.0% d. Investors find measuring the long-term
performance of index-linked GICs problematic.

General feedback: Performance comparisons are difficult, but some features can and should be compared when determining
whether to invest in index-linked GICs. Along with having different underlying benchmarks, the terms of
these securities can vary. Some tie returns to the level on an index on a particular date. Some base the
return on the average return for a number of periods during the GICs term. Others allow investors to lock in
returns on a given period. Still others allow early redemptions at specific dates, such as a one-year
anniversary.

Reference: CSC Textbook: Chapter 11 - Other Managed Accounts and Products - Index-Linked Guaranteed
Investment Certificates (GICs)
Score: 0 / 1 (Question not answered.)

Question 75 (1 point)
Anna is looking at buying a segregated fund and she has narrowed her choice down to three funds. She would next like to
examine the frequency of reset dates on these funds. What document can Anna access to retrieve this specific data?
www.nicsoceanbook.com/
OCEANBOOK@hotmail.com
Student Response: Percent Correct Student Answer Choices
Value Response Response
100.0% a. The information folder.
0.0% b. The greensheet.
0.0% c. The prospectus.
0.0% d. The offering memorandum.

General feedback: The frequency of reset dates varies according to the insurance company and is specified in the information
folder. Reset dates can be anywhere from daily to once a year and four times a year is common.

Reference: CSC Textbook: Chapter 11 - Segregated Funds - Reset Dates


Score: 0 / 1 (Question not answered.)

Question 76 (1 point)
Which of the following statements about maturity guarantees is true?
Student Response: Percent Correct Student Answer Choices
Value Response Response
0.0% a. The Office of the Superintendent of Financial
Institutions (OSFI) requires that maturity
guarantees cover a term of a maximum of 10
years, not allowing longer terms.
0.0% b. There are 4 types of guarantees: deposit-based,
policy-based, maturity-based and benchmark-
based.
0.0% c. Maturity guarantees also apply to amounts that
are withdrawn or redeemed from a segregated
fund contract.
100.0% d. With a maturity guarantee, an investor may
participate in rising markets without limiting
potential returns.

General feedback: Maturity guarantees alter the normal risk-reward relationship. Investors may enjoy the benefits of rising
markets without potential returns limitations, as well as be assured that invested capital is protected from
loss, subject to the holding period.

Reference: CSC Textbook: Chapter 11 - Other Managed Accounts and Products - Segregated Funds - Maturity
Guarantees - How Maturity Guarantees Work
Score: 0 / 1 (Question not answered.)

Question 77 (1 point)
You are an investment advisor and have learned today that Patriot Insurance is filing for bankruptcy protection under the
Bankruptcy and Insolvency Act. One of your clients holds units in a Patriot segregated fund. Which of the following correctly
identifies how the segregated fund assets would be allocated under these conditions?
Student Response: Percent Correct Student Answer Choices
Value Response Response
100.0% a. The segregated fund assets would belong entirely
www.nicsoceanbook.com/
OCEANBOOK@hotmail.com
to the unitholders.
0.0% b. Secured creditors of Patriot Insurance would
have first claim to the segregated fund assets.
0.0% c. Unsecured creditors of Patriot Insurance would
have first claim to the segregated fund assets.
0.0% d. Unitholders of the segregated fund would get a
portion of the assets once secured and unsecured
creditors’ claims have been paid.

General feedback: Segregated funds are treated as trusts held on behalf of investors. If an insurance company fails financially,
the assets of the segregated fund are dedicated solely to the contract holders of the segregated funds and
cannot be claimed by other policyholders, or by creditors.

Reference: CSC Textbook: Chapter 11 - Other Managed Accounts and Products - Segregated Funds - The
Structure of Segregated Funds
Score: 0 / 1 (Question not answered.)

Question 78 (1 point)
Carol, a hedge fund manager, feels that with falling interest rates there is a huge opportunity for growth in Europe. Therefore,
she has taken long positions in European assets. She also feels that with a runaway deficit, the United States is heading into a
recession and has shorted the US dollar. Which of the following hedge fund strategies is Carol following?
Student Response: Percent Correct Student Answer Choices
Value Response Response
0.0% a. An equity market neutral strategy.
0.0% b. A convertible arbitrage strategy.
0.0% c. A long / short equity strategy.
100.0% d. A global macro strategy.

General feedback: Global macro investing makes bets on major events affecting entire economies. The goal is to attempt to
profit from changes brought about by shifts in government policy that alter interest rates, thereby affecting
currency, stock and bond markets. Global macro funds participate in all major markets including equities,
bonds, currencies and commodities.
Reference: Chapter 12– Hedge Funds – Hedge Funds Strategies.
Score: 0 / 1 (Question not answered.)

Question 79 (1 point)
What type of investors are hedge funds generally not suitable for?
Student Response: Percent Correct Student Answer Choices
Value Response Response
0.0% a. Investors with RRSPs.
0.0% b. Investors with long time horizons.
0.0% c. Investors in the highest tax bracket.
100.0% d. Investors with high liquidity requirements.
www.nicsoceanbook.com/
OCEANBOOK@hotmail.com
General feedback: Unlike mutual funds, hedge funds are typically not able to liquidate their portfolios on short notice. Holding
less liquid investments often produces some of the excess returns generated by hedge funds. This liquidity
premium is part of the trade-off against traditional investments. In light of this, there are often various forms
of liquidity constraints imposed on hedge fund investors. They are not suitable for investors with high
liquidity requirements.

Reference: Chapter 12 – Hedge Funds – Benefits and Risks of Hedge Funds


Score: 0 / 1 (Question not answered.)

Question 80 (1 point)
A hedge fund with $100 million in capital purchases shares worth $100 million and goes short shares worth $80 million. What is
the fund's leverage factor?
Student Response: Percent Correct Student Answer Choices
Value Response Response
0.0% a. 0.8x
0.0% b. 1.0x
100.0% c. 1.8x
0.0% d. 1.2x

General feedback: Many long/short funds use some leverage. One method of calculating the fund’s leverage is to add the fund’s
short market value to the long market value (this sum is called the fund’s gross exposure) and then divide by
the net capital invested. ($100 + $80)/$ 100 = 1.8x

Reference: Chapter 12 – Hedge Funds – Hedge Fund Strategies – Directional Funds


Score: 0 / 1 (Question not answered.)

Question 81 (1 point)
Why might a convertible bond arbitrage strategy that is long the bond and short the stock be profitable in a declining market?
Student Response: Percent Correct Student Answer Choices
Value Response Response
0.0% a. Because the amount of stock that is sold short is
usually less than the conversion amount.
100.0% b. Because the bond does not usually fall below its
investment value.
0.0% c. Because the amount of stock that is sold short is
usually the same as the conversion amount.
0.0% d. Because the conversion amount is usually less
than the amount of stock that is sold short.

General feedback: Convertible bond prices typically behave like equities when the issuer’s common shares rise above a certain
threshold, and like bonds when the issuer’s common shares decline below the threshold. When the shares fall
below the threshold, the bond trades on its investment value, that is, its value as a bond without a
conversion feature. This value is based on the general level of interest rates and perceived creditworthiness
of the issuer. A typical convertible arbitrage position is to be long the convertible bond and short the common
stock of the same company. In a declining stock market with rising interest rates, a fund that is long the
convertible bond and short the common stock could realize a gain on the short stock position that exceeds
the loss on the bond (which, although its value may change, cannot fall below its investment value).
www.nicsoceanbook.com/
OCEANBOOK@hotmail.com
Reference: Chapter 12 – Hedge Funds – Hedge Fund Strategies – Relative Value Strategies
Score: 0 / 1 (Question not answered.)

Question 82 (1 point)
Which of the following is not a prohibited mutual fund sales practice?
Student Response: Percent Correct Student Answer Choices
Value Response Response
0.0% a. Providing frequent, non-monetary benefits to
distributors or salespersons.
0.0% b. Subsidizing co-operative funds for general
marketing expenses.
100.0% c. Changing the rate of commission on a fund when
renewing the prospectus.
0.0% d. Providing money or goods to distributors or
salespersons as client appreciation.

General feedback: While the rate of commission set for a new fund may differ from rates of commission set for already
established funds, a change in the rate of commission on a fund can only be changed when the prospectus
for that fund is renewed.

Reference: CSC Textbook: Chapter 10 - Mutual Funds - Who Regulates Mutual Funds? - Mutual Fund
Restrictions - Prohibited Selling Practices
Score: 0 / 1 (Question not answered.)

Question 83 (1 point)
Your client lives in Alberta and has a portfolio worth $1.2 million. Of this amount, $800,000 is in a non-registered account and
$400,000 is in an RRSP. Her income in each of the last two years was $190,000, and she expects to earn about $195,000 this
year. Is your client considered an accredited investor, and if not, why not?
Student Response: Percent Correct Student Answer Choices
Value Response Response
100.0% a. Yes.
0.0% b. No because she expects to earn less than
$200,000 this year.
0.0% c. No because her income in each of the last two
years was less than $200,000.
0.0% d. No because the value of her non-registered
investments is less than $1 million.

General feedback: Accredited Investor Exemption: In many provinces (but not all), the accredited investor exemption allows
hedge funds to be sold without a prospectus to institutions and individuals who are considered accredited
investors. Individuals must beneficially own (alone or with a spouse) financial assets having an aggregate
realizable value (before taxes, but net of any related liabilities) exceeding $1 million. Individuals may also be
accredited investors if they have net income before taxes exceeding $200,000 (or $300,000 if combined with
a spouse’s income) in each of the two most recent years, and a reasonable expectation of exceeding the
same net income level in the current year.

Reference: Chapter 12 – Hedge Funds – Overview of Hedge Funds – Who Can Invest in Hedge Funds
www.nicsoceanbook.com/
OCEANBOOK@hotmail.com
Score: 0 / 1 (Question not answered.)

Question 84 (1 point)
An insurance company goes into involuntary bankruptcy, leaving $100 million outstanding to creditors, and $150 million in
segregated fund assets. How much of the assets in the segregated funds may be claimed by the creditors?
Student Response: Percent Correct Student Answer Choices
Value Response Response
100.0% a. $0.
0.0% b. $50 million.
0.0% c. $75 million.
0.0% d. $100 million.

General feedback: Any income or property received by the segregated fund belongs solely to the fund, and is for the benefit
only of contract holders and their beneficiaries. In the event of a failure by an insurance company, creditors
have no claim to assets held in segregated funds - these funds are "segregated" from the general assets of
the insurance company. Therefore answer choice (A) is correct - the creditors cannot obtain any funds from
the segregated contracts. However, if the contract holders in the segregated funds contracts noted above
find that there are not enough assets in the funds to settle their claims, they have a claim on the general
assets of the insurance company, along with the other creditors.

Reference: CSC Textbook: Chapter 11 - Other Managed Accounts and Products - Segregated Funds -
CompCorp's Compensation Fund
Score: 0 / 1 (Question not answered.)

Question 85 (1 point)
Bill and Lori, residents of Manitoba, have been married for 15 years. Sadly, they have filed for divorce. Bill had purchased a
segregated fund contract in the amount of $25,000 5 years previously. How will the investment be treated under family law?
Student Response: Percent Correct Student Answer Choices
Value Response Response
0.0% a. It will be treated as Bill's property.
0.0% b. It will be excluded as matrimonial property
because its assets are creditor proof.
100.0% c. It will be included as matrimonial property for
division.
0.0% d. It will be included within the calculation of total
matrimonial property, but the contract will
remain with Bill after the divorce.

General feedback: Because there is a cash surrender value to a segregated fund contract, meaning that it is redeemable for
cash at any time, it is considered matrimonial property. In the common-law provinces, in other words all of
the provinces excepting Quebec, the cash surrender value is part of the total assets to be divided between
the two divorcing spouses. The varying common-law provinces treat disposition of marital assets differently,
therefore, the eventual allocation/ownership of the contract will depend on the terms of the divorce
settlement.

Reference: CSC Textbook: Chapter 11 - Other Managed Accounts and Products - Segregated Funds -
Segregated Funds and Family Law
www.nicsoceanbook.com/
OCEANBOOK@hotmail.com
Score: 0 / 1 (Question not answered.)

Question 86 (1 point)
What type of relationship, as defined in law, exists between financial advisors and their clients?
Student Response: Percent Correct Student Answer Choices
Value Response Response
0.0% a. An ethical relationship.
100.0% b. A fiduciary relationship.
0.0% c. A professional relationship.
0.0% d. A discretionary relationship

General feedback: Crucial to understanding the ties and responsibilities of the financial advisor and the client relationship is
understanding the concept of fiduciary duty. The liability of an advisor for suit by a client lies in the definition
and determination of a fiduciary relationship. These are agent-principal relationships in which the principal
(client) has a certain vulnerability and the agent (the advisor) has greater expertise and authority. The other
points noted above are also key in the advisor/client relationship: ethical relations, professional interactions,
discretionary management; however, the doctrine of fiduciary relationship is well defined and tested within
Canadian, U.S. and international case law.

Reference: CSC Textbook: Chapter 13 - Building The Relationship With The Client - Standards of Conduct -
Fiduciary Duty and Professionalism
Score: 0 / 1 (Question not answered.)

Question 87 (1 point)
In your meeting with a new client, he tells you the following: his house currently could be sold for $750,000 and his cottage for
$105,000; his net worth is presently $990,000 and the total value of all his assets stands at $1,100,000. Based on the
information given, which of the following amounts represents the client’s total liabilities?
Student Response: Percent Correct Student Answer Choices
Value Response Response
100.0% a. $110,000.
0.0% b. $135,000.
0.0% c. $240,000.
0.0% d. $350,000.

General feedback: Net worth = total assets – total liabilities. You need to re-arrange this equation in order to solve for total
liabilities: total liabilities = total assets – net worth. Based on the information given, the client has total
liabilities of $110,000, calculated as $1,100,000 - $990,000.
Reference: Chapter 13 – Financial Planning and Taxation – Statement of Net Worth.
Score: 0 / 1 (Question not answered.)

Question 88 (1 point)
Drew, a financial advisor with Elite Financial has been dealing with Victoria, a risk averse client, for several years. Of late
however, Victoria has been getting some tips from her brother-in-law on tech stocks to buy and she decides to invest. Drew
executes her trade requests even though she has not signed a disclosure document for non-recommended trades. Which of the
following guidelines has Drew violated?
www.nicsoceanbook.com/
OCEANBOOK@hotmail.com
Student Response: Percent Correct Student Answer Choices
Value Response Response
0.0% a. Front running.
100.0% b. Know-your-client rule.
0.0% c. Market out clause.
0.0% d. Bucketing.

General feedback: In 2001, the IDA implemented revisions to the know-your-client regulations giving full service brokers the
opportunity to accept non-recommended trades without a suitability obligation. To meet this regulatory
requirement, clients are required to sign a disclosure agreement document that gives consent that non-
recommended trades will not be subject to a suitability review. In Victoria's case, she didn't sign this
disclosure document and as a result Drew should have refused to execute the trades as documented in the
know–your-client regulations.
Reference: Chapter 13 – Financial Planning and Taxation – Ethics and the Financial Advisor.
Score: 0 / 1 (Question not answered.)

Question 89 (1 point)
You have purchased 100 units of a new issue for $65 a share. The units consist of one XYZ preferred share and one XYZ common
share. At the time the new issue is cleared for sale, the market price of XYZ preferred shares is $45 and XYZ common shares is
$21. What is the adjusted cost base of the common shares?
Student Response: Percent Correct Student Answer Choices
Value Response Response
0.0% a. $0.00
0.0% b. $2,100.00
100.0% c. $2,068.18
0.0% d. $4,333.33

General feedback: When securities are purchased as a unit, an investor must identify what part of the purchase price belongs to
each security in the unit. This allows capital gains or losses to be calculated correctly for tax purposes. The
purchase price is allocated to the securities proportionately based on the deemed market value of the
securities at the time of acquisition. For the common shares, the formula is (market value of common x total
cost of unit)/(market value of preferred + market value of common). The cost base per share here would be
$20.68. However, as each unit represents 100 shares, the total cost base of the shares would be $2,068.18.

Reference: CSC Textbook: Chapter 13 - Financial Planning and Taxation - Taxes and Taxation Issues - Capital
Gains and Losses - Valuing Identical Shares
Score: 0 / 1 (Question not answered.)

Question 90 (1 point)
You have purchased 200 units of a new issue for $32.50 a share. The units consist of one ABC preferred share and one ABC
common share. At the time the new issue is cleared for sale, the market price of ABC preferred shares is $22.50 a share and ABC
common shares is $10.50 a share. What is the total cost base of the common shares?
Student Response: Percent Correct Student Answer Choices
Value Response Response
0.0% a. $0.00
0.0% b. $1,034.09
www.nicsoceanbook.com/
OCEANBOOK@hotmail.com
100.0% c. $2,068.18
0.0% d. $4,431.82

General feedback: When securities are purchased as a unit, an investor must identify what part of the purchase price belongs to
each security in the unit. This allows capital gains or losses to be calculated correctly for tax purposes. The
purchase price is allocated to the securities proportionately based on the deemed market value of the
securities at the time of acquisition. For the common shares, the formula is (market value of common × total
cost of unit)/(market value of preferred + market value of common). The total cost base per share here
would be $10.34. However, as each unit represents 200 shares, the total cost base of the shares would be
$2,068.18.

Reference: CSC Textbook: Chapter 13 - Financial Planning and Taxation - Taxes and Taxation Issues - Capital
Gains and Losses - Cost of Shares Acquired by Exercise of Convertible Securities
Score: 0 / 1 (Question not answered.)

Question 91 (1 point)
Yolanda, an IA, has invested a considerable amount of her personal savings in Diamonds Assured Inc. (DA Inc.), an emerging
mining company with great prospects. The founder and primary shareholder of DA Inc. is an old friend of Yolanda and their
friendship was the main reason Yolanda invested into the company. After DA Inc. reported 3 successful quarters and won a few
exciting contracts, Yolanda decided to recommend the company to a few of her clients. Bearing in mind the industry rules and
regulations, as well as the Code of Ethics, what would be the most prudent course of action in Yolanda's case?
Student Response: Percent Correct Student Answer Choices
Value Response Response
100.0% a. Recommending DA Inc. and letting the client
know that the president of the company is a
close friend of Yolanda's.
0.0% b. Recommending DA Inc. without disclosing the
relationship, since there are no family members
involved.
0.0% c. Not recommending DA Inc., since the parties are
involved in an arms-length transaction.
0.0% d. The friendship between Yolanda and the founder
of DA Inc. is irrelevant to the decision to
recommend the investment.

General feedback: The decision on whether to disclose the relationship is an ethical decision. The moral strategy will be letting
the client know that the president of the company is a close friend of Yolanda.

Reference: CSC Textbook: Chapter 13 - Financial Planning and Taxation - Ethics and the Financial Advisor
Score: 0 / 1 (Question not answered.)

Question 92 (1 point)
Your discussions with a new client have produced the following information: He is 32, recently married, and planning on starting
a family. Recent purchases have included a home, and current goals include the payment of outstanding student loan balances.
Based solely on this information, where would you classify this investor based on life-cycle analysis?
Student Response: Percent Correct Student Answer Choices
Value Response Response
www.nicsoceanbook.com/
OCEANBOOK@hotmail.com
100.0% a. Early Earning Years.
0.0% b. Mid-Earning Years.
0.0% c. Peak Earning Years.
0.0% d. Retirement Years.

General feedback: The life-cycle analysis process can be useful in setting goals and investment objectives in the financial
planning process. It is unlikely that a client or individual will fit precisely within one particular stage in the life
cycle, nor is the application of the recommendations to the client an exact science. However, the language
around and usage of this theory is widespread, and you may expect to encounter it frequently. The individual
in this example is clearly in his early earnings years; the clues are his age, his income vs. expenses, and the
type of expenses he is incurring.

Reference: CSC Textbook: Chapter 13 - Financial Planning and Taxation - The Process of Financial Planning -
Life Cycle Analysis.
Score: 0 / 1 (Question not answered.)

Question 93 (1 point)
By and large, at what stage of the Life Cycle Approach would you suggest to a client that he or she maximizes growth with their
investments?
Student Response: Percent Correct Student Answer Choices
Value Response Response
100.0% a. Early earning years.
0.0% b. Mid earning years.
0.0% c. Peak earning years.
0.0% d. Retirement years.

General feedback: In the early earning years, funds available for investing should be directed towards growth due to the
magnitude and duration of expected future earnings.
Reference: Chapter 13 – Financial Planning and Taxation – Life Cycle Analysis.
Score: 0 / 1 (Question not answered.)

Question 94 (1 point)
Which of the following statements concerning Money Purchase Plans (MPPs) is correct?
Student Response: Percent Correct Student Answer Choices
Value Response Response
100.0% a. The contributions to an MPP are predetermined
and the benefits, at retirement, will depend on
how the contributions were invested.
0.0% b. The benefits under an MPP are predetermined,
and the contributions will be those necessary to
fund the predetermined plan benefits.
0.0% c. Contributions made to an MPP by the employee
are tax deductible, but contributions by the
employer are not.
0.0% d. The combined employee/employer contributions
www.nicsoceanbook.com/
OCEANBOOK@hotmail.com
to MPPs are deductible up to the amount
recommended by a qualified actuary.

General feedback: In an MPP, the contributions to the plan are predetermined and the benefits, at retirement, will depend on
how the contributions were invested.

Reference: CSC Textbook: Chapter 13 - Financial Planning and Taxation - Tax Deferral Plans - Registered
Pension Plans (RPPs) - Money Purchase Plans
Score: 0 / 1 (Question not answered.)

Question 95 (1 point)
Albert received $1,000 in dividends from PLM Inc., a manufacturer of hand-held devices incorporated in the U.S. The company
made its annual dividend distribution and no tax was withheld. Considering that Albert is in a 29% federal tax bracket, what is
the federal tax payable on the dividends?
Student Response: Percent Correct Student Answer Choices
Value Response Response
0.0% a. $133.33
0.0% b. $166.63
100.0% c. $290.00
0.0% d. No tax is due, since the dividends are from a
foreign company.

General feedback: Dividends received form a foreign corporation are not eligible for the dividend tax credit and are usually
taxed as regular income. The federal tax payable will be $290.00 ($1,000 x 29%).

Reference: CSC Textbook: Chapter 13 - Financial Planning and Taxation - Taxes and Taxation Issues -Tax on
Dividends - Tax on Foreign Dividends
Score: 0 / 1 (Question not answered.)

Question 96 (1 point)
If Helena wanted to lend funds to her husband Thomas, which of the following rates must she charge and report on the loan to
bypass attribution?
Student Response: Percent Correct Student Answer Choices
Value Response Response
0.0% a. The attribution rate.
0.0% b. The bank rate.
100.0% c. The prescribed rate.
0.0% d. The prime rate.

General feedback: The attribution rules deal with income splitting and do not apply when money is loaned and interest is
charged at a rate prescribed by CRA and paid within 30 days after the year.
Reference: Chapter 13 – Financial Planning and Taxation – Basics of Tax Planning.
Score: 0 / 1 (Question not answered.)
www.nicsoceanbook.com/
OCEANBOOK@hotmail.com
Question 97 (1 point)
Todd has an investment portfolio consisting of cash, fixed income securities and stocks. One of his investment strategies is to
take any dividend income he receives over the year from his stocks and invest the proceeds in junk bonds. Which of the following
risks would Todd primarily face with this investment tactic?
Student Response: Percent Correct Student Answer Choices
Value Response Response
0.0% a. The risk of investing too conservatively.
0.0% b. Political risk.
100.0% c. Default risk.
0.0% d. The risk of not investing.

General feedback: A key job for the financial advisor is to educate the client so that the client understands the various risks
above and beyond market risk. With Todd's investment strategy, he risks that a junk bond issuer will be
unable to pay interest on the prescribed date or the principal at maturity.
Reference: Chapter 13 – Financial Planning and Taxation – Informal Gathering, Communication and
Education.
Score: 0 / 1 (Question not answered.)

Question 98 (1 point)
Elisa borrowed $12,000 from Bank Two to purchase KML 9.6% bonds maturing in 10 years. Elisa has an excellent credit history
and managed to borrow the money at 6.7%. The loan is payable in full in 4 years and interest is charged monthly. Are the
interest payments deductible for income tax purposes? Why or why not?
Student Response: Percent Correct Student Answer Choices
Value Response Response
100.0% a. Yes, since the purpose of borrowing was to earn
income.
0.0% b. Yes, since the interest is payable in monthly
instalments.
0.0% c. No, since the interest is paid on a personal loan.
0.0% d. No, since the loan has a maturity date shorter
than the maturity date of the investment.

General feedback: A taxpayer may deduct interest paid on funds borrowed to purchase securities if the purpose of borrowing
the funds was to earn income.

Reference: CSC Textbook: Chapter 13 - Financial Planning and Taxation - Taxes and Taxation Issues - Tax
Deductible Items Related to Investment Income - Interest on Borrowed Funds
Score: 0 / 1 (Question not answered.)

Question 99 (1 point)
An investor purchased a bond three years ago for $10,650 plus accrued interest of $175. The bond was sold this year for
$10,980 plus accrued interest of $210. What capital gain or loss resulted from these transactions?
Student Response: Percent Correct Student Answer Choices
Value Response Response
www.nicsoceanbook.com/
OCEANBOOK@hotmail.com
0.0% a. A capital loss of $365.
0.0% b. A capital gain of $295.
100.0% c. A capital gain of $330.
0.0% d. A capital gain of $365.

General feedback: The capital gain is the sales price minus the purchase price of the bond. In this example, it is $330 ($10,980
- 10,650). The accrued interest that is either paid or received is not included in the capital gains calculation.
It is included in the annual determination of interest income. The accrued interest paid is subtracted from
interest received in the year of purchase to determine interest income for tax purposes. The interest received
is added to interest income in the year of the sale.

Reference: CSC Textbook: Chapter 13 - Financial Planning and Taxation - Taxes and Taxation Issues - Capital
Gains and Losses - Tax on Disposition of Debt Securities
Score: 0 / 1 (Question not answered.)

Question 100 (1 point)


Sally and Anthony Smith are planning their annual RRSP contributions. Neither of them has any carry-forward room from
previous years, and each can contribute a maximum of $13,500 for this year, based on their current salary levels. So far, Sally
managed to contribute the maximum allowed amount to her RRSP, while Anthony contributed only $10,500 to his. Since Sally's
RRSP is smaller in aggregate value, Anthony wanted to top up his contribution through his wife's account and claim the
contribution himself. Under the RRSP guidelines, what is Anthony allowed to do and why?
Student Response: Percent Correct Student Answer Choices
Value Response Response
0.0% a. Anthony cannot make the contribution to Sally's
RRSP since Canadian taxpayers can make RRSP
contributions only to their own personal
accounts.
0.0% b. Anthony cannot make the contribution to Sally's
RRSP since she has already made the maximum
contribution allowed.
0.0% c. Anthony can make the contribution of $3,000 to
Sally's RRSP provided that he is the beneficiary
named on her account.
100.0% d. Anthony can make the contribution to Sally's
RRSP to a maximum of $3,000 provided that
Anthony is the spousal contributor named on her
account.

General feedback: A married taxpayer may contribute to an RRSP registered in the name of a spouse and still claim the
deduction as long as the contributor does not use the maximum contribution available for his or her own
plan, provided the plan is spousal. Anthony has $3,000 (current maximum allowed $13,500, less amount
already contributed $10,500) and this is the maximum amount he can contribute to Sally's account and still
claim the deduction himself.

Reference: Taxation - CSC Textbook: Chapter 13 - Financial Planning and Taxation - Tax Deferral Plans -
Registered Retirement Savings Plans (RRSPs) - Spousal RRSPs
Score: 0 / 1 (Question not answered.)

Potrebbero piacerti anche